Tổng Hợp Đề Đề Xuất Kì Thi Hsg Khu Vực Duyên Hải Và Đồng Bằng Bắc Bộ Môn Tiếng Anh Khối 10, 11 Năm 2023 (Có Đáp Án Và Phần Nghe) - PDFCOFFEE.COM (2024)

ĐỀ THI DUYÊN HẢI BẮC BỘ MÔN TIẾNG ANH

vectorstock.com/31086119

Ths Nguyễn Thanh Tú eBook Collection

TỔNG HỢP ĐỀ ĐỀ XUẤT KÌ THI HSG KHU VỰC DUYÊN HẢI VÀ ĐỒNG BẰNG BẮC BỘ MÔN TIẾNG ANH KHỐI 10, 11 NĂM 2023 (CÓ ĐÁP ÁN VÀ PHẦN NGHE) WORD VERSION | 2023 EDITION ORDER NOW / CHUYỂN GIAO QUA EMAIL [emailprotected]

Tài liệu chuẩn tham khảo Phát triển kênh bởi Ths Nguyễn Thanh Tú Đơn vị tài trợ / phát hành / chia sẻ học thuật : Nguyen Thanh Tu Group Hỗ trợ trực tuyến Fb www.facebook.com/DayKemQuyNhon Mobi/Zalo 0905779594

I.

LISTENING (50 points)

L

ĐỀ THI MÔN ANH KHỐI 10 NĂM 2023 Thời gian làm bài 180 phút Đề thi gồm …..trang

FI CI A

HỘI CÁC TRƯỜNG THPT CHUYÊN VÙNG DUYÊN HẢI VÀ ĐỒNG BẰNG BẮC BỘ TRƯỜNG THPT CHUYÊN BIÊN HÒA TỈNH HÀ NAM ĐỀ THI ĐỀ XUẤT

Part 1. You will hear part of an interview with a music journalist called Pip Rogers and a musician called Heath Francis about the renewed popularity of vinyl records. For

NH

ƠN

A It has not created as much income as expected. B It is happening only now rather than in the 1990s. C. Some of the old record factories haven’t closed down. D Vinyl manufacturers are struggling to meet demand. 2. In Heath's opinion, what is the main attraction of records? A. They are an object you can possess. B. They can become a serious hobby. C. They are often cheaper than CDs.

OF

questions 1- 6, choose the best answer (A, B, C or D). (20 points) 1. What surprised Pip about the comeback of vinyl records?

M

QU Y

D. They are durable and long lasting. 3. What does Pip say about young people and records? A. They prefer listening to current music on records. B. Vinyl records offer them a novel experience. C. They are frustrated by vinyl because of its limitations. D They have alternative ways of expressing their musical tastes. 4. When asked about the future of records, Pip reveals she is A. confident that people will want to invest in the industry B. unsure whether the requirements for growth will be met. C. concerned people will lose interest in vinyl eventually.

Y

D. surprised that vinyl is perceived as a passing trend. 5. What both Pip and Health appreciate about digital music is A. not having to pay for every song you hear. B. being able to listen to music anywhere. C. having access to a wide variety of music. D. being able to share music instantaneously. Your answers:

DẠ

1.

2.

3.

4.

1

5.

FI CI A

L

Part 2. Listen to a podcast on climate change and decide whether the following statements are True (T), False (F), or Not Given (NG) according to what you hear. (10 points) 1._________ A 1.5 degree rise in temperature makes virtually no difference to the world we are living in. 2._________ There is little likelihood of the aviation undergoing carbon neutral due to some governments’ objections. 3._________ Every year, the number of people suffering from diseases related to exceptional

heatwaves increases by 65 million. 4._________ Notwithstanding the melted ice rivers, the number of people lacking water for daily

Your answers: 2.

3.

4.

5.

ƠN

1.

OF

lives will increase twofold. 5._________ Continually rising carbon emission is synonymous with a world ridden with disasters.

M

QU Y

NH

Part 3. Listen to a talk about the unique characteristics of glass and fill each blanks with NO MORE THAN FOUR WORDS. (20 points) Glass is technically known as any (1)_________________________, whose structures of atoms and molecules are in disorder. In ice, the water molecules (2)____________________________ each other and lock themselves into a repeating crystal pattern. As glass cools, its molecules (3)_________________________ until they stop moving altogether. One study from 2017 estimated that if a cathedral were to stand at room temperature for a billion years, it’s glass would flow (4)____________________________________. Another research team from Spain examined samples of 110 million-year-old amber, a naturally occurring variety of glass (5)_______________________________, and found that over its long existence it had become about (6)__________________________________. Glass, if cooled slowly and infinitely until it hardens, would have (7)_____________________________ like a crystal’s. Non-ideal glass is thought to be riddled with two-level systems, which, when going near absolute

DẠ

Y

zero, can (8)________________________________, absorbing heat in the process. In a very different glassmaking technique that makes use of (9)_________________________, glass is built one molecule at a time. Old cathedral glass is believed to be thicker at the bottom because it’s (10)___________________________. However, that’s just due to the technique used to make the glass.

2

L

II. LEXICO-GRAMMAR (40 points) Part 1. Choose the correct answer (A, B, C or D) to complete the sentences. (20 points)

D. line

3. The dog’s owner has no idea what may have _________ the attack. A. spelled B. provoked C. embodied

D. heralded

OF

4. I’m not sure I believe Jason’s story- it is a bit_________. A. plausible B. inhospitable C. far-fetched

FI CI A

1. Walking down the street, I noticed that_________ had a beautiful garden. A. entire houses B. either of the houses C. each and every house D. enough houses 2. The fire spread quickly, leaving devastation in its _________. A. zone B. blaze C. wake

D. endless

QU Y

NH

ƠN

5. His health was gradually ________ by drink and drugs. A. ravaged B. injured C. destroyed D. rectified 6. ________, I had to buy a new one. A. My dictionary losing B. My dictionary had been lost C. My dictionary having been lost D. Because my dictionary lost 7. The sheep were huddled into a ____________ to protect them from overnight frosts. A. cage B. kennel C. pen D. hutch 8. We stand on the ____________ of a new era in space exploration. A. threshold B. basis C. brink D. surface 9. Three lives were lost in an accident at the____________ of a driver who had been drinking. A. mercy B. expense C. high D. hands 10. The _______ of chickens in cages is cruel, so many farmers allow them to wander freely.

Y

M

A. constraint B. confinement C. distress D. slaughter 11. We can’t always rely on ________ on time. A. the buses that arriving B. the arriving of buses C. the buses to arrive D. the buses’ arriving 12. The ________ of marriage in Viet Nam remains popular in spite of the high divorce rate recently. A. state B. habit C. practice D. institution 13. At the ________ of organizing committee, personnel must wear their identity badges at all time. A. request B. assistance C. demands D. interests 14. IU is known for being a ________ girl who can take the roles of a singer, a song writer and an actress as well. A. versatile B. changeable C. variable D. diverse 15. This isn’t a family any longer, no one seems to care about solving ________ problem.

DẠ

A. anyone’s else B. anyone else’s C. any other D. any other’s 16. Since the torture scandal several months ago, there have been ill ________ between area residents and police. A. emotions B. feelings C. affections D. sensation 3

FI CI A

L

17. The new development project of Phu Ly city has begun to ________ on the surrounding green belt. A. enter B. intrude C. encroach D. reach 18. Ha Nam has been attracting foreign investment due to the present favorable political ________ of the province. A. temperature B. climate C. state D. weather 19. Don’t worry, Will’s been going through a rebellious phase as any other teenager. He’ll gradually ________ it. A. grow into

B. get through to

OF

C. grow out of D. get on with it 20. I’ve ________ as inconsiderate a person as Chander. A. ever to have met B. been met C. yet to meet D. never been meeting

ƠN

Your answers: 1.

2.

3.

4.

5.

6.

7.

8.

9.

10.

11.

12.

13.

14.

15.

16.

17.

18.

19.

20.

3. 4. 5. 6.

effect on who we become as parents. APPRECIATE Employees who have been nursing a ______________ against their manager are encouraged to discuss it with the board. GRIEF

QU Y

2.

NH

Part 2. Give the correct form of each word in the blanket to complete the following sentences. (10 points) 1. How we ourselves were treated by our parents in our youth can have an ______________

Although air travel is admittedly fast, passengers are still subject to the unexpected ______________ which seem to be inherent in any form of travel. HOLD He became an ______________ by defying the law. LAW That old concrete block of flats is a real ______________ in such a modern city. EYE Passing the final exam with flying colour, I was overcome by relief and ______________ joy.

DẠ

Y

M

ADULT 7. The _____________ of the movement of Mars by Kepler led to the formation of his three laws of planetary motion. DOCUMENT 8. I think this printer has _____________ its usefulness and should be sold. LIVE 9. It would be erroneous to believe that they are born weaker, more sensitive or with a greater natural _____________ to cry. Boys have feelings too. INCLINE 10. The headmaster, Bowyer, was a severe ______________ but respected by his pupils. DISCIPLINE Your answers: 1.

2.

3.

4.

5.

6.

7.

8.

9.

10.

4

Part 3. Fill in each blank with one or two prepositions/particles. (10 points)

3. 4. 5. 6.

L

This evening Manchester United takes ________ Barcelona. When John and Frank first met, they hit it ________ immediately. They have been good friends

ever since. You might have gotten a higher grade if you didn't cram ________ that exam at the last minute. Another way of saying 'old-fashioned' is '________ the times'. Growing environmental fears have made climate research all the more important, and Europe is forging ________ in this field. The thing I don't like about my present job is that we often have to work up to 10 hours

FI CI A

1. 2.

1.

2.

3.

6.

7.

8.

ƠN

OF

_______ a stretch. 7. He manipulates people and tries to bend them ________ his will. 8. The authorities plan to bring _________ new regulations on the application of pesticides. 9. If I could do it ______ again, I’d do it differently. 10. I wish you wouldn’t fly_______ me like that every time I make a mistake. Your answers: 4.

5.

9.

10.

QU Y

NH

SECTION C: READING (60 points) Part 1. Read the following article and decide which answer (A, B, C or D) best fits each gap. (15 points) WE REALLY CAN TELL IF WE ARE BEING WATCHED Stories about how people somehow know when they are being watched have been going around for years. However, few (0) ____________ have been made to investigate the phenomenon scientifically. Now, with the completion of the largest ever study of the so-called staring effect, there is impressive evidence that this is a recognizable and (1) __________ sixth sense. The study

DẠ

Y

M

(2) ___________hundreds of children. For the experiments, they sat with their eyes (3) ____________ so they could not see, and with their backs to other children, who were told to either stare at them or look away. Time and time again the results showed that the children who could not see were able to (4) __________ when they were being stared at. In a (5) ____________ of more than 18,000 trials (6) ____________ worldwide, the children (7) ____________ sensed when they were being watched almost 70% of the time. The experiment was repeated with the (8) __________precaution of putting the children who were being watched outside the room, (9) ____________ from the starers by the windows. This was done just in case there was some (10) _________ going on with the children telling each other whether they were looking or not. 0. A. tries B. tests C. attempts D. aims 1. A. genuine B. accepted C. received D. sure 2. A. involved B. contained C. comprised D. enclosed 3. A. shaded 4. A. find

B. wrapped B. notice

C. masked C. tell 5

D. covered D. reveal

C. mass C. carried on C. thoroughly C. connected C. split

10. A. pretending Your answers:

B. lying

C. cheating

2.

3.

4.

6.

7.

8.

9.

D. deceiving 5.

10.

OF

1.

D. total D. carried out D. perfectly D. increased D. divided

L

B. collection B. worked through B. exactly B. added B. parted

FI CI A

5. A. sum 6. A. worked over 7. A. correctly 8. A. attached 9. A. separated

QU Y

NH

ƠN

Part 2. Read the following text and fill in one word which best fits each gap. Use only ONE word in each gap. (15 points) One of the hazards that electronic media like the television, radio or computers (1) _______ these days is the decline in book reading. The concern applies mainly (2) _______ the younger generations who are strongly tempted by the glamour of the silver screen and, (3) _______, don’t recognize the importance of acquiring first-hand information from books. To encourage reading for pleasure and to propagate a wide array of publications (4) _______ encyclopedias, reference books, manuals or fiction, radical solutions should be applied. Firstly, more (5) _____ ought to be put on the educational factor. Youngsters should be made to feel comfortable while reading either for information or self-satisfaction in (6) _____ places like airports, buses or on the beach. Secondly, libraries must be subsidized (7) _____ accurately in order to provide the potential readers with ample choice of publications and to become more publically active so as to put books at people’s (8) _____ rather than keep them under lock and key. Fund collecting actions organized by libraries might also raise the public awareness of the advantages of becoming engrossed in a good book. (9) _____, the mass media themselves might contribute

M

substantially by recommending the purchase of valuable best-sellers (10) _____ inspiring their viewers to enrich knowledge and erudition, and thus helping them to develop the habit of

spontaneous every reading. Your answers: 1.

3.

4.

5.

7.

8.

9.

10.

DẠ

Y

6.

2.

6

FI CI A

L

Part 3. Read the passage and choose the best option (A, B, C, or D) to answer the questions. Write your answers in the corresponding numbered boxes. (15 points) HISTORY OF THE CHICKENPOX VACCINE Chickenpox is a highly contagious infectious disease caused by the Varicella zoster virus; sufferers develop a fleeting itchy rash that can spread throughout the body. The disease can last for up to 14

days and can occur in both children and adults, though the young are particularly vulnerable. Individuals infected with chickenpox can expect to experience a high but tolerable level of

discomfort and a fever as the disease works its way through the system. The ailment was once considered to be a “rite of passage” by parents in the U.S. and thought to provide children

NH

ƠN

OF

with greater and improved immunity to other forms of sickness later in life. This view, however, was altered after additional research by scientists demonstrated unexpected dangers associated with the virus. Over time, the fruits of this research have transformed attitudes toward the disease and the utility of seeking preemptive measures against it. A vaccine against chickenpox was originally invented by Michiaki Takahashi, a Japanese doctor and research scientist, in the mid-1960s. Dr. Takahashi began his work to isolate and grow the virus in 1965 and in 1972 began clinical trials with a live but weakened form of the virus that caused the human body to create antibodies. Japan and several other countries began widespread chickenpox vaccination programs in 1974. However, it took over 20 years for the chickenpox vaccine to be approved by the U.S. Food & Drug Administration (FDA), finally earning the U.S.

M

QU Y

government’s seal of approval for widespread use in 1995. Yet even though the chickenpox vaccine was available and recommended by the FDA, parents did not immediately choose to vaccinate their children against this disease. Mothers and fathers typically cited the notion that chickenpox did not constitute a serious enough disease against which a person needed to be vaccinated. Strong belief in that view eroded when scientists discovered the link between Varicella zoster, the virus that causes chickenpox, and shingles, a far more serious, harmful, and longer-lasting disease in older adults that impacts the nervous system. They reached the conclusion that Varicella zoster remains dormant inside the body, making it significantly more likely for someone to develop shingles. As a result, the medical community in the U.S. encouraged the development, adoption,

DẠ

Y

and use of a vaccine against chickenpox to the public. Although the appearance of chickenpox and shingles within one person can be many years apart - generally many decades - the increased risk in developing shingles as a younger adult (30-40 years old rather than 60-70 years old) proved to be enough to convince the medical community that immunization should be preferred to the traditional alternative. Another reason that the chickenpox vaccine was not immediately accepted and used by parents in the U.S. centered on observations made by scientists that the vaccine simply did not last long enough and did not confer a lifetime of immunity. In other words, scientists considered the benefits of the vaccine to be temporary when given to young children. They also feared that it increased the odds that a person could become infected with chickenpox later as a young adult, when the rash is more painful and prevalent and can last up to three or four weeks. Hence, allowing young 7

FI CI A

L

children to develop chickenpox rather than take a vaccine against it was believed to be the “lesser of two evils.” This idea changed over time as booster shots of the vaccine elongated immunity and countered the perceived limits on the strength of the vaccine itself. Today, use of the chickenpox vaccine is common throughout the world. Pediatricians suggest an initial vaccination shot after a child turns one year old, with booster shots recommended after the child turns eight. The vaccine is estimated to be up to 90% effective and has reduced worldwide cases of chickenpox infection to 400,000 cases per year from over 4,000,000 cases before vaccination became widespread. ■ (A) In light of such statistics, most doctors insist that the potential risks of developing shingles outweigh the benefits of avoiding rare complications

OF

associated with inoculations. ■ (B) Of course, many parents continue to think of the disease as an innocuous ailment, refusing to take preemptive steps against it. ■ (C) As increasing numbers of students are vaccinated and the virus becomes increasingly rarer, however, even this trend among parents has failed to halt the decline of chickenpox among the most vulnerable populations. ■ (D) 1. The word “tolerable” in the 1st passage is closest in meaning to

NH

ƠN

(A) sudden. (B) bearable. (C) infrequent. (D) unexpected. 2. According to paragraph 1, which of the following is true of the chickenpox virus? (A) It leads to a potentially deadly disease in adults. (B) It is associated with a possibly permanent rash. (C) It is easily transmittable by an infected individual.

M

QU Y

(D) It has been virtually eradicated in the modern world. 3. Which of the following best expresses the essential information in the highlighted sentence? (A) U.S. parents believed that having chickenpox benefited their children. (B) U.S. parents believed that chickenpox led to immunity against most sickness. (C) U.S. parents wanted to make sure that their children developed chickenpox. (D) U.S. parents did not think that other vaccinations were needed after chickenpox. 4. what can be inferred from paragraph 2 about the clinical trials for the chickenpox vaccine? (A) They took longer than expected. (B) They cost a lot of money to complete. (C) They took a long time to finish. (D) They were ultimately successful. nd 5. The word “notion” in the 2 passage is closest in meaning to

DẠ

Y

(A) history. (B) findings. (C) fact. (D) belief. 6. According to paragraph 3, which of the following is true of Varicella Zoster? (A) It typically attacks adults who are over 60 years old. (B) It is linked to a serious disease that occurs more commonly in adults. (C) It likely is not a serious enough threat to human health to require a vaccine. (D) It is completely eradicated from the body after chickenpox occurs. 7. According to paragraph 3, all of the following is true about the chickenpox virus EXCEPT: (A) It causes two distinct yet related ailments. (B) People did not view it as a serious public health threat. (C) It tended to quickly become dormant and remain inoperative over time. (D) Vaccination against it would help prevent the onset of shingles. 8

FI CI A

L

8. The author uses “booster shots” as an example of (A) a way to increase the effectiveness of the chickenpox vaccine. (B) a preferred method of chickenpox rash and fever treatment. (C) a scientifically approved medicine to eliminate chickenpox. (D) a strategy for parents to avoid vaccinating their child altogether.

9. According to paragraph 4, many parents did not choose the chickenpox vaccine because (A) they believed that the virus was weak and not especially harmful. (B) they thought that scientists did not have enough data to reach a conclusion. (C) they were unsure about the utility of the vaccine given its expected duration.

2.

3.

6.

7.

8.

NH

1.

ƠN

OF

(D) they were convinced it was potentially very toxic, particularly for older children. 10. Look at the four squares [■] that indicate where the following sentence could be added to the passage. Meanwhile, some continue to remain unconvinced, citing a supposed potential of the vaccine to do harm. Where would the sentence fit best? (A) A (B) B (C) C (D) D Your answers 4.

5.

9.

10.

QU Y

Part 4. Read the text and do the following tasks. (15 points) Stadiums: past, present and future A Stadiums are among the oldest forms of urban architecture: vast stadiums where the public could watch sporting events were at the centre of western city life as far back as the ancient Greek and Roman Empires, well before the construction of the great medieval cathedrals and the grand 19th- and 20th-century railway stations which dominated urban skylines in later eras.

M

Today, however, stadiums are regarded with growing scepticism. Construction costs can soar above £1 billion, and stadiums finished for major events such as the Olympic Games or the FIFA World Cup have notably fallen into disuse and disrepair. But this need not be the case. History shows that stadiums can drive urban development and adapt to the culture of every age. Even today, architects and planners are finding new ways to adapt the mono-functional sports arenas which became emblematic of modernization during the 20th century. The amphitheatre1 of Arles in southwest France, with a capacity of 25,000 spectators, is perhaps the best example of just how versatile stadiums can be. Built by the Romans in 90

DẠ

Y

B

1

amphitheatre: (especially in Greek and Roman architecture) an open circular or oval building with a central space surrounded by tiers of seats for spectators, for the presentation of dramatic or sporting events

9

FI CI A

L

AD, it became a fortress with four towers after the fifth century, and was then transformed into a village containing more than 200 houses. With the growing interest in conservation during the 19th century, it was converted back into an arena for the staging of bullfights, thereby returning the structure to its original use as a venue for public spectacles. Another example is the imposing arena of Verona in northern Italy, with space for 30,000 spectators, which was built 60 years before the Arles amphitheatre and 40 years before Rome’s famous Colosseum. It has endured the centuries and is currently considered one of

yet another impressive example of an amphitheatre becoming absorbed into the fabric of the city. The site evolved in a similar way to Arles and was progressively filled with buildings from the Middle Ages until the 19th century, variously used as houses, a salt depot and a prison. But rather than reverting to an arena, it became a market square, designed by Romanticist architect Lorenzo Nottolini. Today, the ruins of the amphitheatre remain embedded in the various shops and residences surrounding the public square. There are many similarities between modern stadiums and the ancient amphitheatres intended for games. But some of the flexibility was lost at the beginning of the 20th century, as stadiums were developed using new products such as steel and reinforced concrete, and made use of bright lights for night-time matches.

NH

D

ƠN

OF

C

the world’s prime sites for opera, thanks to its outstanding acoustics. The area in the centre of the Italian town of Lucca, known as the Piazza dell’Anfiteatro, is

QU Y

M

E

Many such stadiums are situated in suburban areas, designed for sporting use only and surrounded by parking lots. These factors mean that they may not be as accessible to the general public, require more energy to run and contribute to urban heat. But many of today’s most innovative architects see scope for the stadium to help improve the city. Among the current strategies, two seem to be having particular success: the stadium as an urban hub, and as a power plant. There’s a growing trend for stadiums to be equipped with public spaces and services that serve a function beyond sport, such as hotels, retail outlets, conference centres, restaurants and bars, children’s playgrounds and green space. Creating mixed-use developments such as this reinforces compactness and multi-functionality, making more efficient use of land

and helping to regenerate urban spaces. This opens the space up to families and a wider cross-section of society, instead of catering only to sportspeople and supporters. There have been many examples of this in the UK: the mixed-use facilities at Wembley and Old Trafford have become a blueprint for many other stadiums in the world. The phenomenon of stadiums as power stations has arisen from the idea that energy problems can be overcome by integrating interconnected buildings by means of a smart grid, which is an electricity supply network that uses digital communications technology to detect and react to local changes in usage, without significant energy losses. Stadiums are ideal for these purposes, because their canopies have a large surface area for fitting photovoltaic panels and rise high enough (more than 40 metres) to make use of micro wind turbines.

DẠ

Y

F

10

FI CI A

L

Freiburg Mage Solar Stadium in Germany is the first of a new wave of stadiums as power plants, which also includes the Amsterdam Arena and the Kaohsiung Stadium. The latter, inaugurated in 2009, has 8,844 photovoltaic panels producing up to 1.14 GWh of electricity annually. This reduces the annual output of carbon dioxide by 660 tons and supplies up to 80 percent of the surrounding area when the stadium is not in use. This is proof that a stadium

can serve its city, and have a decidedly positive impact in terms of reduction of CO2 emissions. G

Sporting arenas have always been central to the life and culture of cities. In every era, the stadium has acquired new value and uses: from military fortress to residential village, public

NH

ƠN

OF

space to theatre and most recently a field for experimentation in advanced engineering. The stadium of today now brings together multiple functions, thus helping cities to create a sustainable future. Questions 1–4: Reading Passage 2 has seven sections, A–G. Which section contains the following information? NB You may use any letter more than once. 1. a mention of negative attitudes towards stadium building projects. 2. figures demonstrating the environmental benefits of a certain stadium. 3. examples of the wide range of facilities available at some new stadiums. 4. reference to the disadvantages of the stadiums built during a certain era.

M

QU Y

Questions 5-8: Complete the summary below. Choose ONE WORD ONLY from the passage for each answer. Roman amphitheatres The Roman stadiums of Europe have proved very versatile. The amphitheatre of Arles, for example, was converted first into a 5. __________, then into a residential area and finally into an arena where spectators could watch 6. __________. Meanwhile, the arena in Verona, one of the oldest Roman amphitheatres, is famous today as a venue where opera is performed. The site of Lucca’s amphitheatre has also been used for many purposes over the centuries, including the storage of 7. __________. It is now a market square with 8. __________ and homes incorporated into the remains of the Roman amphitheatre.

DẠ

Y

Questions 9-10: Choose TWO letters, A–E. Which TWO advantages of modern stadium design does the writer mention? A. offering improved amenities for the enjoyment of sports events. B. bringing community life back into the city environment. C. facilitating research into solar and wind energy solutions. D. enabling local residents to reduce their consumption of electricity. E. providing a suitable site for the installation of renewable power generators. Your answers 1.

2.

3.

4.

11

5.

8.

9.

10.

WRITING (50 points)

FI CI A

II.

7.

L

6.

Part 1. 20 points The table below shows the percentage of the population by age groups in one town who rode

NH

ƠN

OF

bicycles in 2011. Summarise the information by selecting and reporting the main features and make comparisons where relevant.

QU Y

Part 2. 30 points Write an essay of about 250 words on the following topic: Some people feel that we should not keep animals in zoos because animals suffer in captivity. Others feel zoos play an important educational role in teaching us about wildlife. Do you believe zoos still have a place in the modern world? Explain your opinion and give specific details and reasons to support your viewpoint.

DẠ

Y

M

---------- THE END ----------

12

ĐỀ THI MÔN ANH KHỐI 10

VÙNG DUYÊN HẢI VÀ ĐỒNG BẰNG BẮC BỘ

NĂM 2023

TRƯỜNG THPT CHUYÊN BIÊN HÒA

HƯỚNG DẪN CHẤM

FI CI A

TỈNH HÀ NAM ĐỀ THI ĐỀ XUẤT I.

L

HỘI CÁC TRƯỜNG THPT CHUYÊN

LISTENING (50 points)

Part 1. You will hear part of an interview with a music journalist called Pip Rogers

OF

and a musician called Heath Francis about the renewed popularity of vinyl records. For questions 1- 6, choose the best answer (A, B, C or D). (20 points) 1. C

2. A

3. B

4. B

5. D

Part 2. Listen to a podcast on climate change and decide whether the following

ƠN

statements are True (T), False (F), or Not Given (NG) according to what you hear. (10 points)

https://www.youtube.com/watch?v=oqMTm5S8FxY#:~:text=every%20year%20every 1. F

NH

%20choice%20matters,that%20we%20can%20barely%20even. 2. T

3. NG

4. T

5. T

Part 3. Listen to a talk about the unique characteristics of glass and fill each blanks with NO MORE THAN FOUR WORDS. (20 points)

QU Y

https://www.youtube.com/watch?v=8Jsbyx3nMfU 1. Rigid amorphous solid 2.

Tug on

3. Contract

4. A single nanometer

M

5. Derived from tree sap 6. 2% denser

7. ( Low )entropy

8. Quantum tunnel between configurations 9. Vapor disposition 10. Sagged over time

Y

II. LEXICO-GRAMMAR (40 points)

DẠ

Part 1. Choose the correct answer (A, B, C or D) to complete the sentences. (20 points) 1. C

2. C

3. B

4. C

5. A

6. C

7. C

8. A

9. D

10. B

11. D

12. D

13. A

14. A

15. B

16. B

17. C

18. B

19. C

20. C

1

Appreciable Grievance

3. 4.

Hold- ups Outlaw

L

1. 2.

FI CI A

Part 2. Give the correct form of each word in the blanket to complete the following sentences. (10 points)

OF

5. Eyesore 6. Unadulterated 7. Documentation 8. Outlived 9. Inclination 10. Disciplinarian Part 3. Fill in each blank with one or two prepositions/particles. (10 points) 2. off

3. for

6. at

7. to

8. in

4. behind

5. ahead

9. over

10. at

ƠN

1. on

NH

III. READING (60 points) Part 1. Read the following article and decide which answer (A, B, C or D) best fits each gap. (15 points) 1. A

2. A

3. D

4. C

5. D

6. D

7. A

8. B

9. A

10. C

QU Y

Part 2. Read the following text and fill in one word which best fits each gap. Use only ONE word in each gap. (15 points) 1. Pose/ cause/ 2. To create

3. Consequently/ 4. Like hence/ therefore

5. Emphasis/ focus

6. Public

8. Disposal

10. And

7. More

9. Finally

M

Part 3. Read the passage and choose the best option (A, B, C, or D) to answer the questions. Write your answers in the corresponding numbered boxes. (15 points) 1. B

2. C

3. A

4. D

5. D

6. B

7. C

8. A

9. C

10. B

Y

Part 4. Read the text and do the following tasks. (15 points)

DẠ

1. A

2. F

3. E

4. D

5. FORTRESS

6. 7. BULLFIGHTS SALT

2

8. 9-10. B, E SHOPS

FI CI A

The table below shows the percentage of the population by age groups in one town who rode bicycles in 2011. Summarise the information by selecting and reporting the main features and make comparisons where relevant. *Striking features: -

L

IV. WRITING (50 points) Part 1. 20 points

This means of transport was most popular among the youngest age group(0-9)

A higher percentage of females cycled every day in comparison with males. 1. Completion (02 points): The report covers the table and has a clear structure of three parts: -

NH

ƠN

OF

Introduction, Overview and Body. 2. Content (10 points) - The report MUST cover the following points: • Introduce the table (01 point) and state the general and striking features (05 points). • Describe main features with relevant data from both charts and make relevant comparisons (05 points). - The report MUST NOT contain personal opinions. (A penalty of 01 point to 02 points will be given to personal opinions found in the answer.) 3. Organisation (02 points) - Ideas are well organised. - The report is sensibly divided into paragraphs.

DẠ

Y

M

QU Y

4. Punctuation and spelling (01 point) The report should demonstrate the correct use of spelling and punctuations. 5. Language use (05 points) The report should: - Demonstrate a wide variety of lexical and grammatical structures. - Make correct use of words (verb tenses, word forms, voice, etc.). Part 2. 30 points Write an essay of about 250 words on the following topic: Some people feel that we should not keep animals in zoos because animals suffer in captivity. Others feel zoos play an important educational role in teaching us about wildlife. Do you believe zoos still have a place in the modern world? Explain your opinion and give specific details and reasons to support your viewpoint. The mark given to part 3 is based on the following scheme: 1. Content (10 points) - ALL requirements of the task are sufficiently addressed. - Ideas are adequately supported and elaborated with relevant and reliable explanations, examples, evidence, personal experience, etc. 2. Organisation and Presentation (08 points) - Ideas are well organised and presented with coherence, cohesion, and clarity.

3

FI CI A

• The body paragraphs are written with unity, coherence and cohesion. Each body paragraph must have a topic sentence and supporting details and examples where necessary.

L

- The essay is well-structured: • The introduction is presented with a clear thesis statement.

• The conclusion summarises the main points and offers personal opinions on the issue. 3. Language (09 points)

ƠN

OF

- Demonstration of a variety of vocabulary and structures appropriate to the level of English language gifted upper-secondary school students. - Good use and control of grammatical structures. - Use of vocabulary must not create confusion for readers. 4. Handwriting, punctuation, and spelling (03 points) - Good punctuation and no spelling mistakes. - Legible handwriting.

DẠ

Y

M

QU Y

NH

---------- THE END ----------

4

ĐỀ THI CHỌN HỌC SINH GIỎI LẦN THỨ XIII

MÔN THI: TIẾNG ANH – KHỐI 11 Ngày thi 15-16/07/2023 Thời gian làm bài 180 phút

ĐỀ THI ĐỀ XUẤT

(Đề thi gồm … trang)

FI CI A

TRƯỜNG THPT CHUYÊN BIÊN HOÀ, T. HÀ NAM ___________________

L

HỘI CÁC TRƯỜNG THPT CHUYÊN KHU VỰC DUYÊN HẢI, ĐỒNG BẰNG BẮC BỘ

QU Y

NH

ƠN

OF

SECTION A. LISTENING (50 points) Part 1. You hear two people speaking about their friendship. Each given question has four options A, B, C and D. Choose the best option for each question. 1. One speaker felt her watershed moment was when_____ A. she watched a film under a blanket. B. she turned fifty. C. she had an argument with a good friend. D. she got married. 2. One speaker made a tongue-in-cheek comment about A. The Tate. B. the food they both like. C. taking up knitting. D. singing in a choir. 3. One speaker expressed the view that a quilt of fear A. was ideal for daydreaming. B. was great for keeping you warm in front of the television. C. protected you from imaginary concerns. D. made your wishes come true. 4. The speakers failed to agree about A. the number of times they fell out. B. where the quilt should be displayed. C. the number of pieces the quilt should have. D. the best age to form lasting friendships. 5. What conclusion did they put forward? A. Friendships made in your teenage years reflect your aspirations. B. Good friendships are hard to find. C. The longer the friendship, the better it becomes. D. It's best not to mix friends. Your answers: 2.

3.

4.

5.

M

1.

DẠ

Y

Part 2. Listen to the recording and decide which of the following statements are True (F) and which ones are False (F)? 1. The queen's personal income is primarily derived from her lucrative investment portfolio. 2. The Sovereign Grant is provided by the government to cover official duties and expenditures of the royal family. 3. The Crown Estate and the Royal Collection Trust are both separate entities that hold valuable assets associated with the royal family. 4. The monarchy's brand contributes over 1 billion to the national economy annually. 5. The Duchy of Cornwall serves as the primary source of income for the Duke of Lancaster. Your answers: 1.

2.

3.

4.

5. 1 of 14

2.

6.

7.

3.

4.

5.

8.

9.

10.

QU Y

1.

NH

ƠN

OF

FI CI A

L

Part 3. Listen to the recording and fill in the gaps with NO MORE THAN THREE WORDS and/or A NUMBER The Shinkansen Bullet Train in Japan faced a noise problem when it exited tunnels due to the (1) _______________ it created. To address this issue, an engineering team used (2) ___________________ to design a quieter and faster train. The redesigned train took inspiration from birds such as owls, whose feathers influenced the design of the (3) ________________. The pantograph's supporting shaft was redesigned based on the smooth body of the Adelie penguin to reduce wind resistance and achieve a (4) _____________________. The Kingfisher's beak served as a model for the train's nose design, which helped minimize noise by reducing pressure waves and splashes, achieving the (5) ________________. The redesigned train, with its biomimetic components, achieved remarkable results, including being 10% faster and using (6) ___________________. However, people who shape the world often lack experience in biology. Consequently, they are (7) ___________________ when it comes to understanding the workings of the world. Biomimicry has influenced various fields, including healthcare, where researchers have looked at shark skin to develop bacteria-resistant surfaces for hospitals, mimicking the shark's (8) ________________________. Mimicking natural processes, such as how ants communicate to efficiently find resources, has been applied in the development of software, including the movement of autonomous cars in a (9) _____________________. The idea of the (10) _______________________ promotes using materials in a way that eliminates waste by continuously upcycling them, similar to how materials are reused and transformed in natural ecosystems. Your answers:

M

Part 4. Listen to a recording about Frederick, the duke of York and answer the following questions. Write NO MORE THAN THREE WORDS and/or A NUMBER for each answer. 1. In what type of composition was Frederick, the duke, made famous? _______________________________________________ 2. Who did his battles end in total disaster against? _______________________________________________ 3. What is the derogatory term employed to depict a collective of individuals perceived as contemptible and devoid of value? _______________________________________________ 4. When did the Duke of York find himself subjected to ridicule through satirical songs? _______________________________________________ 5. During his involvement in a scandalous affair, what specific position did the Duke hold? _______________________________________________

DẠ

Y

SECTION B. GRAMMAR & VOCABULARY (30 points) Part 1: Choose the answer A, B, C, or D that best completes each of the following sentences. Write your answers in the corresponding numbered boxes. (20 points) 1. They regularly hold elections without a _____ of corruption or violence. A. scent B. breath C. sniff D. whiff 2. The lyrics came to him in _____ during this and other catnaps. A. few and far B. length and breadth C. bits and pieces D. leaps and bounds 2 of 14

1.

Y

11.

M

QU Y

NH

ƠN

OF

FI CI A

L

3. Should the longevity link also apply to human beings, it could well lead to the development of drugs that mimic the effects of calorie _____ while allowing people to maintain their normal diet. A. constraint B. restriction C. prevention D. restraint 4. Right after I got married, I got a big promotion at work, so I'm really _____ at the moment! A. on the fence B. on the crest of the wave C. on cloud nine D. B&C are correct 5. Joe's been walking with a _____ ever since he found out he was getting a promotion. A. feather in his cap B. spring in his step C. nail on his head D. chip on his shoulder 6. They would much sooner ____ than _____ by car. A. walk/going B. walking/going C. walking/go D. walk/go 7. Both reporters cut their journalistic _____ on the same provincial newspaper. A. tooth B. teeth C. foot D. feet 8. (NP) It’s _____ as bad as he said A. more like B. much C. nothing like D. a great deal 9. The old bridge fell into _____ several years ago. A. disuse B. misuse C. unuse D. abuse 10. (NP) Lest anyone _____ my story, I have brought documents to attest to its truth. A. must doubt B. should doubt C. doubts D. doubted 11. Let me make a _____ concession at the start. A. square B. triangle C. round D. cross 12. (NP) There has been _____ increasing number of cases of _____ disease. A. an/the B. an/Ø (zero article) C. the/a D. the/the 13. He announced he could see the _____ of recovery in the job market. A. green shoots B. blue chip C. white goods D. golden hello 14. Colin's girlfriend dumped him weeks ago, but the poor guy's heart is still in his _____ . A. shoes B. heels C. boots D. hoods 15. I chose to pursue a career in medical research so that I might someday _____ in the world with a groundbreaking discovery. A. make my grade B. make a move C. make my day D. make my mark 16. There is concern that overfishing could snuff _____ some species. A. up B. out C. in D. about 17. This is a good exercise for toning ____ the thighs. A. up B. out C. in D. at 18. (NP) __________ that it now carries nearly two million passenger each day. A. So popular the system has become B. So popular has the system become C. Such popular was the system D. Such was popular the system 19. (NP) I’ll see you on Saturday. What ____________ in the afternoon? A. will you do B. will you be doing C. do you do D. are you doing 20. The company has changed __________ several times but is still on the verge of bankruptcy. A. places B. tune C. subject D. hands Your answers 2.

3.

4.

5.

6.

7.

8.

9.

12.

13.

14.

15.

16.

17.

18.

19.

DẠ

Part 2: Give the correct form of each bracketed words. Write your answers in the corresponding numbered boxes. (10 points) 1. He had a(n) _____ smile on his face when he saw me. (ENIGMA) 2. In Greek myth, love is _____ by the goddess Aphrodite. (PERSON) 3. I have only a _____ knowledge of Spanish history. (PATCH) 3 of 14

6.

2.

7.

3.

8.

4.

9.

5.

OF

1.

ƠN

FI CI A

L

4. After several setbacks it's hard not to grow _____. (HEART) 5. The book is an _____ summary of issues in pensions, relying on quotes from and references to others. (ORIGIN) 6. Pinga is _____ and begins to cry, to the extent of remaining against a wall. (CONSOLE) 7. The report contained conflicting evidence and plenty of _____. (CONSISTENT) 8. _____ by her own fears, she never left the house. (PRISON) 9. The inclusion of _____ explanations and justifications was by no means standard practice. (SOLICIT) 10. Given the observed difference between men and women as regards to psychological distress, all _____ were conducted separately for men and women. (ANALYSE) Your answers

10.

DẠ

Y

M

QU Y

NH

SECTION C. READING (60 points) Part 1: Read the text below and think of the word which best fits each space. Use only ONE word in each space. Write your answers in the corresponding numbered boxes. (15 points) “Extreme sports” is a broad term to describe any action or adventure sports that include a high dosage of risk, height, speed, natural challenges, and physical struggle. Most extreme sports are considered as an alternative (1) _____ mainstream sports with their adrenaline-rushing thrills. Extreme sports are popularized in (2) _____ 1990s with lots of TV coverage and they keep on increasing their popularity every year. Bungee Jumping is a simple yet thrilling recreational activity that involves head-first jumping from a tall structure (3) _____ an elastic cord. Bungee jump as we (4) _____ it was first practiced in 1979 but its different forms used to be a tribal coming-of-age ritual in many cultures. Now, bungee jumping is one of the most popular and available extreme sports in the world. Paragliding is an aerial extreme sport involving gliding through the air with a parachute that is attached into your body. Participants descend from (5) _____ altitudes such as a mountain, cliff or an aircraft. Abseiling is a recreational activity that involves sliding down through a rope in controlled conditions from a mountain, cliff or a man made structure. In spring the weather is more welcoming and warmer for abseiling and since the (6) _____ is mostly taking place in nature; spring is the most convenient season for it. Kitesurfing is riding and gliding across the water (7) _____ holding onto a large hand-controlled kite that is powered by the wind. Mountain biking is an off-road bicycle racing sport that is set on rough terrain like a mountain, desert, or rocks with specially (8) _____ mountain bikes. Most mountain bikers like to ride on a dry terrain (9) _____ snow or rain. Skateboarding is an actionfilled recreational activity and a professional sport that involves performing tricks on a skateboard. Skateboarding has also created its (10) _____ subculture from its slang to music. (Adapted from https://www.flypgs.com/en/extreme-sports ) Your answers 1.

2.

3.

4.

5.

6.

7.

8.

9.

10.

4 of 14

DẠ

Y

M

QU Y

NH

ƠN

OF

FI CI A

L

Part 2. Read the following passage and choose the answer (A, B, C or D) which fits best according to the text. Write your answers in the corresponding numbered boxes. (10 points) MARCO POLO “Here begins the introduction of this book, which is called “The Description of the World”. Lords, Emperors, and Kings, Dukes, and Marquesses, Counts, Knights, and Burgessess, and all people who wish to know the different generations of men and the diversities of the different regions of the world, then take this book and have it read and here you will fine all the greatest marvels and the great diversities…” So begins Marco Polo’s book, “The Description of the World,” as presented in Arthur Christopher Moule’s masterful English translation of a version of Marco Polo’s book known to scholars and the “F” text. The storied Venetian trader escaped bandits, pirates, rampaging rivers and sandstorms on his epic eastbound journey. Sailing the treacherous coasts of Southeast Asia and India, Marco Polo returned to Venicce in 1295, after 24 years, rich in gems, and wild tales of unimagined lands. Shortly after his return to Venice, Marco Polo was captured at sea, possibly by pirates. One tradition suggests he was imprisoned in Genoa’s Palazzo and that he devoted his prison time to composing his book. On his deathbed in 1324, the legendary adventurer reflected that he had many more stories to tell. “The Description of the World,” the original product of Marco Polo’s collaboration with a romance writer named Rustichello has been lost, and so scholars are left to sift through the some 150 versions known to exist, no two exactly alike. Scholars divide the 150 versions into two groups, labeled “A”, and “B”. The “F” text, which falls into the “A” group, is housed in the Bibliotheque Nationale in Paris. Considered one of the best and very close to the original, it is written in a Franco-Italian language described by one scholars as “uncouth French much mingled with Italian.” Some of these “A” texts are notorious for variations that show the biases, mistakes and editorial judgments of their copiers. For example, when some translators were presented with the news that three Magi were buried at Saveh in Persia rather than in Cologne, they inserted that the people of Saveh tell many lies. As these books were translated from language to language, the opportunities for error multiplied; one text from the early 16th century is a Tuscan translation of a Latin translation of an earlier Tuscan translation of the original Franco-Italian language. Although we have no confirmation of the Marco-Rustichello collaboration other than the book itself, Marco seems to have approved of at least some of its versions, for in 1307 he presented a French translation of it to an envoy of Charles of Valois. The second group of manuscripts, known as the “B” group, provides some provocative material not found in the “A” texts. From this “B” group, for example, we learn that the people around Yarkand in western China suffer from goiter – a problem for them even today. Until the 1930s the only examples of “B” texts were a few odd bits of manuscript and a printed text by Giambattista Ramusio that appeared in 1559, two years after his death. Ramusio tells his readers that his Italian version was produced with the help of different copies.” [A] The foundation of his work appears to be a Latin text dating from before 1320, with influences from other identifiable versions. [B] What is distinctive about Ramusio’s work is that about twenty percent of it was, until 1932, considered unique. [C] That twenty percent is thought to have destroyed in a 1558 fire. [D] A second version containing much of Ramusio’s original material surfaced in Toledo, Spain in 1932. Most of this Latin manuscript agrees with the “F” manuscript, but it also contains some 200 passages not found in “F”. About 120 of those, however, are found in Ramusio’s book. Because the remaining 80 offer valuable historical and geographical material and even help to clarify some obscure passages of “F”, this manuscript is thought to be a copy of something that was very close to an original. In sorting this out, scholars have come to conclude that Marco Polo probably wrote two versions of his book. The second version, presented by the “B” texts, may have been a revision and expansion done for a select group of readers who had already made their way through the first book. It is 5 of 14

Y

M

QU Y

NH

ƠN

OF

FI CI A

L

unlikely that we will ever know exactly what form the first book took, but the versions we have still make for a very good read. (Adapted from CPE express book 2) 1. According to the introduction to the book, readers can expect to A. learn about Marco Polo’s life. B. learn about differences among generations. C. travel to far regions of the world. D. read descriptions of places. 2. According to the second paragraphs, stories about Marco Polo’s life A. are well-supported B. are all imaginary C. take place at sea D. are sometimes unreliable 3. What is the “F” text? A. The authentic text written by Marco Polo and Rustichello. B. The script with the greatest affinity to the original source. C. Not one of the 150 versions of Marco’s original book. D. A good version of the “B” texts written in Franco-Italian. 4. What is one of the main problems with the “A” texts? A. All translators manipulated the truth. B. Editing is now difficult and unreliable. C. The early versions were remote from the original text. D. Later translations distorted the original. 5. The “B” group of manuscripts A. contained previously undocumented information. B. were compilations of manuscripts printed by Ramusio. C. dealt with health and culture in China. D. were published two years after Ramusio’s death. 6. What was found in Spain in 1932? A. A Latin version containing valuable information about Ramusio B. A text which was very close to the “F” manuscript. C. A manuscript of 200 passages that do not appear in the “F” text. D. The original book written by Marco Polo. 7. In relation to the book, “The Description of the World”, the author suggests that A. despite its uncertain origins, it is a fascinating piece of literature. B. scholars should discover who the true author was. C. Marco Polo wrote many versions of the same book. D. Marco Polo intended his original book for an elite readership. 8. Which of the following statements is TRUE A. Marco Polo was captured at sea by pirates for two years. B. There are about 150 versions of the book “The Description of the World”. C. “B” texts have more biases, mistakes and editorial judgments than “B” texts. D. Texts by Ramusio were destroyed in a fire. 9. The word “envoy” in paragraph 4 refers to A. a person B. a dynasty C. a department D. a country 10. Which of the following square brackets [A], [B], [C], or [D] best indicates where in the paragraph the following sentence can be inserted? In any event, the source has never been found. A. [A] B. [B] C. [C] D. [D] Your answers 2.

3.

4.

5.

6.

7.

8.

9.

10.

DẠ

1.

6 of 14

L

OF

FI CI A

Part 3. Read the passage and do the tasks that follow. Write your answers in the corresponding numbered boxes. (13points) List of headings i. How the study of body language has changed ii. A possible business application of body language iii. Using body language as a tool to deceive others iv. Communicating a wide range of messages v. A branch of an older academic field vi. The need for skill when interpreting body language vii. Recognising a positive attitude without realising it viii. How power is linked with certain family roles ix. A form of body language that can be misinterpreted x. Imitating the chief person in a group xi. Ignoring signals from other people

NH

ƠN

1. Paragraph A: _____ 2. Paragraph B: _____ 3. Paragraph C: _____ 4. Paragraph D: _____ 5. Paragraph E: _____ 6. Paragraph F: _____ 7. Paragraph G: _____ 8. Paragraph H: _____

DẠ

Y

M

QU Y

KINESICS A. Psychology is a well-established subject, but one area of it, “kinesics”- the study of body language – was not identified until 1952, and research only began in earnest in the 1960s. Kinesics is based on the behavioral patterns of non-verbal communication. Clinical studies have revealed the extent to which body language can actually contradict verbal communications. A classic example is the young woman who told her psychiatrist that she loved her boyfriend very much while shaking her head from side to side in subconscious denial. B. Body language also sheds light on the dynamics of interfamily relationships. A family sitting together can give a revealing picture of itself simply by the way its members move their arms and legs. If the mother, for example, crosses her legs first and the rest of the family then follows suit, she has set the lead for the family action, though she, as well as the rest of the family may not be aware she is doing it. In fact, her words may deny her leadership as she asks her husband or children for advice. But the unspoken, follow-the-leader clue in the actions of the family members gives the family set-up away to someone knowledgeable in kinesics. C. Another kinesic signal is the unconscious widening of a person’s pupils when their eyes see something pleasant. Experiments have shown that we become aware of how that person feels, although we are conscious neither of seeing the signal nor of giving it meaning. D. This kinesic principle has been used on a commercial level to detect the effect of a television advertisem*nt. While the ad is being shown to a selected audience, their eyes are photographed. Later, the film is carefully studied to detect just when there is any widening of the eye; in other words, when there is any unconscious, positive response to the advertisem*nt. E. Body language can include any volutionary or involutionary movement of a part or all the body, used by a person to communicate an emotional message to the outside world. To understand this unspoken body language, kinesics experts often have to take into consideration cultural and environmental differences. The average person, unschooled in cultural nuances of body language, is often mistaken when decoding what he or she sees. 7 of 14

ƠN

OF

FI CI A

L

F. In addition to sending and receiving messages, body language can also serve to break through defences, if it is used skillfully. Often the swiftest and most obvious type of body language is touch. The touch of a hand, or an arm around someone’s shoulder, can spell a more vivid and direct message of friendliness than dozens of words. But such a touch must come at the right moment and in the right context, or the other person may take it as an intrusion into his or her personal space. For every situation there must be two elements to body language: the delivery of the message and the reception of the message. G. However, some people are “touchers”, compulsive touchers, who seem completely impervious to all messages they may get from friends and acquaintances. They are people who will touch others even though they are bombarded with body-language requests not to. There are also people who avoid touching others altogether. H. We act out our state of being with non-verbal body language. We lift one eyebrow for disbelief, shrug our shoulders for indifference, tap our fingers for impatience. The gestures are numerous, and while some are deliberate and others are almost deliberate, there are some, such as rubbing under our noses for puzzlement or clasping our arms to protect ourselves, that are mostly unconscious. Kinesics is a study of the mixture of all body movements, from the very deliberate to the completely unconscious, from those that apply only in one culture to those that cut across all cultural barriers.

NH

Questions 9-13. Do the following statements reflect the claims of the writer in the reading passage? Write YES if the statement reflects the claims of the writer NO if the statement contradicts the claims of the writer NOT GIVEN if it is impossible to say what the writer thinks about this

QU Y

9. Little work was done in the field of kinesics for the first few years after the subject had been identified. 10. Family leaders consistently show their dominance through speech and body language. 11. The use of kinesics in connection with television advertising has increased sales of products. 12. Touching may be regarded as an unwelcome gesture.

6.

2.

3.

4.

5.

7.

8.

9.

10.

12.

13.

Y

11.

1.

M

Question 13. Choose the correct letter A, B, C or D The writer’s intention is A. to present recent findings in kinesics to specialists. B. to introduce kinesics to general readers. C. to examine weaknesses in kinesics. D. to identify the scientific basis of kinesics. (Adapted from Cambridge Objective IELTS advanced) Your answers:

DẠ

Part 4. You are going to read an extract from an article. Seven paragraphs have been removed from the extract. Choose from the paragraphs A - H the one which fits each gap (1- 7). There is one extra paragraph which you do not need to use. Write your answers in the corresponding numbered boxes. (7 points)

8 of 14

L

OpenAI last week opened up access to ChatGPT, an AI-powered chatbot that interacts with users in an eerily convincing and conversational way. Its ability to provide lengthy, thoughtful and thorough responses to questions and prompts – even if inaccurate – has stunned users, including academics and some in the tech industry.

FI CI A

1.

“There’s a certain feeling that happens when a new technology adjusts your thinking about computing. Google did it. Firefox did it. AWS did it. iPhone did it. OpenAI is doing it with ChatGPT,” Levie said on Twitter. But as with other AI-powered tools, it also poses possible concerns, including for how it could disrupt creative industries, perpetuate biases and spread misinformation.

OF

2.

ƠN

After signing up for ChatGPT, users can ask the AI system to field a range of questions, such as “Who was the president of the United States in 1955,” or summarize difficult concepts into something a second grader could understand. It’ll even tackle open-ended questions, such as “What’s the meaning of life?” or “What should I wear if it’s 40 degrees out today?”

NH

3.

QU Y

But some users are getting very creative. One person asked the chatbot to rewrite the 90s hit song, “Baby Got Back,” in the Style of “The Canterbury Tales;” another wrote a letter to remove a bad account from a credit report (rather than using a credit repair lawyer). Other colorful examples including asking for fairy-tale inspired home décor tips and giving it an AP English exam question (it responded with a 5 paragraph essay about Wuthering Heights.) 4.

M

While ChatGPT successfully fielded a variety of questions submitted by CNN, some responses were noticeably off. In fact, Stack Overflow – a Q&A platform for coders and programmers – temporarily banned users from sharing information from ChatGPT, noting that it’s “substantially harmful to the site and to users who are asking or looking for correct answers.”

5.

Y

“While we’ve made efforts to make the model refuse inappropriate requests, it will sometimes respond to harmful instructions or exhibit biased behavior,” Open AI said on its website. “We’re using the Moderation API to warn or block certain types of unsafe content, but we expect it to have some false negatives and positives for now. We’re eager to collect user feedback to aid our ongoing work to improve this system.”

DẠ

6.

9 of 14

L

“It is very easy for the model to give plausible-sounding but incorrect or nonsensical answers,” he said. “It guessed when it was supposed to clarify and sometimes responded to harmful instructions or exhibited biased behavior. It also lacks regional and country-specific understanding.”

FI CI A

7.

2.

1.

M

QU Y

NH

ƠN

OF

While the DALL-E tool is free, it does put a limit on the number of prompts a user can do before having to pay. When Elon Musk, a co-founder of OpenAI, recently asked Altman on Twitter about the average cost per ChatGPT chat, Altman said: “We will have to monetize it somehow at some point; the compute costs are eye-watering.” (Adapted from https://edition.cnn.com/2022/12/05/tech/chatgpt-trnd/index.html) A. “It depends on what activities you plan to do. If you plan to be outside, you should wear a light jacket or sweater, long pants, and closed-toe shoes,” ChatGPT responded. “If you plan to be inside, you can wear a t-shirt and jeans or other comfortable clothing.” B. The tool quickly went viral. On Monday, Open AI’s co-founder Sam Altman, a prominent Silicon Valley investor, said on Twitter that ChatGPT crossed one million users. It also captured the attention of some prominent tech leaders, such as Box CEO Aaron Levie. C. Still, Lian Jye Su, a research director at market research firm ABI Research, warns the chatbot is operating “without a contextual understanding of the language.” D. In a blog post last week, OpenAI said the “format makes it possible for the tool to answer followup questions, admit its mistakes, challenge incorrect premises, and reject inappropriate requests.” As of Monday morning, the page to try ChatGPT was down, citing “exceptionally high demand.” “Please hang tight as we work on scaling our systems,” the message said. (It now appears to be back online). E. At the same time, however, it does provide a glimpse into how companies may be able to capitalize on developing more robust virtual assistance, as well as patient and customer care solutions. F. Beyond the issue of spreading incorrect information, the tool could also threaten some written professions, be used to explain problematic concepts, and as with all AI tools, perpetuate biases based on the pool of data on which it’s trained. Typing a prompt involving a CEO, for example, could prompt a response assuming that the individual is white and male, for example. G. Like ChatGPT, the new Google Search and Bard are built on a large language model. They are trained on vast troves of data online in order to generate compelling responses to user prompts, but these tools are also known to get responses wrong or “hallucinate” answers. H. ChatGPT is a large language model trained on a massive trove of information online to create its responses. It comes from the same company behind DALL-E, which generates a seemingly limitless range of images in response to prompts from users. It’s also the next iteration of text generator GPT-3. Your answers 3.

4.

5.

6.

DẠ

Y

Part 5. For questions 1-10, select the travel writer (A-E) using the separate answer sheet. Each travel writer may be selected more than once. (15 points) GENETIC ENGINEERING- THE WAY OF THE FUTURE? To examine the issue, we’ve asked the opinions of six experts A. Dr Robert Rodriguez – bioethics lecturer Is it so surprising that there is widespread public suspicion and mistrust? Incidentally, the public’s negative view of GM cannot be attributed to ignorance because mistrust tends to increase with education on the topic. This is despite an ever-growing body of research that can find no evidence of harm. Of course, this doesn’t mean there isn’t any, even if we assume the best intentions 10 of 14

7.

DẠ

Y

M

QU Y

NH

ƠN

OF

FI CI A

L

of the people involved; it’s a subject that is not completely understood. Anyway why should the public assume that best intentions are behind the research? Look at the past. Look what happened with BSE, better known as mad cow disease. Agricultural practices did not protect the public, it endangered them. Look back further to the pesticide DDT. We do not have a track record that encourages public confidence. B. Dr Lisa Khan – geographer There is no doubt that people are starving today in many parts of the world. And with global population growth projections – we seem set to add a billion people every twelve to fifteen years – there is absolutely no debate that we will be unable to feed the population in the future unless things change fundamentally. Genetically modifying crops is certainly one way to achieve this change. But it is not the only way, and it may not be the most effective. Take, for comparison, the “green revolution” of the 1950s, which greatly increased productivity by using new strains of crops, new mechanical tools and petrochemical pesticides and fertilizers; it has created its own set of problems. People may starve because of lack of food, but the food is there; other things – social or economic issues – stand in the way of it getting where it needs to be. C. Dr Sylvia Johnson – doctor I would like to point out that GM organisms have made very important contributions to medicine. I’m not talking about GM foods; that is a separate issue. But if we consider insulin being produced in tobacco plants, there is a tremendous benefit. A difficult to obtain substance is made available safely at a lower cost. The plants are cultivated in a greenhouse, under controlled circ*mstances, the product they synthesise is purified in a stringent process, and the modified plants are destroyed; there is very little risk involved. The potential for creating a wide range of difficult- to- produce and life-saving proteins and pharmaceuticals, even vaccines, at costs low enough for third world utilization should not be ignored. D. Dr Gary Wilson – population geneticist These days there is little question about gene flow. If you plant GM crops, the genes will end up in non-GM crops, in wild weed populations, in soil bacteria; they cannot be contained. They will move; they have moved. There are studies that prove it. In the early research, they were citing probabilities of pollination events or gene transfer events that were miniscule; these studies were used to support the cause of GM. But if you have enough chances, if you cultivate enough acres, the improbable will happen. And it has. And it will continue to. For a population geneticist there is no surprise here. At the risk of being incendiary, I will say the conclusions drawn from this early research could be considered an example of willful misinterpretation, of statistical probability. Well, it has become a profitable industry. E. Dr. Daphne Alexander – ecologist I would like to mention the monarch butterfly. This insect carries out a unique migration from the Northern US and Canada all the way to Mexico, and is reliant on a range of environmental and temporal patterns throughout this whole geographic area. It came out a while ago that one strain of GM corn that was engineered to contain a toxin, originally from bacteria, to kill any insect that attempted to eat its leaves, also expressed this toxin in its pollen. It was not supposed to do this, other strains did not. And this pollen was falling on the leaves of the milkweed plants, on which the monarch larvae are dependent for a food source, killing or stunting the larvae. This is an example of the kind of unintended consequence that is impossible to foresee. F. Dr. Andrew Wright – lawyer In the EU, by law, food products that contain GM ingredients must be labeled. But it is not quite as simple as that. In fact, it is not simple at all; it is unbelievably convoluted. Tomato sauce made from GM tomatoes is simple; but it must be labeled. But what about meat, milk, cheese, or eggs produced from animals fed GM corn or soya in their feed? The feed must be labeled, but not the final product. And what about enzymes, like those used to make cheese for example, that have been produced by GM microorganisms? Furthermore, there can legally be up to 9 percent contamination with GM products, with no labeling required, as long as the producer can prove it was accidental and 11 of 14

Your answers 2.

3.

4.

5.

6.

7.

8.

ƠN

1.

OF

FI CI A

L

unavoidable! I would urge everyone to read widely; there is no other way to keep informed and a lot has already transpired, while most of us were unaware. Which person gives each of the following opinions about genetic engineering? 1. _________ GM genes are already present in wild populations. 2. _________ Some people may have misled others when they did not interpret data correctly. 3. _________ Current research might not be correct. 4. _________ People haven’t been paying attention to developments. 5. _________ Even unlikely events happen. 6. _________ It is impossible to account for all the possible effects in advance. 7. _________ People have no reason to believe what the authorities say. 8. _________ Unintentional presence of GM items in food is not regulated. 9. _________ There is more than one way to solve a problem. 10. _________ In some cases the benefits clearly outweigh the risks.

DẠ

Y

M

QU Y

NH

SECTION D. WRITING (60 points) Part 1. Read the following text and use your own words to summarize it. Your summary should be about 100-120 words long. You MUST NOT copy the original. (15 points) Man is forever changing the face of nature. He has been doing so since he first appeared on the earth. Yet, all that man has done is not always to the ultimate advantage of the earth or himself. Man has, in fact, destroyed more than necessary. In his struggle to live and extract the most out of life, man has destroyed many species of wildlife; directly by sheer physical destruction, and indirectly by the destruction or alteration of habitats. Some species may be able to withstand disruptions to their habitat while others may not be able to cope. Take the simple act of farming. When a farmer tills a rough ground, he makes it unsuitable for the survival of certain species. Every change in land use brings about a change in the types of plant and animals found on that land.When man builds a new town, this means the total destruction of vast areas of farmland or woodland. Here, you have the complete destruction of entire habitats and it is inevitable. It follows therefore, that every form of human activity unavoidably upsets or changes the wildlife complex of the area. Man has destroyed many forms of wildlife for no reasonable purpose. They have also made many great blunders in land use, habitat destruction and the extermination of many forms of wildlife Man's attitude towards animals depends on the degree to which his own survival is affected. He sets aside protection for animals that he hunts for sport and wages a war on any other creature that may pose a danger or inconvenience to him. This creates many problems and man has made irreversible, serious errors in his destruction of predators. He has destroyed animals and birds which are useful to farmers as pest controllers. The tragedy that emerges is that all the killing of predators did not in any way increase the number of game birds. …………………………………………………………………………………………………… …………………………………………………………………………………………………… …………………………………………………………………………………………………… …………………………………………………………………………………………………… …………………………………………………………………………………………………… …………………………………………………………………………………………………… …………………………………………………………………………………………………… …………………………………………………………………………………………………… 12 of 14

9.

M

QU Y

NH

ƠN

OF

FI CI A

L

…………………………………………………………………………………………………… …………………………………………………………………………………………………… …………………………………………………………………………………………………… …………………………………………………………………………………………………… ……………………………………………………………… …………………………………………………………………………………………………… …………………………………………………………………………………………………… …………………………………………………………………………………………………… …………………………………………………………………………………………………… ………………………………………………………………………………………………………… ………………………… Part 2. Report writing (15points) The charts below show the numbers of people in different age groups who go sailing or play basketball in Australia. Summarise the information by selecting and reporting the main features, and make comparisons where relevant. Write at least 150 words. 13

DẠ

Y

…………………………………………………………………………………………………… …………………………………………………………………………………………………… …………………………………………………………………………………………………… …………………………………………………………………………………………………… ………………………………………………………………………………………………………… …………………………………………………………………………………………………… …………………………………………………………………………………………………… …………………………………………………………………………………………………… 13 of 14

DẠ

Y

M

QU Y

NH

ƠN

OF

FI CI A

L

…………………………………………………………………………………………………… …………………………………………………………………………………………………… ……………………………………… ……………………………………… ………………………. …………………………………………………………………………………………………… …………………………………………………………………………………………………… …………………………………………………………………………………………………… …………………………………………………………………………………………………… …………………………………………………………………………………………………… ……………………………………… ……………………………………………………………… Part 3. Write an essay (30points) It is said that the characteristics we are born with have much more influence on our personality and development than any experiences we may have in our life. To what extent do you agree or disagree with that idea? Present your argumentation to highlight your opinion on this matter. Give reasons and specific examples to support your opinion(s). Write an essay (about 350 words) to express your opinion. …………………………………………………………………………………………………… …………………………………………………………………………………………………… …………………………………………………………………………………………………… …………………………………………………………………………………………………… …………………………………………………………………………………………………… …………………………………………………………………………………………………… …………………………………………………………………………………………………… …………………………………………………………………………………………………… …………………………………………………………………………………………………… …………………………………………………………………………………………………… …………………………………………………………………………………………………… …………………………………………………………………………………………………… …………………………………………………………………………………………………… …………………………………………………………………………………………………… …………………………………………………………………………………………………… …………………………………………………………………………………………………… …………………………………………………………………………………………………… …………………………………………………………………………………………………… …………………………………………………………………………………………………… …………………………………………………………………………………………………… …………………………………………………………………………………………………… …………………………………………………………………………………………………… …………………………………………………………………………………………………… …………………………………………………………………………………………………… …………………………………………………………………………………………………… …………………………………………………………………………………………………… …………………………………………………………………………………………………… …………………………………………………………………………………………………… …………………………………………………………………………………………………… …………………………………………………………………………………………………… …………………………………………………………………………………………………… THE END Người ra đề: Lại Thanh Tình (Lexico-grammar, Reading, Writing) Đỗ Hồng Ngọc Diệp (Listening)

14 of 14

ĐỀ THI CHỌN HỌC SINH GIỎI LẦN THỨ XIII MÔN THI: TIẾNG ANH – KHỐI 11 Ngày thi 15-16/07/2023 Thời gian làm bài 180 phút (Đề thi gồm … trang)

OF

FI CI A

L

HỘI CÁC TRƯỜNG THPT CHUYÊN KHU VỰC DUYÊN HẢI, ĐỒNG BẰNG BẮC BỘ TRƯỜNG THPT CHUYÊN BIÊN HOÀ, T. HÀ NAM

1. B

2. A

3. C

ƠN

SECTION A. LISTENING (50 points) Part 1: CPE Your answers: https://www.englishaula.com/en/cambridge-english-test-exam-preparation/cambridgeenglish-proficiency-c2-proficiency/listening/exam-parts/exercise-practice-test/0-3-55317875497697280/ 4. A

5. A

1. F

NH

Part 2: authentic Your answers: https://www.youtube.com/watch?v=pI19xAquCdM 2. T

3. T

4. T

5. F

Part 3: authentic Your answers: https://www.youtube.com/watch?v=iMtXqTmfta0&t=5s 2. biomimicry

3. pantograph

4. drag-reducing 5. lotus effect pattern

6. 15% less electricity

7. Novices

8. same serrations

9. prairie dog burrows

10. circular economy

M

QU Y

1. sonic boom

DẠ

Y

Part 4: IELTS short answer (Simulation Test 8, part 4) 1. (In) Rhyme / song 3. French Revolutionary Army 3. Scum of Earth 4. After military fiasco 5. Commander in Chief SECTION B. GRAMMAR & VOCABULARY (30 points) Part 1: (5 câu ngữ pháp, 15 câu từ vựng nguồn CPE) 1. D

2. C

3. B

4. D

5. B

6. D

7. B

8. C

9. A

10. B

11. A

12. A

13. A

14. C

15. D

16. B

17. A

18. B

19. B

20. D

2. personified

7. inconsistencies

3. patchy

8. Imprisoned

4. disheartened

9. unsolicited

5. unoriginal

10. analyses

FI CI A

6. inconsolable

OF

1. enigmatic

L

Part 2: 10 câu rời, nguồn CPE

SECTION C. READING (60 points) Part 1: (https://www.flypgs.com/en/extreme-sports) 2. the

3. with

6. sport

7. while/when

8. designed

2. D

3. B

4. D

5. high

9. without

NH

Part 2: (CPE Express book 2, test 2) 1. D

4. know

ƠN

1. to

5. A

6. B

7. A

8. B

10. own

9. A

10. D

Part 3: (Cambridge Objective IELTS advanced) 2.x

6.ix

7.xi

11.NG

12.YES

3.vii

4.ii

5.vi

8.iv

9.YES

10.NO

QU Y

1. v

13.B

Part 4: (https://edition.cnn.com/2022/12/05/tech/chatgpt-trnd/index.html) 2. H

M

1. B

3. A

4. D

5. F

6. C

7. E

Part 5: (CPE express Book 3) 2. D

DẠ

Y

1. D

3. A

4. F

5. D

6. E

7. A

8. F

SECTION D. WRITING (60 points)

Part 1: Key points: - Man has done more harm than good to the Earth. - Man has destroyed some wild species both directly and indirectly - Some species survive, but some have gone extinct due to human activities

9. B

10. C

FI CI A

L

- The only protection of animals done by man is only useful to man’s survival, man kills animals that cause harm and inconvenience to him. -> creating the imbalance in the ecology. Part 2: Striking features/trends: - sailing attracts more people from older age groups than younger ones - basketball attracts more pepple from younger age groups than older ones - more men take part in these two sports than women. - the gap between the number of men and women taking part in basketball is negligible in the age groups of 25-34 and 35-44

QU Y

NH

ƠN

OF

Sample:

DẠ

Y

M

\

FI CI A

L

ĐỀ THI ĐỀ XUẤT TRẠI HÈ DUYÊN HẢI – ĐBBB NĂM 2023 TRƯỜNG THPT CHUYÊN SƠN LA – TỈNH SƠN LA MÔN: TIẾNG ANH 10 (Đề thi có 14 trang) SECTION I: LISTENING (50pts)

Part 1: Questions 1-5. Complete the notes below. Write ONE WORD AND/ OR A NUMBER for each answer. (10pts) Customer Satisfaction Survey Name:

Sophie Bird

Occupation:

1………………………………..

OF

Customer details

Journey information Name of station returning to:

Staunfirth

ƠN

Type of ticket purchased: standard 2…………………………….ticket Where ticket was bought: online Satisfaction with the journey

NH

Most satisfied with: the Wi-Fi

Least satisfied with: the 3……………………………….this morning Satisfaction with station facilities

Most satisfied with: how much 4…………………………………….was provided

QU Y

Least satisfied with: lack of seats, particularly on the platforms Neither satisfied nor dissatisfied with: the 5……………………………..available

2.

(Cambridge IELTS ACADEMIC 15)

3.

4.

5.

M

Your Answers: 1.

Part 2. For questions 6-10, listen to a talk about fear and decide whether these statements are True (T)

or False (F). Write your answers in the corresponding numbered boxes provided. (10 points) 6. Fear and worry can be easily overcome once you understand awareness in the mind. 7. Worry and fear exist only in the future, not in the past.

Y

8. The person in the story missed their favorite radio show because they got stuck in the snow. 9. Developing concentration and willpower can help in keeping awareness on what you're doing and

DẠ

prevent it from going into the future in an uncontrolled way.

10. It is beneficial to let awareness go into the future to think about things in a negative way. https://www.youtube.com/watch?v=9WicYGH5X4M

Page 1 of 14

Your answers: 7.

8.

9.

10.

L

6.

Part 3: You will hear part of an interview in which a career coach called Charles Norris is discussing

FI CI A

job-hunting. For questions 11-15, choose the answer (A, B, or C) which fits best according to what you hear. (10pts) 11. What is NOT included in the role of a career coach? A. Recognizing a client’s job-hunting weaknesses. B. Advising clients how to update their CVs D. Assisting clients in improving their interview techniques

OF

C. Locating and securing employment for clients 12. According to Charles, the covering letter is important because______. A. It is an integral part of the CV.

ƠN

B. It contains all the details of the applicant. C. It is the same for every position D. It is the only part of the CV employers read.

NH

13. When discussing the internet, Charles suggests that job seekers should______. A. socialize on the internet to meet employers.

B. Clean up data that exists about them on the internet. C. Search for other job-seekers online.

QU Y

D. Apply to employers that use networking sites. 14. Charles believes one of his clients succeeded in his job search because he______. A. uploaded his CV to his profile.

C. included keywords in his profile.

B. had a detailed profile. D. Had a catchy headline on his profile.

15. What does Charles say about asking questions at an interview?

M

A. That you should only do if you have done your research. B. That you should never ask about a company’s competitor.

C. That you should have a list prepared before you go. D. That you should search for questions online. (On Screen workbook and Grammar book C1)

12.

13.

14.

15.

DẠ

11.

Y

Your answers:

Part 4. For questions 16-25, listen to Listen to a recording and complete the sentences with NO MORE THAN THREE WORDS AND/OR A NUMBER taken from the recording for each answer in the space provided.

- Scientists are searching the hidden corners of the world for viruses that are poised to become Page 2 of 14

(16) __________________________. - In many cases, they’re searching for Disease. That’s not the name of an actual illness. The next big worldwide

L

disease could be caused by what we call (17) ________________________________. in my own research on bacterial communities, so let’s break it down together!

FI CI A

- (18) ___________________ is the main tool they’re using to do that. This is actually something I’m using - In (19) ______________________ we take a sample—that can be soil, ocean water, a bodily fluid—and we purify it down to the genetic material of just the stuff we’re looking for. In this case, let’s say it’s viruses. So now we have the genomes of all the viruses in our sample.

- All of these genomes have to be read by our (20) ______________________________. And, if you can

OF

picture, it’s much faster for lots of people to each read a sentence. And they can each read their sentence at the same time a sentence than it is for one person to read a whole page, one at a time.

- A large number of (21) _______________________ are quickly identified by metagenomic sequencing, an incredibly useful tool.

ƠN

- Scientists have been surprised at how many they’re finding, everywhere, as they have turned that powerful lens on our world to (22) _______________________________.

- In the ocean, in our wastewater, inside the spiders in our gardens and of course, inside us. They’re preying other

marine

microorganisms,

releasing

nutrients.

This

is

probably

the

(23)

NH

on

______________________________ of the ocean’s food web.

- It’s likely that the balance of other organisms would rapidly get out of whack if viruses suddenly disappeared from the planet.

QU Y

- Maybe, a piece of (24) _______________________________ that jumped over to its mammalian host about 130 million years ago enabled modern humans to give birth. That gave us our ability to grow a placenta. So, yep. We got hacked, and it’s probably the reason we don’t lay eggs. - Scientists don’t know how many more viruses are out there, waiting to be discovered. Sequencing at least some of those nonillion viruses out there is the first step of finding how many of them may be harmful to us.

M

Identifying what they’re living inside...and how exactly these tiny, invisible puppeteers are (25)

____________________________ of our natural world is the next step. https://youtu.be/uRABqa8wPQw

Your answers: 16. 20.

DẠ

22.

Y

18.

24.

17. 19. 21. 23. 25.

Page 3 of 14

SECTION II: LEXICO-GRAMMAR (40pts) Part 1: Choose one word or phrase which best completes each sentence. Circle its corresponding letter

L

A, B, C, or D to indicate your answer. (20pts) 1. The teachers are not very qualified, but in all ______ to Principal Montara, the school hasn't received A. fairness

B. tenderness

FI CI A

funding to hire better teachers. C. fondness

D. hardiness

2. Words with strongly positive ______often persuade readers to see your ideas in a favorable way. A. calculations

B. connotations

C. considerations

D. compilations

3. She was kept in ______ of his true identity. B. intolerance

C. annoyance

D. ignorance

OF

A. acceptance

4. My brother says that snails have teeth but I don't know if he's ______ or if he's serious. A. giving me the cold shoulder

B. twisting my arm

C. pulling my leg

D. raising his eyebrows

A. long-lasting

B. absent-minded

ƠN

5. It was a very ______ article that truly made me stop and think.

C. thought-provoking

D. strong-willed

6. He was greatly ______ by the tragic news of the assassination of the Archduke and his wife. B. charmed

C. distressed

NH

A. baffled

D. intrigued

7. Be ______ enough to learn from your mistakes. A. spurious

B. wary

C. humble

D. eloquent

8. She gave him a look that made words ______. B. self-effacting

C. eclectic

QU Y

A. tedious

D. superfluous

9. Moving in together was a mistake, and things rapidly went ______. A. down bad

B. off

C. a long way

D. downhill

10. "______" is an American English metaphorical idiom for an obvious problem or risk that no one wants to discuss.

B. Crocodile tears

M

A. Lone wolf C. Elephant in the room

D. Butterflies in the stomach

A. do I

11. I don’t suppose you are telling us the true version of the story, ______? B. are you

C. aren’t you

D. will you

12. Not having written about the required topic, ________ a low mark. B. the teacher gives me

C. the teacher gave me

D. I was given

Y

A. my presentation was given

DẠ

13. Thomas received a warning for speeding. He _________ so fast. A. shouldn’t have driven

C. must have driven

B. would have driven

D. might have driven

14.________, they resumed their journey. A. Rising the sun

B. The sun having risen Page 4 of 14

C. The sun being risen

D. Having risen the sun

A. did not work too hard for three months

B. take it easy for three months

C. take it easy inside of three months

D. could take a vacation for three months

L

15. The doctor insisted that his patient ________.

A. reflected

B. reflecting

FI CI A

16. She walked indolently along, with a mind at rest, its peace ______ in her innocent face. C. being reflected

D. having reflected

17. ________ technically proficient; it also explores psychological questions. A. Not only is Barbara Astman's artwork

B. Not only Barbara Astman's artwork

C. Barbara Astman's artwork not only

D. Barbara Astman’s artwork not only

OF

18_________ talking of running for election again, after such a crushing defeat, is surely proof of his resilience. A. Should he be

B. That he is

C. Had he been

D. That he were

19. We took the shortcut round the market ______ late for class this morning. B. lest we be not

C. for fear that we should be

D. in order that we not be

ƠN

A. so that we will not be

20. “The President says the company is making more cars this year than last year.” “_______attribute the A. Does he

B. What to

NH

increase in productivity?”

C. To what does he

Your answers: 2.

3.

4.

5.

11.

12.

13.

14.

15.

QU Y

1.

D. What does he

6.

7.

8.

9.

10.

16.

17.

18.

19.

20.

Part 2: Use the word given in capitals at the end of each line to form a word that fits in the gap in the same line.

1. Though her novels do not feature the themes of Romanticism, Jane Austen’s work

M

was ______ with that of Wordsworth and Byron.

TEMPORAL GUARD

3. She spoke with______ of his impudent behaviour.

APPROVE

4. The Princess looked ______ beautiful at the wedding.

STUN

5. They tested the______ of water in the village wells.

TRANSPARENT

6. I counted on your ______ and you let me down.

DISCREET

7. The principal took______ measures against the culprits.

DISCIPLINE

8. During the war, a lot of______ fled to America.

REFUGE

9. After the death of his uncle, he became the______ owner of the castle.

LAW

10. She has had several______ of her illness lately.

RECUR

DẠ

Y

2. A______angel is believed to be a good spirit that protects each of us.

Page 5 of 14

6.

2.

7.

3

8.

4.

9.

5.

10.

FI CI A

1.

L

Your answers:

Part 3. Complete each of the following sentences with suitable preposition(s). Write your answer in the boxes provided on the answer sheet. (10 points) 2. Come sit ____ the fire and I’ll get you a hot cup of coffee.

OF

1. He left the country …………………………threat of arrest if he returned.

3. Many of the passengers looked shaken ____ when the pilot announced that the plane was having mechanical problems.

ƠN

4. Tommy was bummed ____ about his favorite TV show getting cancelled. 5. I was ____ the zone at my basketball game last night. I couldn’t miss. 6. Some of the neighbors’ kids threw eggs ____ our house on Mischief Night.

NH

7. Before I learned how to do my job, I felt like I was just getting ____ the way of my coworkers. 8. Can I substitute something else ____ the eggs in this recipe? I’m a vegan. 9. Before beginning her research career, my aunt went to college and graduated ____ a degree in biology.

Your answers: 1.

2.

6.

7.

QU Y

10. Are organic fruits and vegetables really superior ____ conventional produce?

3.

4.

5.

8.

9.

10.

M

SECTION C: READING (60 points) Part 1. Read the following passage and decide which option (A, B, C, or D) best fits each gap. Write your answers in corresponding numbered boxes on the answer sheet. (15 points) The concept of artificial intelligence (AI) has (1) ________ the boundaries of what was once thought possible. With its ability to process vast amounts of data and perform complex tasks, AI has (2) ________ potential in

Y

various fields.

One area where AI has made significant strides is in the field of healthcare. It has the capacity to (3) ________

DẠ

patterns and anomalies in medical images, aiding in the early detection and diagnosis of diseases. Moreover,

AI-powered virtual assistants can (4) ________ patient data, enabling healthcare professionals to make more informed decisions. However, there are ethical concerns regarding the privacy and security of sensitive

medical information.

Page 6 of 14

In the realm of transportation, AI is shaping the future of autonomous vehicles. Self-driving cars (5) ________ advanced algorithms and sensors to navigate roads, potentially reducing accidents and improving traffic flow.

L

Nevertheless, questions about legal liability and the ethics of decision-making in life-or-death situations (6) ________ unresolved.

FI CI A

AI has also revolutionized the way we interact with technology through natural language processing. Voice assistants like Siri and Alexa can (7) ________ user commands and respond with relevant information. However, concerns about data privacy and the potential for misuse has been raised.

Moreover, AI's impact extends to the job market. Automation powered by AI has led to the (8) ________ of certain jobs, while simultaneously creating new roles that require expertise in AI and data analysis. The (10)

OF

________ between humans and AI in the workforce is a topic of ongoing discussion and debate.

In conclusion, the advancement of AI presents immense possibilities and challenges across various domains. As we harness the power of AI, it is crucial to address ethical, legal, and societal implications to (10)______a balanced and responsible integration.

ƠN

Questions: B. extended

C. stretched

D. pushed

2. A. unlimited

B. boundless

C. immense

D. vast

3. A identify

B. recognize

C. distinguish

D. detect

4. A. analyze

B. assess

5. A. employ

B. utilize

6. A remain

B. persist

7. A. comprehend

B. decode

8. A elimination

NH

1. A expanded

D. examine

C. harness

D. exploit

C. endure

D. linger

C. interpret

D. understand

B. eradication

C. depletion

D. obsolescence

9. A. interaction

B. collaboration

C. integration

D. cooperation

10. A. assure

B. ensure

C. secure

D. confirm

QU Y

C. evaluate

Your answers: 2.

3.

4.

5.

7.

8.

9.

10.

M

1. 6.

Y

Part 2. Fill each of the following numbered blanks with ONE suitable word and write your answers in the corresponding numbered boxes on the answer sheet. (15 points) Organic nutrition or organic food is a phrase that is mentioned a lot nowadays. Organic food is the most (1)______food from housewives because of the health benefits it brings. But (2)______people understand

DẠ

what organic food is, what nutritional benefits this food brings, as well as how to choose these foods. The term “organic food” refers to foods that are grown, handled and prepared in a way that is safe for the environment. For agricultural products, it is a product that is grown (3)______the use of most conventional

Page 7 of 14

pesticides, using artificial fertilizers, sewage sludge, radiation and genetically modified organisms. For animal meat, antibiotics or hormones are not injected.

L

Organic food is usually good for the environment. (4)______, it is also relatively more expensive: the USDA

FI CI A

(US Department of Agriculture) reports that the cost of organic fruits and vegetables is often 20% more than

conventional products. Sometimes the difference can be much higher, (5)______for certain items like organic milk and eggs.

Advocates say organic food is safer, possibly more nutritious, and often tastier than non-organic food. They also say that organic production is (6)______for the environment and is also safer for animals. a lot more — sales in the industry are steadily increasing.

OF

And more and more consumers seem to be convinced by that. Although organic foods often cost more — or

Food scientist Alyson E. Mitchell, and his colleagues at the University of California, Davis have studied compounds (7)______flavonoids. Recent evidence suggests that these micronutrients play an important role

ƠN

in preventing cancer and heart disease.

Flavonoids are also compounds with plant protection effects. They protect against UV (8)______. They help fight fungi and bacteria. In addition, flavonoids do not have a favorable taste for pests. Normally, if a plant is

NH

(9)______by bugs, it starts to produce more flavonoids. Mitchell thinks plants sprayed with insecticides and fungicides won't produce as many flavonoids as plants grown organically. So her research team compared the flavonoid levels in fruits and vegetables grown in the same place but with

QU Y

different methods. The results showed that Flavonoids were found more in (10)______grown vegetables. https://www.vinmec.com/vi/news/health-news/nutrition/what-is-organic-food/

Your answers: 2.

1. 6.

4.

5.

8.

9.

10.

M

7.

3.

Part 3. Read the following passage and circle the best answer to each of the following questions. Write your answers in corresponding numbered boxes provided on the answer sheet. (15 points) 1. There has, in recent years, been an outpouring of information about the impact of buildings on the natural environment; Information which explains and promotes green and sustainable construction design, strives to

Y

convince others of its efficacy and warns of the dangers of ignoring the issue. Seldom do these documents

DẠ

offer any advice to practitioners, such as those designing mechanical and electrical systems for a building, on how to utilise this knowledge on a practical level. 2. While the terms green and sustainable are often considered synonymous, in that they both symbolise nature,

green does not encompass all that is meant by sustainability, which can be defined as minimizing the negative impacts of human activities on the natural environment, in particular those which have long-term and Page 8 of 14

irreversible effects. Some elements of green design may be sustainable too, for example those which reduce energy usage and pollution, while others, such as ensuring internal air quality, may be considered green despite

L

having no influence on the ecological balance.

FI CI A

3. Although there are a good many advocates of ‘green’ construction in the architectural industry, able to cite ample reasons why buildings should be designed in a sustainable way, not to mention a plethora of architectural firms with experience in green design, this is not enough to make green construction come into being. The driving force behind whether a building is constructed with minimal environmental impact lies with the owner of the building; that is, the person financing the project. If the owner considers green design unimportant, or of secondary importance, then more than likely, it will not be factored into the design.

OF

4. The commissioning process plays a key role in ensuring the owner gets the building he wants, in terms of design, costs and risk. At the predesign stage, the owner’s objectives, criteria and the type of design envisaged are discussed and documented. This gives a design team a solid foundation on which they can build their ideas,

ƠN

and also provides a specific benchmark against which individual elements, such as costs, design and environmental impact can be judged.

5. Owners who skip the commissioning process, or fail to take ‘green’ issues into account when doing so,

NH

often come a cropper once their building is up and running. Materials and equipment are installed as planned, and, at first glance, appear to fulfil their purpose adequately. However, in time, the owner realises that operational and maintenance costs are higher than necessary, and that the occupants are dissatisfied with the results. These factors in turn lead to higher ownership costs as well as increased environmental impact.

QU Y

6. In some cases, an owner may be aware of the latest trends in sustainable building design. He may have done research into it himself, or he may have been informed of the merits of green design through early discussion with professionals. However, firms should not take it as read that someone commissioning a building already has a preconceived idea of how green he intends the structure to be. Indeed, this initial interaction between owner and firm is the ideal time for a designer to outline and promote the ways that green design can meet the

M

client’s objectives, thus turning a project originally not destined for green design into a potential candidate.

7. Typically, when considering whether or not to adopt a green approach, an owner will ask about additional costs, return for investment and to what extent green design should be the limiting factor governing decisions in the design process. (1) Many of these costs are incurred by the increased cooperation between the various stakeholders, such as the owner, the design professionals, contractors and end-users. (2) However, in green

Y

design, they must be involved from the outset, since green design demands interaction between these

DẠ

disciplines. (3) This increased coordination clearly requires additional expenditure. (4) A client may

initially balk at these added fees, and may require further convincing of the benefits if he is to proceed. It is

up to the project team to gauge the extent to which a client wants to get involved in a green design project and provide a commensurate service.

Page 9 of 14

8. Of course, there may be financial advantage for the client in choosing a greener design. Case studies cite examples of green / sustainable designs which have demonstrated lower costs for long-term operation,

L

ownership and even construction. Tax credits and rebates are usually available on a regional basis for projects

FI CI A

with sustainable design or low emissions, among others. 1. The writer’s main purpose is to______.

A. explain to professionals how they can influence clients to choose greener designs

B. explain the importance of green building design in reducing long-term damage to the environment C. explain to owners commissioning a building why ignoring green issues is costly and dangerous

OF

D. explain to professionals why it is important to follow the correct procedures when a building is commissioned

2 The examples of green and sustainable designs given in paragraph 2 show that______. A. designs must be sustainable in order for them to be described as green

ƠN

B. for the purposes of this paper, the terms green and sustainable have the same meaning C. some sustainable designs are green, while others are not

D. some designs are termed green, even though they are not sustainable

NH

3 According to paragraph 3, the reason for the lack of green buildings being designed is that______. A. few firms have any experience in design and constructing buildings to a green design B. construction companies are unaware of the benefits of green and sustainable designs C. firms do not get to decide whether a building is to be constructed sustainably

QU Y

D. firms tend to convince clients that other factors are more important than sustainability 4. In paragraph 4, what does the phrase ‘envisaged’ mean? A. envisioned

B. decided

C. fixed

D. enclosed

5 Which of the following is NOT true about the commissioning process? A. It is conducted before the building is designed.

M

B. It is a stage that all clients go through when constructing a building. C. It is a step in the design procedure in which the client’s goals are identified.

D. It provides the firm with a measure of how well they did their job. 6 In paragraph 5, what does the phrase ‘come a cropper’ mean? A. experience misfortune

B. change one’s mind

C. notice the benefits

D. make a selection

Y

7 In paragraph 6, the writer implies that______.

DẠ

A. most clients enter the commissioning process with a clear idea of whether or not they want a green building

B. designers are usually less concerned about green design than the clients are C. the commissioning process offers a perfect opportunity to bring up the subject of green design D. firms should avoid working with clients who reject green designs in their buildings Page 10 of 14

8. Where in paragraph 7 does this sentence belong? In a typical project, landscape architects and mechanical, electrical and plumbing engineers do not become A. 1

B. 2

C.3

D.4

B. agree to pay

C. question the reason for

D. understand the need for

FI CI A

9 In paragraph 7, what does ‘balk at’ mean? A. display shock towards

L

involved until a much later stage.

10. Green buildings are most likely to incur more expense than conventional buildings due to______. B. higher long-term operational costs

C. the higher cost of green construction materials

D. increased coordination between construction teams

OF

A. higher taxes incurred on sustainable buildings

Your answers: 2.

3.

4.

5.

7.

8.

9.

10.

ƠN

1. 6.

Part 4. Read the passage and do the following tasks. Write your answers in the corresponding numbered boxes on the answer sheet. (15 points)

NH

The Maori are the indigenous people of the islands of New Zealand. Their traditional medicine, which is believed to date back as far as the 13th century, was a system of healing that was passed down through the generations orally. It comprised diverse practices and placed an emphasis on the spiritual dimension of health. in the muscles and bones.

QU Y

Its practice included remedies made from herbs, and physical therapies such as massage to relieve discomfort Maori systems for treating illness were well developed before European arrived in New Zealand in the late 1700s: they had quite detailed knowledge of anatomy and recognition of the healing properties of various plants. When Europeans first visited New Zealand, the average age of death for Maori adults was around 30. However, apart from this, the people were fit and healthy, and troubled by few diseases.

M

Illness was often seen as spiritually based. Maori saw themselves as guardians of the earth, and the focus of their existence was to remain at one with the natural and supernatural world. Rather than a medical problem,

sickness was often viewed as a symptom of disharmony with natures. In Maori culture, illnesses were divided into diseases of the gods (mate atua) and physical diseases (mate tangata). Diseases sent by the gods were often attributed to attacks by evil spirits, because the person had

Y

broken a religious rule. For instance, for Maori, Places where people had died, or places where their ancestors

DẠ

were buried were sacred, so if someone took food from a river where someone had died, or took a stick form a tree that had held their ancestor's bones and placed it on a cooking fire, it was believed that the gods could

punish them for their disrespectful acts by making them SICK. More than 200 plants were used medicinally by Maori. The leaves of the flax plant were used to treat skin

infections and food poisoning, and the hard part of the leaf was also used as a splint or brace for broken bones Page 11 of 14

and injured backs. Flax fibers were used along with a sharpened stick to sew up bad cuts. The bark and leaves of the pepper tree were used to heal cuts, wounds and stomach pain. People who had toothache were instructed

L

to chew the leaves of this same tree, and this was found to be of considerable benefit. The pepper tree was also used in vapor baths to treat people with painful joints.

FI CI A

Colonization by European in the 1800s had a significant effect on traditional Maori healing. Europeans brought many new diseases with them which Maori healers had limited ability to combat. Though Western

medicine was also relatively ineffectual at the time, this failure still strongly affected Maori confidence in their healers. Some western missionaries attributed the spread of disease to the fact the Maori did not believe in Christianity, and as Maori healers appeared powerless, many Maori accepted this explanation and turned to

OF

Christianity. Over time the schools of higher learning which ahd trained healers started to close and the tradition of the Maori healer declined.

From the late 20th century, there was renewed Maori interest in their traditional medicine. This was due to several factors. There was a resurgence of all aspects of Maori culture in New Zealand. Furthermore, people

ƠN

started to be less trusting of Western medicine-statistics from the 1970s came out revealing that Maori health continued to be poorer than that of other New Zealanders. There were also problems with access to health care for Maori. Additionally, there was and still a today a perceived lack of a spiritual dimension in Western health

NH

services.

Although Maori today largely accepted Western concepts of health and illness, and use the mainstream health system, there is significant demand for traditional medicine. This is true for unusual illnesses, or those that fail to respond to standard medical treatment, but also for common ailments such as the cold and influenza.

QU Y

Today's healers differ significantly from those of old times. Training is highly variable, usually informal, and often less tribally bound than the rigorous education of the traditional houses of higher learning. Many modern healers work in urban clinics, some alongside mainstream health professionals. They experiment, incorporating knowledge from Western and other medical systems. As a result, their modern day work has no standard system of diagnosis or widespread agreement about treatments. Despite this, many healers are

M

recognized as having knowledge and ability that has been passed down from their ancestors. The Maori language is also seen as important by many of those receiving treatment.

Question 1-5

In boxes 1-5 on your answer sheet, write TRUE if the statement agrees with the information FALSE if the statement contradicts the information

Y

NOT GIVEN if there is no information on this Early Maori healers learned their skills through studying written texts.

DẠ

1.

2. Diseases of the gods were believed to be more serious than physical diseases. 3. The leaves of the pepper tree were used to treat toothache. 4. Western religion was one reason why traditional Maori medicine became less popular. 5. Modern day Maori healers often reach the same conclusion about the type of treatment which is best. Page 12 of 14

Your answers: 2.

3.

4.

5.

L

1.

Questions 6-10

FI CI A

Complete the notes below. Choose ONE WORD ONLY from the passage for each answer. Write your answers in boxes 6-10 on your answer sheet. A short history of Maori healing Pre-European arrival

Maori were using plant based remedies, as well as treatment including massage

Diseases sent from the gods were thought to be caused by disobeying a spiritual 6.______________

Sickness could be attributed to eating food from a sacred 7. _____________or burning sacred wood

OF

After European arrival

ƠN

1800s

The inability of Maori healers to cure new diseases meant the Maori people lost 8 __________in

them.

NH

Eventually the schools for Maori healing began shutting down

1970s

Published 9. _______________showed that Maori were not as healthy as Europeans

2000s

Maori healers can be seen working with Western doctors in 10_________________ in cities

Many patients appreciate the fact that the Maoris language in used by healers

6.

QU Y

7.

8.

D. WRITING (50 points)

9.

10.

Nationality

M

Part 1. (20 points) The table below shows the number of hours per week spent by students of different nationalities studying different subjects at a university in the UK. Summarize the information by selecting and reporting the main features, and make comparisons where relevant. English

Maths

History

Science

Art

4

3

2

4

3

6

5

2

4

1

India

3

2

4

3

1

USA

4

2

2

4

2

Japan

3

3

3

3

3

Y

UK

DẠ

China

Page 13 of 14

Part 2. Essay writing (30 points) Some people believe that children should spend more time on individual studies at home, while others argue

L

that they should participate in group activities and sports.

FI CI A

Discuss both views and give your own opinion.

DẠ

Y

M

QU Y

NH

ƠN

OF

Write at least 250 words.

Page 14 of 14

1. journalist

2. return

FI CI A

L

HƯỚNG DẪN CHẤM ĐỀ THI ĐỀ XUẤT TRẠI HÈ DUYÊN HẢI – ĐBBB NĂM 2023 TRƯỜNG THPT CHUYÊN SƠN LA – TỈNH SƠN LA MÔN: TIẾNG ANH 10 (HDC gồm có: 08 trang) SECTION I: LISTENING (50pts) Part 1: Questions 1-5. Complete the notes below. Write ONE WORD AND/ OR A NUMBER for each answer. (10pts) 3. delay

4. information

5. parking

6. TRUE

7. TRUE

OF

Part 2. For questions 6-10, listen to a talk about fear and decide whether these statements are True (T) or False (F). Write your answers in the corresponding numbered boxes provided. (10 points) 8. FALSE

9. TRUE

10. FALSE

ƠN

Part 3: You will hear part of an interview in which a career coach called Charles Norris is discussing job-hunting. For questions 11-15, choose the answer (A, B, or C) which fits best according to what you hear. (10pts) 11. D 12. A 13. A 14. B 15. D

QU Y

16. human pathogens 17. hypothetical infection 18. Metagenomics 19. metagenomic sequencing 20. sequencing machine

NH

Part 4. For questions 16-25, listen to Listen to a recording and complete the sentences with NO MORE THAN THREE WORDS AND/OR A NUMBER taken from the recording for each answer in the space provided. 21. microbes 22. catalogue viruses 23. bottommost foundation 24. viral genetic code 25. pulling the strings

SECTION II: LEXICO-GRAMMAR (40pts) Part 1: Choose one word or phrase which best completes each sentence. Circle its corresponding letter A, B, C, or D to indicate your answer. (20pts) 3. D 13. A

M

2. B 12. D

4. C 14. B

5. C 15. B

6. C 16. A

7. C 17. A

8. D 18. B

9. D 19. C

10. C 20. C

1. A 11. B

DẠ

Y

Part 2: Use the word given in capitals at the end of each line to form a word that fits in the gap in the same line. 1. contemporaneous 6. discretion 2. guardian 7. disciplinary 3 disapproval 8. refugees 4. . stunningly 9. lawful 5. transparency 10. recover Part 3. Complete each of the following sentences with suitable preposition(s). Write your answer in the boxes provided on the answer sheet. (10 points) 1. under 2. by 3. up 4. out 5. in Page 1|8

6. at

7. in

8. for

9. with

10. to

1. A 6. A

2. C 7. C

3. D 8. D

4. A 9. A

FI CI A

answers in corresponding numbered boxes on the answer sheet. (15 points)

L

SECTION C: READING (60 points) Part 1. Read the following passage and decide which option (A, B, C, or D) best fits each gap. Write your 5. B 10. B

Part 2. Fill each of the following numbered blanks with ONE suitable word and write your answers in the corresponding numbered boxes on the answer sheet. (15 points) 2. few 7. called

3. without 8. radiation

4. However 9. attacked

OF

1. chosen 6. sustainable

5. especially 10. organically

2. D

3. C

7. C

8. B

NH

1. A 6. A

ƠN

Part 3. Read the following passage and circle the best answer to each of the following questions. Write your answers in corresponding numbered boxes provided on the answer sheet. (15 points) 4. A

5. B

9. A

10. D

Part 4. Read the passage and do the following tasks. Write your answers in the corresponding numbered boxes on the answer sheet. (15 points) 2. NOT GIVEN

3. TRUE

4. TRUE

5. FALSE

6. rule

7. river

8. confidence

9. statistics

10. clinics

D. WRITING (60 points)

QU Y

1. FALSE

M

Part 1: 20 points Contents (10 points) The report MUST cover the following points: - Introduce the charts (2 points) and state the overall trends and striking features (2 points)

- Describe main features with relevant data from the charts and make relevant comparisons (6 points) Language use (10 points) The report:

DẠ

Y

- should demonstrate a wide variety of lexical and grammatical structures - should have correct use of words (verb tenses, word forms, voice...) and mechanics (spelling, punctuations,) Part 2: (30 points)

The mark given to part 3 is based on the following criteria: 1. Organization (5 points) a. Ideas are well organized and presented with coherence, cohesion and unity. Page 2|8

b. The essay is well-structured: * Introduction is presented with clear thesis statement. * Body paragraph are written with unity, coherence and cohesion.

FI CI A

L

Each body paragraph must have a topic sentence and supporting details and examples when necessary. * Conclusion summarizes the main points and offers personal opinions (prediction,

OF

recommendation, consideration ...) on the issue. 2. Content (15 points) a. All requirements of the task are sufficiently addressed. b. Ideas are adequately supported and elaborated with relevant and reliable explanations, examples, evidence.... 3. Language use (10 points) a. Demonstration of a variety of topic-related vocabulary.

ƠN

b. Excellent use and control of grammatical structures (verb tenses, word forms, voice...) and mechanics (spelling, punctuations...)

DẠ

Y

M

QU Y

PART 1. IELTS 15. TEST 4. PART 1

NH

TRANSCRIPTS

Page 3|8

L FI CI A OF ƠN NH QU Y M KÈ Y DẠ

PART 2. https://www.youtube.com/watch?v=9WicYGH5X4M So how do you overcome fear and worry? Right this is something we all experience in different degrees from time to time. This is very easy to overcome once you understand awareness in the mind. Let's look at a simple view of time. Right you have the past, you have the present which is now, and the future. Right this is a very simplified way of looking at time. Page 4|8

QU Y

NH

ƠN

OF

FI CI A

L

Worry and fear only exists in the future. You cannot worry anything about anything in the past. It's already happened, but you can be concerned or fearful of what the consequences that may take place in the future. Fear and worry is future based. Fear and worry is awareness leaving the present moment going into the future, creating a situation that has not happened coming back to the present, and worrying about it. I'll tell you a story that my guru shared with me when he was seven years old. This was in 1934 up in Lake Tahoe. His family was coming back in the car and it was snowing really heavily, and he was really worried that he would miss his favorite radio program called captain midnight. And back then if you miss your favorite radio show there's no internet that you can go and like listen to it again. Right, there's somebody on the phone going captain midnight. Right, and that's all you get to listen you get one shot. So sitting in the car, it was snowing really heavily, the car was driving and he was thinking “my god i'm going to miss my show. We're going to get stuck in the snow. I'm not going to get home in time, not going to be able to watch my rad, listen to my radio show.” And then he saw what was happening in his mind. He saw his awareness going into the future, creating a situation in his mind where they were getting stuck in the snow and then he was coming back his awareness, came back to the present and started worrying about this thing that had not happened yet. Then he asked himself “Are we stuck in the snow?” and his answer was “no”. “Are we still moving” “Yes am I still on schedule to get home to watch my show”. “Yes, so what am I worried about?” right and he said to himself I am all right right now. And that's a beautiful affirmation to say every time you're fearful or worry say to yourself. I am all right right now but to understand and overcome fear and worry you have to understand awareness. And the mind fear and worry is based on the fact that awareness is leaving the present moment going into the future in your head, creating a situation that has not happened. coming back to the present and worrying about that or fearing that. So every time awareness goes into the future in an uncontrolled way and creates a situation in your mind. Right that is not wholesome bring it back to the present and say I'm not going to let it do it and that's why you want to develop concentration, and willpower right. So you can keep awareness on what you're doing. It's okay to let awareness go into the future as long as it goes into the future to think out things in a very positive way.

DẠ

Y

M

PART 3. ON SCREEN C1. MODULE 3- 3d. listening

Page 5|8

Page 6|8

Y

DẠ M

KÈ QU Y ƠN

NH

FI CI A

OF

L

L FI CI A OF ƠN

PART 4. https://youtu.be/uRABqa8wPQw

DẠ

Y

M

QU Y

NH

The current pandemic has highlighted how important it is for us to know what viruses may be lurking out there. But while we keep one wary eye out for those potentially harmful ones, we’re also uncovering just how much viruses have actually shaped humans—and the rest of life on Earth—in some really surprising ways. ‘Virus hunters’, as they’re known, are scientists who search the hidden corners of the world for viruses that are poised to become human pathogens. In many cases, they’re searching for Disease. That’s not the name of an actual illness—it's what we call the hypothetical infection that could cause the next big global disease. The idea is to find it before it finds us. But other researchers are delving into what else viruses are doing on our planet besides making people sick. It’s estimated that there are about this many individual viruses on our planet. And we’ve only made the most miniscule dent in that number: we’ve found and classified about 9,000 of the viruses on Earth. And of those, only about 200 are known to cause disease in humans. But over the past decade, scientists have been identifying new viruses by the thousands. And the main tool they’re using to do that...is metagenomics. This is actually something I’m using in my own research on bacterial communities, so let’s break it down together.! In metagenomic sequencing, we take a sample—that can be soil, ocean water, a bodily fluid—and we purify it down to the genetic material of just the stuff we’re looking for. In this case, let’s say it’s viruses. So now we have the genomes of all the viruses in our sample. Lets picture each virus's genome as a piece of paper with sentences on it. Because the next step—and this sounds nuts— is that we rip each piece of paper up into little strips with just sentences on them. We do this because our sequencing machine has to ‘read’ all of these genomes. And, if you can picture, it’s much faster for lots of people to each read a sentence. And they can each read their sentence at the same time a sentence than it is for one person to read a whole page, one at a time. That’s what we’re doing when we divide our genome up into little chunks. Now that we’ve read the sentences in a super efficient way, we can put the sentences back together into the full page, or the full genome. Then we compare that to a library full of known genomes. So we can say, ‘this unknown page is the same as this known page that we have on file’. It’s a match! We’ve ID’ed the virus in our sample. And we do this for every virus in our sample. Metagenomic Page 7|8

DẠ

Y

M

QU Y

NH

ƠN

OF

FI CI A

L

sequencing is an incredibly useful tool that lets us identify huge numbers of microbes really fast. And as scientists have turned that powerful lens on our world to catalogue viruses...they’ve been astonished at just how many they’re finding, everywhere. In the ocean, in our wastewater, inside the spiders in our gardens and of course, inside us. Most of them are totally harmless to humans, and many of them are totally new to science! The viruses in the ocean? They’re preying on other marine microorganisms, releasing nutrients. This may be the bottommost foundation of the ocean’s food web. The viruses in every ecosystem? They help maintain our planet’s biodiversity by keeping other species in check. If viruses suddenly vanished from the planet, it’s likely that the balance of other organisms would rapidly get out of whack. And one other major result of all this sequencing is a new understanding of just how thoroughly viruses have shaped life on Earth as we know it. Because they invade living cells to replicate, they’re equipped to take over a host’s genetic replication machinery— sometimes, viral genetic material gets incorporated into the host’s. It’s estimated that around 8% of human DNA actually came from a virus, including some of our most important skills. Like, the ability of modern humans to give birth probably came from a piece of viral genetic code that jumped over to its mammalian host about 130 million years ago. That gave us our ability to grow a placenta. So, yep. We got hacked, and it’s probably the reason we don’t lay eggs. Scientists don’t know how many more viruses are out there, waiting to be discovered. And of those, we have no idea how many of them may be harmful to us. But sequencing at least some of those nonillion viruses out there is the first step. The next step is to identify what they’re living inside...and how exactly these tiny, invisible puppeteers are pulling the strings of our natural world. If you want more on just how many viruses are in our oceans and what they’re doing there, you can check out this video, and leave us a comment down below if there’s another viral topic you’d like to see us cover.

Page 8|8

FI CI A

L

TRƯỜNG THPT CHUYÊN SƠN LA – TỈNH SƠN LA ĐỀ ĐỀ XUẤT KỲ THI CHỌN HSG KHU VỰC DH VÀ ĐBBB NĂM HỌC 2022-2023 MÔN: TIẾNG ANH – LỚP 11 (Đề thi có 20 trang)

DẠ

Y

M

QU Y

NH

ƠN

OF

I. LISTENING (50 POINTS) Part 1: You will hear part of a discussion in which two business owners, Anya Stern and Vincent Chambers, are talking about their experiences of launching their own businesses. For questions 1-5, decide whether the following statements are TRUE (T) or FALSE (F). (10 points) 1. Anya's parents were successful in adapting to e-commerce. 2. Vincent worked for a large tech giant before starting his own business. 3. Anya wanted to bring Brazilian cuisine to the mass market. 4. Anyone can have a unique business proposition. 5. You should always abandon your plans if you encounter adversity. Your answers: 1. 2. 3. 4. 5. Part 2: Listen to a talk about a new approach in the field of genomics and answer the questions. For questions 6-10, write NO MORE THAN THREE WORDS taken from the recording for each answer in the corresponding numbered spaces provided. (10 points) 6. In what aspect of biology does the scientific community has a limited understanding until now? 7. How is the current process of interpreting scientific outcomes described? 8. What element of the new approach enables the examination of biological systems in great detail? 9. Along with a chemical process, what is utilized to conduct single-cell sequencing? 10. Apart from promoting research within biology, in what way can this advance in genomics contribute scientifically? Your answers: 6. 9. 7. 10. 8. Part 3: You will hear a discussion in which academics Gordon Mackie and Sophie Blackmore talk about how communication has changed in society. For questions 11–15, select the best answer A, B, C or D. (10 points) 11. In his latest book, Gordon aims to challenge the notion that… Page 1 of 20

DẠ

Y

M

QU Y

NH

ƠN

OF

FI CI A

L

A. technology has changed the purpose of communication. B. linguistic standards in society are falling. C. the media influences people’s use of language. D. the evolution of language is inevitable. 12. Gordon says his main duty as an academic is to… A. ensure language traditions survive in modern communication. B. record examples of linguistic patterns and trends. C. explain the fundamental principles of correct language use. D. evaluate different theories about language change. 13. Sophie says the thing she values most in written communication is: A style. B accuracy. C clarity. D tone. 14.Why does Sophie use social media posts in her classes? A. To raise awareness of what catches readers’ attention B. To show how people alter their communication styles C. To highlight the features of informal communication D. To prove that context changes the meaning of a message 15. What aspect of ‘text language’ do Sophie and Gordon disagree about? A. Whether it will ever be accepted in education B. Why it may be favored by young people C. How well it crosses cultural boundaries D. Whether it influences spoken communication Your answers: 11. 12. 13. 14. 15. Part 4: For questions 16-25, listen to a presenter talking about a phenomenon in the nature and complete the summary by writing NO MORE THAN THREE words and/or a number in each gap. (20 points) - Nutrients are the building blocks of plant life. - Plants naturally absorb nutrients from the soil through their root system. However, farmers turn to fertilizer when facing poor soil or massive 16. __________. - Farmers tend to overuse fertilizer as a precautionary measure because it is difficult to 17. _____________the amount of fertilizer needed. - Excess fertilizer can runoff into bodies of water causing 18. ______________, characterized by the rapid accumulation in the population of algae in marine water systems. - Dense layers of algae form an 19. _________ on the surface of the water, blocking other plants in the water from getting the sunlight they need to survive. - Because water cannot support aquatic life, plants die off and sink to the bottom of the water body, where decomposers 20. the dead bodies. This decomposition process consumes plenty of dissolved oxygen and animals that rely on oxygen to breathe can, in fact, 21.

Page 2 of 20

OF

FI CI A

L

________. A vicious cycle of degradation in aquatic life can be observed and a 22. _______is created. - If this process occurs in lakes, native species can give way to 23. _________. If this happens in the ocean, the incidence of coral bleaching is inevitable. - Nutrient-rich runoff, also known as 24. ___________, can also be the result of 25. ___________and farming activities. Your answers: 16. 21. 17. 22. 18. 23. 19. 24. 20. 25.

DẠ

Y

M

QU Y

NH

ƠN

II. LEXICO AND GRAMMAR (30 POINTS) Part 1: Choose the answer A, B, C, or D that best completes each of the following sentences. Write your answers in the corresponding numbered boxes. (20 points) Grammar 26. The problem of pollution is a _____ topic. A. hotly debating B. debating hotly C. debated hotly D. hotly debated 27. I never bought a telephone answering machine. They’re just not ___________ useful. A. that B. too C. that far D. that much 28. The laws of this country apply to everyone, ___________ you are rich or poor. A. either B. whether C. both D. including 29. Using English regularly is often the key ___________ fluent in it. A. in becoming B. to become C. to becoming D. becoming 30. This project is ___________ for the level of this class. A. too much really hard B. much too really hard C. really much too hard D. really hard too much Vocabulary 31. When my sister has an exam to take, she either studies day and night or does nothing at all. Shecan never find the____________. A. a double bind B. a leading light C. the common touch D. the happy medium 32. Their business ____________better than expected and profits were slightly up. A. came about B. brought up C. set off D. made out 33. We can buy some more things if you want – that won’t break____________. A. the mold B. the ice C. the bank D. the budget 34. Tom wanted to be a professional footballer, but he didn't make the________ . A. grade B. point C. mark D. best Page 3 of 20

Y

M

QU Y

NH

ƠN

OF

FI CI A

L

35. This software is called a “firewall” because it is an __________obstacle when someone wants to access the system's data. A. unyielding B. insurmountable C. infrangible D. invulnerable 36. North Korea is a small country but _________ a great punch on the world stage as it possesses nuclear weapons. A. throws B. blows C. packs D. lands 37. I don’t want to make a decision ___________, so I need a couple of minutes to think about it. A. in one go B. there and then C. at a stroke D. on and off 38. It’s their __________ of stupidity to go swimming in the beach in this stormy weather. A. height B. depth C. source D. matter 39. A little ___________ told me that you were accepted to Harvard. Is that correct? A. dog’s B. birdie C. fox D. cat’s 40. The number of foreign tourists coming to Vietnam has been growing __________. A. by leaps and bounds B. from time to time C. slow but sure D. by hook and crook 41. People turned out in _________ into My Dinh Stadium to watch the friendly match between the Vietnamese team and the Japanese team A. volume B. mass C. force D. bulk 42. Not being able to find my email address is a pretty _________excuse for not contacting me. A. fragile B. frail C. faint D. feeble 43. The pollution problems on the beaches have been ___________ by mass tourism in the summer months. A. exacerbated B. developed C. augmented D. contributed 44. You’ll have to _________ yourself to the fact that you can’t always have what you want. A. acknowledge B. reconcile C. concede D. allow 45. It was a fantastic movie. I enjoyed it _________. A. endlessly B. unendingly C. without end D. no end Your answers: 26. 27. 28. 29. 30. 31. 32. 33. 34. 35. 36. 37. 38. 39. 40. 41. 42. 43. 44. 45.

DẠ

Part 2: Give the correct form of each given word in CAPITAL to complete the following sentences. Write your answers in the numbered boxes. (10 points) 46. Chemicals are easily ________if they are laid open in the atmosphere for a long time. (ACTION) 47. His speech is __________ dull! Page 4 of 20

NH

ƠN

OF

FI CI A

L

(SPEAK) 48. The new students will be ________________ classified into different classes. (QUALIFY) 49. A person with an___________ complex is generally quite shy. (INFERIOR) 50. The New Year 2023’s _________ show involves almost every famous face of the showbiz. (STAR) 51. Lot of young people go abroad in ___________ of their study. (PERSUE) 52. Jason was ____________ from all the major social media sites for spreading a fake new. (PLATFORM) 53. The conflict had a seriously ___________ effect on the region. (STABLE) 54. ______________ children often have poor concentration and require very little sleep. (ACTIVE) 55. High __________ among suppliers, distributors and customers via Internet is promoting development of e-commerce in Vietnam. (OPERATE)

QU Y

Your answers: 46. 47. 48. 49. 50.

51. 52. 53. 54. 55.

DẠ

Y

M

III. READING (60 POINTS) Part 1: Read the text below and think of the word which best fits each space. Use only ONE word in each space. Write your answers in the space provided. (15 points) People are often (56) _________ off meditation by what they see as its many mystical associations. Yet meditation is a straightforward technique which merely involves sitting and resting the (57) _________ In addition to its simplicity, meditation offers powerful help in the battle (58) _________stress. Hundreds of studies have shown that meditation, when (59) _________in a principled way, can reduce hypertension which is related to stress in the (60) _________ Research has proved that certain types of meditation can substantially decrease key stress symptoms such as anxiety and irritability. In fact, those who practise meditation with any (61) _________ see their doctors less and spend, on average, seventy per cent (62) _________ days in hospital. They are said to have more stamina, a happier disposition and even enjoy better relationships. Page 5 of 20

FI CI A

L

When you learn to meditate, your teacher will give you a personal 'mantra' or word which you use every time you practise the technique and which is supposedly chosen according to your needs. Initial classes are taught individually but (63) _________ classes usually consist of a group of students and take place over a period of about four days. The aim is to learn how to slip into a deeper (64) _________ of consciousness for twenty minutes a day. The rewards (65) _________ for themselves. Your answers: 56. 57. 58. 59. 60. 61. 62. 63. 64. 65.

DẠ

Y

M

QU Y

NH

ƠN

OF

Part 2: Read the article below and choose the answer (A, B, C or D) which you think fits best according to the text. (10 points) OIL AGAINST CARIBOU… They hiked over mountains and canoed along crashing Arctic rivers. They were guided by a sun that set for only an hour a night and braced by freezing winds. [A] When they finally made it to the log cabins of Arctic Village on Friday, their cries of joy and their dancing and singing for a moment drowned out the deep fear that drove them on their odyssey. [B] The 100,000-strong Gwich’in tribe’s ‘millennium trek’ was a desperate plea for survival. One of the last tribes of native Americans to live by subsistence, they fear that they – and the caribou on which they depend – are about to lose a 25-year battle that could end in one last Klondikelike scramble for oil. [C] It is a battle that has pitched Alaska against the rest of America and the Indians and environmentalists against Britain’s biggest oil company. [D] The age-old conflict between development and environment has nowhere been so stark as in this fragile corner of the world. Nowhere are man’s thirst for oil – and the effects of global warming – having such an impact. The remote Gwich’in community of Arctic Village, 100 miles north of the Arctic Circle and with no access by road, nestles against the southern edge of the vast Arctic National Wildlife Refuge. The refuge is the last true wilderness in North America, so abundant in wildlife it has been dubbed America’s Serengeti. Here are 20 million acres of soaring snow-capped mountains, marshy coastal plains, Arctic tundra and forests of stunted trees, with no signs of man: no roads nor car parks, no visitor centres nor hiking trails. In the short summers the tundra explodes into vibrant reds and yellows. In winter it is smothered in snow. Visitors talk about a primal landscape that takes them back to a time before man, but to the environmentalists and the Gwich’in, it is the wildlife that makes it unique. It has the only population of Alaskan polar bears that live on land; it has the near-extinct shaggy musk ox, the regal moose, grizzly bears, wolverine and lynx. The refuge is the breeding ground for 150 species of birds that migrate to the US, Asia and South America for the bitter winters, when for three months the sun does not rise and temperatures drop below minus 50 centigrade. Above all, it is the 130,000 caribou which dominate the landscape and on which the Gwich’in depend for much of their food. Each year the caribou go on one of the last great Page 6 of 20

DẠ

Y

M

QU Y

NH

ƠN

OF

FI CI A

L

mammal migrations on the planet, trekking from 1,000 miles away in Canada, through the 9,000ft Brooks range and swimming precariously across sweeping rivers, to the narrow coastal strip of the refuge. Here, in the calving grounds sacred to the Gwich’in, the caribou fatten up with cotton grass to get them through the Arctic winter and, protected from predators, the cows give birth. But amid the majestic scenery are a few oil seeps, a tell-tale sign of what is underneath: below the feet of the grazing caribou lies enough crude oil to fill up to 16 billion barrels – worth many tens of billions of pounds – and the oil companies are pushing hard to start drilling. Oil is already produced along much of the rest of the Alaskan coast but is starting to dry up. At the Prudhoe Bay field, just along from the refuge, production has fallen from two million barrels a day a decade ago to a million now. And the momentum to raid the refuge for oil, which will further promote global warming, is building. Although not all shareholders are enthusiastic, preferring money to be spent on renewable energy sources, the oil industry can count on the support of the majority Alaskans. Oil has made this once poor and punishing state rich and far more comfortable, and now accounts for 85 per cent of its economy. Anchorage, not so long ago a small, insignificant town, now boasts soaring, mirrored office blocks, and every village now has a million-dollar school. Oil has also brought many thousands of wellpaid jobs, and opening up the refuge will create up to 700,000 more, and with current oil fields drying up, pressure for new jobs is intense. Prudhoe Bay is one of the largest industrial developments in the world, with hundreds of miles of pipelines snacking across the damp tundra, disappearing off to the horizon in every direction. There are hundreds of well heads and processing facilities, flares, landing strips, roads, sea water processing plants and accommodation blocks. In between, the land is indeed clean and major oil spills have been avoided. However, the company’s record here is far from unblemished: earlier this year it had to pay a $15m fine for toxic waste dumping. For the Gwich’in, there is a far more fundamental issue. Faith Gemmill, their spokeswoman, showed me around her ancestors’ cemetery, on a bend in a river overlooked by the mountains. She then declared: “This is how we have lived for thousands of years. No one has a right to take this away from us for money or greed. This is what they did to the Sioux when they killed the buffalo. When is this country going to learn they can't do that to a people? Questions: 66. Which of the square brackets [A], [B], [C], or [D] best indicates where in the first paragraph the sentence “For 800 miles across the wilderness they trekked, from the Yukon in Canada to remotest Alaska, far above the Arctic Circle.” can be inserted? A. [A] B. [B] C. [C] D. [D] 67. The Gwich’in tribe seem to have… A. travelled to gain publicity. B. taken part in a traditional dance ritual. C. recently become aware of a threat. D. been at war with neighboring tribes. 68. What is meant by the phrase “live by subsistence” in the first paragraph? Page 7 of 20

DẠ

Y

M

QU Y

NH

ƠN

OF

FI CI A

L

A. People depend only on the natural resources available to them in their immediate environment for their survival. B. People deal with a clash between industrial and environmental interest. C. People live in an area where the caribou breed – perhaps one which is being threatened by oil pollution. D. People suffer one of the various forms of anti-environmental activity which threaten the caribou’s habitat. 69.Their destination, Arctic Village… A. falls within a wildlife reserve. B. is home to many wild animals. C. is part of an area under threat. D. is completely inaccessible. 70. According to the article, locals and tourists… A. are struck by the diverse natural surroundings. B. are mainly interested in birds. C. are awed by the area's long history. D. have different perceptions of the area. 71. The caribou are unusual in that… A. they survive in an oil field. B. they make seasonal journeys. C. they are considered sacred. D. they can only eat certain grasses. 72. Greatest support for the drilling to begin comes from… A. oil companies. B. the Alaskan locals. C. the unemployed. D. the Anchorage authorities 73. The writer implies that the installation at Prudhoe Bay… A. is a taste of things to come. B. is larger than it should be. C. is without environmental impact. D. is aesthetically pleasing. 74. What point is Faith Gemmill making when she mentions the Sioux and the buffalo in the last paragraph? A. Just as the traditional life of the Sioux depended on the survival of the buffalo, the traditional life of the Gwich’in will be destroyed if the oil company destroys the habitat of the caribou. B. The local population will support the oil industry because drilling has brought wealth and prosperity to other Alaskan communities. C. The Sioux are a tribe of North American Indians famed for their struggle to protect their territories and buffalo from the white settlers in the mid-nineteenth century. D. The benefits of drilling for oil in the area where caribou graze is: the huge quantity of crude oil which lies beneath the caribou grazing ground; and the wealth which would greatly improve the quality of life for thousands of people living and working in the area. 75. Regarding the clash between the environmentalists and the oil companies, the writer’s attitude can best be described as… A. cynical B. balanced C. balanced D. aloof Your answers Page 8 of 20

67.

68.

69.

70.

71.

72.

73.

74.

75.

L

66.

DẠ

Y

M

QU Y

NH

ƠN

OF

FI CI A

Part 3: Read the passage and do the tasks that follow. (13 points) Adults and children are frequently confronted with statements about the alarming rate of loss of tropical rainforests. For example, one graphic illustration to which children might readily relate is the estimate that rainforests are being destroyed at a rate equivalent to one thousand football fields every forty minutes – about the duration of a normal classroom period. In the face of the frequent and often vivid media coverage, it is likely that children will have formed ideas about rainforests – what and where they are, why they are important, what endangers them – independent of any formal tuition. It is also possible that some of these ideas will be mistaken. Many studies have shown that children harbour misconceptions about ‘pure’, curriculum science. These misconceptions do not remain isolated but become incorporated into a multifaceted, but organized, conceptual framework, making it and the component ideas, some of which are erroneous, more robust but also accessible to modification. These ideas may be developed by children absorbing ideas through the popular media. Sometimes this information may be erroneous. It seems schools may not be providing an opportunity for children to reexpress their ideas and so have them tested and refined by teachers and their peers. Despite the extensive coverage in the popular media of the destruction of rainforests, little formal information is available about children’s ideas in this area. The aim of the present study is to start to provide such information, to help teachers design their educational strategies to build upon correct ideas and to displace misconceptions and to plan programmes in environmental studies in their schools. The study surveys children’s scientific knowledge and attitudes to rainforests. Secondary school children were asked to complete a questionnaire containing five open-form questions. The most frequent responses to the first question were descriptions which are selfevident from the term ‘rainforest’. Some children described them as damp, wet or hot. The second question concerned the geographical location of rainforests. The commonest responses were continents or countries: Africa (given by 43% of children), South America (30%), Brazil (25%). Some children also gave more general locations, such as being near the Equator. Responses to question three concerned the importance of rainforests. The dominant idea, raised by 64% of the pupils, was that rainforests provide animals with habitats. Fewer students responded that rainforests provide plant habitats, and even fewer mentioned the indigenous populations of rainforests. More girls (70%) than boys (60%) raised the idea of rainforest as animal habitats. Similarly, but at a lower level, more girls (13%) than boys (5%) said that rainforests provided human habitats. These observations are generally consistent with our previous studies of pupils’ views about the use and conservation of rainforests, in which girls were shown to be Page 9 of 20

DẠ

Y

M

QU Y

NH

ƠN

OF

FI CI A

L

more sympathetic to animals and expressed views which seem to place an intrinsic value on non-human animal life. The fourth question concerned the causes of the destruction of rainforests. Perhaps encouragingly, more than half of the pupils (59%) identified that it is human activities which are destroying rainforests, some personalizing the responsibility by the use of terms such as ‘we are’. About 18% of the pupils referred specifically to logging activity. One misconception, expressed by some 10% of the pupils, was that acid rain is responsible for rainforest destruction; a similar proportion said that pollution is destroying rainforests. Here, children are confusing rainforest destruction with damage to the forests of Western Europe by these factors. While two fifths of the students provided the information that the rainforests provide oxygen, in some cases this response also embraced the misconception that rainforest destruction would reduce atmospheric oxygen, making the atmosphere incompatible with human life on Earth. In answer to the final question about the importance of rainforest conservation, the majority of children simply said that we need rainforests to survive. Only a few of the pupils (6%) mentioned that rainforest destruction may contribute to global warming. This is surprising considering the high level of media coverage on this issue. Some children expressed the idea that the conservation of rainforests is not important. The results of this study suggest that certain ideas predominate in the thinking of children about rainforests. Pupils’ responses indicate some misconceptions in basic scientific knowledge of rainforests’ ecosystems such as their ideas about rainforests as habitats for animals, plants and humans and the relationship between climatic change and destruction of rainforests. Pupils did not volunteer ideas that suggested that they appreciated the complexity of causes of rainforest destruction. In other words, they gave no indication of an appreciation of either the range of ways in which rainforests are important or the complex social, economic and political factors which drive the activities which are destroying the rainforests. One encouragement is that the results of similar studies about other environmental issues suggest that older children seem to acquire the ability to appreciate, value and evaluate conflicting views. Environmental education offers an arena in which these skills can be developed, which is essential for these children as future decision–makers. Questions 76-83 Do the following statements agree with the information given in Reading Passage 1? In boxes 76-83, write: TRUE if the statement agrees with the information FALSE if the statement contradicts the information NOT GIVEN if there is no information on this 76. The plight of the rainforests has largely been ignored by the media. Page 10 of 20

QU Y

NH

ƠN

OF

FI CI A

L

77. Children only accept opinions on rainforests that they encounter in their classrooms. 78. It has been suggested that children hold mistaken views about the ‘pure’ science that they study at school. 79. The fact that children’s ideas about science form part of a larger framework of ideas means that it is easier to change them. 80. The study involved asking children a number of yes/no questions such as ‘Are there any rainforests in Africa?’ 81. Girls are more likely than boys to hold mistaken views about the rain forests’ destruction. 82. The study reported here follows on from a series of studies that have looked at children’s understanding of rainforests. 83. A second study has been planned to investigate primary school children’s ideas about rainforests. Questions 84-88 The box below gives a list of responses A–P to the questionnaire discussed in Reading Passage 1. Answer the following questions by choosing the correct responses A–P. Write your answers in boxes 84–88. 84. What was the children’s most frequent response when asked where the rainforests were? 85. What was the most common response to the question about the importance of the rainforests? 86. What did most children give as the reason for the loss of the rainforests? 87. Why did most children think it important for the rainforests to be protected? 88. Which of the responses is cited as unexpectedly uncommon, given the amount of time spent on the issue by the newspapers and television?

DẠ

Y

M

A. There is a complicated combination of reasons for the loss of the rainforests. B. The rainforests are being destroyed by the same things that are destroying the forests of Western Europe. C. Rainforests are located near the Equator. D. Brazil is home to the rainforests. E. Without rainforests some animals would have nowhere to live. F. Rainforests are important habitats for a lot of plants. G. People are responsible for the loss of the rainforests. H. The rainforests are a source of oxygen. I. Rainforests are of consequence for a number of different reasons. J. As the rainforests are destroyed, the world gets warmer. K. Without rainforests there would not be enough oxygen in the air. L. There are people for whom the rainforests are home. M. Rainforests are found in Africa. N. Rainforests are not really important to human life. Page 11 of 20

L

FI CI A

O. The destruction of the rainforests is the direct result of logging activity. P. Humans depend on the rainforests for their continuing existence. Your answers: 76. 77. 78. 79. 80. 81. 82. 83. 84. 85. 86. 87. 88.

DẠ

Y

M

QU Y

NH

ƠN

OF

Part 4: Read the text. Seven paragraphs have been removed. Choose from the paragraphs A- H the one which fits each gap (89-95). There is ONE extra paragraph which you do not need to use. (0.7 pt) Would I lie to you? Blatant dishonesty has invaded our culture. Sue Jackson explains how to spot a liar. Who hasn’t told a lie? Even the most upstanding individual probably utters one occasionally to help the day to run more smoothly. But, according to the experts, the extent to which people regularly tell serious untruths has exploded. Lying has pervaded every aspect of our lives. 89 Research in California reveals that people lie up to 20 times a day, while in a poll last year, a quarter or respondents admitted being untruthful on a daily basis. Only 8 per cent claimed they had never lied – although there is always the chance, that even then they weren’t being honest. Many of these will be sweet little lies, the type psychologists refer to as ‘false positives’ and the sort we are all guilty of committing when we want to appear more enthusiastic about something than we really are. 90 Until recently it was thought that only manipulative and Machiavellian characters were prone to excessive fabrication of this sort, but research has proved otherwise. According to experts, anyone under pressure or with the being enough incentive is prepared to say something that isn’t true. 91 That figure rose to one in three among people with university qualifications. Apparently, this sort of background gives people the vocabulary and the confidence to deceive. They are more sophisticated and plausible than you might elsewhere in society. 92 The proliferation of lying in corporate culture means that there are huge profits to be gained by companies who can weed out fraudsters before employing them. Numerous studies have been conducted, including using video cameras, to analyze people who lie. There are two main methods of ousting liars, although one, the mechanical lie detector or polygraph, requires subjects to be trussed up in electrodes, so it hardly lends itself to interviews. That leaves body language and psychological testing. 93 Page 12 of 20

OF

FI CI A

L

However, sometimes the subconscious takes over. Liars often start blinking fast, a visual sight that the brain is concentrating hard on the task in hand, and are likely to frequently touch their body and face with their hands. Liars are also more likely to tap or swing a foot as they speak. 94 Everyone seems to agree that good liars don’t show non-verbal signals, so you need to know what to look for. Lying takes a lot of effort, so often they will rely on past experience to see them through and reduce a cognitive load. 95 Experts, however, agree that the one person you shouldn’t deceive is yourself – and that, once you begin to do so, it is a sure sign that your untruthfulness is getting out of hand.

DẠ

Y

M

QU Y

NH

ƠN

Paragraphs to choose: A. This makes detecting the charlatan who fibs his way through a CV very difficult. In the film Liar, Liar the comedian Jim Carrey played a smooth-talking lawyer and consummate liar who specializes in dealing with untrustworthy clients whom no one else will take on. Only when his young son made a wish to see his father get through an entire day without lying was Carrey’s character forced to tell the truth. Mayhem ensued. B. Things like ‘That was delicious, thank you’, ’You look great in that dress’ and ‘Of course I want to see you’. They are mostly considered harmless social fabricants. But at the other end of the spectrum are compulsive liars who are effortlessly dishonest. C. Visual clues are not wholly reliable, as experienced deceivers are aware of the common give-away signs and take calculated measures to avoid them. Shifty eyes, for instance, are traditionally thought to be a sure way to tell whether someone is being dishonest, but experienced fabricators will capitalize on this myth. D. So, for instance, people who are lying about where they have been may declare they were at the cinema or the gym so that their untruths doesn’t take too much mental planning. It is easier to make up story about something they know well and have done many times. E. ‘We are experiencing an epidemic of lying’ says Professor Leo Damak, an expert in lie detection at a leading university. ‘It has always been around, but we are much more aware of it now.’ In one study of college students, 85 per cent of couples reported that one or both of them had lied about past relationships or recent events. In another, it was found that dating partners lie to each other in about a third of their conversations. F. A recent study found that pathological liars are just as likely to be self-confident, attractive and popular as they are introverted and withdrawn. It also seems that the better educated a person is, the higher level of deceit. It was found that falsehoods typically occurred in one fifth of all ten-minute conversations they have.

Page 13 of 20

FI CI A

L

G. Obviously, many won’t stand for ambiguity any more. By being more aware of how and why someone will tell a lie, they have more chance of catching him before he tells another and causes real harm. H. However, vocabulary and sounds are generally considered more reliable indicators than body movements. Liars tend to use fewer words, take longer to start answering a question and a pause a lot as if to mentally rehearse what they are about to say. Their voices may adopt a high pitch and they are prone to repetition. Your answers: 89. 90. 91. 92. 93. 94. 95.

DẠ

Y

M

QU Y

NH

ƠN

OF

Part 5: You are going to read a set of science book reviews. For questions 96–105, choose from the reviews (A-D). The reviews may be chosen more than once. Write your answer in the numbered boxes. (15 points) Prairie Fever A new book chronicles the efforts of 19th century British aristocrats to create a corner of England in the American west. A. How the British aristocracy was drawn to the frontier lands of 19th-century America is perhaps the most bizarre episode in the country's epic immigration story, and is revealed in a remarkable new book, Prairie Fever, by veteran BBC documentary maker Peter Pagnamenta. Lured by romantic tales of the American outdoors by writers such as James Fennimore Cooper, and the real-life gun-slinging escapades of Wild Bill Hickock, these eccentric newcomers wanted the U.S. on their own terms. In settlements with reassuringly British names, such as Runnymede and Victoria, the British aristocracy set about ensuring that there was one corner of America that was forever England. B. The pioneers started arriving in the 1830s. Some were sportsmen drawn by the promise of unlimited buffalo to hunt, others true adventurers. They were led by Scotsman Sir William Stewart, a Waterloo veteran who spent seven years trekking through the Rockies, rubbing shoulders with mountain men, and fending off marauding bears and Indians. His companion, Charles Murray, son of the Earl of Dunmore, lived for a spell with the Pawnee Indians. The Old Etonian had to swallow his pride when his hosts ate his dog, but he impressed with rockthrowing contests in which he used skills honed in the Highland Games. Sadly, few of the lords that followed were nearly so adaptable. They often treated the locals and their customs with utter contempt. Sir George Gore — a classic example of the breed -- went on a $100,000, three-year hunting expedition beginning in 1854 in Missouri. American officials later accused him of slaughtering 6,000 buffalo, single-handedly endangering the Plains Indians Food supply. Later, the English settlers wound up the Americans even more because of their air of superiority. C. By the 1870s, however, their American hosts had more to complain about than aristocratic rudeness – the British wanted to settle permanently. The British ruling classes had Page 14 of 20

M

QU Y

NH

ƠN

OF

FI CI A

L

realised that the American West wasn't just a good place to hunt and carouse, but also the perfect dumping ground for younger sons with few prospects at home, America, desperate for new settlers to farm prairie states like Kansas and Iowa, welcomed them with open arms. Back in Britain, the Press followed the settlers closely. ‘It was hot but everyone looked happy ... how much more sensible and useful lives they live there than they would live here at home!’ the Times reported. Yet more astute observers noted that the British settlers never grasped the American work ethic. For them, running their farms came a poor second to hunting and enjoying themselves. D. The prairie states were already dotted with ‘colonies’, each made up exclusively of workers from one part of America or one group of immigrants such as Danes or Russians. In 1873, an enterprising Scottish gentleman farmer named George Grant had a brainwave - a colony in western Kansas populated entirely by the British upper classes, by stipulating that they had to have at least £2,000 in funds and would each get no less than a square mile of land, he kept out the rabble. Victoria, as Grant patriotically called his settlement, was talked back home as a ‘Second Eden’, but the new arrivals - many of whom had never farmed in their lives - soon discovered it was a hard place to play the country gentleman. No rain would fall for months and the temperature could soar to 1050F in the shade. Worst of all, nobody had mentioned the dense clouds of grasshoppers that would suddenly arrive and eat everything. Despite their neighbors’ derision at these remittance men’ (so named because they relied on allowances from their parents), the two hundred or so colonists gamely battled on. E. In general, the colonists’ dreams came to nothing, and many headed home. But there was one event that definitively ended the British aristocracy’s love affair with the West. Encouraged by the vast sums to be made from cattle ranching, some wealthy British investors bought huge tracts of land. One investor alone amassed 1.75 million acres and 100,000 cattle. Enough was enough. Tolerant when the British were buffoonish adventurers, Americans felt threatened once they became too rich. U.S. politicians stoked anti-foreigner resentment, aided by widows out of their homes and rumours that some were so snobby they referred to their cowboys as ‘cow-servants’. Congress passed the Alien Land Act limiting foreign companies to buying no more than 5,000 acres in future.

DẠ

Y

In which paragraph is each of the followings mentioned? 86. the opinion that the settlers never got their priorities right 87. the fact the settlers wanted nothing less than a home away from home 88. the wish to maintain exclusivity in the British colonies 89. the inability of the settlers to become truly independent of Britain 90. the view that the English were naive in their expectations of the USA 91. a difference in the locals’ and settlers’ cultural taboos 92. the view that the British settlers were victims of their own success 93. the fact that America offered a solution to a problem

Your answers 96. ________ 97. ________ 98. ________ 99. ________ 100. _______ 101. _______ 102. _______ 103. _______ Page 15 of 20

103.

104.

L

104. _______ 105. _______ 105.

FI CI A

94. a newspaper showing lack of perception 95. the disregard of the settlers for the locals’ way of life Your answers: 96. 97. 98. 99. 100. 101. 102.

DẠ

Y

M

QU Y

NH

ƠN

OF

IV. WRITING (60 POINTS) Part 1: Read the following extract and use your own words to summarize it. Your summary should be about 100-120 words long. (15 pts) The sea turtles, especially the leatherbacks, are undoubtedly one of nature amazing creatures. They roam the warm seas of the world throughout their come ashore only to lay eggs. Malaysia with its long sandy shorelines has destined as one of their breeding grounds. Unfortunately, the number of leatherback turtles landing on the beaches has been declining over the years. According WWF estimates, about 2000 leatherbacks arrived on Malaysian beaches in 19 1989, only between 30 and 60 leatherbacks were found. The drastic drop within a short span is certainly a great cause for concern. Where have they gone to? Are avoiding the beaches of Malaysia? Whatever it is, the message is disturbingly cl they may become an extinct species soon, that is, if no concerted effort is taken to check the decline now. The belief that the declining landings of turtles in Malaysia is the result of increasing landings in others parts of the world is a fallacy. This is because scenario is the same in other countries known to have been visited by the turtles. What has caused the species to dwindle at such a rapid rate? There are many reasons but an obvious one is none other than man's greed. As we know, turtles are killed for their meat. In the days before refrigeration, turtles had been a source of fresh food for sailing ships. Today, turtle soup is a favorite dish among the Asians. Their shells have become coveted items for decorations and jewelry. Their eggs which are meant to hatched into young turtles, are instead harvested and eaten. When deep sea fishing nets inadvertently trap the turtles, fisherman often kill the turtles instead of cutting their nets to release them. Pollution of the sea has also reduced the number of turtles. Many are choked to death by the plastic bags that they mistake for jelly fish. It appears that the turtles are no longer safe in the sea where they spend most of the lives. Neither are they safe when the females come ashore to lay eggs. In fact, this is the time when they are particularly vulnerable as their movements are slow on land. The nesting places for these turtles have also been greatly reduced. As more and more beaches are taken over for tourism with the construction of hotels, chalets and condominiums, the breeding grounds are reduced in the process. The intrusion of tourists into these places make it difficult for the turtles to lay their eggs. They have to look elsewhere to places that are still quiet and undisturbed to lay their eggs. Unfortunately, these ideal places are few to come by now. Too much development has taken place even along the coastline in most countries. Perhaps all is not lost yet. Sincere efforts are being taken to check the decline. In Malaysia, it is heartening to note that concrete steps have been taken to protect the turtles that Page 16 of 20

DẠ

Y

M

QU Y

NH

ƠN

OF

FI CI A

L

come ashore to lay eggs. The indiscriminate collection of turtles’ eggs on the beaches is no more allowed. Turtle sanctuaries have been set up in Rantau Abang in Terengganu. The eggs collected by designated officials are sent to hatcheries in the sanctuaries. In this way, the loss of eggs and the rate of mortality among the baby turtles are reduced. In other words, more baby turtles are now able to return to sea and grow into adulthood. In an effort to discourage the public from eating turtle eggs a Turtle Enactment Act has been introduced to prohibit the sale of leatherback eggs. The WWF has also launched the ‘Save the Turtle Campaign’ to create an awareness among the public to help save the endangered species. In this way, the consumption of turtle eggs and perhaps turtle meat will be discouraged, thus putting a stop to the illegal sale of eggs and trapping of turtles. Let us hope that it is not too late to save these fascinating creatures from becoming extinct. Your answer:

Page 17 of 20

ƠN

OF

FI CI A

L

Part 2: Graph description (15 points) The charts below give information about the way in which water was used in different countries in 2022. Summarize the information by selecting and reporting the main features, and make comparisons where relevant. You should write at least 150 words.

DẠ

Y

M

QU Y

NH

Your answer:

Page 18 of 20

DẠ

Y

M

QU Y

NH

ƠN

OF

FI CI A

L

Part 3: Essay writing (30 points) It is evident that a significant proportion of current high school students engage in cheating during tests and exams, or consider cheating to be acceptable. Some people blame this trend on the intense academic pressure and excessive workload placed upon students. Others think that it is no more than the problem of students’ dishonesty and insufficient selfrespect. Discuss both views and state your opinion. Use specific reasons and examples to support your answer. You should write at least 350 words. Your answer:

Page 19 of 20

L FI CI A OF ƠN NH QU Y M KÈ Y

- THE END -

DẠ

GV ra đề: Hà Huy Khánh – Trường THPT Chuyên Sơn La Số ĐT: Email:

Page 20 of 20

FI CI A

L

TRƯỜNG THPT CHUYÊN SƠN LA – TỈNH SƠN LA HƯỚNG DẪN CHẤM ĐỀ ĐỀ XUẤT KỲ THI CHỌN HSG KHU VỰC DH VÀ ĐBBB NĂM HỌC 2022-2023 MÔN: TIẾNG ANH – LỚP 11 (Hướng dẫn chấm có 06 trang)

ƠN

OF

I. LISTENING (50 POINTS) Part 1: You will hear part of a discussion in which two business owners, Anya Stern and Vincent Chambers, are talking about their experiences of launching their own businesses. For questions 1-5, decide whether the following statements are TRUE (T) or FALSE (F). (10 points) 1. False (They struggled to adapt and their clothes shop never recovered.) 2. True 3. True 4. True 5. False (You should not automatically abandon your plans if you encounter adversity.)

2. incremental and slow 5. fuel medical breakthroughs

QU Y

1. inner workings 4. proprietary tagging method

NH

Part 2: Listen to a talk about a new approach in the field of genomics and answer the questions. For questions 6-10, write NO MORE THAN THREE WORDS taken from the recording for each answer in the corresponding numbered spaces provided. (10 points) 3. technology suite

Part 3: You will hear a discussion in which academics Gordon Mackie and Sophie Blackmore talk about how communication has changed in society. For questions 11–15, select the best answer A, B, C or D. (10 points) 12. B

13. C

14. D

15. A

M

11. B

Part 4: For questions 16-25, listen to a presenter talking about a phenomenon in the nature and complete the summary by writing NO MORE THAN THREE words and/or a number in each gap. (20 points)

DẠ

Y

(https://www.youtube.com/watch?v=mLbDbmmV6Qc&t=26s) 16. erosion or leaching 17. gauge 18. algal bloom 19. impenetrable roof 20. feast on 21. suffocate 22. positive feedback loop 23. invasive 24. eutrophication 25. clear-cutting

Page 1/10

FI CI A

L

II. LEXICO AND GRAMMAR (30 POINTS) Part 1: Choose the answer A, B, C, or D that best completes each of the following sentences. Write your answers in the corresponding numbered boxes. (20 points) 26. A 27. A 28. B 29. C 30. C 31. D 32. D 33. C 34. A 35. B 36. C 37. B 38. A 39. B 40. A 41. C 42. D 43. A 44. B 45. D

OF

Part 2: Give the correct form of each given word in CAPITAL to complete the following sentences. Write your answers in the numbered boxes. (10 points) 46. deactivated 51. pursuit 57. unspeakably 52. deplatformed 48. qualifiedly 53. destabilizing 49. inferiority 54. Hyperactive 50. all – star 55. interoperability

ƠN

III. READING (60 POINTS) Part 1: Read the text below and think of the word which best fits each space. Use only ONE word in each space. Write your answers in the space provided. (15 points) 56. put 57. mind 58. against 59. undertaken 60. body 61. regularity 62. fewer 63. subsequent 64. state 65. speak

NH

Part 2: Read the article below and choose the answer (A, B, C or D) which you think fits best according to the text. (10 points) OIL AGAINST CARIBOU… 66. A 67. A 68. A 69. C 70. D 71. B 72. B 73. A 74. A 75. C

DẠ

Y

M

QU Y

Part 3: Read the passage and do the tasks that follow. (13 points) 76. False. In the third sentence of Paragraph One the opposite is said – the media gives ‘frequent and often vivid’ coverage of deforestation. 77. False. Third sentence of Paragraph one mentions children probably having formed their own opinion on the problem of deforestation ‘independent of any formal tuition’, i. e. the information that influenced their opinions didn’t come from their classrooms, so the opposite is true. 78. True. First sentence of the second paragraph uses a rather difficult phrasing that contains the answer: children ‘harbour misconceptions’ about the pure science taught at school. To harbour misconceptions about something is to have wrong ideas about it. 79. True. Sentence two of Paragraph Two phrases the idea in a complicated way. The ideas about ‘pure’ science are not isolated, but are a part of some bigger system of knowledge. This fact makes it easier to change these ideas, they are ‘accessible to modification’. 80. False. Paragraph Four, second sentence mentions ‘open-form questions’, which is the direct opposite of yes/no questions. Therefore, False should be used for the answer. Remember that when the opposite statement is true, then the answer should be ‘False’. 81. Not given. Paragraph Six focuses on the differences in responses between boys and girls. However, it does not make a comparison of how likely boys or girls are to have mistaken views. Page 2/10

OF

FI CI A

L

82. True. Paragraph Six, second sentence confirms that this study follows the series of studies focused on the importance of rainforests and how school pupils understand it. 83. Not given. No such or similar information can be found in the text. 84. M. Last sentence of Paragraph Four mentions Africa as the most prevalent answer. 85. E. Paragraph Four, the second sentence points out the popular opinion of children about rainforests providing habitats for animals. 86. G. Paragraph Seven, sentence two says that number one reason of rainforests destruction according to pupils is human activity. 87. P. Paragraph Nine focuses on the popular misconception about potential volume of oxygen which wouldn’t be produced were the rainforests completely destroyed. Hence the idea of why it is so important to protect the rainforests. First sentence of Paragraph Ten can also be used to answer the question. 88. J. Second sentence of Paragraph Ten has the author of the text surprised by how few of the pupils are aware of the possible threat of global warming associated with deforestation.

90. B

91. F

92. A

NH

89. E

ƠN

Part 4: Read the text. Seven paragraphs have been removed. Choose from the paragraphs A- H the one which fits each gap (89-95). There is ONE extra paragraph which you do not need to use. (0.7 pt) Would I lie to you? 93. C

94. H

95. D

96. C 101. B

97. A 102. E

QU Y

Part 5: You are going to read a set of science book reviews. For questions 96–105, choose from the reviews (A-D). The reviews may be chosen more than once. Write your answer in the numbered boxes. (15 points) Prairie Fever A new book chronicles the efforts of 19th century British aristocrats to create a corner of England in the American west. 98. D 103. C

99. D 104. C

100. D 105. B

Y

M

IV. WRITING (60 POINTS) Part 1: Read the following extract and use your own words to summarize it. Your summary should be about 100-120 words long. (15 pts) Students at least mention to two ideas: 1. How and why the number of leather back turtles has declined 2. What steps have been taken to prevent the decline

DẠ

Suggested Summary: The declining landings of turtles in Malaysia are due to uncontrolled catching for their meat, shells and eggs, many are inadvertently caught in deep sea fishing nets. Pollution of the sea has also reduced the number of turtles. Many die of suffocation by discarded plastic bags.

Page 3/10

FI CI A

L

Besides, turtles are most vulnerable when they come ashore to lay eggs. Their nesting places have also been reduced due to development for tourism along the coasts. Nevertheless, concrete steps have been taken to arrest the decline. Indiscriminate collection and sale of turtle eggs are banned. Turtle sanctuaries have been set up, thereby reducing the rate of mortality among the baby turtles. Lastly, an awareness campaign has been launched by WWF to discourage the public from eating turtle eggs. (125 words)

QU Y

NH

ƠN

OF

Part 2: Graph description (15 points) The charts below give information about the way in which water was used in different countries in 2022. Summarize the information by selecting and reporting the main features, and make comparisons where relevant. You should write at least 150 words.

DẠ

Y

M

. Suggested Description: The two charts give figures for water use in different parts of the world in 2000. The first indicates that almost three-quarters of world consumption (70%) was for agriculture, while 22% was used for industry and a mere 8% for domestic purposes. This pattern is almost identical to that for China in 2022, whereas South Africa used even more water (92%) for agriculture and only 8% for industrial and domestic sectors. In contrast, New Zealand used almost equal proportions for agriculture and household use, 44% and 46% respectively, and a slightly higher 10% was consumed by industry. The pattern in the USA is almost the reverse of the world average, with a mere 8% of water consumed by agriculture and a massive 80% by industry Only 12% was used by the domestic sector, which was almost a quarter of the New Zealand industrial consumption. Overall, the data show that water use in the two developing countries is closer to the world patterns of consumption. (161 words)

Page 4/10

Discuss both views and state your opinion. Use specific reasons and examples to support your answer. You should write at least 350 words.

FI CI A

L

Part 3: Essay writing (30 points) It is evident that a significant proportion of current high school students engage in cheating during tests and exams, or consider cheating to be acceptable. Some people blame this trend on the intense academic pressure and excessive workload placed upon students. Others think that it is no more than the problem of students’ dishonesty and insufficient selfrespect.

M

QU Y

NH

ƠN

OF

MARKING SCALES: The mark given to Part 3 is based on the following criteria: 1. Organization (5 pt) a. Ideas are well organized and presented with coherence, cohesion and unity. b. The essay is well-structured: * Introduction is presented with clear thesis statement. * Body paragraphs are written with unity, coherence and cohesion. Each body paragraph must have a topic sentence and supporting details and examples where necessary. * Conclusion summarizes the main points and offers personal opinions on the issue. 2. Content (15 pt) a. All requirements of the task are sufficiently addressed. b. Ideas are adequately supported and elaborated with relevant and reliable explanations, examples, evidence.... 3. Language use (10 pt) a. Demonstration of a variety of topic-related vocabulary. b. Excellent use and control of grammatical structures (verb tenses, word forms, voice...) and mechanics (spelling, punctuations...).

DẠ

Y

Suggested Essay Cheating among high school students has become a noticeable and concerning trend in recent years. A considerable number of students are either actively participating in cheating during tests and exams or perceive it as a normal practice. This issue has sparked a debate among experts and observers, with divergent opinions emerging regarding its underlying causes and implications. One school of thought attributes the prevalence of cheating to the intense academic pressure and overwhelming workload experienced by students. In today's competitive educational environment, students face immense pressure to excel academically and secure Page 5/10

M

QU Y

NH

ƠN

OF

FI CI A

L

their future. This pressure often leads to a mindset where the end justifies the means, making cheating appear as a viable option to achieve desired outcomes. Moreover, the heavy burden of assignments, projects, and exams leaves students feeling overwhelmed and drained, pushing some towards cheating as a means of coping with the workload. On the other hand, some argue that cheating among students is primarily a reflection of their dishonesty and lack of self-respect. According to this viewpoint, the problem lies within the moral values and ethics instilled in students. They believe that a strong emphasis on character development and integrity should be in place to deter cheating behavior. Supporters of this perspective argue that students who possess a strong sense of self-respect and moral principles are less likely to resort to cheating, even under high-pressure circ*mstances. In my opinion, both factors contribute to the prevalence of cheating among high school students. The intense academic pressure and heavy workload undoubtedly create an environment where cheating seems like an attractive option. However, the issue of dishonesty and insufficient self-respect cannot be ignored either. It is crucial to address both aspects to effectively tackle this problem. Educational institutions should adopt measures to alleviate academic pressure by promoting a balanced approach to learning. This can be achieved by implementing welldesigned curriculum structures that prioritize understanding and critical thinking over rote memorization. Additionally, teachers and educators must emphasize the importance of ethical conduct and foster a culture of integrity within the school community. Encouraging open discussions about the consequences of cheating and providing guidance on time management and study skills can also play a significant role in reducing cheating incidents. In conclusion, the prevalence of cheating among high school students is a complex issue with multiple contributing factors. While academic pressure and heavy workloads certainly play a role, addressing the problem of dishonesty and promoting self-respect is equally important. By adopting a comprehensive approach that combines academic reform and character development, we can create an environment where cheating is discouraged, and students are encouraged to pursue success with integrity. -THE END-

DẠ

Y

GV ra đề: Hà Huy Khánh – Trường THPT Chuyên Sơn La Số ĐT: 0912.453.903 Email: [emailprotected]

Page 6/10

LISTENING TRANSCRIPTS (Transcripts có 4 trang)

L

Part 1:

FI CI A

Interviewer Today we’re talking about business start-ups. I’m joined by Anya Stern, who set up the Pepper chain of restaurants, and Vincent Chambers, an IT entrepreneur. Anya, where does your entrepreneurial drive come from? Was that encouraged at an early age?

OF

Speaker 1 Well, ‘influenced’ might be a better word. I passively absorbed useful business insights from my parents. Of course, it was mainly things I’d only appreciate much later. Like the importance of doing things your own way rather than worrying about what other people are doing. But one thing has stuck with me since my childhood. I saw my parents struggling to adapt when e-commerce came in. They put in incredibly long hours to salvage their clothes shop but sadly, it never recovered. It showed me that effort alone isn’t enough in business. That sparked my curiosity about why some businesses work and others fail.

NH

ƠN

Speaker 2 Unlike Anya, I didn’t have any business influences in my formative years. Entrepreneurs were just the people I’d seen on TV making million-dollar deals buying and selling things! It didn’t dawn on me that there could be IT entrepreneurs, or that I didn’t have to fit a specific mould to start my own business. That’s why I went down the standard route of working for a large tech giant. Admittedly, it ensured my financial security. And of course, I amassed a lot of invaluable business skills too. But the hardest obstacle was still convincing myself that people like me could run their own business.

QU Y

Speaker 1 Vincent’s point about TV is interesting. There used to be limited opportunities to see business portrayed onscreen. Even now, many programmes purport to be about business, but they’re not really. I’m talking about reality competitions where contestants are trying to win investment. I can see how young people might tune in, especially if there are contestants, they personally relate to. But the contestants do things they’d never get away with in the real world. Presumably, the content is manipulated by the producers for the sake of entertainment. But how many people with great business ideas are discouraged because of the behaviour they see on these shows? I’d prefer broadcasters to focus on more inspirational business programmes. Interviewer But what is it that inspires people to want to get into business in the first place?

M

Speaker 1 One key factor’s probably wanting to be your own boss. We all want a say in how things are managed. But it depends on the individual. I’ve even met people who launched businesses as a hobby after they’d retired and their children had left home. For me, I was desperate to bring Brazilian cuisine to the mass market because I felt the restaurant sector was ready for something new.

DẠ

Y

Speaker 2 Yes, many entrepreneurs are spurred on by spotting an opportunity. For instance, I wanted to devote my time to virtual reality, which was really compatible with my corporate role. That was my personal tipping point, and I could see potential to transform IT. I hadn’t formulated any specific plan at that point, but it felt like it was too important an opportunity to pass up.

Page 7/10

L

Speaker 1 That’s exactly the kind of thinking that makes an entrepreneur. Anyone can have a unique business proposition, but not everyone’s brave enough to take the plunge, or deal with the negative feedback.

FI CI A

Speaker 2 Yes, because even if your business model isn’t particularly innovative, there’ll be times when you encounter adversity. So above all, you must be ready to respond to issues as they arise. I’m not saying you should automatically abandon your plans, but be prepared to analyze things logically and without emotion. Then you can deal with whatever comes your way. Interviewer And I suppose…

Part 2:

M

QU Y

NH

ƠN

OF

Scientists have made incredible advances in recent years. Yet there's still much they don't understand about the inner workings of biology. Even after mapping the human genome, for example an international effort that took more than a decade to complete scientists still don't know why some people with a particular variation in their DNA develop cancer and others with the same variation don't. One reason for the limited understanding is that the tools scientists have been using to study biological systems like cells or tissues are themselves limited. Tools that offer the ability to closely examine biological samples at high resolution don't allow scientists to see the bigger picture. While tools that show the bigger picture lack the resolution necessary to reveal the true biology of a sample. Neither approach is ideal, and the work of piecing together the results is incremental and slow. What a scientist really needs is a clear, full view of a biological system from the start. And that's precisely what Ten x genomics offers new perspectives on the inner workings of biology that can quickly lead to new insights. Our technology suite enables scientists to study biological systems at an unprecedented level of resolution. With our instrument and kits, a scientist can separate and encapsulate tens of thousands of cells from a biological sample into individual droplets. Next, using a unique chemical process and proprietary tagging method It, the scientists can capture DNA, RNA, protein and immunological information from individual cells. A sequencer then decodes the information. It. Finally, our software uses this decoded information to build an interactive reconstruction of the biological sample for interpretation. These combined innovations represent a dramatic shift in the scale and resolution of information that can be derived from the sequencing process, and they advance our mission to accelerate research in critical areas of biology and fuel medical breakthroughs. A in fact, Ten X Genomics has already helped scientists uncover important biological processes and identify previously unrecognized cell types. These discoveries, and others sure to follow will lead to better technologies, better treatments, and a better future.

Part 3:

DẠ

Y

Interviewer Today we’re discussing language, and the role it plays in society. I’m joined by Professor Gordon Mackie and Dr Sophie Blackmore, from Belmont College. Gordon, communication is at the heart of your latest book, isn’t it? Speaker 1 Exactly so. You only need to watch news reports or read newspaper articles from just a few years ago to see that communication has changed dramatically in the last few decades. And I’ve obviously devoted a lot of the book to digital communication. We simply cannot ignore how it’s transformed the way we express ourselves, and even the slang we use. But the point isn’t that these changes are having an adverse impact on language. Not at all. My mission is to show how that view’s a myth because, if anything, our language has never been richer.

Page 8/10

M

QU Y

NH

ƠN

OF

FI CI A

L

Speaker 2 It sounds fascinating! So often in our academic work we deal with the obscure and highly theoretical aspects of linguistics rather than highlight the beauty of everyday communication. Speaker 1 Thanks, Sophie. Yes, and what you say about the nature of academic research is particularly true in my field of expertise, historical linguistics. I spend my time exploring how the so-called rules of grammar and spelling have evolved, and the origins of certain taboo expressions. But it’s important that I never lose sight of my primary task, which is to gather evidence of the type of language being used at particular times. My role isn’t to suggest how language should be, it’s to describe how it is, or in my case, how it once was. I preserve documents, not language usage! Speaker 2 My role is somewhat different. I do discuss the features that make up effective communication because I teach essay-writing classes. Many university students have a tendency to confuse adopting an appropriate tone with introducing an unnecessarily complex style. Now clearly, university essays should fulfil the conventions of academic writing, and be error-free. However, those things are important only insofar as they help you achieve the key goal, which is to be understood. You may have persuasive arguments, clear evidence, maybe even ground-breaking theories, and that’s great. But unless they’re expressed in way that is accessible to the reader, your writing cannot be deemed a success. Speaker 1 And you also research pragmatics, don’t you? You’ve published a lot on the nature of social interaction. Speaker 2 That’s right. Writers choose which information to present, and whether to convey it in an emotional or neutral way. They decide how best to engage the reader. Then, the reader analyses that information and compares it with their existing knowledge. I highlight that point in my classes using the example of social media. When we post something to our network, our followers are already aware of the background. Our intended audience will immediately see the point we’re making without us joining all the dots for them. But a stranger might see the same post and interpret it in a completely different way. Speaker 1 Then there’s text language, which has its own particular style and structure. I know it’s frowned upon by schools currently, but it’ll eventually be studied in the same way that pupils study any other form of writing. After all, it’s very inventive. And it’s remarkable how universal standards are being adopted within text language. For instance, young people from different cultures can use instant messaging and text language to understand one another without language barriers. Speaker 2 Yes, it’s evolved very quickly. But do you really envisage it appearing on a school syllabus? I can’t see any prospect of that. And that’s a shame because of course, text language is increasingly accepted by more of the population. Interviewer And do you…

Part 4:

DẠ

Y

Plants require several things to grow. Most of us know that they need things like water, sunlight, and CO2, but plants also need a variety of nutrients as well, things like nitrogen and phosphorus. Usually, plants will get all their required nutrients from the soil through their roots, but when the soil is bad or there's been a lot of erosion or leaching, farmers, or even just people trying to make their lawns green, will put down something called fertilizer. What makes fertilizer so fertile is that it's been enriched with these nutrients that the plants need, again mostly fixed nitrogen and phosphorus. But plants aren't the best at soaking up every last nutrient in the soil, and it's also hard to gauge just how much fertilizer a field needs as soil quality can vary drastically over short distances. To be safe, farmers will usually

Page 9/10

THE END

DẠ

Y

M

QU Y

NH

ƠN

OF

FI CI A

L

apply excess fertilizer to a given plot of land, better safe than sorry, you know? But instead of staying in the soil for years, most of the excess nutrients will be carried away by the rain or other forms of irrigation. These nutrients mix with the water and find their way into bodies of water, like ponds, lakes, reservoirs, and even the ocean sometimes. With all these nutrients added, the algae, phytoplankton, and even plants in the water do the same thing the crops in the field do. They grow. Well actually, they explode in numbers. This is called an algal bloom. And entire lakes can become covered in layers of plant growth like this. To sum, this might seem like a good thing. Plants are good for the environment, right? Well, not always. First off, this floating layer of algae forms basically an impenetrable roof on the water, not allowing sunlight through to the bottom of the lake. Without the presence of sunlight, all plants below the surface cannot partake in photosynthesis. You know, metabolize, make glucose, live, that sort of stuff? But not even this is the bad part of it all. Many plants can store enough energy in their bodies to weight out these conditions. The real problem comes when all the nutrients are used up and the water can no longer support so much life. When this happens, the excess algae, phytoplankton, and plants die off and sink to the bottom of the body of water. Here, bacteria and other decomposers feast on the dead bodies in a chemical process of decay which consumes oxygen. Now, in a usual ecosystem, the amount of dead matter is relatively constant, so oxygen levels stay relatively constant as well. But when a bloom occurs, far more organic matter is ready to decompose, and so nearly all the oxygen in the water is used in the process of decomposition, and none is left for the animals living in the water. Without this, animals that use the dissolved oxygen to breathe, so things like fish, can actually suffocate. This causes even more death, leading to more decomposition and more oxygen usage. Basically, at this point, a positive feedback loop has been created. It can take a body of water a very long time to recover, though each one is different, and recovery depends on a lot of things, like how many nutrients leaked into the water, how big the body of water is, what organisms are present there, and so on. When this happens in lakes, native species can be suppressed and allow invasives to come in while the environment is still disturbed. If this happens in the ocean, the lack of oxygen can cause corals to bleach and possibly even die. All around, this can greatly damage many ecosystems and leads to a decrease in biodiversity globally. Nutrient -⁠rich runoff can also be caused by things like clear cutting, which releases the nutrients which were kept in the soil by the plants, or also by things like animal farms where nutrient -⁠rich waste materials can leak into local bodies of water. And that's eutrophication, simply put. I hope you enjoyed the video and maybe think twice before using fertilizer or clearing a forest. If you want more people to see this video, give it a like, and if you want to see more.

Page 10/10

ĐỀ XUẤT ĐỀ THI HỌC SINH GIỎI

TRƯỜNG THPT CHUYÊN

KHU VỰC DUYÊN HẢI – ĐBBB NĂM 2023

-------------------------

Môn: Tiếng Anh – Lớp 10

FI CI A

L

SỞ GD & ĐT CAO BẰNG

NH

ƠN

OF

A. LISTENING (50 points) Part 1: Complete the table below. Write NO MORE THAN THREE WORDS for each answer. Write your answers in the corresponding numbered boxes. (10 points) Date Event Importance for art 3000 BC rice farmers from 1. ________ built temples with wood and stone carvings settled in Bali 14th century introduction of Hinduism artists employed by 2. ________ and focused on epic narratives 1906 Dutch East Indies Company art became expression of opposition to established 3. ________ 1920s beginning of 4. ________ encouraged use of new materials, techniques and subjects 1945 independence new art with scene of 5. _________ (e.g. harvests) reflecting national identity Your answers: 1. 2. 3. 4. 5.

DẠ

Y

M

QU Y

Part 2. You will hear part of a discussion between Velm and Andrews, a lawyer, and Sergeant William Bailey, a police officer. For questions 1-5, choose the answer (A, B, C, or D) which fits best according to what you hear. (10 points) 1. How did William feel the first time he gave evidence in court? A. humiliated B. nervous C. furious D. indifferent 2. Velm suggests that police officers giving evidence should ………. A. study the evidence more carefully. B. ignore the lawyer for the defense. C. not take comments personally. D. demonstrate that they are honest and reliable. 3. Velma compares a police officer's evidence to a piece in a jigsaw puzzle because ………. A. it is unimportant unless it is part of a bigger picture. B. it may not fit in with the rest of the evidence. C. the defense lawyer will try to destroy it. D the police officer should only talk about his or her evidence. 4. William suggests that lawyers …………. A adopt a special manner in the courtroom. B. can be detached from a case. C. might actually be close friends. D. do not take their work seriously.

1

FI CI A

L

5. William's main concern is that A. a criminal could get away with his or her crime. B. a court case could be confusing, C. young police officers find courts terrifying. D. police officers might argue with the lawyer.

NH

ƠN

OF

Your answers: 1. 2. 3. 4. 5. Part 3: You will hear two students, Bella and Tom, discussing an article they have read about a woman astronaut. For questions 1-5, decide whether the following statements are True (T) or False (F). (10 points) 1. The speakers agree that being an astronaut is an unexpected job for women. 2. Bella particularly admires the astronaut Ellen Ochoa because of her determination to fulfill a childhood dream. 3. Tom was surprised to learn that people who want to become astronauts should have experience as airplane pilots. 4. Bella thinks the most interesting part of Ellen’s life is coping with unexpected problems. 5. Tom and Bella both now decide to go to some talks on space travel in films and literature. Your answers: 1. 2. 3. 4. 5.

DẠ

Y

M

QU Y

Part 4: For questions 1-10, listen to a piece of news from BBC about “What to wear?” and supply the blanks with the missing information. Write NO MORE THAN THREE WORDS and/or A NUMBER taken from the recording for each answer in the spaces provided. (20 points) One of the biggest shocks, when you arrive in a new country, can be the clothes people are wearing. You may look fashionable at home, but you suddenly find you are 1____________________or simply someone to laugh at when you arrive abroad. With this in mind, let's take a look at teenage fashion in the UK for girls. One of the things that may shock an outsider most is piercings. These days it is not enough to simply wear rings in your ears. You will see many teenagers with rings on their navel, belly button, nose, lip, or even their eyebrows. Some girls go for a 'glam' look. They wear T-shirts; trousers are usually preferred, blue or black, and the look is 2_____________________ with metallic bags and shoes and arms full of bracelets. Another alternative is the 3________________________ You start with a T-shirt of your favorite band and tight jeans or a long skirt. On top of this, you can wear a denim jacket. Jewellery tends to be large and metallic and to add color, wear a scarf. If neither of these is for you, why not 4_________________________? T-shirts are usually tie-dyed in hot colors. Wear long shorts, short jeans or a denim skirt. And on your feet? 5___________________________, of course! If you prefer something more feminine, there's the 6___________________________ Skirts are long, to the floor. Wear a top with butterflies or flowers printed on it!

2

2.

3.

4.

6.

7.

8.

9.

5.

OF

1.

FI CI A

L

Finally, how about the 'Tom Boy' look? Wear 7________________________ and a T-shirt with a logo. Don't forget your 8_______________________ of course! Follow the fashion tips above, and you shouldn't 9______________________However, it's important to remember to wear clothes and choose a look that you feel comfortable with. Don't just be 10_______________- be yourself! Your answers:

10.

ƠN

B. LEXICO-GRAMMAR (40 points) Part 1: For questions 1 - 20, choose the correct answer (A, B, C, or D) to each of the following questions and write your answers in the corresponding numbered boxes. (20 points)

DẠ

Y

M

QU Y

NH

1. In ____________, with the benefits of hindsight, it is clear that this was a bad decision. A. reflection B. retrospect C. status quo D. a second thought 2. Little did I imagine The Amazing Race would entail a long-winded journey and ups and downs ____. A. aplenty B. inexhaustibly C. profusely D. superabundant 3. None of his relatives really understood the full ____________ of his difficulty or his desperation. A. degree B. dimension C. extent D. width 4. It’s quite unreasonable to ask me to organize the conference at such short ____________. A. warning B. notice C. time D. advance 5. I slept badly last night and am feeling particularly ____________ this morning. A. slow-witted B. far-reaching C. off-hand D. top-heavy 6. A few tears were _____________ by both parents when Maria finally left home. A. flowed B. poured C. shed D. streamed 7. John did nine hours’ ____ studying a day for his exam. A. strong B. solid C. heavy D. big 8. I suppose I____ lent the book to Simon, but I’m almost sure I didn’t. A. might B. could C. might have D. must have 9. People were injured by ____ stones the size of tennis balls. A. sleet B. gale C. hail D. snow 10. The police ____ off the street when the bomb had gone off. A. cordoned B. battened C. fastened D. shuttered

3

QU Y

NH

ƠN

OF

FI CI A

L

11. Roger Williams was a clergyman, ______ the colony of Rhode Island, and an outspoken advocate of religious and political freedom. A. founded B. the founder of C. was the founder of D. the founded 12. ______ at home requires only three types of chemicals, several pieces of simple equipment, and running water. A. For the development of film B. To develop film C. When film is developed D. In developing film 13. The bark of a tree thickens ______. A. with age B. it gets older C. as older D. by age 14. Widely reproduced in magazines and books, ______. A. Ansel Adams depicted the Western wilderness in his photographs. B. the Western wilderness was depicted in the photographs of Ansel Adams. C. Ansel Adams’ photographs depicted the Western wilderness. D. it was through his photographs that Ansel Adams depicted the Western wilderness. 15. This new glue is very useful for small repairs as it ______ very rapidly. A. thickens B. stiffens C. sets D. fixes 16. Advertisers often aim their campaigns at young people as they have considerable spending ______. A. power B. force C. energy D. ability 17. We’ve bought some ______ chairs for the garden so that they are easy to store away. A. adapting B. adjusting C. bending D. folding 18. Alan’s photo was slightly too large for the frame so he decided to ______ it. A. hack B. chop C. slice D. trim 19. Although the patient received intensive treatment, there was no ______ improvement in her condition. A. decipherable B. legible C. discernible D. intelligible 20. The painting was a valuable family possession, which had been ___________ from generation to generation.

Your answers:

C. handed across

D. handed out

2.

3.

4.

5.

7.

8.

9.

10.

11.

1.

B. handed down

M

A. handed over

12.

13.

14.

15.

16.

17.

18.

19.

20.

DẠ

Y

6.

Part 2: Write the correct form of each bracketed word in corresponding numbered boxes. (10 points) 4

FI CI A

L

1. A UN force has been sent in to try and ___________ the area worst affected by the civil war. (PEACE) 2. During puberty, my children showed increasing signs of mental ________, which raised lots of concern for their parents. (STABLE) 3. In fact, those who practice meditation with any________ see their doctors less and spend, on average, seventy percent fewer days in the hospital. (REGULAR) 4. There’s been yet another____________ of cholera in Delhi.

(BREAK)

(EXPRESS)

OF

5. Sarah stood there completely ________, so I had no idea at all what she was thinking. 6. The upper layers of Earth’s oceans have cooled ________ over the past two years, even though the planet as a whole is warming up. (SIGNIFY)

1.

2.

6.

7.

QU Y

NH

ƠN

7. This soft drink has a very fruity flavor to it and a_____after-taste. I really like it. (PLEASE) 8. ________ as it may seem, mammoths were alive only five thousand years ago. (CREDIT) 9. The judge was forced to acquit, saying the evidence was wholly_________. (CONCLUDE) 10. Can you think of one______ reason why I should give you your job back? (SOLITUDE) Your answers: 3.

4.

5.

8.

9.

10.

Part 3: Fill in the blank with a suitable preposition or a particle to complete the following sentences. (10 points)

DẠ

Y

M

1. Text messaging caught_____ because kids wanted to pass notes to each other during class. 2. It is true that she plans to give _____ her prize money to charity. 3. If you act _____impulse, you risk making mistakes. 4. Don’t let one spoilt meal put you _____ cooking forever! 5. I got the job _______ the strength of your recommendation. 6. You must be weak _______ the head if you believe that. 7. Prospects of success in the talks were put _______ zero. 8. His appearance was the subject _______ some critical comments. 9. She went _______ the roof when I told her I’d crashed her car. 10. The government’s decision is a real kick _______ the teeth for the unions. Your answers: 5

4. 9.

5. 10.

L

3. 8.

FI CI A

1. 2. 6. 7. C. READING (60 points)

Part 1: Read the following passage and mark the letter A, B, C or D on your answer sheet to indicate the best answer to each of the following questions. (15 points)

In the past, not a very long time ago, most people traveled on foot, by train, or on horseback. (1) _____ had made it possible to travel rapidly over long distances. Bicycles

OF

were also becoming (2) _____, after the invention of the air- filled (3) _____, which made cycling a lot more comfortable. Buses, trams, and underground railways had already been invented, and cities all over the world already had traffic (4) _____. There were very few

ƠN

private cars, and city streets were still full of horses. What a difference a hundred years have (5) _____! Nowadays we have got (6) _____to the problem of private cars, and some cities are so noisy and (7) _____ that in many places (8) _____ have been banned from the

NH

city centre. How will we be travelling in a hundred year’s time? Perhaps by then there will be only personal helicopters. There may be no need to (9) _____ to work or school in the future, (10) _____everyone will have a computer at home. There might even be more

QU Y

people walking and horse – riding, for pleasure and exercise. B. Railways

C. Ways

D. Lines

B. invented

C. popular

D. handlebars

B. brake

C. engine

D. tyre

B. sticks

C. knots

D. jams

Question 5. A. taken

B. done

C. made

D. got

Question 6. A. more

B. them

C. motorists

D. used

Question 7. A. even

B. so

C. polluted

D. poisoned

Question 8. A. traffic

B. vehicles

C. transport

D. trips

Question 9. A. have

B. transport

C. decide

D. commute

B. since

C. when

D. unless

Question 1. A. Tracks Question 2. A. then Question 3. A. boot

Y

M

Question 4. A. blocks

DẠ

Question 10. A. if Your answers: 1. 2. 6. 7.

3. 8.

4. 9.

5. 10.

6

L

Part 2: Fill in the blank with one suitable word. (15 points)

QU Y

NH

ƠN

OF

FI CI A

Of course, we all think our children are the best and brightest in the class, but have you ever seriously thought that your child might be highly intelligent? Does he or she (1) ________ out in class as being far ahead of his or her classmates? How can we be sure? Child psychiatrists and educationists have compiled a profile of (2) ________ gifted children so that parents can have the opportunity to see for themselves which characteristics are true for their own offspring and then take the appropriate (3) ________. Have a look at the profile below (4) ________ we hope will help you to form a truer opinion of your own child's intelligence. Firstly, these children have an unusually wide vocabulary, enjoy reading and show (5) ________ about the world and what makes things work. Secondly, they are perfectionists, have the ability to work (6) _________, and are interested in adult topics such as religion, war, politics, and so on. Furthermore, highly gifted children are sensitive, creative, and adventurous. They have a good sense of humor and usually have the ability to look (7) ________ the bright side of a difficult situation. They often show a desire to improve themselves and they dislike conforming just for the (8) ________ of it. Finally, they are self-confident - especially in the company of adults - adaptable and responsible. All of the above characteristics (9) ________ to making up what is seen as a highly gifted child. If you feel that your child shows this particular combination, then it is advisable to get in (10) __________ with one of the organizations that exist to give guidance to parents where they will no longer feel "different " but, possibly for the first time, on equal footing with their peers. Your answers: 1. 2. 3. 4. 5. 6. 7. 8. 9. 10. Part 3: Read the text and choose the answer (A, B, C or D) which you think fits best

M

according to the text. (15 points)

The attraction of valuable objects from ships sinking in the oceans is always great.

Until recently, hunting for treasure from shipwrecks was mostly fantasy; with recent technological advances, however, the search for sunken treasure has become more popular as a legitimate endeavor. This has caused a debate between those wanting to salvage the

Y

wrecks and those wanting to preserve them.

DẠ

Treasure hunters are spurred on by the thought of finding caches of gold coins or

other valuable objects on a sunken ship. One team of salvagers, for instance, searched the

7

L

wreck of the RMS Republic, which sank outside the Boston harbor in 1900. The search

FI CI A

party, using side-scan sonar, a device that projects sound waves across the ocean bottom and produces a profile of the sea floor, located the wreck in just two and a half days. Before

the use of this new technology, such searches could take months or years. The team of 45 divers searched the wreck for two months, finding silver tea services, crystal dinnerware, and thousands of bottles of wine, but they did not find the five and a half tons of American

OF

Gold Eagle coins they were searching for.

Preservationists focus on the historic value of a ship. They say that even if a shipwreck’s treasure does not have a high monetary value, it can be an invaluable source of historic artifacts that are preserved in nearly mint condition. But once a salvage team

ƠN

has scoured a site, much of the archaeological value is lost. Maritime archaeologists who are preservationists worry that the success of salvagers will attract more treasure-hunting

NH

expeditions and thus threaten remaining undiscovered wrecks. Preservationists are lobbying their state lawmakers to legally restrict underwater searches and unregulated salvages. To counter their efforts, treasure hunters argue that without the lure of gold and million-dollar treasures, the wrecks and their historical artifacts would never be recovered

QU Y

at all.

Question 1: What is the main idea of this passage? A. The popularity of treasure-seeking has spurred a debate between preservationists and salvagers.

M

B. Maritime archaeologists are concerned about the unregulated searching of wrecks. C. The search of the RMS Republic failed to produce the hoped-for coins.

D. Searching for wrecks is much easier with new technologies like side-scan sonar. Question 2: The word “sunken” is closest in meaning to which of the following words? A. underwater

B. broken

C. ancient

D. hollow

Y

Question 3: Which of the following could best replace the phrase “a profile” in the

DẠ

passage?

A. a projection

B. an execution

C. an outline

D. a highlight

Question 4: Which of the following statements is best supported by the author? 8

L

A. The value of a shipwreck depends on the quantity of its artifacts. C. Side-scan sonar has helped to legitimize salvaging. D. The use of sound waves is crucial to locating shipwrecks.

FI CI A

B. Preservationists are fighting the use of technological advances such as side-scan sonar.

Question 5: The author uses the phrase “mint condition” to describe _____ . B. something significant

C. something tolerant

D. something magical

OF

A. something perfect

Question 6: All of the following were found on the RMS Republic EXCEPT _____ . A. wine bottles

B. silver tea services

C. American Gold Eagle coins

D. crystal dinnerware

ƠN

Question 7: From the passage, you can infer that a preservationist would be most likely to ______ . C. put treasures in a museum

B. be a diver

NH

A. shun treasure-seeking salvagers

D. do archaeological research

Question 8: The word “scoured” is most similar to which of the following? A. scraped away

B. scratched over

C. scrambled around

D. searched around

A. knowledge

QU Y

Question 9: What is the closest meaning to the word “lure” in the passage? B. attraction

C. luxury

D. glare

Question 10: The second and third paragraphs are an example of _____ . A. chronological order

1. 6.

D. definition

2.

3.

4.

5.

7.

8.

9.

10.

Your answers:

M

C. specific to general

B. explanation

DẠ

Y

Part 4: Read the passage and do the tasks that follow. (15 points) The evolutional mystery: Crocodile survives A

9

DẠ

Y

M

QU Y

NH

ƠN

OF

FI CI A

L

Crocodiles have been around for 200 million years, but they’re certainly not primitive. The early forms of crocodiles are known as Crocodilian. Since they spent most of their life beneath the water, accordingly their body adapted to the aquatic lifestyle. Due to the changes formed within their body shape and tendency to adapt according to the climate, they were able to survive when most of the reptiles of their period are just a part of history. In their tenure on Earth, they’ve endured the impacts of meteors, planetary refrigeration, extreme upheavals of the Earth’s tectonic surface, and profound climate change. They were around for the rise and fall of the dinosaurs, and even 65 million years of supposed mammalian dominance has failed to loosen their grip on the environments they inhabit. Today’s crocodiles and alligators are little changed from their prehistoric ancestors, a telling clue that these reptiles were (and remain) extremely well adapted to their environment. B The first crocodile-like ancestors appeared about 230 million years ago, with many of the features that make crocs such successful stealth hunters already in place: streamlined bodies, long tails, protective armor, and long jaws. They have a long head and a long tail that helps them to change their direction in the water while moving. They have four legs which are short and are webbed. Never underestimate their ability to move on the ground. When they move they can move at such a speed that won’t give you a second chance to make a mistake by going close to them especially when hungry. They can lift their whole body within seconds from ground. The fastest way by which most species can move is a sort of “belly run”, where the body moves like a snake, members huddled to the side paddling away frenetically while the tail whips back and forth. When “belly running” Crocodiles can reach speeds up to 10 or 11 km/h (about 7mph), and often faster if they are sliding down muddy banks. Other form of movement is their “high walk”, where the body is elevated above the ground. C Crocodilians have no lips. When submerged in their classic ‘sit and wait’ position, their mouths filled with water. The nostrils on the tip of the elongated snout lead into canals that run through bone to open behind the valve – allowing the crocodilian to breathe through its nostrils even though its mouth is underwater. When the animal is totally submerged, another valve seals the nostrils, so the crocodilian can open its mouth to catch prey with no fear of drowning. The thin skin on the crocodilian head and face is covered with tiny, pigmented domes, forming a network of neural pressure receptors that can detect barely perceptible vibrations in the water. This enables a crocodile lying in silent darkness to suddenly throw its head sideways and grasp with deadly accuracy small prey moving close by. D Like other reptiles, crocodiles are endothermic animals (cold-blooded, or whose body temperature varies with the temperature of the surrounding environment) and, therefore, need to sunbathe, to raise the temperature of the body. On the contrary, if it is too hot, they prefer being in water or in the shade. Being a cold-blooded species, the crocodilian heart is unique in having an actively controlled valve that can redirect, at will, blood flow away 10

DẠ

Y

M

QU Y

NH

ƠN

OF

FI CI A

L

from the lungs and recirculate it around the body, taking oxygen to where it’s needed most. In addition, their metabolism is a very slow one, so, they can survive for long periods without feeding. Crocodiles are capable of slowing their metabolism even further allowing them to survive for a full year without feeding. Compared to mammals and birds, crocodilians have slow metabolisms that burn much less fuel, and are ideally suited to relatively unstable environments that would defeat mammals with their high food demands. E Crocodiles use a very effective technique to catch the prey. The prey remains almost unaware of the fact that there can be any crocodile beneath water. It is due to the fact that when the crocodile sees its prey it moves under water without making any noise and significant movement. It keeps only its eyes above water surface. When it feels it has reached sufficiently close to the target it whistles out of water with wide open jaws. 80 percent of their attempts are successful. They have very powerful jaws. Once the prey trapped in its jaws they swallow it. Their power can be judged from the fact they can kill the wild zebras which come to watery areas in search of water. They do not chew their food. They normally feed on small animals, big fish, birds and even human flesh. As like some water creatures that interact by making sounds crocodiles also use many sounds to communicate with other crocodiles. They exist where conditions have remained the same and they are free of human interference. The crocodile is successful because it switches its feeding methods. It hunts fish, grabs birds at the surface, hides among the water edge vegetation to wait for a gazelle to come by, and when there is a chance for an ambush, the crocodile lunges forward, knocks the animal with its powerful tail and then drags it to water where it quickly drowns. Another way is to wait motionless for an animal to come to the water’s edge and grabs it by its nose where it is held to drown. F In many places inhabited by crocodilians, the hot season brings drought that dries up their hunting grounds and takes away the means to regulate their body temperature. They allowed reptiles to dominate the terrestrial environment. Furthermore, many crocs protect themselves from this by digging burrows and entombing themselves in mud, waiting for months without access to food or water, until the rains arrive. To do this, they sink into a quiescent state called aestivation. G Most of (At least nine species of) crocodilians are thought to aestivate during dry periods. Kennett and Christian’s six-year study of Australian freshwater crocodiles – Crocodylus Johnstoni (the King Crocodiles). The crocodiles spent almost four months a year underground without access to water. Doubly labeled water was used to measure field metabolic rates and water flux, and plasma (and cloacal fluid samples were taken at approximately monthly intervals during some years to monitor the effects of aestivation with respect to the accumulation of nitrogenous wastes and electrolyte concentrations. Double found that the crocodiles’ metabolic engines tick over, producing waste and using up water and fat reserves. Waste products are stored in the urine, which gets increasingly concentrated as the months pass. However, the concentration of waste products in the blood changes very little, allowing the crocodiles to function normally. Furthermore, though the 11

FI CI A

L

animals lost water and body mass (just over one-tenth of their initial mass) while underground, the losses were proportional: on emergence, the aestivating crocodiles were not dehydrated and exhibited no other detrimental effects such as a decreased growth rate. Kennett and Christian believe this ability of individuals to sit out the bad times and endure long periods of enforced starvation must surely be key to the survival of the crocodilian line through time.

QU Y

NH

ƠN

OF

Questions 1-5 The reading passage has seven paragraphs, A-G. Choose the correct heading for paragraph A-G from the list below. Write the correct number, i-x, in boxes 1 – 5 on your answer sheet. List of Headings i The competitors with the dinosaur ii A historical event for the Supreme survivors iii What makes the crocodile the fastest-running animal on land iv Regulated body temperature by the surrounding environment v Underwater aid in body structure offered to a successful predator vi The perfectly designed body for a great land roamer vii Slow metabolisms which make crocodiles unique reptiles viii The favorable features of the impact of a drought xi Shifting Eating habits and food intake x A unique finding has been achieved recently Example: Paragraph A – ii 1. Paragraph B _____ 2. Paragraph C _____ 3. Paragraph D _____ 4. Paragraph E _____ 5. Paragraph F _____

DẠ

Y

M

Questions 6 - 10 Complete the summary and write the correct answer (NO MORE THAN TWO WORDS OR A NUMBER) in boxes 6 – 10 on your answer sheet. In many places inhabited by crocodilians, most types of crocodile have evolved a successful scheme to survive in the drought brought by a (6)……………………… According to Kennett and Christian’s six-year study of Australian freshwater crocodiles’ aestivation, they found Aestivating crocodiles spent around (7)……………………… a year without access to (8) ……………………… The absolute size of body water pools declined proportionately with (9) ………………………; thus there is no sign of (10) ……………………… and other health-damaging impacts in the crocodiles even after an aestivation period. This super capacity helps crocodiles endure the tough drought without slowing their speed of growth significantly. Your answers: 12

3. 8.

4. 9.

5. 10.

L

2. 7.

FI CI A

1. 6.

D. WRITING (50 points)

NH

ƠN

OF

Part 1: The table below shows the proportion of different categories of families living in poverty in Australia in 1999. (20 points)

QU Y

………………………………………………………………………………………….… …………………………………………………………………………………………… …….……………………………………………………………………………………… …………….……………………………………………………………………………… …………………….………………………………………………………………………

M

…………………………….………………………………………………………………

…………………………………….……………………………………………………… …………………………………………….……………………………………………… …………………………………………………….………………………………………

Y

…………………………………………………………….………………………………

DẠ

…………………………………………………………………….……………………… …………………………………………………………………………….……………… …………………………………………………………………………………….………

13

L

…………………………………………………………………………………………….

FI CI A

……………………………………………………………………………………………

……….…………………………………………………………………………………… ……………….…………………………………………………………………………… ……………………….……………………………………………………………………

……………………………….……………………………………………………………

OF

……………………………………….……………………………………………………

……………………………………………….…………………………………………… ……………………………………………………….…………………………………… ……………………………………………………………….……………………………

ƠN

Part 2. Essay writing (30 points)

NH

Chat GPT is an advanced natural language processing tool developed by OpenAI. It has gained a lot of popularity since its release in November 2022. Advantageous as it may be, this tool poses lots of threats to our lives in many aspects. Write a 250-word essay to discuss both pros and cons of Chat GPT.

DẠ

Y

M

QU Y

………………….………………………………………………………………………… ………………………….………………………………………………………………… ………………………………….………………………………………………………… ………………………………………….………………………………………………… ………………………………………………….………………………………………… ………………………………………………………….………………………………… ………………………………………………………………….………………………… ………………………………………………………………………….………………… ………………………………………………………………………………….………… ………………………………………………………………………………………….… ……………………………………………………………………….…………………… ……………………………………………………………………………….…………… ……………………………………………………………………………………….…… ……………………………………………………………………………………………… .…………………………………………………………………………………………… ……….…………………………………………………………………………………… ……………….…………………………………………………………………………… ……………………….…………………………………………………………………… ……………………………….…………………………………………………………… ……………………………………….…………………………………………………… ……………………………………………….……………………………………………

14

DẠ

Y

M

QU Y

NH

ƠN

OF

FI CI A

L

…………………………….……………………………………………………………… …………………………………….……………………………………………………… …………………………………………….……………………………………………… …………………………………………………….……………………………………… …………………………………………………………….……………………………… …………………………………………………………………….……………………… …………………………………………………………………………….……………… …………………………………………………………………………………….……… ……………………………………………………………………………………………. ……………………………………………………………………………………………… …….………………………………………………………………………….…………… ……………………………………………………………………………………….…… ……………………………………………………………………………………………… .…………………………………………………………………………………………… ……….…………………………………………………………………………………… ……………….…………………………………………………………………………… ……………………….…………………………………………………………………… ……………………………….…………………………………………………………… ……………………………………….…………………………………………………… ……………………………………………….…………………………………………… ……………………………………………………….…………………………………… …………………………………….……………………………………………………… …………………………………………….……………………………………………… …………………………………………………….……………………………………… …………………………………………………………….……………………………… …………………………………………………………………….……………………… …………………………………………………………………………….……………… …………………………………………………………………………………….……… ……………………………………………………………………………………………. ……………………………………………………………………………………………… …….……………………………………………………………………………………… …………….………………………………………………………………………….…… ……………………………………………………………………………………………… .…………………………………………………………………………………………… ……….…………………………………………………………………………………… ……………….…………………………………………………………………………… ……………………….…………………………………………………………………… ……………………………….…………………………………………………………… ……………………………………….…………………………………………………… ……………………………………………….…………………………………………… ……………………………………………………….…………………………………… ……………………………………………………………….…………… The end

15

16

Y

DẠ M

KÈ QU Y ƠN

NH

FI CI A

OF

L

HƯỚNG DẤN CHẤM

TRƯỜNG THPT CHUYÊN

ĐỀ XUẤT ĐỀ THI HỌC SINH GIỎI

-------------------------

KHU VỰC DUYÊN HẢI – ĐBBB NĂM 2023

FI CI A

L

SỞ GD & ĐT CAO BẰNG

Môn: Tiếng Anh – Lớp 10

OF

A. LISTENING (50 points) Part 1. Complete the table below. Write NO MORE THAN THREE WORDS for each answer. Write your answers in the corresponding numbered boxes. (10 points) 1. China 2. (the) ruling families 3. colonisation 4. tourism 5. everyday life

DẠ

Y

M

QU Y

NH

ƠN

TAPESCRIPTS Part 1: Last week we looked at the traditional art of Japan. In this week’s lecture, we’re going to move south and look at the very special way in which art has developed in the beautiful island of Bali, which is now part of Indonesia. I’ll begin by giving you a brief historical overview. It’s thought that the first inhabitants of Bali were farmers who arrived around 3000 BC … at the beginning of the Iron Age. They probably originally came from China, and in Bali they cultivated rice and built temples ornamented with wood and stone carvings and statues. The Hindu religion was introduced in the 14th century AD, and this has remained the main religion on the island. This was an important period in the artistic development of the island, when sculptors, poets, priests and painters worked together in the service of the ruling families. Rather than painting everyday scenes, artists concentrated on narrative paintings illustrating the epic stories of Hinduism. Bali’s rich natural resources have always made it an alluring goal for merchants, and from the 17th century onwards, Dutch ships visited the island to trade in spices and luxury goods. Gradually the old royal families lost their power, and eventually in 1906 the Dutch East Indies Company was founded and the island became a colony. In the 20th century, art then took on a very different role: as a tool accessible to everyone in the fight of the Balinese people against colonisation rather than as the property of a minority. Shortly after this, in the 1920s, stories of the beauty of the island of Bali began to spread around the world, and Balinese art underwent another vast transformation with advent of tourism to the island. At first, this was only on a small scale, but it had important effects. Expatriate artists from Holland and Germany settled on the island bringing paper, Chinese ink and other new materials with them. They worked with local artists, encouraging them to experiment with concepts like naturalism, expressionism, light and perspective, as well as to move away from the tradition focus on narrative painting towards something closer to their own experience. When independence came in 1945, this desire for an art to match a new national identity became stronger and the traditional narrative paintings started to give way to scenes showing the everyday life of the Balinese people –harvests, market scenes and daily tasks – as well as the myths and legends of their history. Part 2. You will hear part of a discussion between Velm and Andrews, a lawyer, and Sergeant William Bailey, a police officer. For questions 1-5, choose the answer (A, B, C or D) which fits best according to what you hear. ( 10 points) 1

1. A

2. C

3. D

4. B

5. A

DẠ

Y

M

QU Y

NH

ƠN

OF

FI CI A

L

Part 2 (Exam essentials Practice Tests - CAE) You will hear part of a discussion between Velma Andrews, a lawyer, and Sergeant William Bailey, a police officer. For questions 1-5, choose the answer, A, B, C or D which fits best according to what you hear. Interviewer: Today on Legal Issues we have Velma Andrews, a lawyer, and Sergeant William Bailey, a police officer who helps to run a scheme that trains police officers In the art of giving evidence In court. William, perhaps I can start by asking you why this training scheme is necessary. William: Well, you must remember that in a criminal case, the police have gathered evidence to show that someone -the defendant - is guilty of a crime. And the defendant's lawyer is trying to show that this evidence Is wrong or unreliable. Now, the way the defense lawyer goes about doing this can be very tricky. For instance, the first time I gave evidence in court 25 years ago, the lawyer for the defense made me look like a right fool. He annoyed me by interrupting me all the time, and when I tried to argue with him I got confused, and the people in court laughed at me. That made my evidence look bad. I simply had no idea what I was up against. Interviewer: Velma, you are a defense lawyer; do you agree with William? Velma: Absolutely. A police officer has to learn how the system works. You must get used to the idea that the lawyers are just doing a job, and even if It seem s they are attacking your honesty in a rude or brutal manner, they have nothing against you as an individual. Interviewer: It must be hard to think like that when you're giving evidence and some lawyer is trying to trip you up. Velma: It is, but a police officer has to develop the right attitude. You need to think of your evidence as one piece in a jigsaw puzzle, the picture being the whole case against the defendant. If you start giving opinions about other pieces, other parts of the case that aren't your responsibility, it weakens the case as a whole. Your piece of the puzzle is the only thing you should think about! Interviewer: Do you find Velma's advice helpful for police officers on your training scheme, William? William: Definitely. For a young officer, appearing in court is an intimidating experience. It's hard to get used to the system. I mean, there are two lawyers, one acting for the defendant and one for the crown, and in the courtroom they are adversaries but they probably know each other professionally. They may even go off together after the trial and have dinner. As if it were all a game! Interviewer: Would you advise William's trainees to treat a court case as a game, Velma? Velma: I would tell them to remember that the defence lawyer is trying to discredit them and their evidence. One tip to help you develop the right attitude so you don't get drawn into an argument with the lawyer is to stand so you're facing the judge, and direct all your answers to the bench. That should make it easier to avoid any sort of personal exchange with the lawyer. Interviewer: William , is your training scheme having results? William: Yes. I think police officers are more confident in court. And this is not just about making people less nervous! I've seen some pretty terrible things happen in court. You get an inexperienced officer who starts arguing with the lawyer and ends up making the judge and jury think there's something wrong with the police case ” there's a risk that dangerous criminals might be found not guilty and set free. That's the main reason why officers need this training. Interviewer: Velma Andrew s and William Bailey, thank you. 2

2. F

3. F

4. F

5. T

FI CI A

1. T

L

Part 3: You will hear two students, Bella and Tom, discussing an article they have read about a woman astronaut. For questions 1-5, decide whether the following statements are True (T) or False (F). (10 points)

DẠ

Y

M

QU Y

NH

ƠN

OF

Tapescript part 3 Tom: That was a really interesting article the tutor recommended about Ellen Ochoa, wasn't it, Bella? Bella: It certainly was, Tom. I used to dream of going into space when I was a kid. Though I know now I wouldn't be the right kind of person for the job. Tom: But surely they need all sorts of different types of people? Bella: Mm. I'm not sure about that I’d get too panicky if there were problems I think. (1) Anyway,I found if particularly interesting to read about a spacewoman. There aren't too many of those around Tom: You're right there! Bella: Anyway. I'm sure they'd be just as able as men to cope with all the challenges of the job. The article certainly suggests that Ellen was something special, the way she defied all the odds in her determination to do what she’d wanted to do ever since she was small. (2) What I liked best were the sections that quoted her – I thought she was amazingly good at conveying what it was like to go into space. Tom: True. And she's also obviously a very talented scientist Bella: Yes, she did amazing work and I think she sounds as if she must have been a wonderful colleague for the others in her team. Tom: Absolutely, Bella. I also found the article interesting in what it said about the requirements for becoming an astronaut. I knew you'd have to have done loads of flight hours as an ordinary aircraft pilot of course. And l suppose it's not that surprising they want people who are good at sport and who aren't either too small or too tall. Bella: Mm. well, I was surprised, Tom that being shorter than the average was not acceptable I wouldn't have expected that to be an issue. (3) But I never imagined that you needed to have a postgraduate degree I don't think l even realised you had to be a graduate. Tom: Me neither. I never imagined that. Anyway, she does have an interesting life, doesn't she? Bella: Yes, Tom I know. Imagine walking in space and having to work inside and outside the capsule when you're weightless. Tom: Yes, that must be extraordinary. For me I think the most interesting bit would be having to deal with all the little unexpected problems that arise, having to sort things out within your little team. That must be amazing. Bella: Yes, it must. (4) But I was particularly intrigued by her account of the role she sometimes has as one of the people on earth who are in control of the mission. You know when she helps them communicate with other astronauts in space. I thought that sounded really fascinating. Being the one person present on the ground who really understands what life is like for the space crew. She must be able to make things go much more smoothly Anyway, all in all I thought it was a brilliant article. Tom: Yes. it was. Though I thought it was a pity it didn’t tell us as much as it might have done about the less pleasant sides of being in space. I can't believe that it's always straightforward, that all they do is admire the views and carry out lots of ground-breaking scientific research There must be some low points - even if it’s only being irritated by some annoying habit of a fellow crew member or 3

FI CI A

L

getting fed up with the same old food. Bella: Well, we could always try to do a bit more research into that sort of area, if you liked. Tom. I wondered even If Ellen Ochoa’s experiences might make a topic for that science coursework we’ve got to do next term. Or, you know, we could see what we could learn about everyone who’s gone into space from Yuri Gagarin to the present day. Tom: Well, I’m not sure about that. I was quite keen to do something on the funding of space research. Anyway, I suppose we could bear it in mind. (5) But what I do fancy doing is going to a series of lectures I've seen advertised on astronauts and how they’re portrayed in the cinema and in books. Bella: Wow! That sounds brilliant! Do you mind it I come along too? Tom: Of course not, Bella. It'd be good to do it together.

QU Y

NH

ƠN

OF

Part 4. For questions 1-10, listen to a piece of news from BBC about “What to wear?” and supply the blanks with the missing information. Write NO MORE THAN THREE WORDS and/or A NUMBER taken from the recording for each answer in the spaces provided. 20 points 1. Behind the time 2. Finished off 3. Rocker look 4. Go sporty 5. Beach sandals 6. Girly look 7. flared jeans 8. waistcoat 9. feel out of place 10. one of the crowd

7. B

11. B 16. A

8. C

9. C

10. A

12. B

13. A

14. C

15. C

17. D

18. D

19. C

20. B

6. C

M

B. LEXICO-GRAMMAR (40 points) Part 1: For questions 1 - 20, choose the correct answer (A, B, C, or D) to each of the following questions and write your answers in the corresponding numbered boxes. (20 points) 1. B 2. A 3. C 4. B 5. A

Part 2: Write the correct form of each bracketed word in corresponding numbered boxes. (10 points)

DẠ

Y

1. pacify 2. instability 3. regularity 4.outbreak 5. expressionless 6. significantly 7. pleasant 4

L

8. incredible 9. inconclusive 10. solitary Part 3: Fill in the blank with a suitable preposition or a particle to complete the following sentences. 1.on

6. in

2.away

7. at

3. on

8. for

4. off

9. through

5. on

OF

10. in

C. READING (60 points) 2. C

3. D

4. D

5. C

6. D

7. C

ƠN

Part 1: (15 points) 1. B

FI CI A

(10 points)

8. B

9. D

10. B

Part 2: Fill in the blank with one suitable word. (15 points) 2. highly 7. on

3. steps 8. sake

4. which/that 9. contribute

NH

1. stand. 6. Independently

5. curiosity 10. touch

QU Y

Part 3: Read the passage below and choose the best answer to each question. (15 points) 1. A 2. C 3. B 4. D 5. A 6. D 7. C 8. B 9. A 10. A Part 4: Read the passage and do the tasks that follow. (15 points)

M

1. vi 2. v 3. iv 4. ix 5. viii 6. dry season/hot season/ dry period 7. four months 8. water 9. body mass 10. dehydration

Y

D. WRITING

DẠ

Part 1: Chart description. (20 points) Contents (12 points): - The report MUST have at least 2 paragraphs covering the following points: • Introduce the charts (2 points) and state the overall trends & striking features (2 points) 5

FI CI A

L

• Describe the main features with relevant data from the charts and make relevant comparisons (6 points) - The report MUST NOT contain personal opinions. (A penalty of 1 point to 2 points will be given to personal opinions found in the answer.) Language use (8 points) The report: - should demonstrate a wide variety of lexical and grammatical structures, - should have correct use of words (verb tenses, word forms, voice,…); and mechanics (spelling, punctuations,....). Sample:

OF

The table gives information about poverty rates among six types of households in Australia in the year 1999.

ƠN

It is noticeable that levels of poverty were higher for single people than for couples, and people with children were more likely to be poor than those without. Poverty rates were considerably lower among elderly people.

NH

Overall, 11% of Australians, or 1,837,000 people, were living in poverty in 1999. Aged people were the least likely to be poor, with poverty levels of 6% and 4% for single-aged people and aged couples respectively.

QU Y

Just over one-fifth of single parents were living in poverty, whereas only 12% of parents living with a partner were classed as poor. The same pattern can be seen for people with no children: while 19% of single people in this group were living below the poverty line, the figure for couples was much lower, at only 7%. Part 2: Write an essay of about 250 words to express your opinion on the following topic. (30 points)

1. Task achievement: (10 points)

M

a. ALL requirements of the task are sufficiently addressed.

b. Ideas are adequately supported and elaborated with relevant and reliable explanations, examples, evidence, personal experience, etc.

2. Organization: (10 points)

Y

a. Ideas are well organized and presented with coherence, cohesion, and unity.

DẠ

b. The essay is well-structured: • Introduction is presented with clear thesis statement.

6

• Body paragraphs are written with unity, coherence, and cohesion. Each body paragraph must have a topic sentence and supporting details and examples when

L

necessary. recommendation, consideration,…) on the issue. 3. Language use: (5 points) a. Demonstration of a variety of topic-related vocabulary

4. Punctuation, spelling, and handwriting (5 points) a. Correct punctuation and no spelling mistakes b. Legible handwriting

OF

b. Excellent use and control of grammatical structures

FI CI A

• Conclusion summarises the main points and offers personal opinions (prediction,

DẠ

Y

M

QU Y

NH

ƠN

-The end-

7

ĐỀ THI HSG KHU VỰC DH & ĐBBB LẦN THỨ XV – NĂM 2023 MÔN: TIẾNG ANH - KHỐI 11 Thời gian: 180 phút Đề thi gồm: 20 trang

TRƯỜNG THPT CAO BẰNG ĐỀ THI ĐỀ XUẤT

FI CI A

L

HỘI CÁC TRƯỜNG CHUYÊN VÙNG DUYÊN HẢI VÀ ĐỒNG BẰNG BẮC BỘ

PART A. LISTENING (50 pts)

HƯỚNG DẪN PHẦN THI NGHE HIỂU

OF

• Bài nghe gồm 4 phần, mỗi phần được nghe 2 lần, mỗi lần cách nhau 15 giây, mở đầu và kết thúc mỗi phần nghe có tín hiệu.

ƠN

• Mở đầu và kết thúc bài nghe có tín hiệu nhạc. Thí sinh có 3 phút để hoàn chỉnh bài trước tín hiệu nhạc kết thúc bài nghe. • Mọi hướng dẫn cho thí sinh (bằng tiếng Anh) đã có trong bài nghe.

NH

Section 1. You will hear part of a radio interview in which the comedian and writer Jane Clarkson is talking about her. For questions 1-5, choose the answer (A, B, C, or D) which fits best according to what you hear. (10 pts) 1. What did Jane find difficult about writing a book ? A She couldn’t travel around the country. B She didn’t get any instant reaction to her work.

QU Y

C She had to spend time looking after her daughter. D She found the process itself very challenging.

M

2. According to Jane, why did some critics dislike her novel ? A They didn’t think her book was funny. B They were dismissive of her initial success. C They though her male colleagues were better writers. D They thought she should stick to being a comedian 3. Which aspect of Jane’s work as a comedian helped her to write ?

DẠ

Y

A her patience B her ability to learn C her habit of watching people D her rational way of thinking 4. According to Jane, how do many people react to female comedians ? A They’re convinced women can’t tell jokes. B They’re afraid the women will break down. C They find women’s humour too intense. D They find women’s jokes embarrassing. 5. What was the disadvantage of the stage image which Jane developed? A It frightened the audience. B It made the audience angry. 1

Your answers: 1.

2.

3.

4.

FI CI A

L

C People thought it reflected her real personality. D People did not take her seriously anymore.

5.

Section 2. Listen to the recording and decide whether the following statements are true (T) or false (F). (10 pts)

OF

6. The speaker has come from the Theosophical Society. 7. One of the main points of the talk is to save money. 8. She thinks students should do more housework. 9. She argues that plastic containers won’t biodegrade quickly. 10. She warns that asthma sufferers should be careful with her recipes.

6.

7.

8.

ƠN

Your answers: 9.

10.

NH

Section 3. Listen to the talk about women in the workplace and answer the following questions. (10 pts) 11. How is the situation for women in the workplace changing? …..............................................................................................................................

QU Y

12. In which management role that women constitute 17% of the staff? ….............................................................................................................................. 13. How many percent of employees think that gender equality is a priority? …..............................................................................................................................

M

14. What is the action companies should take to understand the problem of gender equality? …..............................................................................................................................

15. What should companies do to make sure opportunities and advancement are equitable? …..............................................................................................................................

Y

Section 4: Listen and fill in the blanks with the missing information Write NO MORE THAN THREE WORDS for each answer. (20 pts)

DẠ

Over the past few years as first lady, I have had the (16)______________ of traveling all across this country and everywhere I’ve gone and the people I've met and the stories I’ve heard, I have seen the very best of the (17)______________. See, our life before moving to Washington was, was filled with simple joys. Saturdays at soccer games, Sundays at grandma’s house, and a 2

L

date night for Barack and me was either dinner or movie because as an exhausted mum I couldn’t stay awake for both. Even back then when Barack was a senator and (18)______________ to me

FI CI A

he was still the guy who picked me up for our dates in a car that was so (19)______________ that I could actually see the pavement going by in a hole in the passenger side door. He was the guy whose (20)______________ was a coffee table he'd found in a dumpster. Well today, after so many (21)______________ and moments that’ve tested my husband in ways I never could have

OF

imagined, I have seen first-hand that being president doesn’t change who you are. No it (22)______________who you are. When it comes to the health of our families, Barack refused to listen to all those folks to told him to leave (23)______________ for another day, another president. He didn’t care whether it was the easy thing to do politically, no that's not how he was raised. He cared that it was the right thing to do. When we were first married our combined monthly student loan bill was actually higher than our (24)______________. Yeah!! We were so young, so in love, and so in debt. If we wanna give all of our children a foundation for their

NH

ƠN

dreams and opportunities worthy of their promise. If we wanna give them that sense of (25)______________, that belief that here in America there was always something better out there if you're willing to work for it. Then we must work like never before, and we must once again come together and stand together for the man we can trust to keep moving this great country forward. My husband, our president, Barack Obama. Thank you, God bless you, God bless America. Your answers:

QU Y

16. 17. 18. 19. 20.

21. 22. 23. 24. 25.

M

PART B. LEXICO – GRAMMAR (30pts) Section 1. Choose the word or phrase that best fits the gap in each sentence (20 pts) 1. I was promised a good job from January this year, but it’s April now and I’m afraid that they are just ___________ me along A. cheating B. swindling C. stringing D. bringing 2. Although we now believe this to be impossible, early scientists tried to produce __________ motion machines, that is, machines which would never stop. A. perpetual B. everlasting C. undying D. forever

DẠ

Y

3. The company was so successful during the 1980s that it _________ 500 new employees in a period of six months A. put on B. took on C. caught on D. laid on 4. Mr. Discontent wanted to complain to the waiter but was afraid of making a(n)_______. B. drama C. play D. scene A. act 5. I could see the tip of his cigarette _________ in the darkness. 3

L

A. glowing B. sparkling C. gleaming D. glinting 6. Is an inexperienced civil servant __________ to the task of running the company ?

FI CI A

A. capable B. skilled C. eligible D. suited 7. After the concert, everyone had to _________ home through the snow. B. trend C. trudge D. trickle A. trace 8. The students paid _________ attention to the distinguished professor.

A. respectable B. respected C. respectful D. respective 9. ______________, the people who came to this club are in their twenties and thirties.

OF

A. By and large B. Although C. To a degree D. Virtually 10. He is late for every conference without __________. B. fail C. doubt D. delay A. suspicion 11. Revenge at last! I always said I would __________ my own back on Steve, and now I have.

NH

ƠN

A. turn B. get C. do D. make 12. Look, will you stop _________ in and let me finish my sentence! A. moving B. pushing C. butting D. plugging 13. It’s the __________ of stupidity to go walking in the mountains in this weather. A. height B. depth C. source D. matter 14. The school was closed for a month because of a serious ________ of fever. A. outcome B. outburst C. outset 15. Social work suits her ______ to the ground. A. down B. for C. out

D. outbreak D. round

M

QU Y

16. The President accused his critics of being oversensitive and of __________. B. cutting to the chase A. playing it by ear C. making a mountain out of a molehill D. splitting hairs 17. My secretary left me in the ________ last month and I haven’t found a replacement yet. B. lurch C. face D. fire A. rock 18. With patience and diplomacy, she eventually _________ the son of the billion-dollar company into marrying her. A. deluded B. inveigled C. cruised D. swindled 19. After our three-hour hike, I had a sandwich and it tasted like the best ________. B. sauce C. seasoning D. flavor A. chef

Y

20. You are being unnecessarily ________ by spending too much time on the details of your CV. A. recalcitrant B. mendacious C. dogmatic D. pedantic Your answers:

DẠ

1. 11.

2. 12.

3. 13.

4. 14.

5. 15.

6. 16.

7. 17.

8. 18.

9. 19.

10. 20.

Section 2. Write the correct form of each bracketed word in each sentence. (10 pts) 1. His childhood spent in the harsh conditions of the civil war has completely ______ (SENSE) 4

FI CI A

L

the boy to the sight of blood and cruelty. 2. “What do you expect from your future wife, Mr. Robson?” - “I’d like one that is faithful and ______ (SUBMIT) so that she can fulfil all my requests.” 3. Our local newspaper is often full of stories that are hardly ______ (NEWS), but they need to fill the pages somehow. 4. Both of the fighting sides have agreed on a temporary ______ (ARM) just to let the civilians

NH

ƠN

OF

safely leave the place. 5. The botanist studied the _____ (FLORAE) of the tropical rainforest, documenting numerous plant species. 6. The orator delivered a _____ (PERSUADE) speech that swayed the opinions of the entire audience. 7. The artist's paintings were filled with ______ (INTRICATE) details, requiring careful observation to appreciate fully. 8. The philosopher's ideas were often ______ (ESOTERIC), understood only by a select few. 9. The archaeologist meticulously _______ (EXCAVATE) the ancient ruins, unearthing valuable artifacts. 10. The professor _______ (PROFICIENT) explained complex mathematical concepts with remarkable clarity. Your answers:

QU Y

1. 2. 3. 4. 5.

6. 7. 8. 9. 10.

PART C. READING (60 pts)

M

Section 1. Choose the word that best fits each of the blanks in the following passage (10 pts) FOOTBALL AS AN ART FORM

When filmmakers Douglas Gordon and Phillipe Parreno set out to make an art house movie about the legendary French footballer Zinedine Zidane, they chose to film just one match between Real Madrid, the club for which he was playing at the (1) …..............., and their great rivals Villareal.

DẠ

Y

But instead of following the progress of the match, the ninety-minute film would show something that had not been seen before; the (2) …............... detailed movements of one man during an entire top-level football match. They hoped that the audience would disengage from the match itself, and focus on this portrait of greatness. Every (3) …............... gesture would be captured and they would see all of the player’s grace, athleticism and competitiveness in great detail.

The film (4) …............... is a fascinating work. Those who are not regular watchers of football will be astonished at how (5) …............... Zidane becomes actively involved in the game. For 5

L

much of the ninety minutes he moves around the field relatively slowly; saying nothing, expressing even less, and only occasionally (6) …............... into a lethargic jog. And then the ball

FI CI A

arrives at his feet, and there is a flurry of bewildering activity. The cameras (and there are seventeen of them (7) …............... on him) struggle to keep up. The defenders don’t (8)

…............... a chance. In a few touches, a couple of checks and feints, Zidane has (9) …............... them all behind. He crosses from the tightest of (10) …............... and his teammate is left with simplest of headers to score a goal.

(Adapted from CAE Practice Tests Plus/88)

A. point

B. moment

C. time

2.

A. clear

B. steady

C. precise

3.

A. one

B. single

C. lone

4.

A. following

B. resulting

C. concluding

5.

A. partly

B. scarcely

6.

A. breaking

B. changing

7.

A. trained

B. looking

8.

A. gain

B. hold

9.

A. missed

B. left

10.

A. places

B. positions

1. 6.

2. 7.

ƠN

3. 8.

D. slow D. sole D. arising

C. rarely

D. hardly

C. opening

D. starting

C. pointed

D. staring

C. stand

D. earn

C. lost

D. dropped

C. areas

D. angles

NH

QU Y

Your answers:

D. occasion

OF

1.

4. 9.

5. 10.

M

Section 2. Read the text below and think of one word which best fits each space. Use only ONE WORD for each space. Write your answer in the space provided (15 pts) Moths count!

Renowned conservationist Sir David Attenborough is launching a campaign today called ‘Moths Count’, to halt the drastically declining number of Britain’s native moths and improve

DẠ

Y

their poor image. A report (1) …............... ‘The State of Britain’s Larger Moths’ revealed last year that in some areas, the moth population has almost (2) …............... since 1968. This has led the charity, ‘Butterfly Conservation’, of which Sir David is president, to develop a new strategy which will provide opportunities for real (3) …............... to broaden their (4) …............... and also generate appreciation among the wider public. Moths, he insists, play an essential role in the environment. Their loss (5) …............... the species of birds, bats and small mammals that (6) 6

L

…............... on them, and the plants they (7) …............... ‘Moths Count’ campaigner Richard Fox says ‘Currently there’s an image problem, partly because there’s a (8) …............... that moths are

FI CI A

night creatures, although many are day-flying and only about half a dozen of Britain’s 2500 species damage clothes.’ Reasons for their decline include climate change and the loss of habitat. Although the (9) …............... of moths has increased with the establishment of new species in Britain, overall their numbers have dropped, and for some, extinction now seems sadly (10) …...............

1. 6.

2. 7.

3. 8.

4. 9.

5. 10.

OF

Your answers:

Section 3. Read the following passage and answer the questions. (15 pts)

DẠ

Y

M

QU Y

NH

ƠN

The history of aspirin is a product of a rollercoaster ride through time, of accidental discoveries, intuitive reasoning and intense corporate rivalry. A. In the opening pages of Aspirin: The Remarkable Story of a Wonder Drug, Diarmuid Jeffreys describes this little white pill as ‘one of the most amazing creations in medical history, a drug so astonishingly versatile that it can relieve headache, ease your aching limbs, lower your temperature and treat some of the deadliest human diseases’. B. Its properties have been known for thousands of years. Ancient Egyptian physicians used extracts from the willow tree as an analgesic, or pain killer. Centuries later the Greek physician Hippocrates recommended the bark of the willow tree as a remedy for the pains of childbirth and as a fever reducer. But it wasn't until the eighteenth and nineteenth centuries that salicylates the chemical found in the willow tree became the subject of serious scientific investigation. The race was on to identify the active ingredient and to replicate it synthetically. At the end of the nineteenth century a German company, Friedrich Bayer & Co. succeeded in creating a relatively safe and very effective chemical compound, acetylsalicylic acid, which was renamed aspirin. C. The late nineteenth century was a fertile period for experimentation, partly because of the hunger among scientists to answer some of the great scientific questions, but also because those questions were within their means to answer. One scientist in a laboratory with some chemicals and a test tube could make significant breakthroughs whereas today, in order to map the human genome for instance, one needs ‘an army of researchers, a bank of computers and millions and millions of dollars’. 7

D.

DẠ

Y

M

QU Y

NH

ƠN

OF

FI CI A

L

But an understanding of the nature of science and scientific inquiry is not enough on its own to explain how society innovates. In the nineteenth century, scientific advance was closely linked to the industrial revolution. This was a period when people frequently had the means, motive and determination to take an idea and turn it into reality. In the case of aspirin that happened piecemeal - a series of minor, often unrelated advances, fertilised by the century’s broader economic, medical and scientific developments, that led to one big final breakthrough. E. The link between big money and pharmaceutical innovation is also a significant one. Aspirin’s continued shelf life was ensured because for the first 70 years of its life, huge amounts of money were put into promoting it as an ordinary everyday analgesic. In the 1970s other analgesics, such as ibuprofen and paracetamol, were entering the market, and the pharmaceutical companies then focused on publicising these new drugs. But just at the same time, discoveries were made regarding the beneficial role of aspirin in preventing heart attacks, strokes and other afflictions. Had it not been for these findings, this pharmaceutical marvel may well have disappeared. F. So the relationship between big money and drugs is an odd one. Commercial markets are necessary for developing new products and ensuring that they remain around long enough for scientists to carry out research on them. But the commercial markets are just as likely to kill off' certain products when something more attractive comes along. In the case of aspirin, a potential ‘wonder drug’ was around for over 70 years without anybody investigating the way in which it achieved its effects, because they were making more than enough money out of it as it was. If ibuprofen or paracetamol had entered the market just a decade earlier, aspirin might then not be here today. It would be just another forgotten drug that people hadn't bothered to explore. G. None of the recent discoveries of aspirin's benefits were made by the big pharmaceutical companies; they were made by scientists working in the public sector. 'The reason for that is very simple and straightforward,' Jeffreys says in his book. 'Drug companies will only pursue research that is going to deliver financial benefits. There's no profit in aspirin any more. It is incredibly inexpensive with tiny profit margins and it has no patent any more, so anyone can produce it.' In fact, there's almost a disincentive for drug companies to further boost the drug, he argues, as it could possibly put them out of business by stopping them from selling their more expensive brands. H. 8

OF

FI CI A

L

So what is the solution to a lack of commercial interest in further exploring the therapeutic benefits of aspirin? More public money going into clinical trials, says Jeffreys. ‘If I were the Department of Health. I would say “this is a very inexpensive drug. There may be a lot of other things we could do with it." We should put a lot more money into trying to find out.' I. Jeffreys' book which not only tells the tale of a 'wonder drug' but also explores the nature of innovation and the role of big business, public money and regulation reminds us why such research is so important. Questions 1-5. The Reading Passage has 9 paragraphs A-I. Choose the correct heading for each paragraph (A-E) from the list of headings below. Write the correct number (i-x) in boxes 1-5.

ƠN

NH

Paragraph A Paragraph B Paragraph C Paragraph D Paragraph E

The discovery of new medical applications

Your answers 2.

M

1.

1. 2. 3. 4. 5.

QU Y

i. ii. iii. iv. v. vi. vii. viii. ix. century x.

List of headings: The most powerful analgesic The profit potential of aspirin Saved from oblivion by drug companies Recognition of an important medicinal property A double-edged sword An unstructured pattern of development Major pharmaceutical companies A wonder drug Roots of the scientific advancements in the 19th

3.

4.

5.

Questions 6-10 Do the following statements agree with the views of the writer in Reading Passage? In boxes 6-10 write: if the statement agrees with the views of the writer

NO

if the statement contradicts the views of the writer

NOT GIVEN

if it is impossible to say what the writer thinks about this

DẠ

Y

YES

6. The 19th century saw significant changes in the way in which scientific experiments were carried out. 9

L

7. For nineteenth-century scientists, small-scale research was far from enough to make important discoveries.

FI CI A

8. The creation of a market for aspirin as a painkiller was achieved through commercial advertising campaigns. 9. In the 1970s sales of new analgesic drugs overtook sales of aspirin. 10. Jeffrey suggests that there should be state support for further research into the possible applications of aspirin. Your answers 7.

8.

9.

Section 4. Read the text and do the task as follow (13 pts)

10.

OF

6.

The Farmers! Parade of history

NH

ƠN

A. History of Fanner trading company: In 1909 Robert Laidlaw establishes mail-order company Laidlaw Leeds in Fort Street, Auckland. Then, Branch expansion: purchase of Green and Colebrook chain store; further provincial stores in Auckland and Waikato to follow. Opening of first furniture and boot factory. In 1920, Company now has 29 branches; Whangarei store purchased. Doors open at Hobson Street for direct selling to public. The firm establishes London and New York buying offices. With permission from the Harbour Board, the Large FARMERS electric sign on the Wyndham Street frontage is erected. B. In 1935, if the merchandise has changed, the language of the catalogues hasn't Robert Laidlaw, the Scottish immigrant who established die century-old business, might have been scripting a

M

QU Y

modern-day television commercial when he told his earliest customers: Satisfaction, or your money back. "It was the first money back guarantee ever offered in New Zealand any firm," says Ian Hunter, business historian. "And his mission statement was, potentially, only the second one ever found in the world." Laidlaw's stated aims were simple to build the greatest business in New Zealand, to simplify every transaction, to eliminate all delays, to only sell goods it would pay the customer to buy.

C. This year, the company that began as a mail-order business and employs 3500 staff across 58 stores turns 100. Its centenary will be celebrated withe release of a book and major community

Y

fundraising project, to be announced next week. Hunter, who is writing the centenary history, says in every 10 people had an account with die company. It was the place where teenage girl shopped for their first bra, where newlyweds purchased their first dinner sets, where first pay cheques were used to pay off hire purchase furniture, where Santa paraded every Christmas. D. Gary Blumenthal's mother shopped there, and so does he. The fondest memory for the Rotorua

DẠ

resident? "We were on holiday in Auckland... I decided that upon the lookout tower on top of the Farmers building would be a unique place to fit the ring on my new fiancee’s finger." The lovebirds, who had to wait for "an annoying youth" to leave the tower before they could enjoy their engagement kiss, celebrate their 50th wedding anniversary in June. 10

L

E. Farmers, say Hunter, has always had a heart. This, from a 1993 North & South interview with a former board chairman, Rawdon Busfield: " One day I was in the Hobson Street shop, and I saw a

FI CI A

woman with two small children. They were clean and tidily dressed, but poor, you could tell. That week we had a special on a big bar of chocolate for one shilling. I heard the woman say to her

boy, 'no, your penny won't buy that'. He wasn't wearing shoes. So I went up to the body said,' Son, have you got your penny?' He handed it to me. It was hot he'd had it in his hand for hours. I took the penny and gave him the chocolate.

F. Farmers was once the home of genteel tearooms, children's playground and an annual sale of celebration for birthday of Hector the Parrot (the store mascot died, aged 131, in the 1970s his

OF

stuffed remains still occupy pride of place at the company's head office). You could buy houses from Farmers. Its saddle factory supplied the armed forces, and its upright grand overstrung

ƠN

pianos offered "the acme of value" according to those early catalogues hand-drawn by Robert Laidlaw himself. Walk through a Farmers store today and get hit by bright lights and big brands. Its Albany branch houses 16 international cosmetics companies. It buys from approximately 500 suppliers, and about 30% of those are locally owned.

QU Y

NH

G. "Eight, 10 years ago," says current chief executive Rod McDermott, "lots of brands wouldn't partner with us. The stores were quite distressed. We were first price point focused, we weren't fashion focused. "Removed the rose-tinted nostalgia, and Farmers is, quite simply, a business, doing business in hard times. Dancing with the Stars presenter Candy Lane launches a clothing line? "We put a trial on, and we thought it was really lovely, but the uptake wasn't what we thought it would be. It's got to be what the customer wants" says McDermott.

M

H. He acknowledges retailers suffer in a recession: "We're celebrating 100 years because we can and because we should" Farmers almost didn't pull though one economic crisis. By the mid 1980s, it had stores across the country. It had acquired the South Island's Calder Mackay chain of stores and bought out Haywrights. Then, with sale topping $375 million, it was taken over by Chase Corporation. Lincoln Laidlaw, now aged 88, and the son of the company's founder, remembers the dark days following the stock market crash and the collapse of Chase. "I think, once, Farmers was

like a big family and all of the people who worked for it felt they were building something which would ultimately be to their benefit and to the benefit of New Zealand... then the business was

DẠ

Y

being divided up and so that kind of family situation was dispelled and it hasn't been recovered." For a turbulent few years, the stores were controlled, first by a consortium of Australian banks and later Deka, the Maori Development Corporation and Foodland Associated Ltd. In 2003, it went back to "family" ownership. with the purchase by the James Pascoe Group, owned by David and Anne Norman the latter being the great-granddaughter of James Pascoe, whose first business interest was jewellery. I. "Sheer power of the brand", says McDermott," pulled Farmers through and now we're becoming the brand it used to be again." Farmers was the company that, during World War n, topped up the wages of any staff member disadvantaged by overseas service. Robert Laidlaw a committed 11

L

Christian who came to his faith at a 1902 evangelistic service in Dunedin concluded his original mission statement with the words, "all at it, always it, win success". Next week, 58 Farmers stores

FI CI A

across the country will announce the local charities they will raise funds for in their centenary celebration everything from guide dog services to hospices to volunteer fire brigades will benefit Every dollar raised by the community will be matched by the company. "It's like a rebirth of an icon," says McDermott.

OF

Question 1-5 The reading Passage has seven paragraphs A-I Which paragraph contains the following information? Write the correct A-I, in boxes provided. 1. Generosity offered in an occasion.

3. Fashion was not its strong point. 4. A romantic event on the roof of farmers.

ƠN

2. Innovation of offer made by the head of company.

NH

5. Farmers were sold to a private owned company. Your answers: 1.

2.

3.

5.

QU Y

Question 6-10

4.

Complete the following summary of the paragraphs of Reading Passage Using NO MORE THAN TWO WORDS from the Reading Passage for each answer. 6. Farmers was first founded as a 36 ______ in Auckland by Mr Laidlaw. 7. Farmers developed fast and bought one 37______ then.

M

8. During oversea expansion, Farmers set up 38______ in cities such as London.

9. Farmers held a 39 ______once a year for the well-known parrot. 10. In the opinion of Lincoln Laidlaw, Farmers is like a 40______ for employees, not just for themselves but for the whole country.

Y

Your answers:

DẠ

6. 7. 8. 9. 10. Question 11-13 Use the information in the passage to match the people (listed A-C) with opinions or deeds below.

12

A Lincoln Laidlaw B Rod McDermott C Ian Hunter 11. Product became worse as wrong aspect focused. 12. An unprecedented statement made by Farmers in New Zealand.

OF

13. Character of the company was changed. Your answer: 12.

13.

ƠN

11.

FI CI A

L

Write the appropriate letters A-C in boxes provided. NB You may use any letter more than once.

NH

Section 5. You are going to read an article. Seven paragraphs have been removed. Choose from the paragraphs A-H the one that fits each gap (1-7). There is one extra paragraph which you do not need to use. (7.0 pts) THE IMPORTANCE OF SLEEP

QU Y

A recent study claims to have found a new explanation for the puzzle that is one of our most essential biological functions: sleep. Apparently, while we have our forty winks, a 'waste management' system pumps cerebral fluid around the brain and flushes out molecular detritus, a natural by-product of neural activity that collects during waking hours. If allowed to pile up, these waste toxins are responsible for a number of neurological disorders, including Alzheimer's 1.

2.

M

As a biological state, sleep is not restricted to humans. It is essential for every living creature and tailored to suit their individual lifestyles as a species - from up to 20 hours a day to brown bats hanging upside down to a 30-minute daily shut-eye for giraffes standing up. Sleep does not, however, appear to make sense from a survival point of view, as, unable to operate our guards and with significantly reduced response to external stimuli, we are vulnerable to predators. Yet, it is hardly a choice for any of us.

DẠ

Y

This is a pattern that nature doesn't let us break. Empirically, we have learnt more about the effects that are triggered by lack of sleep than the benefits we derive from it. Understand it or not, sleep is so vitally important that any deviation from what is within the perceived norms can impair both our mental and physical health.

13

3.

L

Experiments on rats have shown that total sleep deprivation for about two weeks or more

FI CI A

inevitably led to death. In known cases where humans were prevented from sleeping, either due to the existence of the syndrome or other imposed reasons, the body gradually degenerated to the point of multiple organ failure. 4.

The energy conservation theory: Sleep lowers our energy needs by as much as 10 per cent as compared to when we are restful and awake because our body reduced both its temperature and its

OF

caloric consumption. This might have been crucial for the survival of our ancestors when food was not easily available as it gave them the advantage of being more efficient hunters. 5.

ƠN

REM (rapid eye movement), for example, is a deep state of sleep which involves intense brain activity. It allows us to dream and it accounts for about one fifth of our total sleep time. REM sleep plays an important role in restoring mental functions NREM (non-rapid eye movement) sleep account for the remaining four fifths and is conducive to restoring physiological functions.

NH

6.

QU Y

Mental activity during sleep has also been linked to brain plasticity, which is defined as the brain's ability to modify its own structure and function to reflect changes in our body or the external environment. There is evidence that sleep plays a critical role in brain development in infants and young children. 7.

M

Although we have been unable to provide concrete proof for the above theories, it is essential that we recognise the advances that have been made towards comprehending why we sleep and the way in which research has helped us further our understanding of the mechanics of it. The bottom line is that it might be too simplistic to assume there is a single answer to such a complex process

A. The study is just another attempt to gain some insight into what makes us humans tick. Although we have a relatively satisfactory understanding of other life-sustaining activities, such as

Y

eating (to fuel our body with essential nutrients so thit can grow and repair itself) and drinking water (to keep our 70% topped up), we are still searching for a definitive answer to why we need to sleep

DẠ

B. Up to now many theories have been put forward, including this recent one, but science has yet to come up with the definitive explanation of why we sleep. Though research and experimentation we have gained valuable insight into the mechanisms that go to work in our body while we sleep. We now know that sleep is related to our psychology, physiology, metabolism and digestion. Let's have a look at a few of these theories 14

L

C. Finally, there's the theory that maintains that dreams, which only occur when we sleep, are out way of coping with our emotions. Although dream research offers many theories of why we

FI CI A

dream, we know that dreams help us to do some problem solving, and deal with traumatic experiences.

OF

D. Sleep is controlled and dictated by our internal clock, which, in turn, is regulated by the process called Circadian rhythm, inherent in all living beings. It's a daily cycle of biological activity which tells us when it is time to eat, rest or be active. It is set to respond to external stimuli such as the presence of light, which wakes us up in the morning of the absence of it, which prompts us to go to bed at night E. The restorative theory: Sleep give us the opportunity to do some essential repair work and

ƠN

'housekeeping'. It is the time when many of the major restorative function like tissue repair, muscle growth, protein synthesis and the release of growth hormones take place in our body. The same goes for our brain. It is important to note that there are two different states sleep during which we switch from physical to mental maintenance. F. The scientist cautions that the simple brain of a mouse and the more complex brain of the human are two different things, and that what is true for one is not necessarily true for the other. It

NH

is important at this stage not to be either too optimistic or too dismissive of new indications but to continue research until we have enough evidence to from and assumption about its relevance to the human brain.

QU Y

G. The information consolidation theory: During sleep, and particularly during the first hours when deep sleep occurs, our brain has the chance to process and consolidate the information it received the previous day, as well as prepare for the next day. Recently acquired information is either discarded as useless or committed to our long-term memory. The theory is supported by research that indicates that sleep deprivation affects our ability to recall and retain information. H. Chronic sleep deprivation, or, in other words, not enough sleep, which is the most common complaint, has an adverse effect on our cognitive process, compromise our memory and can even

M

make us depressed, it increases the risk of serious illnesses like cardiovascular disease, diabetes and some forms of cancer or even disease-related mortality. PART D. WRITING (60pts)

Y

Section 1. Read the following passage and use your own words to summarize it. Your summary should be between 100 and 120 words. (15 pts)

DẠ

Buying things today is so simple. Just enter a shop, say a book store, choose the desired book and pay for it. Long ago, before the invention of money, how did people trade? The most primitive way of exchange should be the barter trade. In this form of transaction, people used goods to exchange for the things that they had in mind. For instance, if person A wanted a book and he had a spare goat, he must look for someone who had the exact opposite, that is, that 15

FI CI A

book, hence person B may have to offer person A something else, say five chickens.

L

someone, say person B, must have a spare book of person A's choice and is also in need of a goat. Having found such a person, the problem does not end here. A big goat may worth not only one

However, he runs the risk of person A rejecting the offer as he may not need the chickens. The above example clearly illustrates the inefficiency of barter trading. Many years later, the cumbersome barter trade finally gave way to the monetary form of exchange when the idea of money was invented. In the early days, almost anything could qualify as money: beads, shells and even fishing hooks. Then in a region near Turkey, gold coins were used as money. In the beginning, each coin had a different denomination. It was only later, in about 700 BC, that Gyges,

OF

the king of Lydia, standardized the value of each coin and even printed his name on the coins. Monetary means of transaction at first beat the traditional barter trade. However, as time went by, the thought of carrying a ponderous pouch of coins for shopping appeared not only

ƠN

troublesome but thieves attracting. Hence, the Greek and Roman traders who bought goods from people faraway cities, invented checks to solve the problem. Not only are paper checks easy to carry around, they discouraged robbery as these checks can only be used by the person whose

NH

name is printed on the notes. Following this idea, banks later issued notes in exchange for gold deposited with them. These bank notes can then be used as cash. Finally, governments of today adopted the idea and began to print paper money, backed by gold for the country's use. Today, besides enjoying the convenience of using paper notes as the mode of exchange, technology has led man to invent other means of transaction too like the credit and cash cards.

QU Y

……………………………………………………………………………………………………… ………………………………………………………………………………………………………

M

……………………………………………………………………………………………………… ……………………………………………………………………………………………………… ……………………………………………………………………………………………………… ……………………………………………………………………………………………………… ……………………………………………………………………………………………………… ……………………………………………………………………………………………………… ……………………………………………………………………………………………………… ………………………………………………………………………………………………………

Y

……………………………………………………………………………………………………… ……………………………………………………………………………………………………… ……………………………………………………………………………………………………… ………………………………………………………………………………………………………

DẠ

……………………………………………………………………………………………………… ……………………………………………………………………………………………………… Section 2. The chart shows the proportion of renewable energy in total energy supply in 4 countries from 1997 to 2010. Summarize the information and Write at least 150 words. (15pts) 16

L FI CI A OF ƠN

………………………………………………………………………………………………………

QU Y

NH

……………………………………………………………………………………………………… ……………………………………………………………………………………………………… ……………………………………………………………………………………………………… ……………………………………………………………………………………………………… ……………………………………………………………………………………………………… ……………………………………………………………………………………………………… ……………………………………………………………………………………………………… ……………………………………………………………………………………………………… ……………………………………………………………………………………………………… ……………………………………………………………………………………………………… ………………………………………………………………………………………………………

M

……………………………………………………………………………………………………… ……………………………………………………………………………………………………… ……………………………………………………………………………………………………… ……………………………………………………………………………………………………… ……………………………………………………………………………………………………… ………………………………………………………………………………………………………

Y

……………………………………………………………………………………………………… ……………………………………………………………………………………………………… ……………………………………………………………………………………………………… ………………………………………………………………………………………………………

DẠ

……………………………………………………………………………………………………… ………………………………………………………………………………………………………

……………………………………………………………………………………………………… ……………………………………………………………………………………………………… 17

L

……………………………………………………………………………………………………… ………………………………………………………………………………………………………

FI CI A

……………………………………………………………………………………………………… ……………………………………………………………………………………………………… ……………………………………………………………………………………………………… ………………………………………………………………………………………………………

OF

Section 3. Write an essay of 350 words on the following topic. (30 pts) Some people think that the best way to become successful in life is to get a university education, while others disagree and say this is no longer true nowadays. Discuss these both views and give your own opinion.

……………………………………………………………………………………………………… ………………………………………………………………………………………………………

ƠN

……………………………………………………………………………………………………… ………………………………………………………………………………………………………

QU Y

NH

……………………………………………………………………………………………………… ……………………………………………………………………………………………………… ……………………………………………………………………………………………………… ……………………………………………………………………………………………………… ……………………………………………………………………………………………………… ……………………………………………………………………………………………………… ……………………………………………………………………………………………………… ………………………………………………………………………………………………………

M

……………………………………………………………………………………………………… ……………………………………………………………………………………………………… ……………………………………………………………………………………………………… ……………………………………………………………………………………………………… ……………………………………………………………………………………………………… ……………………………………………………………………………………………………… ……………………………………………………………………………………………………… ……………………………………………………………………………………………………… ……………………………………………………………………………………………………… ………………………………………………………………………………………………………

DẠ

Y

……………………………………………………………………………………………………… ……………………………………………………………………………………………………… ……………………………………………………………………………………………………… ……………………………………………………………………………………………………… ……………………………………………………………………………………………………… ……………………………………………………………………………………………………… ……………………………………………………………………………………………………… 18

L

……………………………………………………………………………………………………… ………………………………………………………………………………………………………

FI CI A

……………………………………………………………………………………………………… ……………………………………………………………………………………………………… ……………………………………………………………………………………………………… ………………………………………………………………………………………………………

OF

……………………………………………………………………………………………………… ……………………………………………………………………………………………………… ……………………………………………………………………………………………………… ……………………………………………………………………………………………………… ……………………………………………………………………………………………………… ……………………………………………………………………………………………………… ………………………………………………………………………………………………………

NH

ƠN

……………………………………………………………………………………………………… ……………………………………………………………………………………………………… ……………………………………………………………………………………………………… ……………………………………………………………………………………………………… ……………………………………………………………………………………………………… ……………………………………………………………………………………………………… ……………………………………………………………………………………………………… ………………………………………………………………………………………………………

M

QU Y

……………………………………………………………………………………………………… ……………………………………………………………………………………………………… ……………………………………………………………………………………………………… ……………………………………………………………………………………………………… ……………………………………………………………………………………………………… ……………………………………………………………………………………………………… Giáo viên ra đề Lưu Thị Thúy Ngà

DẠ

Y

Số điện thoại:

19

20

Y

DẠ M

KÈ QU Y ƠN

NH

FI CI A

OF

L

HỘI CÁC TRƯỜNG CHUYÊN VÙNG DUYÊN HẢI VÀ ĐỒNG BẰNG BẮC BỘ

ĐỀ THI HSG KHU VỰC DH & ĐBBB LẦN THỨ XV – NĂM 2023 MÔN: TIẾNG ANH - KHỐI 11 Thời gian: 180 phút Đáp án gồm: 10 trang

ĐÁP ÁN ĐỀ THI ĐỀ XUẤT

PART A. LISTENING: 50 pts 4. B

5. C

Section 2. 5 x 2.0 = 10 pts 1. F 2. T 3. F

4. T

5. F

ƠN

QU Y

Section 4. 10 x 2.0 = 12 pts 1. extraordinary privilege 3. a presidental candidate 5. proudest possession 7. reveals 9. mortgage

NH

Section 3. 5 x 2.0 = 10 pts 1. Not fast enough 2. C-suite 3. 37% 4. Track key metrics 5. Create a level playing field

OF

Section 1. 5 x 2.0 = 10 pts 1. B 2. D 3. C

FI CI A

L

TRƯỜNG THPT CAO BẰNG

2. American spirit 4. wasted out 6. struggles and trials 8. health reforms 10. limitless responsibility

PART B. LEXICO-GRAMMAR: 30 pts Section 1. 20 x 1.0 = 20 pts

2 3 4 5 6 7

A B D A D C

Y DẠ

string someone along: để lừa dối ai đó trong một thời gian dài về những gì bạn đang thực sự có ý định làm perpetual: vĩnh viễn take on: tuyển dụng ai đó make a scene: cư xử một cách ồn ào, tức giận ở nơi công cộng glowing: phát sáng suited: phù hợp trudge through something: để làm công việc hoặc một nhiệm vụ cụ thể chậm và với nỗ lực hoặc khó khăn respectful: tôn trọng By and large: khi mọi thứ về một tình huống được xem xét cùng nhau to do something without fail: không thất bại trong việc làm gì đó

M

C

1

8 9

C A

10

B

A

14

D

15

A

16

C

17

B

18

B

19

D

20

D

L

13

FI CI A

C

OF

12

get your own back (on someone): làm điều gì đó khó chịu với ai đó vì họ đã làm điều gì đó khó chịu với bạn butt in: để làm gián đoạn một cuộc trò chuyện hoặc thảo luận hoặc ai đó đang nói chuyện the height of something: thời gian khi một tình huống hoặc sự kiện là mạnh nhất hoặc đầy đủ nhất của hoạt động outbreak: một thời điểm khi một cái gì đó đột ngột bắt đầu, đặc biệt là một căn bệnh hoặc một cái gì đó nguy hiểm hoặc khó chịu suit someone (right) down to the ground: Nếu một cái gì đó phù hợp với bạn đến tận cùng, thì nó hoàn toàn phù hợp với bạn, thường là vì nó thuận tiện cho bạn making a mountain out of a molehill: để làm cho một khó khăn nhỏ có vẻ như một vấn đề nghiêm trọng leave someone in the lurch: rời bỏ ai đó vào thời điểm họ cần bạn ở lại và giúp đỡ inveigle: để thuyết phục ai đó làm điều gì đó một cách thông minh và không trung thực, khi họ không muốn làm điều đó pedantic: chú ý quá nhiều đến các quy tắc chính thức hoặc các chi tiết nhỏ

ƠN

B

Section 2. 10 x 1.0 = 10 pts

để khiến ai đó trải nghiệm điều gì đó, thường là cảm xúc hoặc nỗi đau, ít mạnh mẽ hơn trước 2. submissive cho phép bản thân bị kiểm soát bởi người hoặc động vật khác 3. newsworthy đủ thú vị để được mô tả trong một báo cáo tin tức 4. armistice một thỏa thuận chính thức giữa hai quốc gia hoặc các nhóm có chiến tranh để ngừng chiến đấu trong một thời gian cụ thể, đặc biệt là để nói về hòa bình có thể 5. flora tất cả các thực vật của một địa điểm cụ thể hoặc từ một thời điểm cụ thể trong lịch sử 6. persuasive khiến bạn muốn làm hoặc tin vào một điều cụ thể 7. intricate có rất nhiều bộ phận nhỏ được sắp xếp một cách phức tạp hoặc tinh tế 8. esoteric rất bất thường và chỉ được hiểu hoặc thích bởi một số ít người, đặc biệt là những người có kiến thức đặc biệt 9. excavate để loại bỏ lớp đất đang bao phủ các đồ vật rất cũ bị chôn vùi trong lòng đất để khám phá những điều về quá khứ 10. proficiently theo cách thể hiện kỹ năng và kinh nghiệm

DẠ

Y

M

QU Y

1. desensitized

NH

11

PART C. READING: 60 pts Section 1. 10 x 1.0 = 10 pts 1. C 2. C 3. B 4. B 5. C 6. A 7. A 8. C 9. B 10. D

Section 3. 10 x 1.5 = 15 pts 2. iv

3. ix

4. vi

6. NG

7. No

8. Yes

9. NG

10. Y

NH

QU Y

Section 4. 13 x 1.0 = 13 pts 1. E 2. B 3. G 4. D 5. H 6. mail-order company 7. chain store 8. buying offices 9. celebration 10. big family 11. B 12. C 13. A

5. x

ƠN

1. viii

OF

FI CI A

L

Section 2. 10 x 1.5 = 15 pts 1. titled 2. halved 3. enthusiatsts 4. expertises 5. threatens 6. feed 7. pollinate 8. perception 9. diversity 10. unavoidable/inevitable

M

Section 5. 7 x 1.0 = 7.0 pts 1. A 2. D 3. H 4. B 5. E 6. G 7. C

Y

PART D. WRITING: 60 pts

DẠ

Section 1. Summarize the following article in not more than 120 words. (15pts)

The mark given to part 1 is based on the following criteria: 1. Write good summary with enough content and clear, logical 5 pts information. 2. Present the key points and main ideas in the right form of a paragraph. 5 pts Make sure that no important points have been omitted or distorted.

L

3. Use your own words or paraphrases with a variety use of synonyms, 5 pts different sentence structures and word class. You can change the order of ideas where necessary.

FI CI A

Sample text

ƠN

OF

This passage focuses on the history of trades and transaction. The earliest method of trading is the barter trade, which entails people using goods they own to exchange for the items they want. This practice is seen as inefficient. A monetary way of exchange is presented many years later as money is invented. Money such as beads, shells and hooks are then reworked to gold coins. This system became more troublesome as time goes on because of thievery and the weight of coin pouches. The Greek and Roman traders came up with paper checks, which can be used as cash to deposit the number of gold in them. Ultimately, present-day governments took on the idea and began the printing of paper money. Besides this, technology's evolution has presented other means of exchange such as credit cards and cash cards. Section 2: (15 pts)

M

QU Y

NH

1. Completion: 3 pts 2. Content: 4 pts - Cover the main information in the chart yet not go into too many details. - Make general remarks and effective comparisons. 3. Organisation: 3 pts - The ideas are well organized - The description is sensibly divided into paragraphs 4. Language: 3pts - Use a wide range of vocabulary and structure - Good grammar 5. Punctuation and spelling: 2 pt Sample text

DẠ

Y

The above chart presents the proportion of renewable energy in the total energy supply in 4 countries from the year 1997 to 2010. Australia and Sweden both have the lowest proportions of renewable energy in total energy supply out of the 4 given countries, as they both average between 10 or lower throughout 1997 and 2010. Australia having a slight decrease from 1997 to 2012 while Sweden has a modest increase. Turkey has a moderate proportion of renewable energy in total energy supply, starting out with an average below 40 in 1997, peaking at about 40 in 2000 and reaching its lowest in 2010 at about 30. Iceland has the highest proportion of renewable energy in total energy supply out of the 4 countries presented, having a great increase from about 50 in 1997, rising to 60 in 2000, before hitting a whopping 70 in 2010.

Section 3. (30 points)

-

3. Organization and Presentation: (10%)

-

L

2. Language: (10%)

Providing all main ideas and details as required Communicating intentions sufficiently and effectively Demonstration of a variety of vocabulary and structures appropriate to the level of English language gifted upper-secondary school students Good use and control of grammatical structures Good punctuation and no spelling mistakes Legible handwriting Ideas are well organized and presented with coherence, cohesion, and clarity The essay is well-structured

FI CI A

-

OF

1. Content: (10%)

ƠN

Sample text

DẠ

Y

M

QU Y

NH

Higher education may be the goal that student and parents are currently aiming for, as evidenced by the annual university entrance exams and the expectations placed on the results. However, some people argued that higher education is no longer worth it. There have been examples of successful businessmen and entreprenuer that forgo higher education such as Bill Gates. Personally, I bellieve this is not a simple issue that can be viewed from only one perspective. Firstly, we have to understand about success in life. The only thing that can provide stable employment and sustain oneself is a job that require a high level of intellectual capacity, as intellectual work is often rewarded much more than ordinary labor. However, investing in developing the intellectual capacity neccessary for high-paying jobs is not a simple task due to the knowledge applied in higher level and broader job is much more extensive than general knowledge, and not everyone has the mentality to acquire it. Most importantly, what demonstate that an applicant can meet the needs of a bussiness the most is higher education, it also provides the shortest path for learners to outline a clear trajectory for developing their own capabilities, and a university degree serves as evidence of learning and to some extend, the competance of the degree holder . Let's pose a simple question: would any hospital hire an unqualified doctor to treat patients? However, this may not be true when we look at other fields such as art, which can be pursued through talent than relying on specific professional like singing or drawing. Even if we consider booming industries like information technology, which offer high salaries but can be learned independently without going through traditional schooling or paying any tuition fees. The field of information technology, which is known for its high salaries, relies heavily on vast open-source resources and a large community that frequently exchanges knowledge and shares extensive documentation. These are the kinds of knowledge that even universities may not necessarily be able to meet.

L

However, currently the tuition fee for higher education are very expensive , and not everyone can afford it. Therefore, higher education is not the optimal choice for everyone considering the financial burden it poses and the limitations it imposes on access and opportunity.

FI CI A

TAPESCRIPT Section 1

Int: Today I’m with the much-loved comedian and writer Jane Clarkson. Obviously Jane, this year has been quite a turning point for you ...

Int: How was your novel received?

ƠN

OF

Jane: Well, I’ll never stop doing comedy, but there were practical reasons for wanting to take some time off and write a book. I felt my daughter had been neglected. She was just about to make the tricky transition from primary to secondary school and I thought she needed her mum around. I seem to have spent most of her life in a van touring from venue to venue for my comedy act. And I did enjoy being at home for a bit, although I missed the applause and the laughter. When I finished writing in the evening, I’d turn the computer off and there’d be nothing, which was hard to get used to.

QU Y

NH

Jane: Well, a lot of male comedians had written books, so there was a bit of a bandwagon waiting to be jumped on, but with my impeccable timing I jumped slightly late, when everyone was starting to get heartily sick of comedians’ books. Also there’s a kind of fury coming from some journalists about comics writing books. They’re absolutely livid, as if they see your book in a bookshop and they jump up and down, shouting, ‘It’s not fair! Why should she make money out of writing as well as performing?’ Int: Was it a difficult transition?

M

Jane: Well, if you think logically, writing is the obvious step. I’ve spent years trying to make people listen to my anecdotes, so that must count for something! Also, if you’ve been an observational comedian, which I am, it’s not a great leap to use those skills you’ve developed, like observing odd mannerisms to use for jokes, and turn them into a book. At least that’s what I felt, but you don’t become a writer instantly. I’ll have to wait and see whether it was just beginners’ luck.

DẠ

Y

Int: I think why people give you a hard time about the novel is surely because we’re so trapped into thinking Jane Clarkson is a comedian. It’s as though, you know, you can’t do anything else, which is quite ridiculous because you’ve been writing radio comedy for years. Jane: Yes, people do become obsessed about what you are. The character I adopted for my comedy act became rather a burden after a while. When I started going on stage alone, I was very young and I wasn’t entirely convincing as a comedian because nervous young women on stage actually frighten audiences.

L

They’re convinced you’re going to fail and burst into tears, which will be very embarrassing. So there’s a palpable tension in the room and some audiences actually boo the female comedians off the stage.

FI CI A

Int: How did you deal with that?

Jane: Well, I had to counteract that stereotype so I started coming on shouting and being madder and crosser than any audience could ever be and that defused the tension. In fact, I rather overdid it and my character got cruder than I ever really intended. I got so good at it that people got confused between the everyday Jane and the stage Jane.

OF

Int: What attracted you in the first place to performing, and particularly to making people laugh?

QU Y

NH

ƠN

Jane: Oh, from an early age, I knew I wanted to be an actress. I innocently thought I was going to be a glamorous film star. The reason I started to do comedy acts was that in the 1980s a lot of small provincial theatres closed down. In the past, girls would’ve come out of drama school and if they had a leaning towards comedy they’d join one of these small theatres and play a variety of comedy roles in all sorts of plays from Shakespeare to contemporary stuff. All of a sudden, with the demise of these theatres, rooms above pubs opened up and comedians started telling jokes and developing their acts there. It was cheap, one performer one microphone, and anyone could do it. In some ways, it’s a healthier performance art than acting, because with acting you’re at the mercy of everybody else deciding whether you can work or not. With standup comedy, you might only get paid peanutsbut nobody can stop you from just driving to a venue, often hundreds of miles in terrible weather, and going on stage. Int: But what is it when you’re actually on stage … [fade] Section 2

M

A talk from a member of the conservation Society about 'green cleaning' Good morning everyone. It's a pleasure to be here as a representative of the Conservation Society, to talk to you about "Green Cleaning", in other words about ways you can help to save the environment at the same time as saving money.

DẠ

Y

I'll start with saving money - as we're all interested in that, especially students who are living on a tight budget. Probably none of you has sat down and calculated how much you spend on cleaning products each year everything from dishwashing detergent, window cleaners and so on through to shampoos and conditioners for your hair, and then those disasters products to get stains out of carpets, or to rescue burnt saucepans. I can see some nods of agreement, even if you don't spend a lot of time on housework you'd end up spending quite a lot of money over a period of time, wouldn't you? We can save money on products

OF

FI CI A

L

and also use products which are cheap, biodegradable and harmless to the environment - these I will call 'green' products. Unfortunately most cleaning products on sale commercially are none of these, and many of our waterways and oceans are polluted with bleach, dioxins, phosphates and artificial colourings and perfumes. Also think how many plastic bottles each household throws away over a year -they'll still be around in land-fill when you are grandparents! So we often feel there's nothing we can do to make a difference, but we can. The actual 'recipes' are on handouts you can take at the end of the talk: The sorts of ingredients I'm referring to are things like bicarbonate of soda, eucalyptus oil, ammonia, vinegar, lemons, pure soap. Lastly many people find they're allergic to modem products, so for all you asthma sufferers keep listening. Nothing in these recipes should cause you any problems, an end to itching and wheezing!

ƠN

So let’s start with spills and stains. Soda water is wonderful as an immediate stain remover: mop up the excess spill don't rub but apply soda water immediately - it's great for tea coffee wine beer and milk - as is salt or bicarbonate of soda, which will absorb the stain - then vacuum when dry and shampoo if necessary.

QU Y

NH

While we are talking about disasters lets quickly look at some others that can be avoided. Bicarbonate of soda is wonderful for removing smells, especially in the fridge - an open box in the fridge will eliminate smells for up to three months. And those terrible burnt saucepans? Either sprinkle with our good friend bicarb again and leave it to stand, or cover with vinegar and a layer of cooking salt.

Y

M

Bring it to the boil and simmer for ten minutes, then wash when cool. Much cheaper than a new saucepan! Then there are heat rings on wooden furniture. Simply rub with a mixture of salt and olive oil, or for scratched furniture use olive oil and vinegar. Now let's look at general cleaning - first the floors. If your floor covering is made of slate, cork or ceramic tiles or lino it probably only needs a mop or a scrub with vinegar in a bucket of water. Carpets can be shampooed using a combination of pure soap washing soda, cloudy ammonia and some boiling water. You put a small amount of this mixture onto the mark on the carpet, rub with a cloth until it lathers and then wipe off the excess. A smelly carpet can be deodorized by sprinkling bicarbonate of soda on the surface, leaving overnight and vacuuming off the next day. Cleaning in the kitchen, bathroom and toilet is the next section....

DẠ

Section 3

Women are graduating from college at higher rates than ever before. Earning more than half of bachelor’s degrees last year. Companies are increasingly focused on the benefits of gender equality in the workplace. However, the situation for women in the workplace is not changing fast enough.

FI CI A

L

Women make up 45% at entry level position across the workforce; which decreases to 32% at management roles. At the top, women only make up seventeen percent of the Csuite. At the current rate of progress, it would be one hundred years before we see real equality in the C-suite. Contrary to popular belief, women are leaving organisations at the same rate as men. However, women are less eager to become a top executive citing stress and pressure as the key deterrents.

OF

Most women see barriers to advancement and feel they have fewer opportunities to advance due to their gender. While 74% of companies claim gender equality is a priority for their CEO, less than half of the employees believe it. And only one-third feel it is a top priority for their direct manager. More than 90% of women and men believe taking extended family leave will hurt their career. So while programs widely exist, participation is low.

ƠN

This inequality even extends to the home. 41 % of working women report doing more childcare than men, 30% report doing more chores but this does not need to be our reality while these facts are a wake-up call we can affect real change. Here are five key actions companies can take.

QU Y

NH

1)Track key metrics to understand the problem: you cannot change why you cannot see. 2)Demonstrate that gender equality is a top priority: employees must believe that their leaders want change. 3)Identify and interrupt gender bias : we all have unconscious biases to overcome. 4)Rethink work: redefine what successful careers look like. 5)Create a level playing field: make sure opportunities and advancement are equitable. It is up to all of us to take action, to speed the process of gender equality so everyone benefits, individuals, organisation and society. To learn more go to the womenintheworkplace.com. Section 4

M

Over the past few years as first lady, I have had the extraordinary privilege of traveling all across this country and everywhere I’ve gone and the people I've met and the stories I’ve heard, I have seen the very best of the American spirit.

DẠ

Y

See, our life before moving to Washington was, was filled with simple joys. Saturdays at soccer games, Sundays at grandma’s house, and a date night for Barack and me was either dinner or movie because as an exhausted mum I couldn’t stay awake for both. Even back then when Barack was a senator and a presidential candidate to me he was still the guy who picked me up for our dates in a car that was so wasted out that I could actually see the pavement going by in a hole in the passenger side door. He was the guy whose proudest possession was a coffee table he'd found in a dumpster.

L

Well today, after so many struggles and trials and moments that‘ve tested my husband in ways I never could have imagined, I have seen first-hand that being president doesn’t change who you are. No it reveals who you are.

FI CI A

When it comes to the health of our families, Barack refused to listen to all those folks to told him to leave health reforms for another day, another president. He didn’t care whether it was the easy thing to do politically, no that's not how he was raised. He cared that it was the right thing to do. When we were first married our combined monthly student loan bill was actually higher than our mortgage.

OF

Yeah!! We were so young, so in love, and so in debt.

ƠN

If we wanna give all of our children a foundation for their dreams and opportunities worthy of their promise. If we wanna give them that sense of limitless possibility, that belief that here in America there was always something better out there if you're willing to work for it. Then we must work like never before, and we must once again come together and stand together for the man we can trust to keep moving this great country forward.

DẠ

Y

M

QU Y

NH

My husband, our president, Thank you, God bless you, God bless America.

Barack

Obama.

ĐỀ XUẤT

FI CI A

ĐỀ THI MÔN: TIẾNG ANH 10 Thời gian: 180 phút (không kể thời gian giao đề) Ngày thi: (Thí sinh làm bài trực tiếp vào đề)

(Đề thi gồm 12 trang) Điểm

Giám khảo 1

Giám khảo 2

Số phách

OF

Bằng chữ

ƠN

Bằng số

L

KỲ THI HỌC SINH GIỎI CÁC TRƯỜNG THPT CHUYÊN KHU VỰC DUYÊN HẢI VÀ ĐỒNG BẰNG BẮC BỘ LẦN THỨ , NĂM 2023

A. LISTENING (50 points): HƯỚNG DẪN PHẦN THI NGHE HIỂU

NH

• Bài nghe gồm 4 phần; mỗi phần được nghe 2 lần, mỗi lần cách nhau 05 giây; mở đầu và kết thúc mỗi phần nghe có tín hiệu. Thí sinh có 20 giây để đọc mỗi phần câu hỏi. • Mở đầu và kết thúc bài nghe có tín hiệu nhạc. Thí sinh có 03 phút để hoàn chỉnh bài trước

QU Y

tín hiệu nhạc kết thúc bài nghe.

• Mọi hướng dẫn cho thí sinh (bằng tiếng Anh) đã có trong bài nghe Part 1. You will listen to a short conversation about easy life cleaning services. For questions 1-5, complete the form below by writing NO MORE THAN ONE WORD OR A NUMBER for each answer. Write your answers in the corresponding numbered boxes

M

provided on the answer sheet. (10 points) Easy Life Cleaning services

Basic cleaning package offered • Cleaning all surfaces • Cleaning the 1 __________ throughout the apartment

Y

• Cleaning shower, sinks, toilet etc.

DẠ

Additional services agreed

• Every week − Cleaning the 2 ______________ − Ironing clothes – 3 ______________ only Page 1 of 15

• Every month − Cleaning all the 4 _______________ from the inside

2.

3.

FI CI A

Your answers: 1.

L

Washing down the 5 ______________ 4.

5.

Part 2. You will listen to a recording about going vegan and its effect on the environment. For questions 6-10, decide whether the following statements are true (T) or false (F). Write your answers in the corresponding numbered boxes provided on the answer sheet. (10 points) 6. Vegans can consume eggs, milk and honey.

ƠN

OF

A. True B. False 7. A large-scale study said that eating more meat plays a pivotal role in saving the planet. A. True B. False 8. Raising livestock for meat results in deforestation. A. True B. False 9. 16% of greenhouse gasses comes from air released from cows’ digestive system. A. True B. False

10.

QU Y

NH

10. Beans and chickpeas can be good alternatives to meat in daily diet. A. True B. False Your answers: 6. 7. 8. 9.

Part 3. You will hear a radio interview in which two young journalists — called Angus Brown and Yolanda Zouche — are talking about their work. For questions 11-15, choose the answers which fit best according to what you hear. Write your answers in the corresponding

M

numbered boxes provided on the answer sheet. (10 points) 11. In Yolanda’s opinion, what is the most challenging aspect of her job? A. finding suitable images to accompany articles B. trying to locate interviewees C. expressing herself coherently within a tight word limit D. working to demanding time constraints 12. What is it about their work that Angus and Yolanda both enjoy? the variety of the projects they get involved in the challenge of reporting news effectively the opportunity to meet interesting people the appeal of searching out information

DẠ

Y

A. B. C. D.

13. In Angus’s opinion, the advantage of online newspapers over print versions is that they _______. Page 2 of 15

FI CI A

D. reach a far wider public. 14. What made getting a job in journalism so hard for Angus? A. the extent of competition for posts B. a low level of encouragement from others

L

A. are able to cover a greater range of topics. B. can keep up with events as they develop. C. allow interaction by readers.

OF

C. a lack of previous professional experience D. the difficulty of establishing useful contacts 15. Yolanda believes that the essential requirement for a journalist entering the profession is ______.

D. a mastery of interviewing techniques. Your answers: 11. 12. 13.

ƠN

A. an ability to write persuasively. B. a clear and logical mind. C. a resourceful and confident character.

14.

15.

NH

Part 4. You will listen to a recording about co-teaching in Herdorn Middle School. For questions 16-25, complete the summary by writing NO MORE THAN THREE WORDS in each gap. Write your answers in the corresponding numbered boxes provided on the answer

QU Y

sheet. (20 points) 16. Co-teaching is a model of ____________ in which two certified teachers work together in a classroom which contains students of varying levels of need and ability. 17. There is no _____________ between the roles of the teachers in the classroom. 18. Each team often comprises a content area _____________ and a special education teacher. 19. The first teacher loves talking about individual students through all _____________. 20. She also appreciates _____________ as a good way to help struggling students.

M

21. The second teacher can get the information or _____________ from another teacher, which is like crosspollinating. 22. Herdorn Middle School receives extra support to meet _____________. 23. Teachers are confident that the _____________ will be further narrowed.

Y

24. The method helps dispel _____________ before students start practicing. 25. By not _____________ students, they can learn together and build on each other’s strengths. Your answers:

DẠ

16. 17. 18. 19.

21. 22. 23. 24. Page 3 of 15

20.

25.

B. LEXICO - GRAMMAR (50 points)

DẠ

Y

M

QU Y

NH

ƠN

OF

FI CI A

L

Part 1. Choose one of the words marked A, B, C, or D which best completes each of the following sentences. Write your answers in the corresponding numbered boxes. (20 points) 1. This picture book, the few pages _____ are missing, is my favorite. A. for which B. of that C. to which D. of which 2. He tried to run away from the police and was charged with _______ arrest. A. suffering B. standing C. tolerating D. resisting 3. Your decision will ______ a great strain on our relationship. A. impose B. propose C. expose D. suppose 4. In _______ to your letter of 30 May, I am writing to accept your offer of £3,575 in compensation. A. connection B. touch C. answer D. middle 5. Scuba diving? Oh no, count me _______ - I hate being under water! A. out B. in C. off D. on 6. The study showed that one in twelve women is_______ to develop breast cancer. A. likely B. alike C. unlike D. like 7. _______ of all modern domestic poultry is the red jungle fowl is widely believed. A. The ancestor B. The ancestor is C. How the ancestor D. That the ancestor 8. Art critics do not all agree on what _______ a painting great. A. qualities to make B. are the qualities for making C. qualities make D. do the qualities that make 9. Farmers supply crops with phosphorus in areas ________ have removed it from the soil. A. because of long years of cultivation B. where long years of cultivation C. with long years of cultivation D. by long years of cultivation 10. By the time his daughter graduates from the college, Mr. Brown ______. A. retired B. will have retired C. will be retiring D. has tired 11. Had it not been for the intolerable heat in the hall, they _____ much longer. A. will stay B. would stay C. would be staying D.would have stayed 12. Having been served dinner, _______. A. the problem was discussed by the members of the committee. B. the committee members discussed the problem. C. it was discussed by the committee members the problem. D. a discussion of the problem was made by the members of the committee. 13. Fast internet service in rural areas provided a new ________ for prosperity. A. convenience B. opportunity C. possibility D. chance 14. I've been on the ________ all day, and I'm really tired. A. go B. do C. rope D. take 15. Why don’t we eat on the porch for a ________? A. similarity B. song C. difference D. change 16. The winning entry will be the first correct answer drawn at ________. A. hand B. random C. least D. last 17. It is high time for the critics _______ their minds to a new approach. A. to open B. had opened C. opened D. to have opened 18. It’s amazing how Jenny act as though she and Darren_______ serious problems at the moment. A. hadn’t been having B. hadn’t had C. weren’t having D. aren’t having 19. But the defence argues that Mr. Kwoyelo is a victim, claiming he was abducted as a child while

Page 4 of 15

L

walking to school and forced to follow ________ for fear of being shot. A. rules B. guidance C. directions D. orders 20. There was widespread outrage when it was discovered that a known pedophile had been given a job at the school. “You don’t let the ______ guard the henhouse” said one of the protesters. A. fox B. cat C. duck D. fish

5.

9.

13.

2.

6.

10.

14.

3.

7.

11.

15.

4.

8.

12.

16.

17. 18. 19. 20.

OF

1.

FI CI A

Your answers:

Part 2. Complete each sentence with one suitable particle or preposition. Write your answers in the box provided. (10 points) 1. Could you look ____ these old newspapers for articles about the environment? 2. Can you hold ____ till I’ve finished talking the manager? Then I’ll be right with you.

ƠN

3. She hasn’t told us the whole story. I’m sure she’s keeping something ____. 4. After the first year, I couldn’t cope with university, so I decided to drop ____. 5. Rubber boots are impervious ____ water.

NH

6. It wouldn’t be compatible ____ the public safety to let animals off with a slight punishment. 7. Leisure for study is congenial ____ his tastes.

8. Geology is a science I am not conversant ____.

9. He may be slow at his work, but he is very quick ____ the uptake.

Your answers: 1. 6.

2. 7.

QU Y

10. The young couple lay ____ money for their old age.

3. 8.

4. 9.

5. 10.

M

Part 3. Write the correct form of the words given in the brackets. Write your answers in the spaces provided below. (10 points)

ESTIMATE

2. There was ice on the pavement which made it very difficult to walk as it was so

SLIP

1. I was late because I _____________ how much time I will need. _____________.

Y

3. Despite the star-studded cast, the film was only _____________ successful. 4. How can you _____________ the fact that some people live in mansions while

PART JUST

DẠ

others live in slums? 5. My brother lives in an attractive _____________ part of Paris.

RESIDENT

6. The car in front was going very slowly, so John__________________ it.

TAKE

7. I _____________ met an old friend last week.

EXPECT

Page 5 of 15

BREAK

9. The lovers stood, hand in hand, gazing at the _____________ sky.

STAR

10. There was a heavy _____________ yesterday afternoon which completely

POUR

L

8. There’s been yet another _____________ of cholera in Delhi.

Your answers: 1. 6.

2. 7.

3. 8.

FI CI A

ruined the church Garden Party.

4. 9.

5. 10.

C. READING (60 points)

M

QU Y

NH

ƠN

OF

Part 1. For questions 1–10, read the text below and decide which answer (A, B, C or D) best fits each gap. FOOTBALL AS AN ART FORM When filmmakers Douglas Gordon and Phillipe Parreno set out to make an art house movie about the legendary French footballer Zinedine Zidane, they chose to film just one match between Real Madrid, the club for which he was playing at the (1) ______, and their great rivals Villareal. But instead of following the progress of the match, the ninety-minute film would show something that had not been seen before; the (2) ______ detailed movements of one man during an entire top-level football match. They hoped that the audience would disengage from the match itself, and focus on this portrait of greatness. Every (3) ______ gesture would be captured and they would see all of the player's grace, athleticism and competitiveness in great detail. The (4) ______ film is a fascinating work. Those who are not regular watchers of football will be astonished at how (5) ______ Zidane becomes actively involved in the game. For much of the ninety minutes he moves around the field relatively slowly; saying nothing, expressing even less, and only occasionally (6) ______ into a lethargic jog. And then the ball arrives at his feet, and there is a flurry of bewildering activity. The cameras (and there are seventeen of them (7) ______ on him) struggle to keep up. The defenders don't (8) ______ a chance. In a few touches, a couple of checks and feints, Zidane has (9) ______ them all behind. He crosses from the tightest of (10) ______ and his team-mate is left with the simplest of headers to score a goal. 1. A. point B. moment C. time D. occasion 2. A. clear B. steady C. precise D. slow 3. A. one B. single C. lone D. sole 4. A. following B. resulting C. concluding D. arising 5. A. partly B. scarcely C. rarely D. hardly 6. A. breaking B. changing C. opening D. starting 7. A. trained B. looking C. pointed D. staring 8. A. gain B. hold C. stand D. earn 9. A. missed B. left C. lost D. dropped 10. A. places B. positions C. areas D. angles

DẠ

Y

Your answers: 1. 2. 3. 4. 5. 6. 7. 8. 9. 10. Part 2. Read the text and think of the word which best fits each gap. Use only ONE word in each gap. Getting ready for Mars The Mars 500 project (1) ______ an experiment that simulated a return mission to Mars. Spending 18 months in a sealed facility in Moscow (2) ______ access to natural light or fresh air, six men were monitored as they attended (3) ______ their daily duties. A study into (4) ______ each of them coped with the psychological and physical constraints of the mission has found that there were wide differences in their wake-sleep patterns. For example, (5) ______ most of the crew began to sleep for

Page 6 of 15

Your answers: 1. 6.

2. 7.

3. 8.

FI CI A

L

longer periods as the mission progressed and boredom set in, one individual slept progressively less, resulting (6) ______ him becoming chronically sleep-deprived towards the end of the (7) ______. Identifying bad sleepers could be important on a real Mars mission, during (8) ______ people are required to be constantly alert even when days are tediously similar. Researchers warn that for any astronaut heading to Mars, exciting as the trip might initially seem, (9) ______ could be problems with stress brought on by the monotony of routine. However, they also report that (10) ______ some personal tensions between crew members, there was overall harmony within the group.

4. 9.

5. 10.

NH

ƠN

OF

Part 3. You are going to read an article about the history of the electric guitar. For questions 1-10, choose the answer (A, B, C or D) which you think fits best according to the text. SUPER HUMANS Sit down with an anthropologist to talk about the nature of humans, and you are likely to hear this pearl of wisdom: 'Well, you have to remember that 99 percent of human history was spent on the open savanna in small bands of hunter-gatherers.' It's a classic scientific cliché, and it's true. Indeed, those millions of ancestral years produced many of our hallmark traits — upright walking and big brains, for instance. Of course, those useful evolutionary innovations come at a price: aching backs from our bipedal stance and existential despair from our large, selfcontemplative cerebral cortex.

QU Y

Compounding the challenges of those trade-offs, the world we have invented is dramatically different from the one to which our bodies and minds are adapted. Have your dinner delivered to you instead of chasing it down on foot; log in to Facebook to interact with your nearest and dearest instead of spending most of the day with them. But this is where the utility of the anthropologist's cliche for explaining the human condition ends.

M

The reason for this mismatch between the setting we evolved to live in and the situations we encounter in our modern era derives from another defining characteristic of our kind, arguably the most important one: our impulse to push beyond the limitations evolution imposed on us by developing tools to make us faster, smarter and longer-lived. Science is one such tool — an invention that requires us to break out of our Stone Age seeing-is-believing mindset so that we can clearly see the next hurdle we have to overcome, be it a pandemic flu or climate change. You could call it the ultimate expression of humanity's singular drive to aspire to be better than we are.

DẠ

Y

To understand how natural selection moulded us into the unique primates we have become, let us return to the ancestral savanna. There the sun was hotter and nutritious plant foods were scarcer. In response, our predecessors lost their hair and their molars dwindled as they abandoned a tough vegetarian diet for one focused in part on meat from grassland grazers. Meanwhile, the selective demands of food scarcities sculpted our distant forebears into having a body that was extremely thrifty and good at storing calories. Now, having inherited that same metabolism, we hunt and gather burgers as diabetes becomes a worldwide scourge. Or consider Page 7 of 15

L

how our immune systems evolved in a world where one hardly ever encountered someone carrying a novel pathogen. Today, if you sneeze near someone in an airport, your rhinovirus could be set free 12 time zones away by the next day.

FI CI A

As regards behavior, our abilities abound. We can follow extraordinarily complex scenarios of social interaction and figure out if a social contract has been violated. And we are peerless when it comes to facial recognition: we even have an area of the cortex in the fusiform gyrus that specializes in this activity.

ƠN

OF

The selective advantages of evolving a highly social brain are obvious. It paved the way for us to finetune our capabilities for reading one another's mental states, to excel at social manipulation and to deceive and attract mates and supporters. Among Americans, the extent of social intelligence in youth is a better predictor of adult success in the occupational world than are academic scores. Indeed, when it comes to social intelligence in primates, humans reign supreme. The social brain hypothesis of primate evolution is built on the fact that across primate species the percentage of the brain devoted to the neocortex correlates with the average size of the social group of that species. This correlation is more dramatic in humans than in any other primate species.

QU Y

NH

The fact that we have created this world proves a point — namely, that it is in our nature to be unconstrained by our nature. Science is one of the strangest, newest domains where we challenge our hominid limits. It also tests our sense of what is the norm, what counts as better than well and it challenges our sense of who we are. Thanks to science, human life expectancy keeps extending, our average height increases, our intelligence test scores improve and we eventually break every world record. But when it comes to humans becoming, on average, smarter, taller and better at athletics, there is a problem: Who cares about the average? As individuals, we want to be better than other individuals. Our brain is invidious, comparative and more interested in contrasts, a state that begins with sensory systems that do not normally tell us about the quality of a stimulus but instead about the quality relative to the stimuli around it.

DẠ

Y

M

1. According to the writer, the anthropological cliché to explain the nature of mankind ______. A. needs some slight modifications B. requires little analysis C. should be considered paradoxical D. is limited in scope 2. Humankind will only be able to use science to progress if ______. A. ethical considerations are ignored B. we discard an outdated approach to acquiring knowledge C. our drive to eliminate barriers continues D. the philosophy we adopt can be widely understood 3. Our ancient ancestors lived in a world where ______. A. the necessity to hunt for food led to good health B. it was vitally important to have a balanced diet Page 8 of 15

DẠ

Y

M

QU Y

NH

ƠN

OF

FI CI A

L

C. isolation allowed them to develop immunity from disease D. their restricted movement protected them from illness 4. The word “peerless” in paragraph 5 is closest in meaning to ______. A. friendless B. unsurpassed C. uncompetitive D. flawless 5. Having a highly social brain ______. A. allows us to create groups with more members B. prevents us from being misunderstood C. causes us to be more devious D. helps us to read other people's minds 6. The way we tend to think ______. A. forces us to overlook our shortcomings B. has enhanced our understanding of sense perception C. distorts our perception of the notion of average D. makes us less likely to be concerned with absolutes 7. The word “invidious” in paragraph 7 mostly means ______. A. prejudiced B. preordained C. unfathomable D. attitudinal 8. Which of the following square brackets [A], [B], [C], or [D] best indicates where in the paragraph the sentence "We are no strangers to going out of bounds." can be inserted? [A] The fact that we have created this world proves a point — namely, that it is in our nature to be unconstrained by our nature. [B] Science is one of the strangest, newest domains where we challenge our hominid limits. It also tests our sense of what is the norm, what counts as better than well and it challenges our sense of who we are. [C] Thanks to science, human life expectancy keeps extending, our average height increases, our intelligence test scores improve and we eventually break every world record. [D] But when it comes to humans becoming, on average smarter, taller and better at athletics, there is a problem: Who cares about the average? As individuals, we want to be better than other individuals. Our brain is invidious, comparative and more interested in contrasts, a state that begins with sensory systems that do not normally tell us about the quality of a stimulus but instead about the quality relative to the stimuli around it. A. [A] B. [B] C. [C] D. [D] 9. It can be inferred from the passage that A. there are no limits to human capabilities B. we will be able to adapt to harsh environments C. humankind's evolutionary path will not be smooth D. our knowledge of the past is crucial to our future 10. Which of the following is the main idea of the passage? A. Social intelligence enables Americans to be both academically and professionally successful.

Page 9 of 15

FI CI A

L

B. Science helps prolong human life and improve human intelligence to break all world records. C. Our evolutionary limits can be exceeded and that's what sets us apart from other species. D. A highly evolved social brain paved the way for humans to be able to read and distort others' thinking. Your answers: 1. 2. 3. 4. 5. 6. 7. 8. 9. 10.

DẠ

Y

M

QU Y

NH

ƠN

OF

Part 4. Read the following passage and do the tasks that follow. The strange journey that made Great Britain As evidenced by its geological make-up, the island of Great Britain has a rich and varied prehistory. First of all, to clear up a common misconception, Great Britain refers, to the largest island of the British Isles, (Great in this case meaning Big). It is made up of England, Scotland and Wales, and is the eighth largest island in the world. There are between around 4,000 and 6,289 other islands in the group (depending on the size of rock we are talking about and whether you can count it as an island if it is only one at high tide). Of these, around 200 are inhabited. In order to grasp some of the more difficult ideas behind the prehistory of Great Britain, it helps to look at the Earth's geological past, starting with its formation 4.5 billion years ago. It is currently thought that the Earth and the planets were formed from dust left over from when the Sun was created. What is significant is that the Earth in its infancy was in a largely molten state, with constant volcanic activity and regular collisions with other bodies (one of which is thought to have created the Moon). Although scientific opinion varies on many of these points, the generally accepted sequence of events is as follows: the temperature began to fall, forming the Earth's crust. At the same time, water appeared in the atmosphere. The volcanic activity released gases, creating an atmosphere, and the water condensed, forming the oceans. Molten material solidified, forming continents which broke apart and re-formed many times over the next couple of billion years. These continents migrated around the surface of the Earth and it is in the study of this continental drift that we can trace Great Britain's past. At three stages in the Earth's geological past, supercontinents were formed, so that there was only one major land mass, while most of the rest of the Earth was covered with water. These supercontinents have been named Rodinia, Pannotia and Pangaea. The earliest of these, Rodinia, broke apart 750 million years ago, and 200 million years later, the land masses joined together again to form the second supercontinent, Pannotia. Around 500 million years ago, in the period known as the Cambrian, Great Britain was largely underwater in the southern hemisphere, about the same distance from the South Pole as New Zealand is today. Scotland was attached to the plate with North America, while England and Wales were attached to Scandinavia. Around the time of the Silurian period, the two plates collided, bringing Scotland into contact with England. The collision caused what we call the Caledonian orogeny? - what are now the Scottish Highlands. In the Devonian period of about 400 million years ago, the continuing rise of the Caledonian orogeny caused the majority of Great Britain to be above sea level. Africa and South America then collided with North America and Europe. Around 100 million years later, during what we Page 10 of 15

DẠ

Y

M

QU Y

NH

ƠN

OF

FI CI A

L

call the Carboniferous period, Great Britain was in the equatorial forest belt of Pangaea, the last supercontinent. It was the vegetation from this forest that created the coal mined in Great Britain for centuries. Then, around 200 million years ago, Britain migrated to the northern desert belt, which explains why sandstone is so prevalent in the bedrock of Great Britain. During the Jurassic and Cretaceous periods, between 190 and 65 million years ago, Pangaea broke apart and, due to rifting, Great Britain was flooded by the newly created Atlantic Ocean. Evidence of this can be found in the chalk under the soil, mainly in southern England. The chalk is made up of trillions of tiny sea creatures that died and formed a sediment at the bottom of the sea. After sea levels fell and rose several times, the continents we know today were established and the island of Great Britain was more or less where it is now. Except that it was not an island. Throughout all of this movement, it was part of a larger continent. It was as recently as about 9,000 years ago that it became an island, when the end of the last ice age made sea levels rise yet again to create the English Channel and cut Great Britain off from continental Europe. The highlands and lowlands, the chalk cliffs, the layers of sandstone, the seams of salt, the precious metals, and the fossil fuels; all of these provide clues to the many, many changes that Great Britain went through in geological history. Questions 1-5 Complete each sentence with the correct ending A-I below. 1. Currently, the island of Great Britain 2. The supercontinent of Rodinia 3. The Scottish Highlands mountain range 4. The coal under the ground in Great Britain 5. Apart from fossil fuels, the land under Great Britain A. broke apart and later re-formed as Pannotia. B. caused a rise in sea levels. C. dates back to a time when it was in an equatorial forest. D. led to the cooling of the Earth. E. contains salt, sandstone and chalk. F. is surrounded by several thousand smaller islands. G. used to be named Pangaea. H. was caused by flooding and rifting. I. is the result of two land masses colliding. Questions 6-10 Complete the timeline below. Choose NO MORE THAN THREE WORDS from the text for each answer. Around 4.5 billion years ago: The Earth was formed, remaining in a molten form with a constantly changing surface due to frequent impacts and 6. ______________________. The Earth's crust was formed due to the cooling of the planet.

Oceans were formed when water from the atmosphere 7. ______________________. The first continents were formed when molten material solidified. Page 11 of 15

FI CI A

Around 300 million years ago: Great Britain was in a(n) 9. ______________________.

L

Around 500 million years ago: 8. ______________________ were united at the time when the Scottish Highlands were formed.

Around 200 million years ago: The presence of sandstone today reveals that Britain was in the northern desert belt.

ƠN

OF

Around 100 million years ago: The chalk in Britain today comes from a build-up of dead sea creatures from the time when the area was 10. ______________________ after the Atlantic Ocean was formed. Your answers: 1. 2. 3. 4. 5. 6. 7. 8. 9. 10.

M

QU Y

NH

D. WRITING (50 points) Part 1. The table below shows information about age, average income per person and population below poverty line in three states in the USA. (15 points) Summarise the information by selecting and reporting the main features, and make comparisons where relevant.

…………………………………………………………………………………………………… ……………………………………………………………………………………………………

Y

…………………………………………………………………………………………………… ……………………………………………………………………………………………………

DẠ

…………………………………………………………………………………………………… ……………………………………………………………………………………………………

Page 12 of 15

…………………………………………………………………………………………………… ……………………………………………………………………………………………………

L

……………………………………………………………………………………………………

FI CI A

…………………………………………………………………………………………………… …………………………………………………………………………………………………… …………………………………………………………………………………………………… …………………………………………………………………………………………………… ……………………………………………………………………………………………………

OF

…………………………………………………………………………………………………… …………………………………………………………………………………………………… …………………………………………………………………………………………………… ……………………………………………………………………………………………………

ƠN

…………………………………………………………………………………………………… …………………………………………………………………………………………………… ……………………………………………………………………………………………………

NH

…………………………………………………………………………………………………… …………………………………………………………………………………………………… ……………………………………………………………………………………………………

QU Y

…………………………………………………………………………………………………… …………………………………………………………………………………………………… …………………………………………………………………………………………………… …………………………………………………………………………………………………… ……………………………………………………………………………………………………

M

……………………………………………………………………………………………………

…………………………………………………………………………………………………… Part 2. Write an essay of about 250 words to express your opinion on the following topic. (35 points)

DẠ

Y

Some people think that the main purpose of school is to turn children into good citizens and workers, rather than benefit them as individuals. To what extent do you agree or disagree? …………………………………………………………………………………………………… …………………………………………………………………………………………………… …………………………………………………………………………………………………… Page 13 of 15

…………………………………………………………………………………………………… ……………………………………………………………………………………………………

L

……………………………………………………………………………………………………

FI CI A

…………………………………………………………………………………………………… …………………………………………………………………………………………………… …………………………………………………………………………………………………… …………………………………………………………………………………………………… ……………………………………………………………………………………………………

OF

…………………………………………………………………………………………………… …………………………………………………………………………………………………… …………………………………………………………………………………………………… ……………………………………………………………………………………………………

ƠN

…………………………………………………………………………………………………… …………………………………………………………………………………………………… ……………………………………………………………………………………………………

NH

…………………………………………………………………………………………………… …………………………………………………………………………………………………… ……………………………………………………………………………………………………

QU Y

…………………………………………………………………………………………………… …………………………………………………………………………………………………… …………………………………………………………………………………………………… …………………………………………………………………………………………………… ……………………………………………………………………………………………………

M

…………………………………………………………………………………………………… ……………………………………………………………………………………………………

…………………………………………………………………………………………………… …………………………………………………………………………………………………… ……………………………………………………………………………………………………

Y

……………………………………………………………………………………………………

DẠ

…………………………………………………………………………………………………… …………………………………………………………………………………………………… ……………………………………………………………………………………………………

Page 14 of 15

…………………………………………………………………………………………………… ……………………………………………………………………………………………………

L

……………………………………………………………………………………………………

FI CI A

…………………………………………………………………………………………………… …………………………………………………………………………………………………… …………………………………………………………………………………………………… …………………………………………………………………………………………………… ……………………………………………………………………………………………………

OF

…………………………………………………………………………………………………… …………………………………………………………………………………………………… …………………………………………………………………………………………………… ……………………………………………………………………………………………………

ƠN

…………………………………………………………………………………………………… …………………………………………………………………………………………………… ……………………………………………………………………………………………………

NH

…………………………………………………………………………………………………… ……………………………………………………………………………………………………

DẠ

Y

M

QU Y

-------------- THE END -------------(Thí sinh không được sử dụng tài liệu. Cán bộ coi thi không giải thích gì thêm)

Page 15 of 15

IA L

KỲ THI HỌC SINH GIỎI CÁC TRƯỜNG THPT CHUYÊN KHU VỰC DUYÊN HẢI VÀ ĐỒNG BẰNG BẮC BỘ LẦN THỨ , NĂM 2023

IC

HƯỚNG DẪN CHẤM MÔN: TIẾNG ANH 10 Thời gian: 180 phút (không kể thời gian giao đề) Ngày thi: (Đáp án gồm 07 trang)

OF F

A. LISTENING (50 points): LISTENING Part 1: 2 points/ correct answer

1. FLOOR(S)

3. SHIRTS

4. WINDOWS

5. BALCONY

Tapescript Hello, Easy Life Cleaning Services, Jacinta speaking. Oh hello. I'm looking for a cleaning service for my apartment - do you do domestic cleaning?

Sure. Well, it’s just a one-bedroom flat. Do you have a basic cleaning package? Yes. For a one-bedroom flat we’re probably looking at about two hours for a clean. So we’d do a thorough clean of all surfaces in each room, and polish them where necessary. Does your apartment have carpets? CLIENT: JACINTA:

NH

JACINTA: CLIENT: JACINTA:

ƠN

JACINTA: CLIENT:

2. FRIDGE

No, I don’t have any, but (1) the floor would need cleaning. Of course - we’d do that in every room. And we’d do a thorough clean of the

Y

kitchen and bathroom. OK.

JACINTA: so for CLIENT: JACINTA: CLIENT: do? JACINTA: CLIENT:

Then we have some additional services which you can request if you want example, we can clean your oven for you every week. Actually, I hardly ever use that, but (2) can vou do the fridge? Sure. Would you like that done every week? Yes, definitely. And would ironing clothes be an additional service you can

JACINTA: CLIENT: JACINTA: every

That’s fine. And we could also clean your microwave if you want. No, I wipe that out pretty regularly so there's no need for that. We also offer additional services that you might want a bit less often, say

KÈ M

QU

CLIENT:

Yes, of course. It wouldn’t be much, (3) just my shirts for work that week.

DẠ

Y

month. So for example, if (4) the inside of your windows need cleaning, we could do that. CLIENT: Yes, that'd be good. I’m on the fifteenth floor, so the outside gets done regularly by specialists, but the inside does get a bit grubby. JACINTA:

And we could arrange for your curtains to get cleaned if necessary.

CLIENT: balcony?

No, they’re OK. But (5) would vou be able to do something about the It’s quite small and I don't use it much, but it could do with a wash

every month or so. JACINTA:

Yes, we can get the pressure washer onto that.

6. FALSE

7. FALSE

8. TRUE

9. TRUE

Tapescript Johnah: Ruby, happy World Vegan Day. I made you this cake.

IA L

Part 2: 2 points/ correct answer

10. TRUE

OF F

IC

Ruby: Thanks Johnah. It's definitely vegan. Vegans can be hard to cook for, like vegetarians. They don't eat meat. (6) But they also rule out other animal products like eggs, milk, and sometimes even honey. There are different reasons people choose to be vegan, like animal welfare or as a way to eat healthier. But there's another reason that's been

getting a lot of attention recently. (7) Last month, a big study came out looking into the effect our food has on the environment. And it found that eating less meat is one of the most important things we can do to help the planet. You see, a huge amount of the Earth's resources goes into (8) raising livestock for meat. In some places, forests are cleared to make space for animals and to grow their food. They also use a lot of water,

ƠN

and these ones in particular create a lot of carbon emissions. (9) Cow burps and farts account for about 16% of global greenhouse gas emissions. The study says to prevent permanent damage to our planet, the average world citizen needs to eat 75% less beef, 90% less pork, and 50% fewer eggs. Of course, not everyone's going to go vegan or give up

Part 3. 2 points/ correct answer

11. D

12. A

NH

eating meat completely. (10) But swapping your beef for beans or your chicken for chickpeas every so often might not be such a bad idea.

13. B

14. A

15. C

Y

Tapescript

KÈ M

QU

Interviewer: Today I’m talking to two young journalists - Angus Brown, a news reporter on a national daily, and Yolanda Zouche, a features writer on a London evening paper. You’re both not only successful in your careers, but enthusiastic about them too. What would you say is the most challenging aspect of your work. Yolanda? Yolanda: I’m tempted to say nothing really — I like it all, some things more, some less. Our features are a mix of things that have been thought of and researched and written in advance, and more urgent, topical pieces with a quick turnaround. I’m sometimes sent out on a story that’s needed for the next day. (11) It’s pretty scary when you know you’ve got just a few hours and it can involve finding people whose addresses you don’t know. That’s the same with any story, of course. You’ll have to think up ideas for pictures to go with it and, write your piece to a fixed word count - but you soon find yourself doing

DẠ

Y

this automatically. Interviewer: And what do you both enjoy about your work? Yolanda: Well, I suppose I’m quite a nosy kind of person, so I love digging out stories that haven’t been reported — I’ll go from interviewing a singer about a forthcoming tour to investigating some crime, all in one day perhaps. I’d be bored otherwise, I think, and then it’s all got to be presented in a way that will make people want to read it. Interviewer: And you, Angus? Angus: There’s no way I could describe ‘my typical working day’ that’s the sort of question people often ask. And that’s really why I enjoy it so much I suppose (12) so many fascinating people to talk to, and so much to learn.

IC

IA L

Interviewer: Like several other papers nowadays, Angus, yours has an online version. What do you think is the greatest significance of the change to digital? Angus: It’s completely altered the way we think about the news. Things move so very quickly, and we really do need to stay receptive to all the opportunities the medium has opened up. I think maybe more people are better informed these days. We’ve certainly become (13) a rolling news operation I can now file a story as soon as it breaks, early

and then update digitally as I find out more. And if I get anything wrong, people are all

OF F

too quick to point this out on social media. Interviewer: Finding a job is not easy for anyone - what was it like for you Angus? Angus: It certainly wasn’t easy. People continually told me that print journalism was dead,

ƠN

and there was no money in it, (14) but in fact I had to beat 1,800 applicants to get on the graduate scheme. Before that I’d been on a couple of temporary work experience placements with a local paper - you can do these to get a taster of the work, but there’s no salary ... I managed financially because I was doing some part-time teaching at the same time. You really need some sort of support network of people with influence too luckily the work experience provided that. Interviewer: Tell us what qualities you think a would-be journalist needs, Yolanda. Yolanda: There’s no straightforward answer to that - I’ve got an English degree and Angus

NH

is a historian, I believe ... that’s his academic background. There are plenty of good courses around, and eventually you’ll have to get to grips with some of the technical stuff - like media law, and so on. (15) But remember no one is ever going to employ a shy retiring type with no ideas. Being able to write clearly and quickly, and think through a tangle of information is obviously helpful but these are things you can develop with practice.

QU

PART 4 16. instructional delivery 17. clear delineation 18. general educator 19. phases

Y

Part 4: 1 point/ correct answer

KÈ M

20. flexible grouping 21. lesson plan ideas 22. student achievement benchmarks 23. achievement gap 24. misconceptions 25. separating Tape script

DẠ

Y

On first glance, this may look like a traditional classroom with one teacher in charge, but on closer inspection, it appears that there are two teachers leading instructions. Well, that's because there are. This is Herndon middle school in this class is an example of co teaching. (16) Co teaching is a model of instructional delivery in which there are two certified teachers in a classroom which contains students of varying levels of need and ability. The great thing about Lisa and Jennifer is when you walk into their classroom, (17) there's no clear delineation between this is the general ed teacher in the classroom, this is the special ed teacher in the classroom. There are two teachers in that classroom and they both have shared responsibility for every student. We really are making strides at Herndon to moving away from that mine and their perception of students and who belongs to who. If

IC

IA L

they're a Herndon middle school student, they belong to every teacher in this building. And Lisa and Jennifer exemplify that. (18) Herndon has more than 20 co teaching teams, which usually pair a content area general educator with a special education teacher. These teams allow teachers to share expertise and actively engage, challenge and motivate students. Most of my teaching career has been in a traditional model where I was a teacher for the entire classroom. (19) And what I love about teen teaching is through all phases, we're talking about individual students and what their needs are. And I love working

OF F

with a special education teacher who has the background for (20) ways to help students that are struggling. So we plan there and then in the classroom with flexible grouping with students that need different things at the same time. We're able to do that together

ƠN

in the classroom. So I feel much more effective through the entire process. And I'm a firstyear teacher and it's made the transition really easy. And I love how I get the English expertise from Miss Merrick and then I can kind of bring my special ed learning strategies expertise to that as well. So it's very helpful because I also co teach with another teacher and I share a room with another teacher. (21) I also get information from them or lesson plan ideas from them and I can bring them to her and vice versa. So we're kind of like crosspollinating, which is really great. As part of FCPS’s commitment to closing the achievement gap, Herndon is one of 30 schools designated as priority schools by the school

NH

system. (22) Herndon middle gets additional support in order to meet student

QU

Y

achievement benchmarks. The staff at Herndon believes that co teaching is an integral part of student success. (23) I absolutely think this will help close the achievement gap because we are able to focus more on individual student needs and with more eyes and more ideas, looking at what students need and then being able to work with them, it helps tremendously. And I think the students really like it as well. Because they have two different approaches. They see us cooperating. And I think that lends to a feeling that we're all in this together. We're all trying to help everyone achieve the most that they can. I definitely think using co teaching helps close the achievement gap because we have two minds thinking of lessons, two minds thinking about how we can teach the students in the most effective manner. (24) It decreases wait time, transition time, and really allows us as teachers to

KÈ M

get to the students and see if there's any misconceptions at all before they start practicing. And the reason I think that is because when you have two teachers who are working together to truly build and deliver dynamic instruction, that you can meet those individual needs of every student in the classroom. (25) And I also think that by mixing students, by not creating 21st century segregation, by here are our achieving kids, here are non-achieving kids, and separating them, that those students can then learn

DẠ

Y

together and build on each other's strengths. That's a good.

B. LEXICO - GRAMMAR (50 points)

FI CI A

L

Part 1. Choose the best option A, B, C, or D to complete the following sentences and write your answers in the corresponding numbered boxes. (20 points) 1. C 5. A 9. B 13. B 17. A 2. D 6. A 10. B 14. A 18. C 3. A 7. D 11. D 15. B 19. D 4. C 8. C 12. B 16. B 20. A

NH

ƠN

OF

Part 2. Complete each sentence with one suitable particle or preposition. Write your answers in the box provided. (10 points) 1. through 2. on 3. back 4. out 5. to 6. with 7. to 8. with 9. on 10. aside Part 3. Write the correct form of the words given in the brackets. Write your answers in the spaces provided below. (10 points) 1. underestimated 6. overtook 7. unexpectedly 2. slippery 3. partially 8. outbreak 4. justify 9. starry 5. residential 10. downpour

1. C 6. A

2. C 7. A

QU Y

C. READING (60 points) Part 1. Read the following passage and decide which answer (A, B, C, or D) best fits each gap. Write your answers in corresponding numbered boxes. (15 points) – (1pt for each correct answer) 3. B 8. C

4. C 9. B

5. C 10. D

Part 2. Read the text below and think of the word which best fits each space. Use only ONE word in each space. Write your answers in the corresponding numbered boxes. (15 points) – (1.5 pts for each correct answer) 2. without 7. mission

M

1. was 6. in

3. to 8. which

4. how 9. there

5. while/whilst/whereas/although/though 10. Despite

DẠ

Y

Part 3. Read the passage and choose the best option A, B, C, or D to answer the questions. Write your answers in the corresponding numbered boxes. (15 points) – (1pt for each correct answer) 1. D 2. B 3. D 4. B 5. A 6. D 7. A 8. B 9. C 10. C Part 4. Read the text and do the following tasks. (15 points) – (1.5pts for each correct answer) 1. F 2. A 3. I 4. C 5. E 6. volcanic activity 7. condensed 8. Scotland and England 9. equatorial forest (belt) 10. flooded Page 5 of 7

D. WRITING (50 points)

L

Part 1: Graph writing (15 points)

ƠN

OF

FI CI A

Contents (1.0 point) - The report MUST cover the following points: •Introduce the chart (0.2 point) and state the striking features (0.2 point) •Describe main features with relevant data from the charts and make relevant comparisons (0.6 point) - The report MUST NOT contain personal opinions. (A penalty of 1 point to 2 points will be given to personal opinions found in the answer.) Language use (0.5 points) The report should: - demonstrate a wide variety of lexical and grammatical structures, - have correct use of words (verb tenses, word forms, voice,…); and mechanics (spelling, punctuations,...). MODEL REPORT The table provides a comparison of three American states in terms of average income per capita, age distribution, and the proportion of population living under the poverty line.

NH

Overall, Utal had the youngest population, whereas Florida had the highest percentage of elderly citizens. Furthermore, even though California turned out to have a higher income per capita, it registered the highest percentage of population living below the line of poverty.

QU Y

In terms of age-related statistics, 28% of Utal’s population was under the age of 18, compared to 17% in California and 16% in Florida. Conversely, Florida had the highest percentage of citizens above the age of 60, at 24%, while California and Utal had only 13% and 8% respectively.

M

With regards to the economic status, the average income per person amounted up to 23 000 and 22 000 in California and Florida respectively, while the average inhabitant in Utal was earning up to 17 000 per month. However, despite having the highest income per capita, California also registered 16% of population under the poverty line, which was significantly higher than 12% in Florida and 9% in Utal. Part 2: Essay writing (35 points)

DẠ

Y

The mark given to this part is based on the following criteria: 1. Content: (35% of total mark) a. Providing all main ideas and details as required b. Communicating intentions sufficiently and effectively 2. Organization & Presentation: (30% of total mark) a. Ideas are well organized and presented with coherence, cohesion, and clarity b. The composition is well - structured

Page 6 of 7

FI CI A

L

3. Language: (30% of total mark) a. Demonstration of a variety of vocabulary and structures appropriate to the level of English language gifted upper - secondary school students b. Good use and control of grammatical structures 4. Punctuation, and spelling and handwriting (5% of total mark) a. Good punctuation and no spelling mistakes b. Legible handwriting MODEL ESSAY Schools are often considered as places where students learn how to become responsible, productive members of society, rather than prioritize their individual needs and desires. Despite this, I would argue that schools should strike a healthy balance between community-oriented and individual-oriented approaches of teaching children.

ƠN

OF

There are clear benefits to transforming children into good citizens and productive employees. The primary advantage has to do with safety. Educating children to understand the need to obey rules and respect others would make the community safer, as well as fostering a sense of belonging and unity. Moreover, school teachers ought to equip their students with a wide cluster of skills, such as discipline, punctuality and co-operation. Having acquired these crucial abilities, school graduates can meet industry requirements and survive in today’s hyper-competitive job market, thus contributing to the economy.

QU Y

NH

The role of schools, however, goes beyond this limited view. Instead of turning children into “obedient slaves” and “robotic workers”, education is crucial for raising well-rounded personalities who pursue meaningful goals, such as happiness and personal fulfillment. This is because while a law-abiding citizen stays quiet in case of propaganda, indoctrination, and enslavement, which is currently the case in Communist North Korea, an active member of society challenges the status quo and reacts as promptly as possible. Teaching children creativity, critical thinking and logical reasoning skills also helps to ensure that we have considerate and reliable future generations, who would make new discoveries and breakthroughs to improve people’s lives.

DẠ

Y

M

In conclusion, I believe that schools should aim to provide a balanced curriculum that not only converts children into law-abiding inhabitants and effective workers but also fosters the development of active and able members of society.

Page 7 of 7

FI CI A

L

KỲ THI HỌC SINH GIỎI CÁC TRƯỜNG THPT CHUYÊN KHU VỰC DUYÊN HẢI VÀ ĐỒNG BẰNG BẮC BỘ LẦN THỨ XIV, NĂM 2023

ĐỀ ĐỀ NGHỊ

(Đề thi gồm 22 trang)

OF

ĐỀ THI MÔN: TIẾNG ANH 11 Thời gian: 180 phút (Không kể thời gian phát đề) Ngày thi: /7/2023 (Thí sinh làm bài vào Phiếu trả lời)

A. LISTENING (50 points) HƯỚNG DẪN PHẦN THI NGHE HIỂU

ƠN

• Bài nghe gồm 4 phần, mỗi phần được nghe 2 lần, mỗi lần cách nhau 15 giây, mở đầu và kết thúc mỗi phần nghe có tín hiệu.

NH

• Mở đầu và kết thúc bài nghe có tín hiệu nhạc. Thí sinh có 3 phút để hoàn chỉnh bài trước tín hiệu nhạc kết thúc bài nghe. • Mọi hướng dẫn cho thí sinh (bằng tiếng Anh) đã có trong bài nghe.

DẠ

Y

M

QU Y

Part 1: You will hear an extract from a lecture about solar eclipses in history. For question 1-5, choose the answer (A, B, C or D) which fits best according to what you hear. (10 pts) 1. The speaker compares a solar eclipse today to a _________. A. religious experience B. scientific event C. popular spectacle D. cultural event 2. The speaker says that the dark spot of an eclipse is ___________. A. simple to predict B. easy to explain C. randomly occurring D. unable to depict 3. Concerning an eclipse, the ancient Chinese were __________. A. fascinated B. rational C. disturbed D. indifferent 4. For the speaker, the most impressive aspect of an eclipse is the __________. A. exceptional beauty of the sky B. chance for scientific study C. effect of the moon on the sun D. effect of the sun on the moon 5. In predicting eclipses, the Babylonians were restricted by the ___________. A. limited ability to calculate B. inaccurate observations C. religious attitudes D. cultural beliefs Your answer: 1 2 3 4 5 Part 2: You will listen to TV program about Gordon Ramsay sharing about his job and decide whether the following sentences are true (T) or false (F). (10 pts) https://www.youtube.com/watch?v=NMTA6JoU_D8 Page 1 of 22

FI CI A

L

6. An absolutely stunning dish can be perfected in even 40 or 50 hours. 7. He travelled to France to discover the origin of cookery they do in France. 8. He could communicate fluently in French after one and a half year living in the country. 9. His parents bought him his first set of knives before sending him to cooking school. 10. When he started out as a chef, his goal was to learn about every ingredient and never feel frightened or unsure of how to use it. Your answers 6. 7. 8. 9. 10.

NH

Your answer: 11

ƠN

OF

Part 3: You will hear a short conversation between two students. Listen and give short answers to the questions. Write NO MORE THAN THREE WORDS AND/OR A NUMBER taken from the recording. (10 pts) https://ielts-up.com/listening/ielts-listening-sample-5.3.html 11. The total course duration is ________. 12. During the final project students will work in teams of ________. 13. The professor said that the key thing in marketing strategy is to _______. 14. The next lecture is in the big classroom on the ________. 15. Students need to take their last week ________.

12

13

14

15

Y

M

QU Y

Part 4. For questions 16–25, listen to a 5-Minute-Food-Fix Podcast about Tofu puffs and fill in the missing information. Write NO MORE THAN THREE WORDS taken from the recording for each answer in the spaces provided. (20 points) (https://player.fm/series/5minute-food-fix/tofu-puffs ) The host of the program adores tofu because she thinks tofu is 16 ___________. One 17__________ of tofu that people can get easily at a grocer is tofu puffs. Tofu puffs are usually in the 18____________ of the store. Personally, the host thinks one tofu puff is similar to a 19__________ because it soaks up any soup. Treat it like a 20__________ ingredient or a pre-cooked soup ingredient. When it comes to shape, tofu puffs are like 21________. Be careful when you 22____________. If you overdo it, the tofu puffs will retain all of that. In that case, you can add some sake or water or stock to 23_______________. You can add chili, and a couple of handfuls of 24___________. A dish from tofu puffs and a side of rice can help you get a really 25____________ meal with very little stress.

DẠ

Your answer: 16

17

18

19

20

Page 2 of 22

22

23

24

25

L

21

FI CI A

B. LEXICO – GRAMMAR (30 points) Part 1. For questions 26-45, choose the best option A, B, C or D to complete the following sentences and write your answers in the corresponding numbered boxes on the answer sheet. (20 points) 26. Let’s _______ the place, it looks so gloomy and unpleasant. A. miss

B. abandon

C. depart

D. disappear

A. escaping

OF

27. The first thing for all of you to remember is that _________ your duties may result in an instant dismissal. B. neglecting

C. resisting

D. missing

A. capture

ƠN

28. Several soldiers of the squad were taken_______ by the enemy forces. B. hostage

C. kidnap

D. torture

29. Patrick is too _______ a gambler to resist placing a bet on the final game. B. spontaneous

C. compulsive

NH

A. instant

D. continuous

30. Mr. Tanner did his best to fix the faulty oven, but his _______ at repairing electrical devices wasn’t good enough to succeed. A. service

B. skill

C. technique

D. craft

A. inflicted

QU Y

31. The inconsiderate driver was _______ for parking his vehicle in the wrong place. B. condemned

C. harassed

D. fined

32. Mrs. Hurston was in deep _______ after her husband’s unexpected death. A. regret

B. grief

C. lament

D. disturbance

B. possess

A. derive

M

33. My older brother is extremely fond of astronomy; he seems to _______ a lot of pleasure from observing the stars. C. seize

D. reach

34. I can accept criticism in general, but George really ______ it too far, so I had no other option but to show my disapproval. B. pushed

C. put

D. made

Y

A. carried

DẠ

35. When Mr. Barnaby died, several people ________ their claim to the substantial legacy that he left.

A. placed

B. drew

C. assumed

D. laid

36. Our karate master is as quick as _______. Page 3 of 22

A. wave

B. thunder

C. current

D. lightning

B. pace

C. rush

D. haste

FI CI A

A. speed

L

37. Mr. Smith ate his breakfast in great ________ so as not to miss the bus to Liverpool.

38. The whole situation is getting out of ____________. Let’s do something before it turns into a bitter row. A. capacity

B. charge

C. hand

D. discipline

39. I know it works in theory, but try putting it into _______and you’ll find out it’s a failure. B. exercise

C. performance

D. procedure

OF

A. operation

40. I wouldn’t have realized the possible risk of setting the factory on fire if Bob hadn’t _______ my attention to it. B. drawn

C. showed

ƠN

A. sought

D. caught

41. Eric’s father works as an interpreter. The man is _______ in three languages. A. fluent B. outspoken C. speechless D. knowledgeable A. up

B. to

NH

42. How exactly did you set _______ training the horse to work so well together? C. about

D. out

43. I could tell at a _______ that nothing had changed between Barbara and Edward. B. blink

QU Y

A. glimpse

C. wink

D. glance

44. I had no chance to defend myself: the dog ______ for me as soon as I opened the door. A. went

B. ran

C. fell

D. stood

45. She should have been here but she’s ______ flu. C. come in for

M

A. gone through with

B. gone down with D. come up against

Part 2. Write the correct form of each bracketed word in the numbered space provided in the column on the right. (10 pts) 46. The number of the nails Harry had bought at the ironmonger’s wasn’t _________ and we had to borrow a few from our neighbor. (SUFFICE)

DẠ

Y

47. Our grandfather is so _________ of the modern technologies that he doesn’t even want to have a TV set in his house. (FEAR) 48. At first I was against keeping the news secret, but Frank’s _________ arguments finally convinced me. (SENSE)

Page 4 of 22

L

49. Big loans on very ________ conditions are offered by the bank to those who make a largescale investment. (FAVOUR)

FI CI A

50. It was a _________afternoon for all the representatives, who were busy preparing the necessary documents. (REST)

51. He is the third _________ caught by the guard today intruding into the no-entry territory. (PASS) 52. Alfred’s irrational reactions may result from the inadequate __________ that he was given by his foster parents. (BRING)

OF

53. Everything they did was illegal as they hadn’t received any official ______ for excavating this land. (AUTHOR)

ƠN

54. There’s no logical accounting for Malcolm’s _________ given that his brothers and sisters are really generous and considerate. (SELF) 55. Because of a sudden fall in temperature the ___________ was so frozen that all the morning flights had to be cancelled. (RUN)

NH

C. READING (60 points) Part 1. For questions 56-65, fill each of the following numbered blanks with ONE suitable word and write your answers in the corresponding numbered boxes on the answer sheet. (15

QU Y

points) June 8th is World Oceans Day – an idea instituted by the United Nations to celebrate and protect our natural heritage. Just as our own central nervous (56)______ controls every part of our body, so the oceans control every part of our planet. They regulate climate, weather and ecosystems.

Unfortunately, human activities have led to pollution and the destruction of marine habitats.

M

Another result of human (57)______ in the natural environment is climate change with its

accompanying extreme weather (58)______. A rise in greenhouse gas emissions has led to an increase in global temperatures and as a consequence sea levels have risen. Plastic pollution is also a big issue.

Y

In 2016, as (59)______ of World Oceans Day, thousands of people took part in ‘the better bag challenge’ and promised to use reusable bags instead of plastic ones. 80 per cent of plastic

DẠ

rubbish (60)______ on land ends up in the sea, usually washed there via (61)______ – where rivers empty into the sea. You may have seen such debris washed up onto your local beach at low (62)______. Most plastic bags are made of polyethene – a (63)______ compound which is Page 5 of 22

not biodegradable. Plastic bags break down into tiny visible (64)______, which are ingested by

L

fish and marine mammals. ‘The better bag challenge’ aims to halt this devastating process and

FI CI A

give new (65 )______ to our oceans.

Part 2. For questions 66-75, read the passage below and choose the answer A, B, C or D that fits best according to the text. Write your answers in the corresponding numbered boxes provided on the answer sheet. (10 points) In orbit high above Earth, a multi-billion-dollar formation of communications satellites stands

OF

ready to instantly connect pilots, seamen and navigators of all kinds to every available aid when they find themselves in an emergency. But what if the communication computers on board these ships and aircraft started acting up or even broke down? The world could go silent,

ƠN

and leave travellers groping around for directions. Not to worry. There is a backup plan, using technology that was invented in 1835. It's Morse code, the language of dots and dashes that has survived the assault of higher technology for a century and a half.

NH

Named after its inventor, Samuel F B Morse, the code is a series of combinations of short and long tones (dots and dashes) representing letters of the alphabet that can be transmitted manually by a key operator. A telegrapher combines the dots and dashes to form letters and

QU Y

words. It is a seemingly tedious procedure, but skilled operators can transmit and receive faster than most secretaries can type. The fastest Morse transmission ever recorded is an amazing 84 words per minute, sent by an operator named T L McElroy in 1951. Morse telegraphy may seem like a quaint anachronism, with its brass sounder and key

M

operated by the world's most basic tool, the human finger. However, it is sometimes vital to worldwide communications. When the Mexico City earthquake occurred in 1985 and all the

power went off, calls for help were transmitted in Morse by an amateur radio operator. "We see the Morse code as a dying art, but we refuse to let it die completely," says Major General Leo M Childs, the US Army's Chief Signal Officer. "Newer is not always better. Even though it is

Y

old and slow, Morse is still the most reliable in difficult conditions. "

DẠ

Every merchant vessel bearing the US flag must carry a radio officer who can both transmit and receive Morse code. Under US law, the officer must spend eight hours at sea every day monitoring the radio for Morse distress signals. Should you ever find yourself adrift at sea in a lifeboat launch from a sink passenger cruise ship, it will be equipped with a single Page 6 of 22

communications device: a Morse transmitter that automatically signals a distress call, but is

L

also equipped with a keyboard in case you happen to know Morse code. Perhaps the best-

FI CI A

known bit of Morse code is the call for help - SOS. In the code, these letters form a distinctive pattern (dot dot dot, dash dash dash, dot dot dot) easily recognised in an emergency.

The enduring use of Morse telegraphy is the legacy of a burst of industrialisation in 19thCentury America, when the railway and telegraph developed side by side. Most of those railway telegraph lines were used well into the mid-20th century, well after radio, television

OF

and computers became commonplace. Until 1985, the Milwaukee Road had a Morse telegraph line between Milwaukee and La Crosse that was routinely used to relay orders to train crews. This Milwaukee operation was shut down quietly in the late Eighties. In many other countries,

ƠN

however, Morse railway is still used.

The military services continue to be the most serious users of Morse telegraphy. While billiondollar satellites and sophisticated ground networks are good in theory, such communication

NH

systems can break down on the battlefield. As a matter of prudence, the Army keeps a functional Morse capability. Morse code signals require much less power to transmit broadcasting than voice messages. In addition, even an unclear Morse signal can be interpreted,

QU Y

whereas a distorted voice transmission is virtually useless. The Army annually trains about 2,800 men and women in Morse code for a variety of signal jobs in infantry, artillery, intelligence and even Special Forces. A Morse transmission will get through when all else fails, and especially in military conflicts, "he who communicates first, no matter how primitively, will come out on top," says Major General Childs.

M

In that case, retired railway telegraph operators will Probably take over the world. Each

evening, the amateur radio waves come alive with the Morse transmissions of the retired operators known among themselves as 'old heads'. "I get on the air and use the code every night to chat to other old heads," says Craig Becker, the retired Milwaukee telegraph operator who received the railway's final telegraph message in 1985. "There are a lot of telegraphers around.

Y

Every night you hear them pecking. "Experienced Operators say the Morse code is not so much

DẠ

a clatter of sounds as a language, because operators do not hear dots and dashes. "You can sit back and hear a conversation," says Becker.

Page 7 of 22

When Morse inaugurated the telegraph service in 1844, he wired from Baltimore to

L

Washington the now-famed message: "What hath God wrought!" Ever since, the death of

FI CI A

Morse code has been regularly predicted. However, although the surface has receded from public view, experts say that they cannot envision an end to its use any time soon. "I can carry a very small Morse key in my pocket and transmit around the world," says Burke Stinson, a public relations man for American Telephone & Telegraph Co. "I do not think you will ever see Morse code die. It is going to be difficult to find another method that is as flexible and

OF

reliable. "

66. The reason the writer mentions the possibility of satellite–based communication systems failing is to

ƠN

A. underline the importance of a reliable alternative. B. suggest that satellite communications are unreliable.

C. emphasise the drawbacks of satellite communications.

NH

D. compare Morse to communication satellites.

67. The process of communicating using Morse code A. involves more than transmitting tones.

D. is simple enough for anyone who can

QU Y

C. is not as laborious as people think.

B. is tedious even for experienced users.

type.

68. In the third paragraph, the writer implies that the use of Morse code is A. old-fashioned, but attracts many enthusiasts. B. only useful to the armed forces and some hobbyists.

M

C. being revived by amateur radio operators.

D. often the only way to communicate during a crisis. 69. In the fifth paragraph, the writer A. implies that Morse telegraphy developed faster than the railway. B. implies that Morse has survived for so long because of the railway.

Y

C. suggests that the railway developed because of Morse telegraphy.

DẠ

D. explains why Morse telegraphy eventually became redundant. 70. The American military A. needs simple communication methods Page 8 of 22

B. has recently neglected its Morse capability. D. offers training for recognising distorted Morse. 71. The first message to be transmitted in Morse code

FI CI A

L

C. allows its forces some Morse equipment.

A. symbolised the code’s flexibility.

B. was used for commercial purposes.

C. took a long time to reach its destination.

D. was sent by Samuel Morse himself.

72. What does the writer mean by saying that Morse code “has survived the assault of higher

OF

technology for a century and a half” in the first paragraph?

A. Even though in the last 150 years newer and more sophisticated ways of doing things have been invented, Morse is still used.

ƠN

B. As modern communication systems have progressed rapidly, it is surprising that Morse telegraphy could survive for 150 years.

C. Thanks to the backup plan, Morse telegraphy has survived for 150 years in the face of

NH

technology.

D. Morse code could not have survived for 150 years without the support of modern technology.

QU Y

73. What does the word “It” in paragraph 2 refer to? A. a series of combinations

B. sending a message in Morse code

C. representing letters of the alphabet

D. the transmission sent by McElroy

74. The phrase “quaint anachronism” in paragraph 3 refers to _______. A. unimportant thing

M

C. old-fashioned tool

B. monotonous cycle D. irrelevant invention

75. Which is the best title for the passage? A. Morse Code – a Revolutionary Invention B. Dots and Dashes Still Alive C. The Comeback of a Forgotten Name

DẠ

Y

D. The Applications of Morse Telegraphy Part 3. For questions 76-88, read the following passage and do the tasks that follow. (13 points)

Page 9 of 22

New innovations are expanding our concept of flight

L

Where is Air Travel Going?

FI CI A

Between 2006 and 2016, the number of airline passengers worldwide went up by around a billion and a half, to nearly four billion. Given the environmental impact of air travel, it is imperative that the industry develop aircraft that produce less noise and fewer carbon emissions than those of the past. Fortunately, we have a number of new technologies and new, lightweight building materials at our disposal.

OF

Electricity could be key to the future of air travel. Just as electric, driverless cars are set to revolutionise personal transport, battery-powered automated flying machines could soon replace traditional taxis. A number of companies worldwide are working on them, convinced

ƠN

that they will take off both literally and metaphorically. One of the things that make the concept of 'sky taxis' suitable for dense urban environments is the ability to start and end journeys vertically, with no runway required.

NH

A number of inventors have also shown that it is possible to build flying machines for personal use on both the roads and in the air, proving that motor pioneer Henry Ford was right when he predicted the appearance of flying cars back in 1940. The problem is that these vehicles have

QU Y

very complicated designs, are very expensive and are subject to very strict regulations about where they can be used; they cannot land just anywhere. They are likely to remain something of a wealthy person’s toy for some time yet. However, some passenger planes on scheduled routes could soon be electric, as long as we find ways of making batteries lighter and more efficient. The most advanced electric aeroplanes

M

today do not carry passengers and have a maximum range of around 100 km, but at least one

company is hoping to introduce a commercial, battery-powered electric aircraft within a decade. Its plan is for a nine-seat aeroplane with a range of around 500 km. Since electric aeroplanes need to fly more slowly and at lower altitudes than those with jet engines, it seems unlikely they will ever be used for long-distance journeys. A mix of fuel and

Y

battery power is certainly possible, however. In fact, this is a very sensible arrangement, as

DẠ

aeroplanes are required to fly with 45 minutes of reserve power available at all times in case an emergency arises.

Page 10 of 22

In the near future, even the passengers themselves could generate at least some of the power an

L

aircraft needs to fly. Seats that have the ability to turn body heat into electrical energy are under

FI CI A

development. If you feel uncomfortable about the idea of becoming a human battery, the seats might also have plenty of features to help you relax. It has been suggested that aeroplane seats – which are notoriously lacking in comfort – could one day change shape to match an individual body. Massage or acupuncture treatments could even be built in. Such luxuries would be provided to passengers who are willing to pay the top fares, anyway.

OF

Other design innovations seem to be aimed at helping people who find flying unpleasant or frightening. If you wish you were somewhere else every time you board an aircraft, how would you like to use a virtual-reality helmet to transport yourself to a different environment? There is

ƠN

a good chance that ‘sensory headsets’ will be fitted to the headrests of aeroplane seats before long. A more outlandish virtual-reality idea is to make the plane walls seem invisible. By covering them with special materials that give off light, airlines will be able to show passengers

NH

images of the sky outside the plane. Of course, that is not everyone’s idea of a relaxing experience – so some people might be very grateful for the sensory headset option! For those who do enjoy flying, however, a new kind of holiday that puts the plane at the centre

QU Y

of the experience has already become available – but it does not come cheap. You can fly around the world in a luxury Boeing 777 aircraft, spending time in ten of the world’s most exciting cities, for around $160,000. The ‘air cruise’, as the name suggests, is an attempt to recreate in the air the experience of travelling by sea for pleasure. The aeroplane has lounge seating, beds, fine dining, a bar and even butlers, and is used exclusively for air cruises lasting

M

between two weeks and a month.

Clearly, standard aircraft are not likely to become so luxurious, but ordinary flyers can at least expect to get more room for themselves. That is because the shape of commercial aircraft is expected to change quite fundamentally in the future, as today’s designs are not ideal for fuel efficiency and smooth movement. Future aircraft will likely be more triangular in shape, with

Y

larger wings that blend into the middle of the plane rather than sticking out from it. These

DẠ

aeroplanes will be less noisy and they will have more interior space, but there will also be fewer windows. Indeed, if you think about how much cars have changed since their early days, it is hard to believe that the basic design of aircraft has stayed the same for as long as it has. Page 11 of 22

L

Questions 76-88

FI CI A

Do the following statements agree with the information given in the Reading Passage. For questions 76-85, choose: TRUE if the statement agrees with the information FALSE if the statement contradicts the information NOT GIVEN if there is no information on this

OF

Write your answers in the corresponding numbered box provided. The popularity of air travel is threatening the environment.

77

Electric power is unlikely to be used in the flight industry.

78

Sky taxis are viewed by some as an opportunity for profit.

79

Sky taxis are not a good idea for crowded, big city locations.

80

Henry Ford designed a prototype flying car.

81

It is illegal to land a flying car on a roadway.

82

There are still technical limitations which electric planes must overcome.

83

In the next decade, electric planes are expected to almost double their range.

NH

ƠN

76

QU Y

Questions 84-88

Choose the correct letter A, B, C or D. Write your answers in the corresponding numbered box provided.

84. What is the writer doing in the fifth paragraph? A describing why electric planes are not widely used

M

B pointing out the advantages of battery power

C discussing future applications of battery technology D warning of potential risks associated with electric planes 85. What are we told about innovations in seats in the sixth paragraph?

Y

A They will benefit the airlines more than the passengers.

DẠ

B They will each make flying a more relaxing experience. C They will do little to ease the discomfort of current seats. D They are unlikely to be made available to everyone. Page 12 of 22

86. Future virtual reality innovations could affect anxious passengers B in several different ways. C by obscuring the flight experience completely. D because the thrills of flight are emphasised. 87. An air cruise seeks to make B luxury travel accessible to people from all walks of life. C the destination more important than the journey. D air travel emulate a more traditional holiday experience.

OF

A round-the-world travel attainable for those with little time.

FI CI A

L

A minimally, as there will be little change.

ƠN

88. What is the main point that the writer makes about change in aircraft in the final paragraph? A It is long overdue. C It will be at the expense of comfort. D It is unlikely to actually take place.

NH

B It will be mostly cosmetic.

QU Y

Part 4: In the passage below, seven paragraphs have been removed. Read the passage and choose from paragraphs A-H the one which fits each gap. There is ONE extra paragraph which you do not need to use. Write your answers in the corresponding numbered boxes provided on the answer sheet. (7 points) Living in a Dream World

M

Daydreaming can help solve problems, trigger creativity, and inspire great works of art and science. By Josie Glazier.

Most people spend between 30 and 47 per cent of their waking hours spacing out, drifting off, lost in thought, wool-gathering or building castles in the air. Yale University emeritus psychology professor Jerome L. Singer defines daydreaming as shifting attention “away from

Y

some primary physical or mental task toward an unfolding sequence of private responses” or,

DẠ

more simply, “watching your own mental videos.” He also divides daydreaming styles into two main categories: “positive-constructive,” which includes upbeat and imaginative thoughts, and “dysphoric,” which encompasses visions of failure or punishment.

Page 13 of 22

L

89.

FI CI A

Such humdrum concerns figured prominently in one study that rigorously measured how much time we spend mind wandering in daily life. In a 2009 study, Kane and his colleague Jennifer McVay asked 72 students to carry Palm Pilots that beeped at random intervals eight times a day for a week. The subjects then recorded their thoughts at that moment on a questionnaire. The study found that about 30 per cent of the beeps coincided with thoughts unrelated to the task at

OF

hand and that mind wandering increased with stress, boredom or sleepiness or in chaotic environments and decreased with enjoyable tasks. That may be because enjoyable activities tend to grab our attention.

ƠN

90.

We may not even be aware that we are daydreaming. We have all had the experience of “reading” a book yet absorbing nothing—moving our eyes over the words on a page as our

NH

attention wanders and the text turns into gibberish. “When this happens, people lack what I call ‘meta-awareness,’ consciousness of what is currently going on in their mind,” he says. But aimless rambling can be productive as they can allow us to stumble on ideas and associations

QU Y

that we may never find if we intentionally seek them. 91.

So, why should daydreaming aid creativity? It may be in part because when the brain is floating in unfocused mental space it serves a specific purpose. It allows us to engage in one

M

task and at the same time trigger reminders of other, concurrent goals so that we do not lose sight of them. There is also the belief that we can boost the creative process by increasing the brains.

Y

92.

amount of daydreaming we do or replaying variants of the millions of events we store in our

The mind's freedom to wander during a deliberate tuning out could also explain the flash of

DẠ

insight that may coincide with taking a break from an unsolved problem. A study conducted at the University of Lancaster in England into this possibility found that if we allow our minds to ramble during a moderately challenging task, we can access ideas that are not easily available Page 14 of 22

to our conscious minds. Our ability to do so is now known to depend on the normal functioning

L

of a dedicated daydreaming network deep in our brain.

FI CI A

93.

It was not until 2007, however, that cognitive psychologist Malia Fox Mason, discovered that the default network — which lights up when people switch from an attention-demanding activity to drifting reveries with no specific goals, becomes more active when mind wandering

OF

is more likely. She also discovered that people who daydream more in everyday life show greater activity in the default network while performing monotonous tasks. 94.

ƠN

The conclusion reached in this ground-breaking study was that the more complex the mind wandering episode is, the more of the mind it is going to consume. This inevitably leads to the problem of determining the point at which creative daydreaming crosses the boundary into the

NH

realms of compulsive fantasising. Although there is often a fine dividing line between the two, one question that can help resolve the dilemma relates to whether the benefits gained from daydreaming outweigh the cost to the daydreamer’s reputation and performance.

QU Y

95.

On the other hand, there are psychologists who feel that the boundary is not so easily defined. They argue that mind wandering is not inherently good or bad as it depends to a great extent on context. When, for example, daydreaming occurs during an activity that requires little

M

concentration, it is unlikely to be costly. If, however, it causes someone to suffer severe injury

or worse by say, walking into traffic, then the line has been crossed.

Y

A Although these two findings were significant, mind wandering itself was not measured

DẠ

during the scans. As a result, it could not be determined exactly when the participants in her study were “on task” and when they were daydreaming. In 2009 Smallwood, Schooler and Kalina Christoff of the University of British Columbia published the first study to directly link Page 15 of 22

mind wandering with increased activity in the default network. Scans on the participants in

L

their study revealed activity in the default network was strongest when subjects were unaware

FI CI A

they had lost focus.

B However, intense focus on our problems may not always lead to immediate solutions. Instead allowing the mind to float freely can enable us to access unconscious ideas hovering underneath the surface — a process that can lead to creative insight, according to psychologist Jonathan W. Schooler of the University of California, Santa Barbara

OF

C Yet to enhance creativity, it is important to pay attention to daydreams. Schooler calls this “tuning out” or deliberate “off-task thinking.”, terms that refer to the ability of an individual to have more than just the mind-wandering process. Those who are most creative also need to

ƠN

have meta-awareness to realise when a creative idea has popped into their mind. D On the other hand, those who ruminate obsessively—rehashing past events, repetitively analyzing their causes and consequences, or worrying about all the ways things could go wrong

NH

in the future - are well aware that their thoughts are their own, but they have intense difficulty turning them off. The late Yale psychologist Susan Nolen-Hoeksema does not believe that rumination is a form of daydreaming, but she has found that in obsessive ruminators, the same

QU Y

default network as the one that is activated during daydreaming switches on. E Other scientists distinguish between mundane musings and extravagant fantasies. Michael Kane, a cognitive psychologist at the University of North Carolina at Greensboro, considers “mind wandering” to be “any thoughts that are unrelated to one's task at hand.” In his view, mind wandering is a broad category that may include everything from pondering ingredients

M

for a dinner recipe to saving the planet from alien invasion. Most of the time when people fall

into mind wandering, they are thinking about everyday concerns, such as recent encounters and items on their to-do list. F According to Schooler, there are two steps you need to take to make the distinction. First, notice whether you are deriving any useful insights from your fantasies. Second, it is important

Y

to take stock of the content of your daydreams. To distinguish between beneficial and

DẠ

pathological imaginings, he adds, “Ask yourself if this is something useful, helpful, valuable, pleasant, or am I just rehashing the same old perseverative thoughts over and over again?” And

Page 16 of 22

if daydreaming feels out of control, then even if it is pleasant it is probably not useful or

L

valuable.

FI CI A

G Artists and scientists are well acquainted with such playful fantasizing. Filmmaker Tim Burton daydreamed his way to Hollywood success, spending his childhood holed up in his bedroom, creating posters for an imaginary horror film series. Orhan Pamuk, the Turkish novelist who won the Nobel Prize in Literature in 2006, imagined “another world,” to which he retreated as a child, Albert Einstein pictured himself running along a light wave—a reverie that

OF

led to his theory of special relativity.

H Like Facebook for the brain, the default network is a bustling web of memories and streaming movies, starring ourselves. “When we daydream, we're at the center of the universe,”

ƠN

says neurologist Marcus Raichle of Washington University in St. Louis, who first described the network in 2001. It consists of three main regions that help us imagine ourselves and the thoughts and feelings of others, draw personal memories from the brain and access episodic

NH

memories.

QU Y

Part 5: For questions 96-105, you are going to read an article about an art exhibition that focuses on the subject of whether paintings are authentic or fake. Answer the questions by choosing from the sections of the article (A - F). The sections may be chosen more than once. Write your answers in the corresponding numbered boxes provided on the answer sheet. (15 points)

Napoleon Bonaparte

M

What characteristics marked out one of history’s greatest military geniuses?

A Napoleon Bonaparte stirs the emotions. Some find him heroic and regard his opponents as reactionary and unimaginative. Others think him mad with ambition and responsible for many of the sins of his era. Between these extremes there are those who find some aspects of the man

Y

admirable and others regrettable. Of course, Bonaparte was anything but pure, anything but modest, anything but democratic, and anything but a peacemaker. But in the end, who else that

DẠ

sat on a throne in Europe could claim to be? Should he be assailed for sins that were so sadly common? What is it about Napoleon Bonaparte that makes him the object of such unique criticism? Is it because he holds a special place in our imaginations, a place that we hope would Page 17 of 22

be an example of our better selves? Was his genius, good fortune, and opportunity enough to

FI CI A

disappointment in Bonaparte simply that he was merely human?

L

condemn him - not so much for what he did, but what he failed to do? In the end is our greatest B Napoleon was a military genius in the strategic and tactical handling of armies and although he provided no large scale reforms of armies (or, indeed, their equipment and techniques), he excelled at the refinement of an art that already existed. One of the most important factors of Napoleon's personality and its effect on his abilities as a military commander was his genius to

OF

inspire others. He believed in the maxim that ‘spirit and drive is to the physical as three is to one’. It was through his system of awards (an appealing to soldiers’ ‘soul in order to electrify the man’) that Napoleon was so successful in obtaining unquestionable obedience, loyalty and

ƠN

devotion from his rank and file.

C The name Napoleon Bonaparte is surrounded by so vast an array of myth and legend that it is quite difficult to separate fact from fiction. It is said that Napoleon displayed outstanding

NH

leadership qualities while still at school. But this is undoubtedly the product of the school of Napoleonic mythology that was systematically promoted for political reasons in 19th century France, since it hardly squares with the general picture of the taciturn child with an inferiority

QU Y

complex that has come down to us. Napoleon

was, in truth, a reserved child, resentful of his peers. On the other hand, he excelled at maths – a qualification that determined his specialisation as an artillery officer. This was a stroke of luck (one of many that Napoleon benefited from) inasmuch as the artillery was the most prestigious branch of the army under the old regime. But the biggest stroke of fortune

M

Napoleon had was to be born when he was – in the age of the French Revolution. The

Revolution turned the whole world upside down and presented an ambitious young man (Napoleon was always ambitious – a consequence of his resentment at his inferior status) with new and vast opportunities. D No man previously ever concentrated authority to such a point, nor showed mental abilities

Y

at all comparable to Napoleon’s: an extraordinary power of work; prodigious memory for detail

DẠ

and fine judgment in their selection; a luminous decision-making capacity and a simple and rapid conception - all placed at the disposal of a sovereign will. And no head of state gave expression more imperiously than this Corsican to the popular passions of the French of that Page 18 of 22

day: abhorrence for the emigrant nobility, fear of the ancient régime, dislike of foreigners,

L

hatred of England, an appetite for conquest evoked by revolutionary propaganda, and the love

FI CI A

of glory.

E Psychological studies of ‘great men and women’ frequently serve as a fig leaf to disguise the absence of an understanding of broad socio-historical processes. The study of history is replaced by trivial personal observations. Instead of science, we have gossip. A careful study of the character and background of Napoleon Bonaparte can furnish us with many useful insights

OF

into his behaviour. But we are left with a small amount of useful information that can help us to attain a deeper understanding of Bonaparte. Men and women make their own history, but they do not make it freely, in the sense that the scope and results of their actions are strictly limited

ƠN

by the given socio-economic context that is prepared independently of their will. In which section are the followings mentioned?

Your answers:

NH

the idea that an individual’s actions cannot be separated from their historical 96. context

97.

the characteristics that made Napoleon an exceptional leader

98.

QU Y

the view that the writing of history is sometimes agenda-driven

the fact that Napoleon was eminently aware of the impact of morale on modern 99. warfare

the fact that Napoleon is as divisive a character as he is famous

100.

the uncertainty as to whether detractors of Napoleon simply lament the fact he 101.

M

had the vices and virtues of the common man

the view that Napoleon was no great revolutionary of warfare itself

102. 103.

the fact that Napoleon’s views reflected those of his countrymen

104.

the event that made Napoleon’s ascent possible

105.

Y

the notion that we are all a product of the time we live in

DẠ

D. WRITING (60 points) Part 1. Read the following extract and use your own words to summarize it. Your summary should be about 140 words. You MUST NOT copy the original. (15 pts) Page 19 of 22

L

Part 1: Read the following extract on how camouflaging and mimicry help insects and use your own words to summarize it. Your summary should be no more than 120 words long. (1.0pt)

FI CI A

Have you ever wondered why soldiers are always clad in green? This is to enable them to camouflage themselves during wartime. Hiding in the jungles, their green attire blend into the surrounding trees and shrubs, making it difficult for the enemies to spot them.

OF

Long before man make use of camouflaging, insects have already adopted the tactic of disguise to escape from the clutches of their predators. By having body colors close to those of the rocks and dried leaves, they catch less attention from the predators and hence escape from being pursued. However, this kind of disguise works only if the insects remain still in the presence of their predators.

NH

ƠN

Butterflies and moths have developed a variety of camouflage strategies since they are quite defenseless and their predators - birds are abundant in supply. Many moth caterpillars resemble dead twigs while the young of certain species of butterflies appear like bird droppings. Adult butterflies and moths camouflage themselves too, in attempts to escape from their hunters birds who are superior gliders. Possessing wings which resemble dried leaves help certain butterflies and moths to hide among heaps of dried leaves when predators are around.

QU Y

Fortunately, not all insects choose the art of disguise to escape from their predators; otherwise, the world would be so dull and colorless. There are insects which assimilate the bright body colors of bees and wasps to escape from being pursued by their predators. The concept of mimicry was derived, owing to the bees and wasps. Long ago, birds have already learnt to avoid brilliantly colored wasps and bees in fear of their painful stings. Hence, over millions of years, many harmless insects have assimilated the bees and wasps by imitating their bright body colors and shapes. In this way, they appear dangerous to their predators and hence ward them off.

M

Mimics of the wasps and bees are most commonly found in the gardens. The furry, plump beefly not only appears like the bumble bee in terms of body colors, even its hums sound similar too. The only difference is that the bee-fly does not have a sting and is hence harmless. The hoverfly is another insect which imitates the body colors of the wasps. Their bodies are striped yellow and black. The only deviations are that hoverflies do not have stings and they have only one pair of wings each while wasps have two pairs each. These variations are hardly noticed by the predators and hence help them to escape.

Y

Part 2: The graph below shows radio and television audiences throughout the day in 1992. Summarise the information by selecting and reporting the main features, and make

DẠ

comparisons where relevant. Write at least 150 words

Page 20 of 22

Page 21 of 22

Y

DẠ M

KÈ QU Y ƠN

NH

FI CI A

OF

L

-------------- HẾT --------------

FI CI A

L

Part 3. Essay writing (30 points) Some people think that children should begin their formal education at a very early age and should spend most of their time on school studies. Others believe that young children should spend most of their time playing. Compare these two views. Which view do you agree with? Why?

OF

(Thí sinh không được sử dụng tài liệu. Cán bộ coi thi không giải thích gì thêm)

DẠ

Y

M

QU Y

NH

ƠN

Họ và tên thí sinh: .............................................................. Số báo danh: ...............................

Page 22 of 22

FI CI A

HƯỚNG DẪN CHẤM MÔN: TIẾNG ANH 11 Thời gian: 180 phút (không kể thời gian giao đề)

6F

7T

OF

ĐỀ XUẤT

(Đáp án gồm 10 trang) A. LISTENING (50 points): LISTENING 1C 2B

L

KỲ THI HỌC SINH GIỎI CÁC TRƯỜNG THPT CHUYÊN KHU VỰC DUYÊN HẢI VÀ ĐỒNG BẰNG BẮC BỘ LẦN THỨ XIV , NĂM 2023

3C

4B

5A

8T

9F

10 T

12. five people

13. grab people's attention

14. ground floor

15. assignment

16 versatile

17 common kind

18 fridge section

19 sponge

20 stir-fry

21 cubes

22 add flavors

23 loosen it up

24 cashew nuts

25 enriched, delicious, vegetarian

QU Y

NH

ƠN

11. seven weeks

DẠ

Y

M

LISTENING SCRIPT Part 1: You will hear an extract from a lecture about solar eclipses in history. For question 15, choose the answer (A, B, C or D) which fits best according to what you hear. (10 pts) Good evening and welcome to this month's Observatory Club lecture. I'm Donald Mackey and I'm here to talk to you about the solar eclipse in history. A thousand years ago, a total eclipse of the sun was a terrifying religious experience. But these days, an eclipse is more likely to be viewed as a tourist attraction than as a scientific or spiritual event. People will literally travel miles to be in the right place at the right time to get the best view of their eclipse. Well, what exactly causes a solar eclipse? When the world goes dark for a few minutes in the middle of the day, scientifically speaking, the dark spot itself is easy to explain. It is the shadow of the moon streaking across a different and, to all intents and purposes, a seemingly random part of the globe. In the past people often interpreted an eclipse as a danger signal heralding disaster, and in fact the Chinese were so disturbed by these events that they included among their gods whose job was to prevent eclipses. But whether or not you are superstitious, or take a purely scientific view, our earthly eclipses are special in three different ways. Firstly, there can be no doubt that they are very beautiful. It's as if a deep blue curtain has fallen over the daytime sky, as the sun becomes a black void surrounded by the glow of its outer atmosphere. But beyond this, total eclipses possess a second more compelling beauty in the eyes of us scientists. For they offer a unique opportunity for research. Page 1 of 10

DẠ

Y

M

QU Y

NH

ƠN

OF

FI CI A

L

Only during an eclipse can we study the corona and other dim things that are normally lost in the sun's glare. And thirdly, they are rare. Even though an eclipse of the sun occurs somewhere on earth, if you sit in your garden and wait, it will take 375 years on average for one to come to you. If the moon were any larger, eclipses would become a monthly bore. If it was smaller, they simply would not be possible. The ancient Babylonian priests, who spent a fair bit of time staring at the sky, had already noted that there was an 18-year pattern in their recurrence, but they didn't have the mathematics to predict an eclipse accurately. It was Edmund Halley, the English astronomer, who knew his maths well enough to predict the return of the comet, which incidentally bears his name. Part 2: You will listen to TV program about Gordon Ramsay sharing about his job and decide whether the following sentences are true (T) or false (F). (10 pts) https://www.youtube.com/watch?v=NMTA6JoU_D8 GORDON RAMSAY: Cooking is a life skill. For me, it's fundamentally important. It's just as crucial as keeping face because Latin, history, geography-- no disrespect, but if you're not going to teach that for the rest of your life, it doesn't come into play. Cooking does, three times a day, seven days a week, for the rest of your life to eat. So we don't cook three meals a day, but there's one meal across that day that needs to be absolutely you on a plate, done properly, healthily cooked, and sets you up, because you are what you eat. So we can never underestimate the importance of food because that's the fuel, and especially in sport. This is my little nicoise-- a specialized nicoise, and it's the kind of thing that I pick myself up after cycling 200K. A beautiful piece of tuna. Brush that first with a little bit of mustard. Stops it from drying out. Now, sesame Just beautifully roll that down. I always think about foods everywhere I go. So when I'm training, I'm never very far away from food. I had that extraordinary moment at the start of Kona two years ago where I was paddling out sort of half past five in the morning waiting for that amazing cannon to go off. To get rid of the nerves, I just started thinking about recipes and dishes and how do I get through this 2.8K swim. The garnish-- it's very, very simple. We got potatoes. Let's start with a little teaspoon of olive oil. We'll get these potatoes nice and crispy. Cut it in half, and put that side down. So, in my bowl, got some really nice shallot rings. Olive nicoise, some black olives, some green beans. And then in there, I'm just going to open up some of these little anchovies. A little touch of vinaigrette. This is classic vinaigrette made with lots of lemon juice. And then we'll just mix that up nicely with some fresh parsley. Your tomatoes, balsamic vinegar, fresh lemon juice in-- that lifts everything. You work 14, 15 hours a day to perfect an absolutely stunning dish. It disappears in 2 and 1/2 minutes. And that's why I said it loses you. You get on that journey and nothing else matters except what you put on the plate. Because you start off with these raw ingredients and you go through that journey. 60 minutes later, you got this bit of magic. Also, it's an incredible passion. It's a huge canvas. Just take your salad leaves. I want to use the little hearts. Just open them up gently without sort of ripping them. Get down to the heart, and cut the hearts into fourths. Potatoes-- nice and crispy. And then from there, as they start to cool down, just put a little touch of vinaigrette over there and hit them with your parsley. Food was my calling, I think, because that was the way I could sort of disappear. Disappear, travel, learn, and get really excited about something. And my first dream Page 2 of 10

DẠ

Y

M

QU Y

NH

ƠN

OF

FI CI A

L

was to go to France. To understand why were they the sort of foundation of cooking. Why did they start it? How did they start it? And I disappeared. I became French. Within 18 months, I was fluent in the language. I was holding my own in a foreign kitchen, and I was seriously cooking my ass off. Now, for the dressing. Very, very simple. Touch of the vinaigrette, and then some fresh lemon juice. That gives a nice lightness, vibrancy to the dish as well. Just lightly dress them. My first set of knives was bought by a charity because my parents couldn't afford to send me to cooking school, and this knife wallet was a "Swimbo." It was with bright, yellow handles. And trust me, they couldn't cut butter, but they looked the part. My mom had a tear in her eye because she was upset she couldn't afford it. My first set of whites and my first set of knives. I still got them. Finally, the tuna. Get the pan nice and hot. Put a teaspoon of oil in there. Tuna into the pan. 90 seconds each side. Out. And literally squeeze some fresh lemon juice on there. When you slice it, let the knife do the work. Fingers on top and just slice through nice and carefully. On. Three nice slices. Foods your journey, and there's something quite amazing about the way you cook. Of giving pleasure. My dream, when I started out as a chef, was to discover every ingredient and never be intimidated and not know what to do with it. That level of perfection. I think I'm about 97% of the way there. There must be about 3% or 4% of the ingredients that I still haven't discovered yet, and that's the exciting thing about food. That is what I call my specialized tuna nicoise. Part 3: You will hear a short conversation between two students. Listen and give short answers to the questions. Write NO MORE THAN THREE WORDS AND/OR A NUMBER taken from the recording. (10 pts) https://ielts-up.com/listening/ielts-listening-sample-5.3.html Julie: Hi Kate. How did you like the first lecture? Kate: Hi Julie. You mean that lecture about brand identity. Julie: Exactly. When our professor described how brand names are created. As I understood, he'll teach us practical tips and techniques to create better brands. And what about course duration? Kate: I heard it's five weeks. No, five weeks is without the final project. The total duration is seven weeks. Julie: Oh, I forgot about it. It's the first time I'll be doing a group project. And I'm a bit nervous to work in a five-people team. Kate:I think it's going to be okay. He was talking about the right marketing strategy. Did you hear that? Julie:Yes, that was the most exciting part of the presentation. The key thing is to grab people's attention. Kate: That's true. And how do you like the curriculum? Julie: I think it's nice. It's great that we'll be learning marketing in detail, as I've always been interested in this subject. Kate: Me too. And after that, we'll be training to design custom logos. The professor told that the curriculum has been modified, and we won't learn design this year. What a pity! The same with product management. But you shouldn't worry, they just combined these two sections into a larger one called branding. Then it's good. Moreover, they've added Page 3 of 10

DẠ

Y

M

QU Y

NH

ƠN

OF

FI CI A

L

something called e-commerce this year. It's a great alteration and the course is kept up to date. Online trading is so popular now. And after that we'll learn advertising. Yes, and also analytics. Really? Won't analytics be replaced with customer attraction? Julie:Yes, you're right. Kate: I forgot about that. Julie: I think that's all. The professor also teaches a course called Business Strategies. So, we may take his other course next year. Dear, and where is the next lecture? Kate: It's in the big classroom on the ground floor. Julie:Thanks. Kate: You're welcome. Don't forget to take your last week assignment. Julie:Okay. Goodbye. Kate: Okay. Goodbye. Part 4. For questions 16–25, listen to a 5-Minute-Food-Fix Podcast about Tofu puffs and fill in the missing information. Write NO MORE THAN THREE WORDS taken from the recording for each answer in the spaces provided. (20 points) (https://player.fm/series/5minute-food-fix/tofu-puffs ) I think about this job sometimes as being about demystifying food. Yeah, so Simon, you've been a food writer, you've edited countless cookbooks, and that's kind of the job, right? Very much. Yeah, like people get scared, people think some ingredients are intimidating, and our job is to be conduits of information to say, no, you can actually do this. To peel back the curtain and reveal the wondrousness of that food. When you just said peel back the curtain, it sounded really gross. It is. How awful. Can we just cut that? Can we start again? No, that was my favourite bit. I'm Yumi Stein. And I'm Simon Davis. And this is your peeling back the curtain five-minute food bit. Oh, dear God. So one of the things I wanted to peel back the curtain on was tofu puffs. So tofu in general, and we've talked about this before, there's such a myriad of options and varieties and flavours and ways to use tofu, but as somebody who really doesn't eat a lot of meat, tofu is something that I just adore. I find it so versatile and I think there's a lot of mystery around it for people who aren't familiar with it. I really want to illuminate one kind of tofu that you can buy pretty commonly. Places like Harris Farm, also Coles and Woolies, I've seen it there, but definitely Asian Grocers if you're lucky enough to have one in proximity to your home. It's called fried tofu puffs or tofu puffs and they are usually in the fridge section of the grocer. They're almost ready to eat, like you could cut open the bag and put one in your mouth and you would not drop dead. That's always good. Which is a bonus for a food staff. But I did want to talk you through how to get the most out of them. So different sorts of tofu have different properties. The soft silky custardy one doesn't take on lots of flavor. These ones are spongy. So picture it like a sponge. It'll soak up any soup that it sits in. It'll be like you could literally squeeze soup out of it. It will Page 4 of 10

36. D

27. B

28. D

29. C

30. B

31. D

32. B

33. A

34. A

35. D

37. D

38. C

39. A

40. B

41. A

42. C

43. D

44. A

45. B

26. B

M

QU Y

NH

ƠN

OF

FI CI A

L

capture a lot of flavors and a lot of wetness and moisture of anything that you cook. And given that it's already in a cooked state and you don't have to do much to it, treat it like a stir-fry ingredient or a pre-cooked soup ingredient. That's my advice for tofu puffs. If you've got, say, some bok choy, one of our favorite ingredients, it's just so cheap, it's so green, it's always fresh and available. So those two things together make something really, really delicious. Let's talk about what you could do with bok choy plus tofu puffs. With the tofu puffs, they come as a cube, sometimes big, two or three centimeters square, and sometimes smaller, like one centimeter square. If you want to really maximize how much flavor they'll suck up, then I would cut them in half. It's a way of handling that you might not be up for. If you just cannot be bothered, then don't worry. But if you want to think, oh, how can I just shove as much flavor in that, definitely cutting them in half will do that. And then you want to stir fry the bok choy in a little bit of oil, say a neutral canola or grapeseed type oil, and then add your flavours, so soy sauce, ginger, oyster sauce. Don't overdo it though, because they're going to retain all of that. It's not going to spread. So you might want to even add some sake or water or stock, just kind of loosen it up. And then toss your fried tofu puffs through, give it a good stir, let them warm up. Corners might burnish and get a little bit brown and delicious. That sounds good. I know. And then if you want chili, of course, because chili goes well in everything, and then a couple of handfuls of cashew nuts. And that's actually kind of like an almost complete meal. That's a whole dinner, isn't it? Yeah. I mean, if you wanted a side of rice, by all means. But if you couldn't be bothered, you're kind of getting a really enriched, delicious, vegetarian meal with very little stress. That sounds perfect, Stevie. I know. If you don't really quite know what a tofu puff is. It sounds like something you could perhaps rest your feet on while watching TV. It does a bit, doesn't it? It does a bit. I'm going to put up a photo of what tofu puffs look like in the wild in the shops on our 5-Minute Food Fix Instagram. Check it out there. And don't forget to tag the shops on our 5-Minute Food Fix Instagram. Check it out there. And don't forget to tag your friends in your favourite food podcast. B. LEXICO - GRAMMAR (30 points) Part 1. Choose the correct answer A, B, C or D to each of the following questions. Write your answers in the corresponding numbered boxes provided.(20 pts)

DẠ

Y

Part 2. Write the correct form of each bracketed word in the numbered space provided in the column on the right. (10 pts) 46. sufficient

47. fearful

51.tresspasser

52. upbringing 53.authorization 54. selfishness

48. sensible

49. favourable

50. restless 55. runways Page 5 of 10

C. READING (60 points) Part 1: 57. Intervention

59. part

63. chemical

64. particles 65. life

Part 2: 66 A 67 C 68 D

OF

69 B 70 C 71 D

ƠN

72 A 73 B 74 C

NH

75 B PART 3: 76 T

QU Y

77 F 78 T 79 F 80 F 81 NG

M

82 T

85 D 86 B

83 F 84 A

60. produced

FI CI A

61. estuaries 62. tide

58. conditions

L

56. system

Y

87 D

DẠ

88 A

PART 4: 89 E

90 B

Page 6 of 10

91 G 92 C

L

93 H

FI CI A

94 A 95 F Part 5. 96 E 97 C

OF

98 D 99 B 100 A 101A

ƠN

102B 103E 104D

NH

105C

DẠ

Y

M

QU Y

D. WRITING (60 points) Part 1: (15 pts) Contents (10 points) - The summary MUST NOT contain personal opinions. Language use (5 points) The summary should: - show attempts to convey the main ideas of the original text by means of paraphrasing (structural and lexical use), - demonstrate correct use of grammatical structures, vocabulary, and mechanics (spelling, punctuations,...), - maintain coherence, cohesion, and unity throughout (by means of linkers and transitional devices). Penalties - A penalty of 1 point to 2 points will be given to personal opinions found in the summary. - A penalty of 1 point to 2 points will be given to any summary with more than 30% of words copied from the original. - A penalty of 1 point will be given to any summary longer than 130 words or shorter than 90 words. Model answer Camouflaging and mimicry have helped defenseless insects escape from their predators. Camouflaging requires insects to have body colors close to the surroundings so as to appear less eye-catching to predators. The moth caterpillars look like dead twigs while certain butterfly caterpillars resemble bird droppings. Some butterflies and moths have wings that resemble dried Page 7 of 10

FI CI A

L

leaves. Mimicry requires harmless insects to adopt the body colors and shapes of the wasps and bees so as to fool their predators into thinking that they are dangerous. Predators usually avoid them, thinking they have stings too. The bee-fly and hoverfly assimilate the body colors of the bumble bee and wasps respectively and the bee-fly even hums like the bumble bee. ( 118 words )

NH

ƠN

OF

Part 2: 15 pts Contents (10 points) - The report MUST cover the following points: •Introduce the chart (2 points) and state the striking features (2 points) •Describe main features with relevant data from the charts and make relevant comparisons (6 points) - The report MUST NOT contain personal opinions. (A penalty of 1 point to 2 points will be given to personal opinions found in the answer.) Language use (5 points) The report should: - demonstrate a wide variety of lexical and grammatical structures, - have correct use of words (verb tenses, word forms, voice,…); and mechanics (spelling, punctuations,...). Model answer The provided line graph delineates the proportion of people watching TV and listening to the radio who were over 4 years old in a 24 - hour period in the UK from October to December 1992.

QU Y

From an overall perspective, it is readily apparent that the majority of people listened to the radio in the morning, while most people watched TV at night. Another striking feature is that watching TV was the more popular activity compared to listening to the radio for most of the period in question.

M

At 6 am, nobody watched TV, yet the figure for this activity started to increase, albeit slightly, to 5% after two hours only to decline by around 2% at 10 am. The figure then regained its upward streak and surpassed that for the radio at roughly 15% by 2 o’clock, prior to a surge to the peak at approximately 48% at 8 pm. Just after midnight, however, only 1 out of 10 people

Y

was reported to engage in this pastime. Watching TV began to lose people’s interest as time went by, yet a marginal rise was recorded before 6 am, culminating the graph at around 4%. Commencing at around 7% higher, the data of the radio precipitously reached 27% just after 8

DẠ

am, before witnessing a dramatic drop to 10% nearly 5 hours later. Despite a negligible recovery at 4 pm, the figure went down to the 8% marker, before another minimal rise was registered. Nevertheless, it fell yet again and after minor fluctuations, ended at the same figure as the other leisure pursuit. Page 8 of 10

NH

ƠN

OF

FI CI A

L

Part 3: 30 pts The mark given to part 3 is based on the following criteria: 1. Task achievement (10 points) a. All requirements of the task are sufficiently addressed. b. Ideas are adequately supported and elaborated with relevant and reliable explanations, examples, evidence, personal experience, etc. 2. Organization (10 points) a. Ideas are well organized and presented with coherence, cohesion, and unity. b. The essay is well-structured: •Introduction is presented with a clear thesis statement introducing the points to be developed. •Body paragraphs develop the points introduced with unity, coherence, and cohesion. Each body paragraph must have a topic sentence and supporting details and examples when necessary. •Conclusion summarizes the main points and offers personal opinions (prediction, recommendation, consideration,…) on the issue. 3. Language use (5 points) a. Demonstration of a variety of topic-related vocabulary b. Excellent use and control of grammatical structures 4. Punctuation, spelling, and handwriting (5 points) a. Correct punctuation and no spelling mistakes b. Legible handwriting

QU Y

SUGGESTED ANSWER Some people think that children should begin their formal education at a very early age and should spend most of their time on school studies. Others believe that young children should spend most of their time playing. Compare these two views. Which view do you agree with? Why?

M

The question of when children should begin their formal education and how they should spend their time is a topic of ongoing debate. There are two contrasting views on this matter. Some people argue that children should start their formal education at a very early age and prioritize academic studies, while others believe that young children should spend most of their time playing. Both perspectives have their merits, and the choice between them depends on various factors.

DẠ

Y

Proponents of early formal education argue that starting at a young age provides children with a head start in acquiring foundational knowledge and skills. They believe that structured learning environments can foster cognitive development, enhance early literacy and numeracy skills, and lay the groundwork for future academic success. Advocates also suggest that early education can promote discipline, socialization, and a sense of routine, which are valuable for a child's overall development. On the other hand, supporters of play-based learning emphasize the importance of allowing children to explore and engage in unstructured activities during their early years. They argue that play is a natural and essential part of childhood, enabling children to develop creativity, problem-solving abilities, and social skills. Play-based approaches encourage self-discovery, imagination, and emotional development, allowing children to learn at their own pace and Page 9 of 10

develop a love for learning.

FI CI A

L

In considering these perspectives, it is important to recognize that both formal education and play have valuable roles in a child's development. A balanced approach that integrates both elements can be beneficial. Young children have a natural curiosity and an innate desire to learn, and play can serve as a vehicle for acquiring knowledge and skills. Combining play-based activities with intentional learning experiences can provide a well-rounded education that addresses different aspects of a child's development.

NH

ƠN

OF

Personally, I lean towards the view that emphasizes the importance of play in early childhood education. Play is an integral part of a child's development, fostering creativity, imagination, and social interactions. Through play, children explore and make sense of the world around them, developing problem-solving abilities and emotional resilience. While formal education has its benefits, I believe that a play-based approach in the early years allows children to cultivate a love for learning, which serves as a strong foundation for future academic pursuits.

DẠ

Y

M

QU Y

-------------- THE END --------------

Page 10 of 10

TRƯỜNG THPT CHUYÊN NGUYỄN TRÃI

FI CI A

L

ĐỀ ĐỀ XUẤT

KỲ THI CHỌN HỌC SINH GIỎI CÁC TRƯỜNG THPT CHUYÊN KHU VỰC DUYÊN HẢI VÀ ĐỒNG BẰNG BẮC BỘ LẦN THỨ XIV, NĂM 2023

ĐỀ THI MÔN TIẾNG ANH – LỚP 10

Đề thi gồm 11 trang

Thời gian: 180 phút (Không kể thời gian giao đề)

SECTION A: LISTENING (50 points) HƯỚNG DẪN PHẦN THI NGHE HIỂU

Bài nghe gồm 4 phần; mỗi phần được nghe 2 lần, mỗi lần cách nhau 05 giây; mở đầu và kết thúc mỗi phần nghe có tín hiệu. Thí sinh có 20 giây để đọc mỗi phần câu hỏi.

Mở đầu và kết thúc bài nghe có tín hiệu nhạc. Thí sinh có 03 phút để hoàn chỉnh bài trước tín hiệu nhạc kết thúc bài nghe.

Mọi hướng dẫn cho thí sinh (bằng tiếng Anh) đã có trong bài nghe

ƠN

OF

Part 1: You will listen to a short conversation about jobs in restaurants. For questions 1-5,

NH

complete the table using NO MORE THAN TWO WORDS AND/ OR A NUMBER for each answer. (10 points) Job title

Wivenhoe Street

Breakfast supervisor

M

(3) __________

QU Y

Location

Junior chef

Responsibilities

Pay and conditions

Checking (1) ________ Making sure equipment is clean

Starting salary 9.50 per hour Start work at (2) ________

Supporting the senior staff and (4) ______ Maintaining stock and arranging deliveries

Annual salary £23,000 No work on every (5) _______

Part 2: You will listen to a talk about Scandinavia. Listen and decide if the following statements are true (T) or false (F). (10 points) 6. The presence of state-controlled monopolies on alcohol in both Norway and Denmark contributes to elevated prices.

DẠ

Y

7. The Scandinavian nations need substantial taxation to sustain their welfare states. 8. Ikea has been transferred to a parent company in Scandinavia because of lower taxation rates in Sweden. 9. The substantial welfare state of the region is thought to guarantee its financial security, job stability, and equitable economic distribution.

Page 1

FI CI A

L

10. Scandinavian citizens receive highly affordable education, access to a free public transportation system , and a well-operating healthcare service. Part 3: For questions 11–15, listen to a radio discussion about children who invent imaginary friends and choose the correct answer A, B, C or D which fits best according to what you hear. (10 points) 11. In the incident that Liz describes A. her daughter asked her to stop the car. B. she had to interrupt the journey twice. C. she got angry with her daughter.

OF

D. her daughter wanted to get out of the car. 12. What does the presenter say about the latest research into imaginary friends?

QU Y

NH

ƠN

A. It contradicts other research on the subject. B. It shows that the number of children who have them is increasing. C. It indicates that negative attitudes towards them are wrong. D. It focuses on the effect they have on parents. 13. How did Liz feel when her daughter had an imaginary friend? A. always confident that it was only a temporary situation B. occasionally worried about the friend's importance to her daughter C. slightly confused as to how she should respond sometimes D. highly impressed by her daughter's inventiveness 14. Karen says that one reason why children have imaginary friends is that A. they are having serious problems with their real friends. B. they can tell imaginary friends what to do. C. they want something that they cannot be given. D. they want something that other children haven't got. 15. Karen says that the teenager who had invented a superhero is an example of A. a very untypical teenager.

M

B. a problem that imaginary friends can cause. C. something she had not expected to discover. D. how children change as they get older. Part 4: For questions 16-25, listen to a recording about the skin care industry in Korea. Write NO MORE THAN THREE WORDS AND/OR A NUMBER taken from the recording for each answer. (20 points)

DẠ

Y

It is anticipated that the skin care industry's revenue will experience growth over the next four years to more than (16) __________. Skin care is a big moneymaker for big (17) ___________. Skin care makes up (18) __________ of the annual revenue of the world beauty industry. According to Larissa Jensen, skin care is actually growing the fastest of all the beauty categories tracked, including (19) ___________.

Page 2

FI CI A

L

Wellness is associated with clean eating, the latest (20)___________ and the no-makeup look. In South Korea, the latest skin care innovations are (21)___________ and large manufacturing takes place first. Charlotte Cho is a Korean (22) ___________, an entrepreneur, and the author of "The Little Book of Skin Care”. She firmly believes Korea has allowed (23)___________ to come to the forefront of a lot of the skin care trends. South Korea is both a huge (24) __________ and a large exporter of beauty products.

OF

Many beauty brands from Europe and America are making their products with (25) ___________ chemists and manufacturing plants.

NH

ƠN

SECTION B: LEXICO-GRAMMAR (40 POINTS) Part 1: Choose the best answer (A, B, C or D) to each of the following questions and write your answer (A, B, C or D) in the corresponding numbered boxes. (20 points) 26. Major political party is ______ the campaign for tighter gun controls in the wake of last month’s shooting. A. procuring B. solidifying C. spearheading D. fulfilling 27. Despite his poor performance in last month’s athletics trials, Jack is still ______ for a medal in

M

QU Y

next week’s race. A. in the running B. by a mile C. on his plate D. on the go 28. In its ______, the city of Bath was a magnet for wealthy and fashionable people from all over the country. A. dream B. foothold C. prestige D. heyday 29. Jenny ______ her audition and was immediately offered the part! A. went up B. scraped through C. took off D.waltzed through 30. The money laundering scandal lead to the minister’s ______ fall from political power. A. prodigious B. precipitous C. dismal D. persistent

DẠ

Y

31. The CEO was found guilty of ______; nobody could believe that he had been deceiving the company for so many months. A. trafficking B. arson C. embezzlement D. hooliganism 32. The ______ windows of the house seemed to indicate that it had been derelict for a long time. A. faced-up B. long-awaited C. semi-detached D. boarded-up 33. The office was closed for a week for refurbishment and now the staff have to deal with the _____ that built up during their absence. A. backstage B. backlash C. backlog D. backdrop 34. Garlic being sold as organic was found to contain pesticide _____ and was removed from the market.

Page 3

FI CI A

L

A. elements B. residues C. filaments D. variables 35. Sam has no ______ for the night of the crime and, therefore, he is the main suspect in this case. A. verdict B. surveillance C. alibi D. witness 36. I don’t think it would be wise to try to make Max change his mind about divorcing Barbara. Well, in his place I _____ her at all. A. would never have married B. needn’t have married C. would never marry D. must never have married 37. I think Candy _____ the prize if she plays this well during the competition. A. is in for winning B. is bound to win C. may as well win D. is set for winning

NH

ƠN

OF

38. Find someone who will let you talk things through, or _____ that, write down your thoughts. A. except B. failing C. for all of D. given 39. _____, water is required by all living things on Earth and life cannot survive without it. A. It is known as far B. Known as far as it is C. As known as far D. As far as is known 40. _____, he tried to fix the machine. A. Not being a computer expert notwithstanding B. Despite of not being a computer expert C. Though no computer expert D. In spite of being no computer expert

QU Y

41. For the first few months the babies looked so alike I couldn't tell _____. A. who is whom B. which is which C. which from which D. whom with whom 42. _____, the meeting began. A. After we have sat down B. All of us having taken the seats C. Our having seated D. Once we had seated 43. They must have gone away, _____? A. shouldn’t they B. haven’t they C. mustn’t they D. didn’t they 44. Kate: ‘Did the minister approve the building plans?’

M

Bill: ‘Not really. He turned them down _____ that the costs were too high.’ A. on the grounds B. provided C. in case D. supposing 45. The draw took place yesterday but the competition winners _____. A. are yet to be announced B. haven’t been yet announced C. are as yet to have been announced D. haven’t announced yet Part 2: Give the correct form of each given word to complete the following sentences. Write your

DẠ

Y

answers in the numbered boxes. (10 points) 46. I am glad that the children have taken advantage of the long walk so ___________. (felicity) 47. The _________ of the technology comes at the end of a long period of innovation. (calcium) 48. Is your name_____________? (hyphen) 49. Many people without jobs are living on the___________. (bread) 50. The inspection of a vehicle's steering mechanism and tyres is designed to ensure that it is ________ in terms of its steering characteristics. (road)

Page 4

FI CI A

L

51.The majority of infected persons are ____________ and unaware of their condition. (symptom) 52.The two losers got to follow in the footsteps of their fellow ____________ by walking the plank. (patriotic) 53. Casting a vote in a _________ is not exactly the same as advancing an opinion in a survey. (refer) 54. The new system __________ with existing telephone equipment. (face) 55. The principal has taken the students’ petition under _____. (advise)

in the boxes provided on the answer sheet. (10 points) 56. It was too late to back ___________ the contract.

OF

Part 3. Complete each of the following sentences with suitable preposition(s). Write your answer

57. Their warnings about the recent robbery put her ___________ the defense. 58. Alice is going to put ________ the job of a sales assistant in Newcastle.

NH

ƠN

59. The conflict in the production department boils _______ the employees’ dissatisfaction with their wages. 60. Tiredness can easily creep ________ you while you’re driving. 61. I can only admire his conscientiousness. It’s over and _________ the call of duty. 62. The house was near a park but there was a road in ___________. 63. You’ll have to tell me the truth. Stop trying to pull the wool __________ my eyes.

QU Y

64. His jokes seemed to go _________ very well with his audience, if their laughter was any indication. 65. He put it _________ that he was thinking of leaving the company and, as a result, he received several offers from rival organisations.

M

SECTION 3: READING (60 points) Part 1: For questions 66-75, read the following passage and decide which option (A, B, C, or D) best fits each gap. Write your answers in corresponding numbered boxes on the answer sheet. (15 points) NATURE’S CLOCKS

DẠ

Y

Our biological clocks govern almost every aspect of our lives. Our sensitivity to stimuli (66)__________over the course of the day, and our ability to perform certain functions is subject to fluctuations. Consequently, there is a (n) (67) ___________ time for tasks such as making decisions: around the middle of the day. Anything that (68) ___________ physical co-ordination, on the other hand, is best attempted in the early evening. What is more, there is a dramatic drop in performance if these activities are (69) __________ out at other times. The risk of accident in a factory, for example, is 20% higher during the night shift. Primitive humans lived their lives in tune with daily cycle of light and dark. Today we are (70)__________convinced that we can impose schedules on our lives at (71)___________ Sooner or later, however, we pay a price for ignoring our natural rhythms. A good example is jet lag, caused when we confuse our body’s biological clocks by (72) _________several time zones.

Page 5

take antidepressant drugs. 66. A. modifies B. ranges

C. varies

67. A. peak 68. A. requests

B. summit B. demands

C. maximum C. dictates

69. A. made 70. A. powerfully 71. A. whim 72. A. landing 73. A. errors 74. A. peak 75. A. triggered

B. done B. firmly B. determination B. penetrating B. mistakes B. pinnacle B. developed

C. carried C. steadily C. will C. crossing C. inaccuracies C. top C. created

FI CI A

L

People suffering from jet lag can take several days to adjust to new time zones, and have a reduced ability to make decisions, which is a worrying thought, as serious (73)__________ of judgment can be made. And this may be just the (74) __________ of the iceberg. An increasing number of people suffer from seasonal affective disorder (SAD), a form of depression that can be (75) _________ by living in artificial conditions SAD can be serious, and suffers may even need to D. wavers

D. optimum D. stipulates

ƠN

OF

D. performed D. highly D. desire D. travelling D. fallacies D. tip D. launched

NH

Part 2: For questions 76-85, fill each of the following numbered blanks with ONE suitable word and write your answers in the corresponding numbered boxes on the answer sheet. (15 points)

M

QU Y

If a picture is (76) ______ a thousand words, a map may equate (77) ______ a million. Maps, infographics, and photographs blur the (78) ______ between information and art, at once documenting the state of the world in which information is presented in a (79) _______ dynamic way. Unlike photographs, however, no map can (80) ______ to be a direct portrait of reality. Every map, no (81) _______ how simple it appears, is shaped by the viewpoint and the choices of the person who made it. On (82) ______ of that, every person who reads a map will (83) ______ it differently than the next. Our interpretations of maps are influenced (84) ______ as much by our perceptions and personal experiences as by what's actually on the page. For this reason, understanding subjectivity has become an important part of both geography and cartography. The long history of cartography has led people to trust maps as a (85) ______ of information about the

DẠ

Y

world. Part 3: For questions 86-95, read the following passage and circle the best answer to each of the following questions. Write your answers in corresponding numbered boxes provided on the answer sheet. (15 points) Simply ticking the boxes isn’t enough I have been asked what I think about the idea of ‘Investing in People’. The best answer I can give is that I think what it tries to achieve - basically making the link between business improvement and focusing on the needs of the people who work for an organization - is great. My problem is with organizations who subscribe to it as a way to help the ‘get better’, when they don’t bother to understand where they went wrong in the first place. They need to ask what explicit and implicit policies and procedures they have in place that prevent their people from being able to do

Page 6

FI CI A

L

the right thing for the right reasons. I am sure that there are managers out there who don’t know any better, and assume that to manage they simply need to put pressure on their people to perform. But people don’t demonstrate high performance because they see the need to do it, and make the choice to do so. They do it because they are connected to the business goals and they see how their contributions can help achieve them. Such managers may tell themselves they can put a ‘tick’ in the ‘we care about people’ box. But simply putting ticks in boxes is no good if it doesn’t reflect reality.

I know of a company that was so concerned that its people were doing the ‘right thing’ that it put in place a series of metrics to measure their effectiveness. So far, so good. But one of the

QU Y

NH

ƠN

OF

objectives - making successful sales calls - manifested itself in the metric ‘Number of potential customers seen in one day’. The sales people obviously focused their efforts on going from one customer’s office to another, and not on closing deals. Instead of the employees becoming more effective, they focused on getting the boxes ticked. Good intent; poor thinking. Another company wanted to improve the speed with which it was able to introduce new products. Competition was beating it to the market place, and consequently the company was losing market share. Senior management sent out the message to reduce the time spent in getting products into customers' hands, with the explanation that they couldn't afford delays. This was a relatively easy task, especially since the time spent testing the products was cut in half to accomplish the time reduction. The result was new products were introduced in less time than those of competition – but soon rejected by customers for poor quality. Good intent; reckless implementation. A third company I know is trying hard to help employees see that they have some control over their future. The company instituted a programme with a title like ‘Creating our own future’ or something like that. A good idea; get the people involved in the future of the company. But instead of the employees becoming motivated to contribute, they saw it as a hollow exercise on

M

the part of senior management who, in the past, had paid little attention to anything other than getting the job done so they could report great earnings. Yes, the programme was a big ‘tick the box’ effort, but that was all it was in the minds of the people that it was designed for. A final example is of a company that brought in one of their ‘Investing in People’

programmes to change the way the company was run. Assessors were running around like crazy, heling managers examine how they managed. They told managers how they could manage better. And when the programme was over, the company was able to say that they had done it - it had invested in its people and life was now good. But the managers simply went back to business as

DẠ

Y

usual. After all, the assessors were gone, and they had targets to hit. [95A] All these examples are representative of senior management who see the need to improve things in their organization, but don't see how to do it. [95B] And when the employees simply see the programme as a box-ticking exercise, then it's hopeless. If a company is going to go through the effort implied in investing in people, it should make it worthwhile. [95C] Defaulting on the choice to improve the decision-making process by going through the motions is as lame as senior management saying their people’s poor performance is not the senior management’s fault.

Page 7

L

FI CI A

[95D] 86.The writer thinks that putting the concept of “Investing in People” into practice………. A. frequently results in confusion among the people it is supposed to help B. involves more effort than some organizations are prepared to make C. may create problems where previously there had not been any problems

D. is something that some organizations should not attempt to do 87. The writer’s main point in the second paragraph is that the performance of employees……… A. may be very good even if management is poor B. cannot be accurately measured by any box-ticking exercise

ƠN

OF

C. is related to their knowledge of the organization as a whole D. is not as unpredictable as some managers believe it to be 88. What point does the writer make about the first company he describes? A. It was not really interested in measuring the effectiveness of employees. B. The targets that it set for staff were unrealistic. C. It failed to understand the real needs of its employees. D. The data that it collected did not measure what it was supposed to measure. 89. The word “objectives” in paragraph 3 is closest in meaning to………….

DẠ

Y

M

QU Y

NH

A. purposes B. goals C. motives D. reasons 90. What point does the writer make about the second company he describes? A. It made what should have been an easy task into a complicated one. B. It failed to foresee the consequences of an instruction. C. It misunderstood why a new approach was required. D. It refused to take into account the views of employees. 91.What does the writer say about the programme introduced by the third company he mentions? A. Employees did not believe that it had been introduced for their benefit. B. Employees felt that it was in fact a way of making their jobs even harder. C.The reason given for introducing it was not the real reason why it was introduced. D. It was an inappropriate kind of programme for this particular organization. 92. The word "hollow” in paragraph 5 is closest in meaning to……... A. not sincere B. without real value C. empty D. deep 93. The writer says that the programme in his final example………... A. was too demanding for managers to maintain long-term B. was treated as a self-contained exercise by managers C. involved some strange ideas on how managers could improve D. cause managers to believe that their previous methods had been better 94. The word "it" in the last paragraph refers to……….. A. senior management B. improving things in things in the organiztion C. organization D. the need

Page 8

FI CI A

L

95. Look at the four options [95A], [95B], [95C] and [95D], which indicate where the following sentence could be added to the passage. Where would the sentence best fit? For a start, a programme targeted at improving things is only as good as management's ability to motivate their people. A. [95A] B. [95B] C. [95C] D. [95D] Part 4. For questions 96-105, read the passage and do the following tasks. Write your answers in the corresponding numbered boxes on the answer sheet. (15 points) MENDING BROKEN HEARTS

Although hearts suffer many maladies-valves leak, membranes become inflamed – coronary heart disease, which can lead to heart attack and ultimately to heart failure, is the number one killer of both men and women in the United States, where 500,000 the annually. Worldwide, it kills 7.2 million people every year. Exacerbated by the Western lifestyle – motorized transport, abundant meat and cheese, workdays conducted from the comfort of a well-padded chair – incidence of the disease is soaring. To help stem this lethal tide, cardiologists can prescribe such cholesterol-lowering drugs as statins to help keep arteries clear. They can advise patients to change their habits, or they can operate to fix a immediate problem. Angioplasty is one procedure, and surgery to bypass the diseased arteries is another – each year more than 400,000 bypasses are

NH

B.

ƠN

OF

A.

QU Y

M

C.

performed in the U. S. Transplants can replace severely damaged hearts, and artificial ones can keep people alive while they wait for a donor heart. But in the face of an impending global epidemic, none of these stopgap measures addresses the essential question: Who gets heart attacks and why? The human heart beats 100,000 times a day, propelling six quarts (5.6 liters) of blood through 60,000 miles (96,560.6 kilometers) of vessels – 20 times the distance across the U. S. from coast to coast. The blood flows briskly, surging out of a ten-ounce (283.5 gram) heart so forcefully that large arteries, when severed, can send a jet of blood several feet into the air. Normally the relentless current helps keep blood vessels clean. But where an artery bends, tiny eddies form, as in a bend in a river. This is where bits of sticky, waxy

DẠ

Y

D.

cholesterol and fat can seep into the artery wall and oxidize, like butter going rancid. Other matter piles up too. Eventually, the whole mas calcifies into a land of arterial stucco, or plague. Until recently, cardiologists approached heart diseases as a plumbing problem. Just as mineral deposits restrict the flow of water through a pipe, an accretion of plague impedes the flow of blood through an arterial channel. The more crud in the system, the greater likelihood that a dammed artery will trigger a heart attack. Doctors now dismiss this “clogged-pipes model” as an idea whose time has passed. It’s just not that simple. Most heart attacks are caused by plague embedded within the artery wall that ruptures, cracking the wall and triggering the formation of a blood clot. The clot flocks the flow of

E.

Page 9

L

FI CI A

F.

blood to the heart muscle, which can form the lack of oxygen and nutrients. Suddenly, the pump stops pumping. Contrary to the clogged pipes model, heart attacks generally occur in arteries that have minimal or moderate blockage, and their occurrence depends more on the kind of plague than on the quantity. Scientists have been struggling to figure out what type is most responsible. Paradoxically, findings suggest that immature, softer plague rich in cholesterol are more unstable and likely to rupture than the hard, calcified, dense plagues that

extensively narrow the artery channel. But understanding the root cause of the disease will require much more research. For one thing, human hearts, unlike plumbing fixtures, are

OF

NH

H.

ƠN

G.

not stamped from a mold. Like the rest of our body parts, they are products of our genes. Don Steffensen was putting duck-hunting decoys out on a small lake one fall afternoon in South Western Iowa when his heart attack hit. The infarction was massive and unexpected. It’s likely that Steffensen survived only because a buddy was carrying nitroglycerin tablets and quickly slipped under his friend’s tongue. Nitroglycerin is used to make dynamite; in the body, a heavily diluted form releases nitric oxide, which signals the smooth muscle cells in veins and arteries to relax, dilating the vessels. The Steffensen clan is enormous: more than 200 relatives spread over three generations. many of the youngest are now dispersed from Iowa to New York and beyond. Although heart trouble is common in the family, it had never struck anyone as unusual. “I attributed

M

QU Y

it to diet,” shrugs Tina, a slim 38-year-old and the family’s only vegetarian. I. It was a reasonable conclusion. The Steffensens were raised on the kind of farm food that the state is famous for – ham balls, meatloaf, pie, macaroni and cheese – and still popular even as careers have moved indoors. Driving north through cornfields to meet some of the family in Buffalo Center, I dined at a restaurant offering deep-fried sandwiches. A single ham and cheese hoagie – clunked in hot fat and served sizzling – seemed capable of stopping a heart all on its own. Questions 96 – 102: Do the following statements agree with the information given in the passage? WRITE YES if the statement agrees with the information

DẠ

Y

NO if the statement contradicts the information NOT GIVEN if there is no information on this 96. Coronary heart disease is the largest culprit behind the deaths in the United States. 97. The Western lifestyle is the largest cause for the increase in the diseases. 98. Measures taken by experts have successfully answered the essential questions about heart attack. 99. Blood in human body could travel much more distance throughout the body on a daily basis than that across the U. S. from coast to coast. 100. Cholesterol is stored in the arteries to provide energy for various functions of the body. 101. The clogged pipes model is accepted by most doctors and specialists. 102. Scientists have yet to decide the most likely factor that leads to heart attack.

Page 10

FI CI A

L

Questions 103-105: Complete each sentence with the correct ending A – G below based on the information in paragraphs A – E. 103. Cardiologists prescribe cholesterol-lowering drugs in order to ___________. 104. Artificial hearts are introduced to ______________. 105. Heart diseases in arteries are compared to ___________. A. reduce the costs of transplantation surgery B. the blood clots – a kind of arterial stucco, or plague

ƠN

OF

C. deposits of minerals limiting the flow of water through a pipe D. smooth the arteries E. cope with difficult situations F. braincase the blood from flowing to the heart muscle and interfere with the absorption of oxygen and nutrients G. help patients survive until the availability of a donated human heart

NH

SECTION D: WRITING (50 points) Part 1: (20 points) The chart shows the percentage of male and female teachers in six different types of educational

QU Y

setting in the UK in 2010. Summarize the information by selecting and reporting the main features, and make comparisons where relevant. Part 2 (30 points): Write an essay to elaborate your opinion on the topic: What should teenagers do to become responsible digital citizens? You should use your own ideas, knowledge and experience and support your arguments with examples and relevant evidence. Write about 250 words.

DẠ

Y

M

--------------------------------------- HẾT-------------------------------------------

Page 11

TRƯỜNG THPT CHUYÊN

KỲ THI CHỌN HỌC SINH GIỎI CÁC TRƯỜNG THPT CHUYÊN KHU VỰC DUYÊN HẢI VÀ ĐỒNG BẰNG BẮC BỘ LẦN THỨ XIV, NĂM 2023

NGUYỄN TRÃI

FI CI A

L

ĐỀ ĐỀ XUẤT

HƯỚNG DẪN CHẤM: MÔN TIẾNG ANH – LỚP 10

SECTION A: LISTENING (50 points)

2. 5.30 am

3. City Road

4. sous chef/ sous-chef

ƠN

1. portions

OF

Part 1: You will listen to a short conversation about jobs in restaurants. For questions 1-5, complete the table using NO MORE THAN TWO WORDS AND/ OR A NUMBER for each answer. (10 points) 5. Monday

Part 2: You will listen to a talk about Scandinavia. Listen and decide if the following statements are true (T) or false (F). (10 points) 7. T

8. F

NH

6. F

9.T

10. F

Part 3: For questions 11–15, listen to a radio discussion about children who invent imaginary friends and choose the correct answer A, B, C or D which fits best according to what you hear.

QU Y

(10 points) 11. B

12.C

13.A

14.C

15.C

Part 4: For questions 16-25, listen to a recording about the skin care industry in Korea. Write NO MORE THAN THREE WORDS AND/OR A NUMBER taken from the recording for each answer in the spaces provided. (20 points) 17. beauty 18. about 24.9% 19. makeup and 20. fitness fads brands fragrance

21. concocted

22. beauty guru

23. indie brands 24. manufacturing hub

25. R&D

Korean

Y

M

16. 379 billion dollars/ $379 billion

DẠ

SECTION B: LEXICO-GRAMMAR (40 POINTS) Part 1: Choose the best answer (A, B, C or D) to each of the following questions and write your answer (A, B, C or D) in the corresponding numbered boxes. (20 points) 26. C

27. A

28. D

29. D

30. B

Page 1

33. C

34. B

35. C

36. B

37. B

38. B

39. D

40. C

41. B

42. B

43. D

44. A

45. C

L

32. D

FI CI A

31. C

Part 2: Give the correct form of each given word to complete the following sentences. Write your answers in the numbered boxes. (10 points) 47. calcification

48. hyphenated

49. breadline

50. roadworthy

51. asymptomatic

52. compatriots

53. referendum

54. interfaces

55. advisem*nt

OF

46. felicitously

57. on

58. in for

61. above

62. between

63. over

59. down to

60. up on

64. down

65. about

NH

56. out of

ƠN

Part 3. Complete each of the following sentences with suitable preposition(s). Write your answer in the boxes provided on the answer sheet. (10 points)

QU Y

SECTION C: READING (60 points) Part 1: For questions 66-75, read the following passage and decide which option (A, B, C, or D) best fits each gap. Write your answers in corresponding numbered boxes on the answer sheet. (15 points) 66. C

67. D

68. B

69. C

70. B

71. C

72. C

73. A

74. D

75. A

M

Part 2: For questions 76-85, fill each of the following numbered blanks with ONE suitable word and write your answers in the corresponding numbered boxes on the answer sheet. (15 points)

76. worth

77. to

78. line

79. visually

80. claim

81. matter

82. top

83. interprete

84. just

85. source

DẠ

Y

Part 3: For questions 86-95, read the following passage and circle the best answer to each of the following questions. Write your answers in corresponding numbered boxes provided on the answer sheet. (15 points) https://engexam.info/cae-reading-and-use-of-english-practice-tests/cae-reading-and-use-ofenglish-practice-test-2/5/

Page 2

87. C 92. B

88. D 93. B

89. B 94. B

90. B 95. B

L

86. B 91. A

in the corresponding numbered boxes on the answer sheet. (15 points) 1.YES

2.NG

3.NO

4.YES

6.NG

7.YES

8.D

9. G

5.NG 10.C

OF

SECTION D: WRITING (50 points) Part 1: (20 points)

FI CI A

Part 4. For questions 96-105, read the passage and do the following tasks. Write your answers

The chart shows the percentage of male and female teachers in six different types of educational setting in the UK in 2010.

Contents (10 points)

QU Y

NH

ƠN

Summarize the information by selecting and reporting the main features, and make comparisons where relevant.

M

The report MUST cover the following points: - Introduce the bar chart (2 points) and state the striking features (2 points)

+ female teachers dominated in nursery, primary and secondary schools + male teachers dominated in private training institutes and universities reversed. + equal rates in colleges - Describe main features with relevant data from the charts and make relevant comparisons (6 points)

DẠ

Y

Language use (10 points) The report: - should demonstrate a wide variety of lexical and grammatical structures - should have correct use of words (verb tenses, word forms, voice...) and mechanics (spelling, punctuations,)

Page 3

Part 2 (30 points): Write an essay to elaborate your opinion on the topic:

examples and relevant evidence. Write about 250 words. The mark given to part 3 is based on the following criteria: 1. Organization (5 points)

FI CI A

L

What should teenagers do to become responsible digital citizens? You should use your own ideas, knowledge and experience and support your arguments with

a. Ideas are well organized and presented with coherence, cohesion and unity. b. The essay is well-structured:

OF

* Introduction is presented with clear thesis statement. * Body paragraph are written with unity, coherence and cohesion.

Each body paragraph must have a topic sentence and supporting details and examples when necessary.

ƠN

* Conclusion summarizes the main points and offers personal opinions (prediction, recommendation, consideration ...) on the issue. 2. Content (15 points) a. All requirements of the task are sufficiently addressed.

NH

b. Ideas are adequately supported and elaborated with relevant and reliable explanations, examples, evidence.... 3. Language use (10 points) a. Demonstration of a variety of topic-related vocabulary.

Listening transcript

QU Y

b. Excellent use and control of grammatical structures (verb tenses, word forms, voice...) and mechanics (spelling, punctuations...)

DẠ

Y

M

Part 1 Source: Part 1, test 2, Cmbridge ielts 18

Page 4

L FI CI A OF ƠN NH QU Y

Part 2: https://www.youtube.com/watch?v=-VJX2W3O9Ek&t=174s Across Scandinavia, the average price of everything from a knitted roll neck to pastries from the local bakery are some of the highest in the world. Scandinavia is a region in northern Europe that was historically made up of three kingdoms:

M

Norway, Sweden, and Denmark. They’re united by their Viking heritage but also their shared reputation as one of the most expensive regions to live and visit. Whether it’s buying a car or a

TV, consumers in Scandinavia pay some of the highest prices in the world. Even buying groceries is costly with all three countries’ food and drink prices above the European average. All three countries also have a standard VAT rate of 25%. Amongst all the European nations, only Hungary has a higher rate. Heading to the bar is also pretty pricey. In Norway, having a

Y

beer or glass of wine will cost 2.5 times the EU average. Both Norway and Sweden have staterun monopolies on alcohol which helps keep the prices up. This social market model, rather than

DẠ

a liberal market model like in the U.S., is common across Scandinavia and helps explain why the cost of living is so high. Scandinavian countries have large welfare states with their social expenditures, as a percentage of GDP, among the highest in the world and this requires high levels of taxation.

Page 5

Sweden has a top rate of personal income tax of over 60%, while Denmark’s is more than 55%,

L

both of which are well above the OECD average. In Denmark, if you want to buy a car you have to pay anywhere between 85% and 150% tax on top of the cost of the vehicle.

FI CI A

And the taxes don’t stop there. Let’s say I want to buy a sweater in Denmark.

I pay 300 kronor for it, but how much of that money actually goes to the vendor? Well, first there’s the 25% VAT - leaving the seller with 240 kronor.

The clothes shop also has to pay a minimum 22% of corporate income tax. That means that a big portion of the money I paid for my sweater goes to the Danish

OF

government. Not to mention there’s a hefty payroll tax on employees' wages and the store still has to pay for rent, electricity, and cleaning - all of which are taxed, too.

These taxes mean that for Scandinavian businesses to make a profit, they need to charge their consumers high prices. For some companies, this has proved too much of a burden for their business model.

ƠN

The world’s biggest furniture company IKEA was founded in Sweden but has moved its headquarters to the Netherlands. Through corporate restructuring, the business is now owned by a non-profit Dutch parent company. In part due to the high taxes in Sweden.

NH

But according to some experts, Scandinavia’s social democrat tendencies have led to a strong social cohesion and has helped provide political stability. This, in turn, has made their economies safe havens for outside investors - which is one reason why the Danish, Swedish and Norwegian currencies called krona are so strong.

QU Y

This can be tough on tourists with the exchange rate doing them no favors. If the cost of living in Scandinavia is so expensive, why then are its citizens some of the happiest in the world? In the past five years, Scandinavian countries have regularly topped the World Happiness Report. That’s an index that measures overall life satisfaction based on different contributing factors. Some experts attribute these high satisfaction levels with Scandinavia’s large welfare state which they say ensures financial security, job security, and economic distribution. In return

M

for high taxes, citizens get free state education, very cheap child care, a functioning public transport and a free health service.

But a large and expensive social welfare state doesn’t necessarily mean the best. For example, Norway is the only Scandinavian country that ranks in the top 10 for adult education levels, amongst OECD countries. And yet Scandinavians remain happy with the status quo, living within a social corporatist

Y

economy that provides reliable economic welfare. Having a beer at the end of the day does cost a small fortune, but for people living

DẠ

and working in Scandinavia, high prices provide a quality of life that’s worth paying for. Part 3

Presenter: Today we're talking about children and their tendency to have imaginary friends. Liz McManus has a daughter called Caitlin, who's eight now. When she was three, she had an imaginary friend called Tytner. Liz, tell us about Caitlin and Tytner.

Page 6

Mother: Well, give you an example. One day I was driving Caitlin and Greg, her baby brother,

L

home when she solemnly informed me that Tytner was hitting the baby. So I said: 'You tell Tytner that if he does that again, he'll be walking home.' Fifteen seconds later came the inevitable news:

FI CI A

'He's just done it again, Mummy.' So I found myself in the embarrassing position of having to pull over, open the back door and say to this imaginary little boy. `Tytner, out, now!' And of course, as

we drove off, Caitlin started crying because her friend was standing on the pavement all alone. I had to turn back and go through the rigmarole of pulling over and opening the door to pick him up again. Presenter: Wow, that's some story! But in fact, Caitlin is no different from many children and her

OF

invented, make-believe friend is far from unusual. As many as 65% of children have had an imaginary friend at some point in their lives. The latest research suggests that invisible friends, far from being a cause for concern, should be welcomed by parents because they can help children to be more creative, confident and articulate, and have more advanced communication skills. It is thought that these findings will help reverse misconceptions about

ƠN

children with imaginary friends and that they will come to be seen as having an advantage, rather than a problem that needs to be worried about. Did it worry you, Liz? Mother: I know it does lots of parents but I never fretted about it, I think I was just amused.

NH

I'd be reading to her and I'd say, 'Is Tytner around?' and she'd say, 'Yes, he's just sitting at the end of the bed.' He became the centre of her life. She'd have tea parties with him, and he'd go to bed with her. She was shy and this was her answer. I knew she would grow out of it. Presenter: Now Liz is one of 15 people taking part in a study of imaginary friends at the Institute

QU Y

of Education in London, run by Karen Majors, an education psychologist and lecturer at the institute. Karen, should parents worry about it? Expert: Well, parents sometimes think, 'Is this healthy and how long should it go on for?' But it is a normal phenomenon for normal children. And it's very healthy. Presenter: Why do children invent imaginary friends? Expert: I think that children create pretend friends for many reasons: as safe, trustworthy best

M

friends at a time when they are just starting to make real friends; as someone to confide in; and as someone to play with. Sometimes it is about wish fulfilment; children who cannot have a pet,

for example, will invent one. I interviewed one little girl, aged six, who had a pony called Minty for several years. It went to school with her and the teachers knew all about it. It was a really strong relationship. Presenter: Presumably, when they get older, children no longer have these imaginary friends.

Y

Karen? Expert: Well, my most surprising finding is that children don't always stop having these made-up

DẠ

playmates when they start school. The imaginary friends often stay with them through their teenage years, providing comfort and escape - although in secret. One teenager I talked to had invented a superhero to help him through tricky patches. When things hadn't gone well at school, he would come home and play with the superhero, for whom everything always went well. Presenter: How should parents treat these invisible people, Karen?

Page 7

Expert: Well, sometimes of course, parents get irritated by them - for instance, if a child insists

L

on having the playmate at the dinner table with an imaginary setting and glass. Actually I myself had a friend called Tiger when I was young, who would sit beside me at mealtimes. But I don't

FI CI A

think parents should tell children off for this kind of thing, or tell them that their friends are not real. Perhaps the best way is Liz's down-to-earth approach.

Presenter: How did you handle it, Liz? Mother: Well, I patiently acknowledged Caitlin's playmate but I tried not to get involved. I never used to have to get out of the friend's way or anything. Other than that one incident in the car, Caitlin's imaginary friend didn't impinge on my life.

OF

Expert: Yes, I agree that parents should recognize imaginary friends, but they shouldn't try to overly influence the friendship. Parents who interfere too much risk driving their children's playmates away. If they try to direct the friends, they could spoil the fantasy altogether. Presenter: Fascinating subject, thanks for coming in to talk about it, Liz and Karen.

ƠN

Part 4: https://www.youtube.com/watch?v=rS6Ifm_YecQ&t=274s The global cosmetics industry makes a ton of money. That includes shampoo, makeup, perfume, cologne, deodorant, soap.

NH

The list goes on. Analysts expect the industry's revenue to grow within the next four years to more than $379 billion. But when you break down that number, it's skin care that's driving much of the growth. And it's

QU Y

not slowing down. I've used moisturizers upwards in like 250 dollar range.

The most expensive single item would be around 50 dollars. I try not to spend more than 50 for in essence or moisturizer. Probably like this really expensive moisturizer that I kind of got conned into buying for about 150 dollars.

M

Skin care is an increasingly lucrative business for cosmetic companies. That's why big names like Amazon and the Kardashians are trying to cash

in. And there's one country that's been playing an outsized role on skin care. South Korea. Here's why. Why is skincare so popular Skin care is a big moneymaker for big beauty brands.

Y

Over the last five years, skin care has grown so much that it's become the largest piece of the pie when you break apart the beauty industry by

DẠ

product category. It makes up about 24.9 percent of a total $52.4 billion in annual revenue. And the global skin care products market size is expected to reach more than $196 billion by 2024. But what is making skin care so popular? And, why are these bottles of face wash and tubs of lotion bringing in so much money?

Page 8

This is Larissa Jensen.

L

She's a skin care and beauty industry analyst with the NPD Group. Skin care has been growing very fast.

So, it's been the strongest performer for the past couple of years. What's really driving a lot of that performance is actually wellness and health, and you know, really natural brands. That's what's really driving the performance of skincare.

FI CI A

It's actually growing the fastest of all the beauty categories that we track, including makeup and fragrance.

OF

Wellness doesn't mean just being healthy. In 2019, it means clean eating, the latest fitness fads and the no-makeup look, which actually does involve some makeup. At any rate, the wellness trend is driving consumers to take better care of their skin. This skin-first philosophy did not originate in the U.S.

ƠN

Instead, it was born on the other side of the world, where skin care is a part of the culture. It's where the latest skin care innovations are concocted and where a large chunk of manufacturing happens in the first place.

NH

- Hi guys. It's Charlotte.

This is Charlotte Cho. She's an entrepreneur, Korean beauty guru and the author of "The Little Book of Skin Care."

QU Y

She started Soko Glam, an online marketplace full of curated K-beauty products for U.S. consumers. I was born and raised in California. I didn't know anything about skin care until I started to live and work in Seoul, South Korea, and I was completely amazed at the product selection there was in Korea. And also the real focus on a skin-first philosophy. So, every woman and man in Korea it seemed really knew how to take care of their skin and at

M

an early age. They had categories that just simply did not exist in the U.S., so it was really intriguing to me

and I saw results on my own skin. I firmly believe that K-beauty created a skin care wave. They opened the door to innovations. They've allowed indie brands to come to the forefront of a lot of the skin care trends. They've also widened the appetite for new products, new categories.

Y

They've also been a big part of the education around skin care. Koreas Cosmetics Exports

DẠ

South Korea's cosmetics exports have exploded in recent years. In 2014, exports were over 1.7 billion, and then in 2017, they had grown nearly 5 billion. Not only is South Korea a large exporter of beauty products, but is also a huge manufacturing hub, even for American brands.

Page 9

to more than 13 billion in 2017. I think the impact of K-beauty goes far beyond just brands that are now introduced into this

FI CI A

global marketplace. It's actually allowing Korean manufacturing companies to grow rapidly as

L

Local cosmetic production there went from 8.5 billion dollars worth of goods in 2014 and grew

DẠ

Y

M

QU Y

NH

ƠN

OF

well. For example, a lot of European brands and U.S.-based brands are formulating their products with Korean R&D chemists and manufacturing plants.

Page 10

FI CI A

L

HỘI CÁC TRƯỜNG CHUYÊN VÙNG ĐỀ THI CHỌN HỌC SINH GIỎI LẦN THỨ DUYÊN HẢI VÀ ĐỒNG BẰNG BẮC BỘ XVI TRƯỜNG THPT CHUYÊN NGUYỄN MÔN: TIẾNG ANH - KHỐI 11 TRÃI Thời gian: 180 phút TỈNH HẢI DƯƠNG Đề thi gồm: 18 trang ĐỀ ĐỀ XUẤT

DẠ

Y

M

QU Y

NH

ƠN

OF

SECTION 1: LISTENING (50 points) I. You will hear an introduction to a radio phone-in programme about modern lifestyles. Listen and indicate true (T) or false (F) statements. (10 points) Statements T F 1. Ron is a record-breaking athlete. 2. Ron thinks an accountant can lead a health and fulfilled life. 3. “Total Living” is believed to be good for athletes. 4. “Total Living” means that we should develop one aspect of our life to the full. 5. According to Ron Clarke, some current health trends are harming us. II. You will hear a short radio report about how technology is helping archaeologists who want to learn more about some texts written over 2,000 years ago known as Roman tablets. Listen and give short answers to the questions. Write NO MORE THAN FIVE WORDS AND/OR A NUMBER taken from the recording. (10 points) 1. Where did archaeologists discover about 200 tablets? ……………………………………………………………………… 2. When did Roman soldiers often use tablets? ……………………………………………………………………… 3. In what field is Professor Mike Brady a leading figure? ……………………………………………………………………… 4. What were panels on the tablets once filled with? ……………………………………………………………………… 5. Which type of texts has the new technology already been applied to? ……………………………………………………………………… III. You will hear part of an interview with someone who has been having ballet lessons. Listen and choose the correct answer. (10 points) 1. What does Rupert say about the fact that he is doing ballet classes? A. Other people have ridiculed him for it. B. He expects to be mocked for it. C. It is not as unusual as people might think. D. People might think it isn’t really true. 2. Rupert says that before he started doing ballet lessons 1

DẠ

Y

M

QU Y

NH

ƠN

OF

FI CI A

L

A. he had been doing routine physical fitness training. B. his knowledge of ballet had been growing. C. ballet had taken over from football as his greatest interest. D. he had been considering doing ballroom dancing again. 3. Rupert say that when the idea of ballet lessons was suggested to him, A. he thought it was a joke. B. he was unsure exactly what would be involved. C. he began to have unrealistic expectations of what he could achieve. D. he initially lacked the confidence to do it. 4. One of the advantages of ballet that Rupert mentions is that A. it leads to fewer injuries than other physical activities. B. it has both physical and mental effects. C. it is particularly good for certain parts of the body. D. it is more interesting than other forms of exercise. 5. What does Rupert say about his progress at ballet? A. It has been much more rapid than he had expected. B. It has made him consider giving up his other training. C. It has given him greater appreciation of the skills of professionals. D. It has led him to enroll for certain exams. IV. Listen to a piece of BBC news about the World War One centenary and fill in the missing information. Write NO MORE THAN THREE WORDS taken from the recording for each answer in the spaces provided. (20 points) People have criticised Germany's government for not doing enough to (1) ___________ the World War One centenary. Germany hasn’t spent as much on events as some other (2) ________________ countries. And the events which have taken place have been seen as (3) ________________by critics. Traditionally in Germany the First World War is (4)________________the Second World War. History teaching in German schools tends to focus on (5)______________ of the Nazis rather than what happened (6) ____________ earlier. And since 1945 there's been a strong (7)_____________ in Germany to anything that might be seen as (8) _______________ . So many people here are uncomfortable with any anniversary of a war or a battle. There's still (9)_________________ among historians about who was responsible for World War One. But having spent the last 70 years atoning for Nazi guilt, many Germans have (10)______________ to now take on the blame for the First World War, too. SECTION 2: LEXICO - GRAMMAR (30 points) I. Choose the word or phrase that best fits the gap in each sentence (20 points) 1. Although the company seems very successful and popular, it has _______ actual money. Everything is built off loans and debts. 2

DẠ

Y

M

QU Y

NH

ƠN

OF

FI CI A

L

A. less or no B. little or no C. many or not D. not any or little 2. There is no point in phoning him. He’s certain _______by now. A. to leave B. to have left C. left D. having left 3. So little _____ that the neighbors could not settle their differences. A. did they agree B. they agreed C. agreed did they D. they did agree 4. Something’s ____________ up, so I’m afraid I won’t be able to make it this afternoon. A. shown B. pulled C. cropped D. cut 5. Could you lend me some money to _________ me over to the end of the month? A. hand B. tide C. get D. make 6. Have you considered applying for a job with the ________ service? A. civil B. civic C. national D. governmental 7. I wonder if you could let me know what the _________ fare to Berlin is? A. round B. double C. two-way D. return 8. Maria was just walking along the road when someone on a motorbike ________ her handbag. A. clutched B. grabbed C. clenched D. fumbled 9. Fred Astaire is one of my favourite dancers and I love the way he seems to ________ across the floor. A. glide B. march C. skid D. dash 10. The new Arts Centre seems to have been ________ construction for quite some time. A. in B. under C. on D. below 11. The new regulations have _________ up a number of problems for the company. A. come B. thrown C. got D. moved 12. We’re saving for our holiday, so we’re __________ back on luxuries this month. A. dropping B. moving C. giving D. cutting 13. Cars have been banned from the city centre, which makes the area much safer for _________. A. passersby B. onlookers C. pedestrians D. footmen 14. There has been a real boom in _________ electronics over the last three decades. A. consumer B. customer C. buyer D. client 15. When I realized that I’d left my homework at home, I quickly ________ back to get it. A. crept B. dashed C. crawled D. drifted 16. I know it’s got his name on the cover, but he used a ________. A. correspondent B. model C. ghostwriter D. fellow 17. The text doesn’t give you the answer explicitly – you have to _________ it from the evidence. A. convert B. grasp C. reckon D. deduce 18. I couldn’t decide what to write about, when I suddenly _________ upon the idea of doing something on writer’s block. A. thought B. chanced C. hit D. arrived 19. This fabric has the __________ of silk but it’s very cheap. A. stroke B substance C. friction D. texture 3

DẠ

Y

M

QU Y

NH

ƠN

OF

FI CI A

L

20. You’ll have to __________ your jacket quite hard to get the grass stain off it. A. polish B. scrub C. pat D. crush II. Fill each gap with the correct form of the word in brackets. (10 points) 1. She stood there completely _______, so I had no idea at all what she was thinking. (EXPRESS) 2. Any actor who becomes known for one role is in danger of becoming _______. (TYPE) 3. This school was once _______ as a military hospital during the war. (REQUIRE) 4. So far, the United States has said it cannot agree to _______ all types of antipersonnel mines. (LAW) 5. In Scotland, there is greater emphasis on _______ by individual schools. (VALUE) 6. The role of clouds is one of the big conundrums of _______. (CLIMATE) 7. Whatever happens, don't let this failure _______ you. (HEART) 8. It is _______ summer, but it's rather autumnal today. (THEORY) 9. She thinks that with a little ________ she can persuade her husband to change his mind. (FLATTER) 10. The branch of a tree knocked one of the riders _________. (SENSE) SECTION 3: READING (60 points) I. Fill each gap in the passage below with ONE appropriate word in the space provided. (15 points) Stuart Hayes had launched himself on a promising career as a swimmer when something odd happened (1) ______ him at the local pool. Flogging up and down for the umpteenth time, he suddenly realized how bored he had become with the monotony. Wasn’t there a more interesting way of (2) ______ sporty, for heaven’s sake? There was and there is: the colour, sweat and sheer emotion of triathlons. On August 5 and 6, Stuart, 27, will join almost 10,000 athletes in the London Triathlon, the biggest event of (3) ______ kind in the world. Triathlons are (4) ______ but boring. Combining swimming, cycling and running in one physical onslaught, they offer huge variety within a single racing framework. The classic Olympic distances are a 1,500 m swim, followed by a 40 km bike ride and 10 km run. Hayes, a world-class triathlete, won last year’s London Triathlon in an impressive 1 hour 47 minutes. “The worst part is the last 5 km of the run – you’re starting to get really fatigued by (5) ______ ,”he says. So why does he do it? He shrugs. “It’s interesting.” In Britain, the sport is growing (6) ______ 10 per cent a year. “People are moving away (7) ______ just running, and are looking for new challenges,” says Nick Rusling, event director for the London Triathlon. “Triathlons are a great deal more interesting to train for and you can vary training to fit busy lifestyles, swimming in your lunch break and (8) ______ on. On top of (9) ______ , people are much more aware of their bodies. They know running is potentially bad (10) ______ the joints. A triathlon is actually very easy and very smooth, particularly the cycle and swim.” II. Read the following passage and choose the best answer. (10 points)

4

DẠ

Y

M

QU Y

NH

ƠN

OF

FI CI A

L

Over the last century the world has become increasingly smaller. Not geographically, of course, but in the sense that media, technology and the opening of borders has enabled the world’s citizens to view, share and gain access to a much wider range of cultures, societies and world views. In this melting pot that the world has become, today’s child is privy to facets of the human experience that his immediate predecessors had no inkling even existed. It stands to reason that in order to absorb, configure and finally form opinions about this information-laden planet, children must be supplied with certain tools. Included in this list of ‘tools’ are: education, social skills, cultural awareness and the acquisition of languages, the most important of these being the latter. Until recently, a child who had the ability to speak more than one language would have been considered a very rare entity. This one-language phenomenon could be attributed to a combination of factors. Firstly, the monolingual environment in which a child was raised played a strong role, as did the limited, biased education of the past. With regard to immigrants, the sad fact was that non-native parents tended to withhold the teaching of the mother tongue so that the child would acquire the ‘more prestigious’ language of the adopted country. Nowadays, the situation has undergone an almost complete reversal. In the majority of North American and European countries, most children are given the opportunity to learn a second or even a third language. Children acquire these foreign languages through various and diverse means. In many countries, learning a foreign language is a compulsory subject in the state school curriculum. Other children rely on language schools or private tuition to achieve their goal. In other instances, children are born to bilingual parents, who, if they so desire, may teach the children two languages. Bringing up one’s child bilingually is not a decision to be taken lightly. Both parents must consider long and hard the implications involved in raising a child in a two-language home. This decision is one of those all-important choices which will affect not only the parents’ lives but also the life of the child. Raising a child bilingually has a two-fold effect. Firstly, of course, the child learns the two languages of the parents. Secondly, the parents’ decision will influence factors which will have a far-reaching effect on the child’s life. Some of these factors include: style and place of education; diameter of social circle; employment potential and preference; and, most importantly, the way in which the child views himself and his global environment. One of the more advantageous by-products of being a member of a bilingual family is the inherent awareness of two different cultures. This bicultural child inherits a wealth of knowledge brought about by an exposure to: historical backgrounds; traditional songs and folklore; rituals of marriage; models of social interaction; and therefore, two varying interpretations of the world. The monolingual child seems to be at a disadvantage in comparison to the bilingual child, who has a set of languages and an accompanying set of abstract cultural ideas. Practically speaking, when a child comes from a two-language family, he must be taught both languages in order to communicate with the extended family members. When, for example, the grandparents speak a language which differs from that of the child’s locale, a monolingual child would be deprived of the interaction which occurs between grandparents and grandchildren. On the other hand, a 5

DẠ

Y

M

QU Y

NH

ƠN

OF

FI CI A

L

bilingual child will not only be able to speak to grandparents, but will also comprehend where these people have ‘come from’. There will be a shared cultural empathy within the family. Because all family members can communicate, on both a verbal and cultural level, no one will feel excluded and the child will develop a sense of rootedness. On a more abstract level, it has been said that a bilingual child thinks differently from a monolingual child. Current research in linguistics indicates that there may be a strong correlation between bilingualism and cognitive skills. This new research concerns itself with the fact that a bilingual child has two lexical structures for any given physical or abstract entity. This leads logically to the assumption that the child also has two associations for many words, as a word can mean different things in different languages. For example, the word ‘fire’ in many western hemisphere languages connotes warmth and relaxation. In the Inuit language however, where fire is a necessity of life, it may connote heat and survival. For the bilingual child, then, vocabulary items and the abstract idea behind them are both dual in nature and more elastic. Researchers maintain that this elasticity of ideas may allow the child to think more flexibly and, therefore, more creatively. 1. In the author’s view, the world is becoming a _____. A. more culturally diverse place. B. place where only privileged children will prosper. C. less complex place to live in. D. much more integrated place. 2. According to the first paragraph, which of the following was true of immigrants? A. Children were reluctant to use their mother tongue. B. The mother tongue was considered less important. C. Parents encouraged children to use their mother tongue. D. Most parents made it a priority for children to grow up bilingual. 3. The phrase “privy to” in paragraph 1 mostly means _____. A. acquainted with B. advised of C. apprised of D. in the know about 4. The phrase “attributed to” in paragraph 1 mostly means _____. A. ascribed to B. associated with C. connected with D. held responsible for 5. According to the writer, second or foreign language learning is something _____. A. people are still apathetic towards. B. mainly associated with private sector education. C. that few people take seriously. D. about which general attitudes have evolved considerably. 6. According to the article, the decision to raise bilingual children is difficult because ______. A. it may limit the child’s choice of friends. B. though simple for parents, it can impact negatively on children. C. it may cause children to lose their sense of identity. D. it needs to be considered from many different angles. 6

NH

ƠN

OF

FI CI A

L

7. With regard to the extended family in immigrant situations, the writer feels it is important that _____. A. adults try to understand the child’s difficult cultural situation. B. children are not pressured to speak their parents’ native language. C. adults recognize the child’s need to identify more with local culture. D. children can relate to all aspects of their parents’ native culture. 8. The word “by-products” in paragraph 4 mostly means _____. A. entailments B. knock-on effects C. side effects D. spin-offs 9. The word “connotes” in paragraph 5 mostly means _____. A. underpins B. implies C. signifies D. smacks of 10. According to current research, the benefit of learning two languages is that _____. A. different types of knowledge can be accessed in different languages. B. bilinguals become more aware the origin of words in languages. C. it helps to develop different capabilities of the mind. D. bilinguals develop a greater sense of the value of culture. III. Read the passage and do the tasks that follow. (13 points) Questions 1 - 6 The reading passage has six paragraphs A-F. Choose the correct heading for each paragraph from the list below. List of Headings

M

QU Y

i. The productive outcomes that may result from boredom ii. What teachers can do to prevent boredom iii. A new explanation and a new cure for boredom iv. Problems with a scientific approach to boredom v. A potential danger arising from boredom vi. Creating a system of classification for feelings of boredom vii. Age groups most affect by boredom viii. Identifying those most affected by boredom

DẠ

Y

1. Paragraph A 2. Paragraph B 3. Paragraph C 4. Paragraph D 5. Paragraph E 6. Paragraph F

______ ______ ______ ______ ______ ______

7

OF

DẠ

Y

D

M

QU Y

C

NH

ƠN

B

FI CI A

L

A

Why being bored is stimulating – and useful, too We all know how it feels – it’s impossible to keep your mind on anything, time stretches out, and all the things you could do seem equally unlikely to make you feel better. But defining boredom so that it can be studied in the lab has proved difficult. For a start, it can include a lot of other mental states, such as frustration, apathy, depression and indifference. There isn’t even agreement over whether boredom is always a low-energy, flat kind of emotion or whether feeling agitated and restless counts as boredom, too. In his book, Boredom: A Lively History, Peter Toohey at the University of Calgary, Canada, compares it to disgust – an emotion that motivates us to stay away from certain situations. ‘If disgust protects humans from infection, boredom may protect them from “infectious” social situations,’ he suggests. By asking people about their experiences of boredom, Thomas Goetz and his team at the University of Konstanz in Germany have recently identified five distinct types: indifferent, calibrating, searching, reactant and apathetic. These can be plotted on two axes – one running left to right, which measures low positive or negative the feeling is. Intriguingly, Goetz has found that while people experience all kinds of boredom, they tend to specialize in one. Of the five types, the most damaging is “reactant” boredom with its explosive combination of high arousal and negative emotion. The most useful is that Goetz calls “indifferent” boredom: someone isn’t engaged in anything satisfying but still feels relaxed and calm. However, it remains to be seen whether there are any character traits that predict the kind of boredom each of us might be prone to. Psychologist Sandi Mann at the University of Central Lancashire, UK, goes further. “All emotions are there for a reason, including boredom,” she says. Mann has found that being bored makes us more creative. “We’re all afraid of being bored but in actual fact it can lead to all kinds of amazing things,” she says. In experiments published last year. Mann found that people who had been made to feel bored by copying numbers out of the phone book for 15 minutes came up with more creative ideas about how to use a polystyrene cup than a control group. Mann concluded that a passive, boring activity is best for creativity because it allows the mind to wander. In fact, she goes so far as to suggest that we should seek out more boredom in our lives. Psychologist John Eastwood at York University in Toronto, Canada, isn’t convinced. “If you are in a state of mind-wandering you are not bored,” he says. “In my view, by definition boredom is a undesirable sate.” That doesn’t necessarily mean that it isn’t adaptive, he adds. “Pain is adaptive – if we didn’t have physical pain, bad things would happen to us. Does that mean that we should actively cause pain? No. But even if boredom has evolved to help us survive, it can still be toxic if allowed to fester.” For Eastwood, the central feature of boredom is a failure to put our “attention system” into gear. This causes an inability to focus on anything, which makes time seem to go painfully slowly. What’s more, your efforts to improve the situation can end up making you feel worse. “People try 8

L

NH

F

ƠN

OF

FI CI A

E

to connect with the world and if they are not successful there’s that frustration and irritability,” he says. Perhaps most worryingly, says Eastwood, repeatedly failing to engage attention can lead to a state where we don’t know what to do any more, and no longer care. Eastwood’s team is now trying to explore why the attention system fails. It’s early days but they think that at least some of it comes down to personality. Boredom proneness has been linked with a variety of traits. People who are motivated by pleasure seem to suffer particularly badly. Other personality traits, such as curiosity, are associated with a high boredom threshold. More evidence that boredom has detrimental effects comes from studies of people who are more or less prone to boredom. It seems those who bore easily face poorer prospects in education, their career and even life in general. But of course, boredom itself cannot kill – it’s the things we do to deal with it that may put us in danger. What can we do to alleviate it before it comes to that? Goetz’s group has one suggestion. Working with teenagers, they found that those who “approach” a boring situation – in other words, see that it’s boring and get stuck in anyway – report less boredom than those who try to avoid it by using snacks, TV or social media for distraction. Psychologist Francoise Wemelsfelder speculates that our over-connected lifestyles might even be a new source of boredom. “In modern human society there is a lot of overstimulation but still a lot of problems finding meaning,” she says. So instead of seeking yet more mental stimulation, perhaps we should leave our phones alone, and use boredom to motivate us to engage with the world in a more meaningful way.

Y

M

QU Y

Questions 7-10 Look at the following people and the list of ideas below. Match each person with the correct idea, A-E. 7. Peter Toohey 8. Thomas Goetz 9. John Eastwood 10. Francoise Wemelsfelder List of ideas A. The way we live today may encourage boredom. B. One sort of boredom is worse than all the others. C. Levels of boredom may fall in the future. D. Trying to cope with boredom can increase its negative effects. E. Boredom may encourage us to avoid an unpleasant experience.

DẠ

Questions 11-13 Complete the summary below. Choose ONE WORD ONLY from the passage for each answer.

9

Responses to boredom

FI CI A

L

For John Eastwood, the central feature of boredom is that people cannot (11) _________, due to a failure in what he calls the “attention system”, and as a result they become frustrated and irritable. His team suggests that those for whom (12) _________ is an important aim in life may have problems in coping with boredom, whereas those who have the characteristic of (13) _________ can generally cope with it.

DẠ

Y

M

QU Y

NH

ƠN

OF

IV. You are going to read an extract from an article. Seven paragraphs have been removed. Select from the paragraphs (A-H) the one that fits each gap (1-7). There is one extra paragraph that you do not need to use. (7 points) A Race to Remember Imagine, if you will, driving across sand dunes at a speed of 100 kilometers per hour, with sand flying up at the windscreen while you try to maintain control of the wheel and your navigator attempts to maintain their compass steady at 25 degrees north-northwest, so you don't divert from your course. Now, imagine that scenario taking place over eight hours in a single day and over a period of around 12 days. That is the reality of the world-renowned Dakar Rally. 1 This year the race took place in Saudi Arabia with a total of 310 vehicles, made up of motorbikes and cars, amongst many other categories, and only 206 of those vehicles crossed the finish line. One individual that made it through this brutal event was Frederique Le Salles for whom it was his second rally. “Although my first Dakar Rally was five years ago, I’ve dreamed about coming back ever since. It took me all that time to raise the funds to enter again, and I’m overjoyed to finally return.” 2 As these examples show, the Rally truly does get under people’s skin, with the same competitors returning multiple times. This gives them the benefit of experience, which is an essential attribute in this competition. Without the modem necessities of GPS, knowing what the race takes in terms of the trials and tribulations, gives old-timers an important mental advantage in the race, and many taking part more than once because of this. 3 The event originated from the experience of a man called Thierry Sabine who became so adrift from the course in the 1977 Abidjan-Nice Rally that he realized his detour would make for a great race in itself. The first Dakar Rally started in Dakar in Africa and finished in Paris, skirting around the edges of the Sahara Desert, a grueling 10,000 - kilometre trip. 4 It is here that I spoke to competitors about their expectations of and concerns for the race, as well as finding out a bit about their preparation for an unyielding few weeks of fierce motor racing. The responses I received didn’t vary wildly - participants’ preparation was always meticulous. 5 10

OF

FI CI A

L

One rally driver, who is on his fifth race, told me that he believes a lot of the rally is down to luck. “It takes just a split second to make a mistake, and so, it doesn’t matter if you’ve driven brilliantly for thousands of kilometers, because all you need is one meter to go wrong, and you crash out. It’s happened to me twice and it’s a completely soul- crushing experience.” 6 Whether this is truly enough to be ready, only time will tell. While the financial rewards for winning aren't publicized (although rumor has it that they are relatively low), the accolade of winning is certainly something money can't buy. However, this doesn't mean that the rally is unprofitable for all, and there are some enterprising routes to make the rally a little more lucrative. 7

DẠ

Y

M

QU Y

NH

ƠN

A. Such is the commitment to compete that one racer, Chris Cork, sold his house in the UK in order to secure a place on the 2015 Dakar Rally. Unfortunately, he crashed out on the fourth day and had to undergo a difficult recovery procedure for his injuries. Undeterred, he came back the next year to compete, with financial help from well- wishers who were touched by his story. B. Although some entrants go it alone, with the bare-minimum requirements needed to participate, there are many other participants who sign up with companies and receive sponsor bonuses depending on their performance in each round. While finishing a round may be in the hands of fate, it is far more beneficial to do it with sponsorship, as many successful rally winners know. C. As a motorsport reporter, I’ve been fortunate enough to follow this competition for the last five years, and it never ceases to amaze me how tough and committed each racer is. Racers are required to fix their vehicles themselves while on a stage, and there is not even a great probability that they will be able to finish, with most competitors crashing out or breaking down on the route. D. Perhaps the main appeal for these riders is that it offers a sense of adventure that is hard to parallel. The Rally covers lots of harsh terrain and the course is simply a series of written directions. In addition to that, if they don’t have any overnight repairs to keep them busy, many drivers find themselves sleeping in tents alongside their vehicles, giving them just enough time to recuperate and get back on the ride. Indeed, unlike many racing events, there is little luxury to have, and this has always been the case with the Dakar Rally. E. There was lot of focus, as with every rally before, on the terrain, and the best ways to cope with it. 75% of the rally would take place on sand, and a great deal on sand dunes. This means hill climbs and sheer drops, and little ability to spot landmarks to help you know where you are. And remember, if you get your directions wrong, it can take you hours to catch up. F. Fortunately, for many people, the Dakar Rally is a labour of love, but budget can make a significant difference to your opportunities of succeeding. While amateurs may be sleeping aside

11

QU Y

NH

ƠN

OF

FI CI A

L

their vehicles relying on themselves for any repairs, professionals will have a team that can take care of their every need at the end of each stage. G. Although for many years Africa was its home, the location of the Rally has not remained static, it transferred to a different continent altogether, South America, from 2009 to 2019, and more lately to Saudi Arabia, where the vehicles have had to compete with the Middle Eastern deserts, and their scorching days and nearly freezing nights. H. Others are more concerned with how they’ll keep their momentum up. “This will be our third race, and we think we’ve finally discovered what we need to really compete with the best. We’ve come to realise that the race is about resilience and technical ability, and we've been training with sleep deprivation and motor repair exercises to make sure we’ve got both these qualities.” V. You are going to read an article by a psychologist about laughter. For questions 1 - 10, choose from the sections (A – D). The sections may be chosen more than once. (15 points) Which section … 1. comments on which person laughs within a verbal exchange? 2. uses a comparison with other physical functions to support an idea? 3. gives reasons why understanding laughter supplies very useful insights? 4. refers to someone who understood the self-perpetuating nature of laughter? 5. cites a study that involved watching people without their knowledge? 6. describes laughter having a detrimental effect? 7. criticizes other research for failing to consider a key function of laughter? 8. explains that laughing does not usually take precedence over speaking? 9. describes people observing themselves? 10. encourages checking that a proposition is correct? Why do people laugh? Psychologist Robert Provine writes about why and when we laugh.

DẠ

Y

M

A. In 1962, what began as an isolated fit of laughter in a group of schoolgirls in Tanzania rapidly rose to epidemic proportions. Contagious laughter spread from one individual to the next and between communities. Fluctuating in intensity, the laughter epidemic lasted for around two and a half years and during this time at least 14 schools were closed and about 1,000 people afflicted. Laughter epidemics, big and small, are universal. Laughter yoga, an innovation of Madan Kataria of Mumbai, taps into contagious laughter for his Laughter Yoga clubs. Members gather in public places to engage in laughter exercises to energise the body and improve health. Kataria realised that only laughter is needed to stimulate laughter – no jokes are necessary. When we hear laughter, we become beasts of the herd, mindlessly laughing in turn, producing a behavioural chain reaction that sweeps through our group. 12

FI CI A

L

B. Laughter is a rich source of information about complex social relationships, if you know where to look. Learning to ‘read’ laughter is particularly valuable because laughter is involuntary and hard to fake, providing uncensored, honest accounts of what people really think about each other. It is a decidedly social signal. The social context of laughter was established by 72 student volunteers in my classes, who recorded their own laughter, its time of occurrence and social circ*mstance in small notebooks (laugh logbooks) during a one-week period. The sociality of laughter was striking. My logbook keepers laughed about 30 times more when they were around others than when they were alone – laughter almost disappeared among solitary subjects.

QU Y

NH

ƠN

OF

C. Further clues about the social context of laughter came from the surreptitious observation of 1,200 instances of conversational laughter among anonymous people in public places. My colleagues and I noted the gender of the speaker and audience (listener), whether the speaker or the audience laughed, and what was said immediately before laughter occurred. Contrary to expectation, most conversational laughter was not a response to jokes or humorous stories. Fewer than 20% of pre-laugh comments were remotely jokelike or humorous. Most laughter followed banal remarks such as ‘Are you sure?’ and ‘It was nice meeting you too.’ Mutual playfulness, ingroup feeling and positive emotional tone – not comedy – mark the social settings of most naturally occurring laughter. Another counterintuitive discovery was that the average speaker laughs about 46% more often than the audience. This contrasts with the scenario in stand-up comedy – a type of comedy performance in which a non-laughing speaker presents jokes to a laughing audience. Comedy performance in general proves an inadequate model for everyday conversational laughter. Analyses that focus only on audience behaviour (a common approach) are obviously limited because they neglect the social nature of the laughing relationship.

DẠ

Y

M

D. Amazingly, we somehow navigate society, laughing at just the right times, while not consciously knowing what we are doing. In our sample of 1,200 laughter episodes, the speaker and the audience seldom interrupted the phrase structure of speech with a ha-ha. Thus, a speaker may say ‘You are wearing that? Ha-ha,’ but rarely ‘You are wearing … ha-ha… that?’ The occurrence of laughter during pauses, at the end of phrases, and before and after statements and questions suggests that a neurologically based process governs the placement of laughter. Speech is dominant over laughter because it has priority access to the single vocalisation channel, and laughter does not violate the integrity of phrase structure. Laughter in speech is similar to punctuation in written communication. If punctuation of speech by laughter seems unlikely, consider that breathing and coughing also punctuate speech. Better yet, why not test my theory of punctuation by examining the placement of laughter in conversation around you, focusing on the placement of ha-ha laughs. It's a good thing that these competing actions are neurologically orchestrated. How complicated would our lives be if we had to plan when to breathe, talk and laugh.

13

DẠ

Y

M

QU Y

NH

ƠN

OF

FI CI A

L

SECTION 4: WRITING (60 points) I. Read the following extract and use your own words to summarize it. Your summary should be about 140 words long. (15 points) NATIONALISM Until about 1800, people in most of the world were loyal to the places where they and their families lived. Most did not see themselves as part of a larger state or nation. But the rise of industry and the need to raise armies prompted rulers to try to inspire a sense of national identity and common cause. This drive toward lifting the interests of the nation above those of individuals or groups became known as nationalism. Many historians date modern nationalism from the French Revolution in the late 1700s. The French monarchy was replaced by a republic, in which the citizens no longer were expected to see themselves as subjects of the king. Rather, they found their identity in the abstract concept of France as their mother country. The process was repeated throughout Europe during the next century. The philosophical basis of nationalism is that the nation is the most important unit of social and economic life to which all other human activities and desires must yield. Helping to secure national pride are flags, foods, sports, traditions, histories, folk tales, music, literature, and culture. There may even be a national religion. Nationalists point to criteria that distinguish nations from each other, such as a common language, culture, and value. These traits are often represented by a single ethnic group to which almost all citizens of a nation belong. Many nations, however, host different ethnic groups side by side, sometimes with violent or politically disruptive results. Some ethnic groups refuse to recognize their nation, seeking to secede in order to rule themselves. Separatist movements in Quebec, Canada, and in the Basque region of Spain have been active for many years but have not yet succeeded. Requiring that all speak the same language has been an important means of enforcing national identity. New nations often attempt to outlaw minority languages. The national language tends to be the one spoken by the upper classes, resulting in the high-status language replacing the low-status ones.

14

NH

ƠN

OF

FI CI A

L

II. Writing task 2: (15 points)

DẠ

Y

M

QU Y

IV. Writing an essay: (30 points) Write an essay of about 350 words to express your opinion on the following topic: “Some people think that you can never become fluent in a language unless you have spent time living or working in that country. To what extent do you agree?”

15

Đề thi gồm: 18 trang

L

ĐỀ THI CHỌN HỌC SINH GIỎI LẦN THỨ XVI MÔN: TIẾNG ANH - KHỐI 11 Thời gian: 180 phút

FI CI A

HỘI CÁC TRƯỜNG CHUYÊN VÙNG DUYÊN HẢI VÀ ĐỒNG BẰNG BẮC BỘ TRƯỜNG THPT CHUYÊN NGUYỄN TRÃI TỈNH HẢI DƯƠNG ĐÁP ÁN

SECTION 1: LISTENING (50 points)

OF

I. You will hear an introduction to a radio phone-in programme about modern lifestyles. Listen and indicate true (T) or false (F) statements. 1. T 2. F 3. F 4. F 5. T II. You will hear a short radio report about how technology is helping archaeologists who want to learn more about some texts written over 2,000 years ago known as Roman tablets.

ƠN

Listen and give short answers to the questions. Write NO MORE THAN FIVE WORDS AND/OR A NUMBER taken from the recording. (10 points) 1. in one Roman fort 2. writing letters or/and legal documents

QU Y

NH

3. computer vision 4. wax 5. texts in ink III. You will hear part of an interview with someone who has been having ballet lessons. Listen and choose the correct answer. (10 points)

M

1. B 2. A 3. D 4. B 5. C IV. Listen to a piece of BBC news about the World War One centenary and fill in the missing information. Write NO MORE THAN THREE WORDS taken from the recording for each answer in the spaces provided. (20 points) 1. commemorate 2. European 3. half-hearted 4. overshadowed by 5. the crimes 6. a generation 7. aversion 8. glorifying militarism 9. some disagreement 10. little appetite SECTION 2: LEXICO - GRAMMAR (30 points) I. Choose the word or phrase that best fits the gap in each sentence (20 points) 3. A

4. C

5. B

6. A

7. D

8. B

9. A

10. B

12. D

13. C

14. A

15. B

16. C

17. D

18. C

19. D

20. B

DẠ

11. B

2. B

Y

1. B

II. Complete the sentences with the correct form of words (10 points). 1. expressionless

1. 2. typecast

3. requisitioned

4. outlaw

5. self-evaluation

6. climatology

7. dishearten

8. theoretically

9. flattery

10. senseless

1. A

10. C

2. B

3. D

4. A

5. D

6. D

7. D

III. Read the passage and do the tasks that follow. (13 points) 1. iv

2. vi

3. i

4. v

5. viii

FI CI A

1. to 2. being 3. its 4. anything 5. then 6. by 7. from 8. so 9. that 10. for II. Read the following passage and choose the best answer. (10 points)

L

SECTION 3: READING (60 points) I. Fill each gap in the passage below with ONE appropriate word in the space provided. (15 points)

8. D

9. B

6. iii

7. C

8. D

9. B

NH

1. C 2. D 3. B 4. A 5. C 6. A SECTION 4: WRITING (60 points) I. Students’ answers

ƠN

OF

7. E 8. B 9. D 10. A 11. focus 12. pleasure 13. curiosity IV. You are going to read an extract from an article. Seven paragraphs have been removed. Select from the paragraphs (A-H) the one that fits each gap (1-7). There is one extra paragraph that you do not need to use. (7 points) 1. C 2. A 3. D 4. G 5. E 6. H 7. B V. You are going to read an article by a psychologist about laughter. For questions 1 - 10, choose from the sections (A – D). The sections may be chosen more than once. (15 points) 10. D

QU Y

The summary must cover the following points: + A sense of national pride, or nationalism, arose around the time of the French Revolution.

M

+ Industrialization and the need to raise armies led to people identifying themselves as being from a country as opposed to from a tribe or region. + Nations are often united by a common language, culture, and value. + When a nation includes different ethnic or religious groups, however, political conflict or violence can result. II. Students’ answers III. Students’ answers

DẠ

Y

Audio scripts I. You will hear part of an interview with someone who has been having ballet lessons. Listen and choose the correct answer. (10 points) Today I’m talking to opera critic Rupert Christiansen, who is in his forties and has recently started doing ballet classes. It’s a pretty unusual thing to do, isn’t it?

Well, yes, but one consolation of growing old is that you cease to care what other people think of your views or activities. So here I am, coming out on the radio as probably the only balding middle-aged man on the entire planet to take up ballet lessons. Go on, snigg*r.

FI CI A

L

I’m not laughing, I’m just interested. Tell me, how did it all start? Well, I have long been an infatuated ballet fan. The way most men think about their football team, I think about the Royal Ballet company – they’re my team and I follow their every move with nerdy fascination. Recently, I’ve become increasingly frustrated at my lack of

OF

technical knowledge, but the thought of having a bash myself hadn’t crossed my mind since a schoolboy attempt at ballroom dancing culminated in disaster. Anyway, many years after that unfortunate event, to stave off bodily decay, I discovered Balance, a wonderful physiotherapy gym in London, where I began personal training with Hans Ektvedt and his colleagues. Hans nobly takes me through the necessary boring stuff – weights, pull-ups, crunches – and his patience and good humour make it endurable and most enjoyable.

ƠN

So what led you from that to ballet lessons? One day earlier this year, Hans mentioned a newcomer to the Balance team and suggested I try him for some supplementary lessons. His name is Julien Diaz, and he turns out to be a remarkable character. A graduate of the Rambert Dance School, he ended up dancing

NH

professionally in Berlin and Amsterdam, before returning to England to set up as a trainer, specializing in posture and the freeing up of body language. He told me I should start doing ballet and his disarming manner soon persuaded me to drop my inhibitions. I was under no illusions

QU Y

about my abilities but I wanted to find out about the art of ballet by trying it myself. Isn’t it a bit dangerous for someone of your age, if you don’t mind me saying so? Well, as Julien says, it’s a superb form of exercise. He’d like to see the whole world doing

M

ballet. It’s only dangerous to muscles if you do it day in day out at the very top level. For almost anyone else, it’s an injury-preventative activity that tones and elongates every part of your body, from toes to fingers to head. It’s a training in balance, co-ordination and flexibility, which gets you to engage your body and brain at the same time. It’s fantastic for getting you to walk with a

spring in your step, and it also cleanses the mind – there’s no way you can do a ballet class without concentrating 100 per cent on what you’re doing. So what do your lessons consist of? The sessions last for 90 minutes. We start with what are basically bending and stretching

DẠ

Y

exercises. Then we move on to the hopping, skipping and turning movements, finishing with some jumps in both the closed first and open second positions, before cooling down with some excruciating but vital stretches. This is basic stuff, but, believe me, it isn’t easy, and you have to

take it slowly. I’m not unfit, but I can’t do more than a couple of minutes without gasping for rest, and it’s incredibly difficult to programme it all into one’s muscle memory. You can’t approximate – you have to get it right, and that involves focusing simultaneously on the correct angle of

turnout, the complementary shaping of the arms, a firm finish to any sequence of movements, and the fluent placing of the head, neck and shoulders. Are you going to get really good at this? What kind of progress are you hoping to make?

FI CI A

L

How much further can I get? Well, I’m not expecting a call from by beloved Royal Ballet. My main aim is to find out something of what it feels like to dance, from the inside – already I find that, as a spectator, my admiration for the professionals has sharpened considerably. Julien has a potty notion of moulding some of the movements I am painfully acquiring into a dance, and

OF

believes that most people of my age could reach a point at which they could take the lower grades of Royal Academy of Dance exams. And Hans has noticed that my work with Julien has resulted in improvements in my regular training. He says that I’m moving better and walking taller, with firmer core stability and more flexibility in my hips and back. I wonder whether he’s tempted to have a go himself. II. You will hear an introduction to a radio phone-in programme about modern lifestyles.

ƠN

Listen and indicate true (T) or false (F) statements. Good afternoon and welcome to our programme “Modern Lifestyles”. Regular listeners will remember the Health and Diet programme we broadcast earlier in the year featuring Ron Clarke, an Australian accountant turned record-breaking athlete.

NH

Ron’s now Managing Director of the five successful Cannons Health Clubs in London and he’s a firm believer in being positive about life. His philosophy is that, in order to have a healthy and fulfilled life (which he obviously felt being an accountant, even in Australia, didn’t

QU Y

offer him), you have to enjoy everything you do. He advocates a healthy diet and exercise as a means of supporting one’s work, family and social life. With this in mind he devised the term “ Total Living”. It certainly stood him in good

M

stead during his successful career as an athlete – and it’s an obvious feature of the health clubs Ron runs in the city – but he believes it can help everybody. His latest venture’s a book he’s just written, also entitled “Total Living”, which isn’t just another book of physical exercises, but a guide to how physical exercise can augment a timetable already filled with a pressurised job and a

exercise!

hectic social life. As the term “Total Living” implies, we should see our lives as a whole, not in isolated compartments – and this means integrating all the different aspects of our lives. Ron thinks that too often we don’t build in time for what we need most – in this case, physical

DẠ

Y

You may think that combining work, play, and exercise sounds daunting, but Ron also argues very much against some current health trends; for example, assuring us that the sun is beneficial for our health and not the danger to our health and longevity which the anti-sun lobby would have us believe! And then, there’s dieting. How many of you can honestly say you’ve never considered going on a diet? If you talk to Ron, he will insist that slimming diets should be avoided at all costs.

Well, we’re fortunate to have Ron back in the studio with us today and he’s going to answer some of your questions during the next half an hour or so but before … III. You will hear a short radio report about how technology is helping archaeologists who

FI CI A

L

want to learn more about some texts written over 2,000 years ago known a Roman tablets. Listen and give short answers to the questions. Write NO MORE THAN FIVE WORDS AND/OR A NUMBER taken from the recording. (10 points) At the time of the Roman Empire in Europe, around 2,000 years ago, it was common for

OF

information to be written, not on paper, but on things called “tablets”. These were pieces of wood about the size and thickness of a typical modern envelope. Hundreds of such tablets have been unearthed from archaeological sites throughout Europe and the Mediterranean world – nearly 200 were found in one Roman fort alone – and like most of these discoveries, they have been placed in public collections, mainly in museums in northern Europe, to be viewed but not, unfortunately, to be read.

ƠN

This is because, although in some cases traces of writing can still be seen, most are now illegible to the naked eye. But that’s all soon to change because archaeologists hope that with the help of new technology, their secrets may soon be revealed. Many of the tablets took the form of legal documents and letters written by Roman soldiers. An example, now at the British Museum,

NH

bears the name of the person who wrote it and the name of the person who received it, plus the word “transportation”, which you can just make out, but the rest remains a mystery. Now, with the help of computer techniques, experts hope eventually to be able to read the whole letter.

QU Y

Professor Mike Brady, a leading figure in what’s known as “computer vision” for many years, admits that this is the hardest project he’s ever worked on. But the excitement of seeing the latest ideas in computing applied to such a very ancient problem has the archaeological community

M

buzzing. So, in simple terms, why has the writing been preserved and how will it be possible to “undo” the ageing process? Well, the tablets were made with thin, hollow panels cut across them. Wax was poured into these and the test was then written into this soft surface using an instrument

with a fine metal point. In virtually all cases, the wax has perished and all that can be detected on the surface of the tablet underneath are scratched. These are too faint to be read, because they are distorted. For some time, scientists have attempted to study them with laser photography, but this has

DẠ

Y

proved fruitless. However, it is now hoped that by enhancing images of the tablets on computer, their original messages will become legible again. If this is the case, a whole new source of historical information will be opened up, and this promises advances and new knowledge for many decades to come. The new technology has already been used on texts in ink as well, and in the future, it will be applied to damaged surfaces of many kinds.

IV. Listen to a piece of BBC news about the World War One centenary and fill in the missing information. Write NO MORE THAN THREE WORDS taken from the recording for each

FI CI A

L

answer in the spaces provided. (20 points) People have criticised Germany's government for not doing enough to commemorate the World War One centenary. Germany hasn’t spent as much on events as some other European countries. And the events

OF

which have taken place have been seen as half-hearted by critics. Traditionally in Germany the First World War is overshadowed by the Second World War. History teaching in German schools tends to focus on the crimes of the Nazis rather than what happened a generation earlier. And since 1945 there's been a strong aversion in Germany to anything that might be seen as glorifying militarism. So many people here are uncomfortable with any anniversary of a war or a

ƠN

battle. There's still some disagreement among historians about who was responsible for World War One.

DẠ

Y

M

QU Y

NH

But having spent the last 70 years atoning for Nazi guilt, many Germans have little appetite to now take on the blame for the First World War, too.

KỲ THI CHỌN HỌC SINH GIỎI CÁC TRƯỜNG THPT CHUYÊN

L

KHU VỰC DUYÊN HẢI VÀ ĐỒNG BẰNG BẮC BỘ

FI CI A

LẦN THỨ XIV, NĂM 2023

ĐỀ THI MÔN: TIẾNG ANH - LỚP 10 ĐỀ ĐỀ XUẤT

Thời gian: 180 phút (Không kể thời gian giao đề)

Đề thi gồm 11 trang

Ngày thi: 16/7/2023

ƠN

OF

SECTION A: LISTENING (50 points) HƯỚNG DẪN PHẦN THI NGHE HIỂU • Bài nghe gồm 4 phần; mỗi phần được nghe 2 lần, mỗi lần cách nhau 05 giây; mở đầu và kết thúc mỗi phần nghe có tín hiệu. Thí sinh có 20 giây để đọc mỗi phần câu hỏi. • Mở đầu và kết thúc bài nghe có tín hiệu nhạc. Thí sinh có 03 phút để hoàn chỉnh bài trước tín hiệu nhạc kết thúc bài nghe. • Mọi hướng dẫn cho thí sinh (bằng tiếng Anh) đã có trong bài nghe Part 1. For questions 1-5, listen to a student asking for information about a healthy eating society and supply the blanks with the missing information. Write NO MORE THAN TWO WORDS OR NUMBERS taken from the recording for each answer in the space provided.

Day

Thursday Friday Saturday

Time

Location of meal

Restaurant: 1.______________ food

7.30

High Street

Prepare a meal together

8.00

2.______________

Restaurant: European food

3.______________

4.______________ in town

5.______________

12.00

canteen

QU Y

Wednesday

Activity

NH

Healthy Eating Society Weekly Plan

No. 6.

Statements

True

False

Susanna feels shy about playing her violin in public. Carl and Susanna share the same opinion about practising their instruments regularly.

Y

7.

M

Part 2. You will hear a conversation between a boy, Carl, and a girl, Susanna, about a school concert. For questions 6-10, decide whether the following statements are true (T) or false (F). Write your answers in the corresponding numbered boxes provided on the answer sheet. (10 points)

Susanna's parents refuse to allow her to give up violin lessons.

9.

Carl's aim is to have a career in music.

10.

Susanna thinks she would enjoy working in another country.

DẠ

8.

Page 1 of 11

NH

ƠN

OF

FI CI A

L

Part 3. For questions 11-15, listen to a radio interview in which a psychologist, Colin Fraser, talks about cultural identity and choose the correct answer A, B, C, or D which fits best according to what you hear. Write your answers in the corresponding numbered boxes provided. 11. When discussing his own cultural identity, Colin reveals______ A. his resilience to changing cultures. B. his unorthodox family background. C. his ability to adapt. D. his feeling of alienation. 12. What does Colin regard as the defining aspect of a person’s cultural identity? A. the sense of birth right B. the emotion it generates C. the physical proximity to heritage D. the symbols of tradition 13. What is the influence of a culture attributed to? A. the dissemination of wisdom B. connection between societies C. knowledge of one’s background D. the practice of archaic rituals 14. According to Colin, what makes a culture successful on the global scene? A. its capacity for tolerance B. its isolation from the mainstream C. its aptitude for resolving conflicts D. its ability to be self-effacing 15. During the conversation, Colin is_______ A. distinguishing between birthplace and residence. B. advocating the celebration of heritage. C. highlighting the differences in societies. D. addressing the issues raised by conflicting cultures.

QU Y

Part 4. For questions 16-25, listen to a report on G7 Summit held in France and supply the blanks with the missing information. Write NO MORE THAN FOUR WORDS taken from the recording for each answer in the space provided. - The leaders of the major industrialized democracies have gathered in France for the G7 summit in an attempt to 16.____________________ amid sharp differences over a clutch of global issues that risk further dividing a group of countries already struggling to 17.____________________.

M

- Speaking before the presumably 18.____________________ summit, the European Council President acknowledged it would be hard to find common ground at a time when 19.____________________ has never been more important

- A grim array of disputes and problems await the leaders, with a trade war between China and the United States deteriorating, European governments struggling to uphold the collapsing 20.___________________, and global condemnation growing over illegal fires which are 21.___________________ the Amazon.

DẠ

Y

- Trudeau sat down with British Prime Minister for discussions that focused on post-Brexit 22.___________________. Trudeau also met with Japanese Prime Minister, where he highlighted strong ties forged with G7 allies as a 23.___________________. - Donald Trump threatened to introduce high tariffs on French 24.___________________ in retaliation for French president Emmanuel Macron’s tax on global technology companies. Emmanuel Macron has said he hopes to convince world leaders to pull back from trade war and 25.___________________ at the G7 summit, despite signs that will be a daunting task

Page 2 of 11

FI CI A

L

SECTION B: LEXICO- GRAMMAR (40 points) Part 1. For questions 26-45, choose the best option A, B, C, or D to complete the following sentences and write your answers in the corresponding numbered boxes provided on the answer sheet. (20 points)

DẠ

Y

M

QU Y

NH

ƠN

OF

26. Unfortunately, Jamie’s plans to tour around Australia didn’t _______ due to a lack of finances. A. pan out B. pull off C. knuckle down D. waltz through 27. If you are given a ________ anaesthetic during an operation, you are still aware of what is going on around you. A. local B. tropical C. varied D. released 28. Insurance companies had to ________ £10 million in storm damage claims. A. dip in B. rip off C. bail out D. cough up 29. The well-known ________ clash between the President and the rebel leader is not making things easier. A. character B. mood C. enemy D. personality 30. Their ________ religious beliefs were of great comfort in troubled times. A. age-long B. long-gone C. strongly-held D. time-honored 31. They are a real ________ organization; they are only interested in making a profit. A. devil-may-care B. fly-by-night C. open-handed D. down-to-earth 32. Although he came to work the day before his retirement, everyone knew he was just ________. A. going with the flow B. going through the motions C. going against the grain D. going along with them 33. She was ________ away from the company with promises of better terms and conditions from a rival firm. A. enhanced B. enticed C. manipulated D. raided 34. Lawrence suggested ________ ourselves at the beginning of the long project in order to avoid exhaustion. A. pacing B. rushing C. plodding D. racing 35. All three siblings agreed to ________ their resources in order to start their new business venture. A. fuse B. blend C. mingle D. pool 36. Although citizen-centered schemes involve residents in debates, full political ________ is kept by local councilors and MPs. A. autonomy B. autarchy C. autocracy D. authority 37. Our guests are from a _______ of society and so should reflect most point of views. A. cross-section B. cross-fire C. cross-purpose D. cross-reference 38. Mr Jones is _______, unfortunately, so you’ll have to call again tomorrow. A. indifferent B. inimitable C. indisposed D. incongruous 39. The only room available was, to say the least, _______. There was no carpet, no curtains, and the only furniture was a bed and a small beside table. A. snug B. dreary C. stark D. cushy 40. My daughter has a very sunny ________, unlike my son who is often moody and uncommunicative. A. behavior B. condition C. disposition D. nurture 41. I’m in a real ________ and I just don’t know what to do. A. dilemma B. paradox C. query D. hunch 42. The Minister was accused of ________ the truth. A. distorting B. revising C. reforming D. shifting 43. Lionel Messi was ________ over his team’s victory against Brazil. A. delighted B. ecstatic C. jubilant D. blissful 44. Something’s ________ up, so I’m afraid I won’t be able to make it this afternoon. A. shown B. pulled C. cropped D. cut Page 3 of 11

L

45. We don’t want him to suspect we’re giving him a surprise party. Make sure you don’t ____________. A. break the ice B. kick the bucket C. spill the beans D. sweep the board

ƠN

OF

FI CI A

Part 2. For questions 46-55, fill each gap with the correct form of the words in brackets. Write your answer in the boxes provided on the answer sheet. (10 points) 46. Like his famous _______, young Washington had a brave, adventurous spirit. (NAME) 47. She has a softly ___________voice that would melt anyone listening to her singing. (FLUENT) 48. The history of the territory is ___________ illustrated in the book. (PICTURE) 49. Don’t you think she is ______? She always looks good in her photographs. (PHOTO) 50. Hardly a day goes by without Tim being __________ of eating sweets. (DESIRE) 51. She resembled an army commander whom nothing could put the __________ on. (FRIGHTEN) 52. In an overscheduled world, children and caregivers are encouraged to take part in open-ended plays, a ____________ experience that creates intergenerational conversations. (FREE) 53. Halloween has been grossly _________ over the last years. (COMMODITY) 54. Until now, there is hardly any measure to lessen the _________ regulation and taxation on small businesses. (ONUS) 55. About $200 million in taxes weren’t paid because of ____________ income. (REPORT)

QU Y

NH

Part 3. For questions 56-65, complete each of the following sentences with suitable preposition(s). Write your answer in the boxes provided on the answer sheet. (10 points) 56. The station is _____________ walking distance of the hotel. 57. I’m so tired after work that I often drop _____________ in front of the TV. 58. The decision was deferred _____________ a later meeting. 59. That awful new office block is a real blot _____________ the landscape. 60. I don’t know how Nicole survives, living all away _____________ the sticks. 61. Carl and I just spent the whole day lazing _____________ listening to music. 62. Unfortunately, most of the photos Terry took were ______________ focus. 63. Why don’t we meet _____________ here again on the 12th, when I get back from Germany? 64. The prime minister has come _____________ fire during this election campaign for being slow to respond to events. 65. Keep plodding _____________ and you’ll finish your novel eventually.

M

SECTION C: READING (60 points) Part 1. For questions 66-75, read the following passage and decide which option (A, B, C, or D) best fits each gap. Write your answers in corresponding numbered boxes on the answer sheet. (15 points)

Y

Despite the continued 66.________ of those early town perks, it wasn't until the Depression that modern Hershey started to take shape. Perhaps the only town in the country actually to 67_________ during the 1930s, it thrived because Hershey vowed his Utopia would never see a breadline. lnstead he 68_________ a massive building boom that gave rise to the most visited buildings in today's Hershey and delivered wages to more than 600 workers. He admitted that his 69____________ were partly selfish: "lf I don't provide work for them, I'll have to feed them. And since building materials are now at their lowest cost levels, I'm going to build and give them jobs."

DẠ

He seems to have 70__________ no expense; most of the new buildings were strikingly 471________. The first to be finished was the three-million-dollar limestone Community Center, home to the 1,904-seat Venetian-style Hershey Community Theater, which has played 72______ since 1933 to touring Broadway shows and to music, dance, and opera performances. lt offers just as much to look at when the lights are on and the curtains closed. The floors in the 73_________ named Grand Lobby are polished ltalian lava rock, surrounded by marble walls and capped with a

Page 4 of 11

B. rigidity B. decline B. funded B. objections B. spent B. unattractive B. housing B. inappropriately B. gaudy B. studded

C. elasticity C. get on C. accounted C. preoccupation C. allowed C. poor C. host C. seemingly C. dazzling C. supported

D. resilience D. flower D. stocked D. intentions D. justified D. opulent D. hom*ogeneously D. frightfully D. bland D. magnified

FI CI A

A. flexibility A. prosper A. trusted A. pretensions A. spared A. impoverished A. hosting A. aptly A. dizzying A. holed

OF

66. 67. 68. 69. 70. 71. 72. 73. 74. 75.

L

bas-relief ceiling showing sheaves of wheat, beehives, swans, and scenes from Roman mythology. With the 74________ inner foyer, Hershey thumbed his nose even harder at the ravages of the Depression: The arched ceiling is tiled in gold, the fire curtain bears a painting of Venice, and the ceiling is 75_______with 88 tiny lightbulbs to re-create a star-lit night.

Part 2. For questions 76-85, fill each of the following numbered blanks with ONE suitable word and write your answers in the corresponding numbered boxes on the answer sheet. (15 points)

ƠN

THE CHANGING FACE OF WORKING LIFE

QU Y

NH

The accepted concept of a career 76._______ followed a similar pattern for decades. After completing their education, people would enter the adult world of work, 77.___________ down on to a job which they would likely remain from that point 78.____________. Not only would this occupation provide their income for their entire working life, it would also allow them a healthy pension when they retired and moved into 79.___________ age. Over the past twenty years, 80.________, the relationship between a wage earner and their chosen profession has changed enormously. Today, the idea of a ‘job-for-life’ has all 81._________ disappeared, to be replaced by an unforgiving world of unstable employment. Some observers even argue that current society to pit old 82._________ young in a constant battle to find work of some description, all against a 83.__________ of increasing debt and economic difficulties.

M

At the same time, the government regularly releases figures that suggest the economy is prospering, evidencing this claim with the fact that the unemployment rate continues to fall annually. There are indeed more jobs available. However, a huge number of these are casual, temporary or short-term positions, all of 84.__________ are low-paid and create little in the way of tax income for the government. This has a number of debilitating long-term effects, not 85.___________ because this assurance of a growing economy is based more in myth than fact.

Part 3. For questions 86-95, read the following passage and circle the best answer to each of the following questions. Write your answers in corresponding numbered boxes provided on the answer sheet. (15 points) The Human Immune System

DẠ

Y

The human immune system is composed of both an innate and an adaptive immune system. First, humans have an innate immune system that is intrinsic in all organisms, and it functions particularly through establishing biological barriers and creating biochemical reactions that immediately respond with a maximal effort in order to destroy infectious microbes. [A] Second, humans have an adaptive immune system, which can only be found in vertebrates with jaws. [B] The adaptive immune system gains an immunological memory from previously encountered germs, so it is able to prevent these specific microbes from causing further infection. [C] With these dual capacities of fighting infection and acquiring resistance to germs, humans can maximize their immunity. [D]

Page 5 of 11

FI CI A

L

A person’s innate immune system has many complex barriers and biochemical reactions designed to ward off infections. The most visible one is the skin, which keeps most bacteria, fungi, and viruses from ever entering the body, but humans also have mucus, which traps germs that reside in the body’s tissue. In addition to such biological material, there are other internal barriers like gastric acids, tears, saliva, urine, and various chemicals that either destroy or flush out germs. Even involuntary functions like sneezing and coughing are barriers that serve to expel germs. Beyond these, there are biochemical reactions that come from leukocytes, which are found in the blood. Leukocytes are white blood cells that effectively clear out cellular debris, create inflammation near an infection, summon immune cells to the inflammation, activate several other chemical reactions, and even destroy tumors. However, perhaps the most important action these cells perform is activating a human’s adaptive immune system, which is essential in not only curing current diseases but also preventing future infections.

ƠN

OF

With an adaptive immune system, cells learn how to best combat pathogens and develop a higher resistance to them. Like the innate immune system, this involves chemical reactions and cellular cooperation. Unlike the innate immune system, this system doesn’t respond very quickly or with its full strength all at once. Instead, it uses its time and energy to provide cells with an immunological memory to the pathogens they encounter, making them more resistant to recurring infections (similarly to how a vaccination works). Certain white blood cells called T-cells are the principal actors in this system; these identify “self” cells with the same DNA and distinguish them from any foreign cells with different DNA. After this, they seek and destroy these foreign cells, whether they are invading microbes or infected host cells. T-cells also mediate the responses from the innate and adaptive immune systems so that the body can effectively exterminate the infection.

QU Y

NH

After destroying infectious cells, the body uses B-cells to develop antibodies, or specialized proteins that prevent future infections. A B-cell is designed to connect with an individual type of antigen created by an infectious cell. The B-cell uses this antigen to produce antibodies that seek out and neutralize infectious bacteria, fungi, and viruses. However, the most important process comes after the infection disappears: these B-cells will duplicate, and their progeny will manufacture the same antibodies. Thus, the body will constantly produce antibodies that successfully fight off a specific infection, and the body can successfully fight off any subsequent infections from this pathogen. In addition to this, B-cells also mark antigens for leukocytes to attack, thus making them and microbes easier targets for the biochemical reaction.

M

An interesting feature of the human immune system is how it affects infants both before and after birth. When babies are first born, they do not have very many previously formed antibodies, so they have a greater risk of infection than adults do. However, they ward off many infections by temporarily obtaining the mother’s antibodies from breast milk and nutrients passed through the placenta. Also interesting is the very inception of the fetus among such an aggressive immune system: somehow, the fetus, which doesn’t have its mother’s exact DNA, is ignored by the mother’s T-cells and B-cells. Scientists currently have a few theories about this phenomenon. For instance, the uterus may not be monitored by white blood cells, or it may produce special proteins that suppress any local immune responses. Nonetheless, the fact that the immune system restrains its programming for reproductive development continues to puzzle many scientists. 86. The word intrinsic in the passage is closest in meaning to ________. B. auxiliary

C. detrimental

D. extraordinary

Y

A. fundamental

DẠ

87. Which of the following square brackets [A], [B], [C], or [D] best indicates where in the paragraph the sentence ‘However, this particular action never changes to counter specific threats of infection’ can be inserted? A. [A]

B. [B]

C. [C]

D. [D]

Page 6 of 11

88. According to passage 2, which bodily fluid initiates biochemical reactions in a human’s innate immune system? B. blood

C. saliva

D. tears

L

A. urine A. interrupt

B. magnify

FI CI A

89. The word mediate in the passage is closest in meaning to _________. C. contemplate

D. coordinate

90. Based on the information in paragraph 3, what can be inferred about the adaptive immune system?

A. Because it takes so long to act, it is less effective in purging infectious cells than the innate immune system.

OF

B. Even though it takes longer to act, it is more effective in long term immunity than the innate immune system.

C. Because humans already have an innate immune system, this system is unnecessary and only used as a substitute.

ƠN

D. It works differently from the innate immune system, so the two are completely independent of one another. 91. The word progeny in the passage is closest in meaning to _________. A. willingness

B. mechanism C. offspring

D. mutation

92. According to paragraph 4, what do B-cells produce? B. antibodies

C. leukocytes

NH

A. antigens

D. pathogens

93. The word inception in the passage is closest in meaning to __________. A. conception

B. invulnerability

C. contamination

D. consumption

94. According to paragraph 5, where do newborn babies get most of their antibodies? B. from immune cells in the uterus

C. from mucus and other barriers

D. from breast milk and the placenta

QU Y

A. from their own white blood cells

95. Based on the information in paragraph 5, what can be inferred about how the mother’s immune system should scientifically be affecting the fetus? A. It should protect the fetus from infection. B. It should help develop cells in the fetus.

M

C. It should be attacking foreign fetal cells.

D. It should be exposing the fetus to microbes. Part 4. For questions 96-105, read the passage and do the following tasks. Write your answers in the corresponding numbered boxes on the answer sheet. (15 points) THE SWIFFER

DẠ

Y

For a fascinating tale about creativity, look at a cleaning product called the Swiffer and how it came about, urges writer Jonah Lehrer. In the story of the Swiffer, he argues, we have the key elements in producing breakthrough ideas: frustration, moments of insight and sheer hard work. The story starts with a multinational company which had invented products for keeping homes spotless, and couldn't come up with better ways to clean floors, so it hired designers to watch how people cleaned. Frustrated after hundreds of hours of observation, they one day noticed a woman do with a paper towel what people do all the time: wipe something up and throw it away. An idea popped into

Page 7 of 11

L

lead designer Harry West's head: the solution to their problem was a floor mop with a disposable cleaning surface. Mountains of prototypes and years of teamwork later, they unveiled the Swiffer, which quickly became a commercial success.

FI CI A

Lehrer, the author of Imagine, a new book that seeks to explain how creativity works, says this study of the imagination started from a desire to understand what happens in the brain at the moment of sudden insight. 'But the book definitely spiralled out of control,' Lehrer says. 'When you talk to creative people, they'll tell you about the 'eureka' moment, but when you press them they also talk about the hard work that comes afterwards, so I realised I needed to write about that, too. And then I realised I couldn't just look at creativity from the perspective of the brain, because it's also about the culture and context, about the group and the team and the way we collaborate.'

OF

When it comes to the mysterious process by which inspiration comes into your head as if from nowhere, Lehrer says modern neuroscience has produced a 'first draft' explanation of what is happening in the brain. He writes of how burnt-out American singer Bob Dylan decided to walk away from his musical career in 1965 and escape to a cabin in the woods, only to be overcome by a desire to write. Apparently 'Like a Rolling Stone' suddenly flowed from his pen. 'It's like a ghost is writing a song,' Dylan has reportedly said. 'It gives you the song and it goes away.' But it's no ghost, according to Lehrer.

QU Y

NH

ƠN

Instead, the right hemisphere of the brain is assembling connections between past influences and making something entirely new. Neuroscientists have roughly charted this process by mapping the brains of people doing word puzzles solved by making sense of remotely connecting information. For instance, subjects are given three words - such as 'age', 'mile' and 'sand' - and asked to come up with a single word that can precede or follow each of them to form a compound word. (It happens to be 'stone'.) Using brain-imaging equipment, researchers discovered that when people get the answer in an apparent flash of insight, a small fold of tissue called the anterior superior temporal gyrus suddenly lights up just beforehand. This stays silent when the word puzzle is solved through careful analysis. Lehrer says that this area of the brain lights up only after we've hit the wall on a problem. Then the brain starts hunting through the 'filing cabinets of the right hemisphere' to make the connections that produce the right answer.

M

Studies have demonstrated it's possible to predict a moment of insight up to eight seconds before it arrives. The predictive signal is a steady rhythm of alpha waves emanating from the brain's right hemisphere, which are closely associated with relaxing activities. 'When our minds are at ease-when those alpha waves are rippling through the brain - we're more likely to direct the spotlight of attention towards that stream of remote associations emanating from the right hemisphere,' Lehrer writes. 'In contrast, when we are diligently focused, our attention tends to be towards the details of the problems we are trying to solve.' In other words, then we are less likely to make those vital associations. So, heading out for a walk or lying down are important phases of the creative process, and smart companies know this. Some now have a policy of encouraging staff to take time out during the day and spend time on things that at first glance are unproductive (like playing a PC game), but day-dreaming has been shown to be positively correlated with problem-solving. However, to be more imaginative, says Lehrer, it's also crucial to collaborate with people from a wide range of backgrounds because if colleagues are too socially intimate, creativity is stifled.

DẠ

Y

Creativity, it seems, thrives on serendipity. American entrepreneur Steve Jobs believed so. Lehrer describes how at Pixar Animation, Jobs designed the entire workplace to maximise the chance of strangers bumping into each other, striking up conversations and learning from one another. He also points to a study of 766 business graduates who had gone on to own their own companies. Those with the greatest diversity of acquaintances enjoyed far more success. Lehrer says he has taken all this on board, and despite his inherent shyness, when he's sitting next to strangers on a plane or at a conference, forces himself to initiate conversations. As for predictions that the rise of the Internet would make the need for shared working space obsolete, Lehrer says research shows the opposite has occurred; when people meet face-to-

Page 8 of 11

FI CI A

L

face, the level of creativity increases. This is why the kind of place we live in is so important to innovation. According to theoretical physicist Geoffrey West, when corporate institutions get bigger, they often become less receptive to change. Cities, however, allow our ingenuity to grow by pulling huge numbers of different people together, who then exchange ideas. Working from the comfort of our homes may be convenient, therefore, but it seems we need the company of others to achieve our finest 'eureka' moments. Questions 96-100

TRUE

if the statement agrees with the information

FALSE

if the statement contradicts the information

NOT GIVEN

if there is no information on this

Write your answers in the corresponding numbered box provided.

OF

Do the following statements agree with the information given in Reading Passage? For questions 96-100, choose:

96. It did not take long for the Swiffer to develop once the idea for it was conceived. 97. Lehrer was driven by his own experience of the ‘eureka’ moment.

ƠN

98. Lehrer refers to the singer Bob Dylan in order to propose particular approaches to regaining lost creativity. 99. Neuroscientists discovered from the word puzzle experiment that one part of the brain only becomes active when a connection is made suddenly.

Your answers 96. ______

97. ______

NH

100.Scientists know a moment of insight is coming because there is greater activity in the right side of the brain. 98. ______

99. ______

100. ______

• •

QU Y

For questions 101-105, complete the notes below. Choose ONE WORD ONLY from the passage for each answer. Write your answers in the corresponding numbered box provided. HOW OTHER PEOPLE INFLUENCE OUR CREATIVITY Steve Jobs: made changes to the (101)__________ to encourage interaction at Pixar. Lehrer: company owners must have a wide range of (102)__________ to do well. it’s important to start (103)__________ with new people

M

the (104)__________ has not replaced the need for physical contact. Geoffrey West: living in (105)__________ encourages creativity.

Your answers

101. ______________

102. ______________

103. ______________

104. ______________

105. ______________

DẠ

Y

D. WRITING (50 points) Part 1: The charts below show reasons for travel and the main issues for the travelling public in the US in 2019. (20 points) Summarise the information by selecting and reporting the main features and make comparisons where relevant. You should write about 150 words.

Page 9 of 11

L FI CI A OF ƠN NH

________________________________________________________________________________ ________________________________________________________________________________

QU Y

________________________________________________________________________________ ________________________________________________________________________________ ________________________________________________________________________________ ________________________________________________________________________________

M

________________________________________________________________________________

________________________________________________________________________________ ________________________________________________________________________________ ________________________________________________________________________________

Y

________________________________________________________________________________

DẠ

________________________________________________________________________________ ________________________________________________________________________________

Page 10 of 11

FI CI A

Discuss both views and give your opinion. You should write an essay of about 250 words.

L

Part 2. With the advent of Chat GPT, people think that students would enormously benefit from this A.I chatbot for their studies. However, many others believe that without teachers, students will do more harm than good when they use this type of chatbot. (30 points)

________________________________________________________________________________ ________________________________________________________________________________ ________________________________________________________________________________ ________________________________________________________________________________

OF

________________________________________________________________________________ ________________________________________________________________________________

ƠN

________________________________________________________________________________ ________________________________________________________________________________ ________________________________________________________________________________

NH

________________________________________________________________________________ ________________________________________________________________________________ ________________________________________________________________________________

QU Y

________________________________________________________________________________ ________________________________________________________________________________ ________________________________________________________________________________ ________________________________________________________________________________

M

________________________________________________________________________________

________________________________________________________________________________ ________________________________________________________________________________ ________________________________________________________________________________

Y

________________________________________________________________________________

DẠ

________________________________________________________________________________ -------------- HẾT -------------(Thí sinh không được sử dụng tài liệu. Cán bộ coi thi không giải thích gì thêm.

Page 11 of 11

FI CI A

L

KỲ THI HỌC SINH GIỎI CÁC TRƯỜNG THPT CHUYÊN KHU VỰC DUYÊN HẢI VÀ ĐỒNG BẰNG BẮC BỘ LẦN THỨ XIV, NĂM 2023

HƯỚNG DẪN CHẤM: MÔN TIẾNG ANH – LỚP 10

3. 7.00

4. covered market

5. Coffee Club

ƠN

(10 points – 2 points/a correct answer) 1. vegetarian 2. Wednesford

OF

A. LISTENING (50 points) Part 1. For questions 1-5, listen to a student asking for information about a healthy eating society and supply the blanks with the missing information. Write NO MORE THAN TWO WORDS OR NUMBERS taken from the recording for each answer in the space provided.

NH

Part 2. You will hear a conversation between a boy, Carl, and a girl, Susanna, about a school concert. For questions 6-10, decide whether the following statements are True or False by putting a tick (√) in the corresponding column. (10 points) 6. F 7. F 8. T 9. F 10. T

11. C

12. B

QU Y

Part 3. For questions 11-15, listen to a radio interview in which a psychologist, Colin Fraser, talks about cultural identity and choose the correct answer A, B, C, or D which fits best according to what you hear. Write your answers in the corresponding numbered boxes provided. 13. C

14. A

15. B

M

Part 4. For questions 16-25, listen to a report on G7 Summit held in France and supply the blanks with the missing information. Write NO MORE THAN FOUR WORDS taken from the recording for each answer in the space provided. 21. ravaging

17. speak with one voice

22. trade relations/ties

18. confrontational

23. key priority

19. cooperation

24. wine (imports)

20. Iran nuclear deal

25. heal growing divisions

DẠ

Y

16. show a united front

26.A 33.B 40.C

27.A 34.A 41.A

28.D 35.D 42.A

29.D 36.A 43.C

FI CI A

L

B. LEXICO – GRAMMAR (40 points) Part 1. For questions 26-45, choose the best option A, B, C, or D to complete the following sentences and write your answers in the corresponding numbered boxes provided on the answer sheet. (20 points – 1 point/a correct answer) 30.C 37.A 44.C

31.B 38.C 45.C

32.B 39.C

OF

Part 2. For questions 46-55, fill each gap with the correct form of the words in brackets. Write your answer in the boxes provided on the answer sheet. (10 points – 1 point/a correct answer) 51. Frighteners

47. Mellifluous

52. Freewheeling

48. Pictorially

53. Commodified/commoditized

49. Photogenic

54. Onerous

50. Desirous

55. Underreported

ƠN

46. Namesake

57. off

61. around

62. out of

58. to

QU Y

56. within

NH

Part 3. For questions 56-65, complete each of the following sentences with suitable preposition(s). Write your answer in the boxes provided on the answer sheet. (10 points – 1 point/a correct answer)

63. up

59. on

60. in

64. under

65. away

M

C. READING (60 points) Part 1. For questions 66-75, read the following passage and decide which option (A, B, C, or D) best fits each gap. Write your answers in corresponding numbered boxes on the answer sheet. (15 points – 1.5 points/a correct answer) 67. A 72. C

66. D 71. D

68. B 73. A

69. D 74. C

70. A 75. B

DẠ

Y

Part 2. For questions 76-85, fill each of the following numbered blanks with ONE suitable word and write your answers in the corresponding numbered boxes on the answer sheet. (15 points – 1.5 points/a correct answer) 76. path

77. settling

78. onward(s)

79. old

80. however

81. but

82. against

83. backdrop

84. which

85. least

FI CI A

L

Part 3. For questions 86-95, read the following passage and circle the best answer to each of the following questions. Write your answers in corresponding numbered boxes provided on the answer sheet. (15 points – 1.5 points/a correct answer) 86. A 87. A 88. B 89. D 90. B 91. C 92. B 93.A 94. D 95. C Part 4. For questions 96-105, read the passage and do the following tasks. Write your answers in the corresponding numbered boxes on the answer sheet. (15 points – 1.5 points/a correct answer) 97. NOT GIVEN

98. FALSE

99. TRUE

100. TRUE

101. workplace

102. acquaintances

103. conversations

104. Internet

105 cities

ƠN

D. WRITING (50 points) Part 1: 20 points Contents (10 points) The report MUST cover the following points:

OF

96. FALSE

QU Y

NH

- Introduce the charts (2 points) and state the overall trends and striking features (2 points) - Describe main features with relevant data from the charts and make relevant comparisons (6 points) Language use (10 points) The report: - should demonstrate a wide variety of lexical and grammatical structures - should have correct use of words (verb tenses, word forms, voice...) and mechanics (spelling, punctuations,)

DẠ

Y

M

Part 2: (30 points) The mark given to part 3 is based on the following criteria: 1. Organization (5 points) a. Ideas are well organized and presented with coherence, cohesion and unity. b. The essay is well-structured: * Introduction is presented with clear thesis statement. * Body paragraph are written with unity, coherence and cohesion. Each body paragraph must have a topic sentence and supporting details and examples when necessary. * Conclusion summarizes the main points and offers personal opinions (prediction, recommendation, consideration ...) on the issue. 2. Content (15 points) a. All requirements of the task are sufficiently addressed.

DẠ

Y

M

QU Y

NH

ƠN

OF

L

FI CI A

b. Ideas are adequately supported and elaborated with relevant and reliable explanations, examples, evidence.... 3. Language use (10 points) a. Demonstration of a variety of topic-related vocabulary. b. Excellent use and control of grammatical structures (verb tenses, word forms, voice...) and mechanics (spelling, punctuations...) THE END

KỲ THI HỌC SINH GIỎI CÁC TRƯỜNG THPT CHUYÊN KHU VỰC DUYÊN HẢI VÀ ĐỒNG BẰNG BẮC BỘ LẦN THỨ XIV, NĂM 2023 ĐỀ THI MÔN: TIẾNG ANH 11 Thời gian: 180 phút (Không kể thời gian phát đề)

L

TRƯỜNG THPT CHUYÊN TỈNH TUYÊN QUANG

FI CI A

ĐỀ THI ĐỀ XUẤT (Đề thi có 20 trang)

Ngày thi: 15/7/2023

A. LISTENING Part 1. For questions 1-5, you will hear a talk about epidemiology. Listen and decide whether

1.

2.

NH

ƠN

OF

the following sentences are true (T) or false (F). Write your answers in the corresponding numbered boxes on the answer sheet. (10 points) 1. Epidemiology involves a interdisciplinary approach to the study of human health and disease. 2. Epidemiology is commonly identified with both infectious and non-infectious diseases. 3. The epidemiologic triangle is made up of three components: an external agent, a host and an environment. 4. Large-scale social distancing recommended by a team of scientists from Imperial College London aimed to break the link between the external agent and the host. 5. The team’s recommendations became public policy in many countries. Your answer 3.

4.

5.

M

QU Y

Part 2. For question 6-10, listen to a talk about the future of fashion and answer the questions. WRITE NO MORE THAN FOUR WORDS taken from the recording for each answer in the recording numbered boxes provided. 6. What is capable of forcasting fashion trends with high precision? 7. As evidenced by H& M’s unsold clothes, what could have untold repercussions? 8. Besides superfluous production, what aspect of fashion could be diminished thanks to the adoption of machine learning? 9. To create new clothes, what will Amazon’s AI designer be doing aside from duplicating well-

known styles? 10. What is ZOZO? Your answer

DẠ

Y

6.______________________ 7.______________________ 8.______________________ 9.______________________ 10.______________________ 1

FI CI A

L

Part 3. You will hear an interview with Maria Stefanovich, co-founder of a creativity group which organises workshop executives. For questions 11-15, choose the answer (A, B, C or D) which fits best according to what you hear. 11. Corporations appreciate mask-making workhops because______ A. no one wants negative faces at the office. B. unhappy employees won’t come to work. C. they realise how their employees see them. D. their employees change their approach.

NH

ƠN

OF

12. Companies are turning to creative workshops because they have acknowledged that ______ A. unproductive employees are a financial burden. B. the traditional work environment has its limitations. C. there is an increase in absenteeism. D. employees are working too hard without enjoying it. 13. The employees at the firm “Play” ______ A. change positions frequently to lessen boredom. B. have business cards indicating their jobs. C. do not have stereotyped ideas about their jobs. D. dress up like comic book characters.

11.

112.

Your answer

M

QU Y

14. The companies that show most interest in creative workshops are surprising because ______ A. they usually have creative employees to begin with. B. their employees are the one who have to present regularly. C. there are many other exciting workshops they would prefer. D. their employees should be used to being funny. 15. Maria mentions the traditional companies that have held workshops in order to ______. A. boast about the clients her company has helped. B. show that they have a narrow list of clients. C. downplay the serious reputations of the films. D. point out the diversity of those trying different approaches.

13.

14.

15.

DẠ

Y

Part 4. For questions 16-25, you will listen to a presenter talking abour a phenomenon in the nature. Complete the summary by writing NO MORE THAN THREE WORDS in each gap. Write your answers in the corresponding numbered boxes on the answer sheet. (20 points – 2 points/a correct answer) (16)_________ are the building blocks of plant life. Plants naturally absorb nutrients from the soil through their root system. However, farmers turn to (17)_________ when facing poor soil or massive erosion or leaching. Farmers tend to overuse fertilizer as a precautionary measure because it is difficult to (18)_________ the amount of fertilizer needed. Excess fertilizer can 2

FI CI A

L

runoff into bodies of water causing (19)_________, characterized by the rapid accumulation in the population of algae in marine water systems. Dense layers of algae form an (20)_________ on the surface of the water, blocking other plants in the water from getting the sunlight they need to survive. Because water cannot support aquatic life, plants die off and sink to the bottom of the water body, where (21)_________ feast ont the dead bodies. This decomposition process consumes plenty of (22)_________ and animals that rely on oxygen to breathe can, in fact, suffocate. A vicious cycle of degradation in aquatic life can be observed and a (23)_________ is

created. If this process occurs in lakes, native species can be (24)_________ and give way to invasives. If this happens in the ocean, the incidence of coral bleaching is inevitable.

OF

(25)_________ and farming activities can lead to nutrient-rich runoff, also known as clearcutting. Your answer

NH

QU Y

22.______________________ 23.______________________ 24.______________________ 25.______________________

ƠN

16.______________________ 17.______________________ 18.______________________ 19.______________________ 20.______________________ 21.______________________

B. LEXICO – GRAMMAR (30 points)

Part 1. For questions 26-45, choose the best option A, B, C or D to complete the following sentences and write your answers in the corresponding numbered boxes provided. (20 points)

DẠ

Y

M

26. Google is the first major company to ______ refuse China’s demands for control. A. blissfully B.sorely C. steadfastly D. woefully 27. Although it takes quite a bit of time at the begginning, once you have acquired the basic knowledge, a quick learner like you will surely ______. A. forge ahead B. plunge ahead C. plough ahead D. press ahead 28. Do you think it’s better to ______ your anger than to supress it? A. assert B. repress C. arouse D. vent 29. Breaking his leg dealt a ______ to his chances of becoming a professional footballer. A. thump B. strike C. blow D. hit 30. I don’t know ______ the new manager. A. what to get B. what to make of C. how to get of D. how to make of 31. Jenny decided to draw a line ______ her recent breakup and move on. A. at B. under C. over D. with 3

FI CI A

L

32. It’s hard to believe that so many completely untrue stories are spread on social media. We are living in an era where ______ information is more credible than fact. A. full- scale B. low-key C. post-true D. short-lived 33. I only asked to move her car but she made such a(n) ______ about it. A.song and dance B. short and sweet C.cut and dried D. open and shut 34. 25 people have replied to the invitation, but I’ve ______ and put out 30 chairs. A. teetered on the brink B. erred on the side of caution

C. been on a zaror’s edge D. been in the teeth 35. She’s a bit down in the ________ at the moment – her husband has just lost his job.

NH

ƠN

OF

A. world B. bottom C. heart D. dumps 36. The company cannot accept_________for injuries resulting from improper use of rental equipment A. validity B. liability C. compensation D. privilege 37. You’ll just have to _________ yourself to the fact that you can’t always have what you want. A. acknowledge B. concede C. allow D. reconcile 38. A whole _______ of measures was tried in an attempt to get them to give up cigarettes. A. battery B. wood C. generation D. stream 39. Only the most basic and essential facts are required, stop adding more information, you are making _______ of the presentation, Emily!

M

QU Y

A. a song B. a dish C. a meal D. a scale 40. _______ tests assume that individuals have instrinc talents and limitations as well as a natural predisposition toward success or failure in various areas based on their innate qualities. A. Achievement B. Perception C. Acumen D. Aptitude 41. If you dare to go against everyone’s expectations, you will be _______. A. off for it B. for one C. for it D. off and on 42. Jason has _______ a fantastic job with one of our top newpapers. A. earned B. launched C. won D. landed 43. Despite dismal failures in the past, James still _______ his ambitions of playing professional soccer.

DẠ

Y

A. nurses B. cradles C. breeds D. rears 44. As a man with an eye for neatness around the place, Tom always ensured that things were _______. A. hale and hearty B. spick and span C. bright and breezy D. short and sweet 45. The books are a good cover to cover read for the recommended age group, with enough detail to ________ the imagination of inquiring young minds. A. incense B. foment C. kindle D. pique Your answer 26.

27.

28.

29.

30.

31.

32.

33.

34.

35.

36.

37.

38.

39.

40.

41.

42.

43.

44.

45. 4

FI CI A

L

Part 2. For questions 46-55, give the correct form of each given word to complete the following sentences and write your answers in the corresponding numbered provided. (10 points) 46. The loveliest scenes, he found, were comprised of the simplest, most natural _______________ of native plants. (POSITION) 47. Living in cities and towns, kids these days are much more than we ever were at their age. (STREET) 48. As in all the chapters, David maintain the excitement visually with pictures and page designs

that make such a book a _______________. (PAGE) 49. There are islands in Antartica and parts of northern Canada that are uninhabitable due to the

QU Y

NH

ƠN

OF

________________ of the weather. (CLEMENT) 50. A(n) ________________ political outlook can be misguided or even dangerous, because it doesn’t take the lessons of the past into account. (HISTORY) 51. Mind-alerting drugs weren’t necessary, since this was an evening of pure ______________ spontaneous and harmless fun. (ADULT) 52. The ________________ headed by Mr.Smith would be looking to develop hotel facilities adjoining the stadium. (CONSORT) 53. People living in very hot climates need to ensure that they get enough fluids into their bodies to prevent ________________. (HYDRATE) 54. Once a fine-looking manor, and easily the largest and grandest building for miles around, the Riddle House was now damp, ______________, and unoccupied. (RELIC) 55. I believe he has committed a(n) _____________ sin, so his punishment is well-deserved. (PARDON) Your answer 46. 47. 48. 49.

M

50.

51. 52. 53. 54. 55.

C. READING (60 points)

DẠ

Y

Part 1. For questions 56-65, fill each of the following numbered blanks with ONE suitable word and write your answers in the corresponding numbered provided. (15 points) Concentration is good in exams, bad in orange juice. Concentration (56) ______ when you manage to focus on one thing to the exclusion of all others, and concentrating on that one thing allows you to stop worrying about a lot of other things. Sometimes your mind concentrates when you don’t want it to. Maybe you can’t get something (57) ______ of your head, such as a problem you have to face up to, or an embarrassing situation you’ve been in. That’s why collecting things as a hobby is popular; it (58) ______ your mind off other things. Indeed, some people seem to prefer looking after and cataloguing their collections to actually (59) ______ anything with them, because this is when the absorbing, single- minded concentration happens. 5

FI CI A

L

The natural (60) ______ for concentration is 45 minutes. That’s why half an hour for a television programme seems too short (61) ______ an hour seems too long. But many people's lives are devoid of concentration. Modern culture is served up in small, easily digestible chunks (62) ______ require only a short attention span although young people can concentrate on computer games for days at a (63) ______ . Sticking out the tongue can aid concentration. This is because you can’t (64) ______ yourself with talking at the same time and other people won’t dare to (65) ______ your thoughts, because you look like an idiot! It would probably be better to concentrate your own mind before having it concentrated for you, but only people with a will of iron choose that route.

56.

58.

60.

62.

57.

59.

61.

63.

OF

Edited from https://www.theguardian.com/lifeandstyle/2006/sep/23/weekend.guybrowning Your answer 64.

65.

ƠN

Part 2. For questions 66-75, read the passage below and choose the answer A, B, C or D that fits best according to the text. Write your answers in the corrresponding numbered boxes provided. (10 points) LEARNING TO RUN

QU Y

NH

An article published recently in the prestigious scientific journal Nature is shedding new light on an important, but hitherto little appreciated, aspect of human evolution. In this article, Professors Dennis Bramble and Daniel Lieberman suggest that the ability to run was a crucial factor in the development of our species. According to the two scientists, humans possess a number of anatomical features that make them surprisingly good runners. ‘We are very confident that strong selection for running (A) ____was instrumental in the origin of the modern human body form,’ says Bramble, a biology professor at the University of Utah. Traditional thinking up to now has been that the distinctive, upright body form of modern humans has come about as a result of the ability to walk, and that running is simply a by-product of walking. Furthermore, humans have usually been regarded as poor runners compared to such

M

animals as dogs, horses or antelopes. However, this is only true if we consider fast running, or sprinting, over short distances. Even an Olympic athlete can hardly run as fast as a horse can gallop, and can only keep up a top speed for fifteen seconds or so. Horses, antelopes and greyhounds, on the other hand, can run at top speed for several minutes, clearly outperforming us in this respect. But when it comes to long-distance running, humans do astonishingly well (B)_____ They can maintain a steady pace for miles, and their overall speed compares favourably

DẠ

Y

with that of horses or dogs. Bramble and Lieberman examined twenty-six anatomical features found in humans. One of the most interesting of these is the nuchal ligament, a band of tissue that extends from a ridge on the base of the skull to the spine. When we run, it is this ligament that prevents our head from pitching back and forth or from side to side. Therefore, we are able to run with steady heads, held high. The nuchal ligament (C)_____is not found in any other surviving primates, although the 6

FI CI A

L

fossil record shows that hom*o erectus, an early human species that walked upright, much as we do, also had one. Then there are our Achilles tendons at the backs of our legs, which connect our calf muscles to our heel bones - and which have nothing to do with walking. When we run, these behave like springs, helping to propel us forward. Furthermore, we have low, wide shoulders, virtually disconnected from our skulls, an anatomical adaptation which allows us to run more

efficiently. Add to this our light forearms, which swing out of phase with the movement of our legs to assist balance, and one begins to appreciate the point that Bramble and Lieberman are trying to make.

But what evolutionary advantage is gained from being good long-distance runners? One

NH

ƠN

OF

hypothesis is that this ability may have permitted early humans to obtain food more effectively. ‘What these features and fossil facts appear to be telling us is that running evolved in order for our direct ancestors to compete with other carnivores for access to the protein needed to grow the big brains that we enjoy today,’ says Lieberman. Some scientists speculate that early humans may have pursued animals for miles in order to exhaust them before killing them. Running would also have conferred an advantage before weapons were invented: early humans might have been scavengers, eating the meat and marrow left over from a kill by lions or other large predators. They may have been alerted to the existence of a freshly-killed carcass by vultures (D)____, and the faster they got to the scene of the kill, the better. ‘Research on the history of human locomotion has traditionally been contentious,’ says

M

QU Y

Lieberman. ’At the very least, I hope this theory will make many people have second thoughts about how humans learned to run and walk and why we are built the way we are.’ 66. According to the text, the human ability to run ________ A. was only recently described in a scientific journal. B. is now regarded as more important than the ability to climb trees. C. played an important part in human evolution. D. is surprising when we consider evolutionary trends. 67. According to the text, scientists used to believe ________ A. that the human body owes its form to the ability to walk. B. the human ability to walk adversely affected the ability to run.

DẠ

Y

C. that only modern humans could walk upright. D. that humans can run because they stand upright. 68. According to the text, humans________ A. are better runners than most other animals. B. are not good at running short distances. C. cannot run at top speed for long distances. D. compare unfavourably with horses and dogs. 69. It appears that the nuchal ligament________ A. is found only in modern primates. B. is associated with the ability to run. C. prevents the head from moving D. is a unique anatomical feature. 70. The text implies that________ A. we do not need calf muscles in order to walk. 7

C. competed with other animals for food. walk.

FI CI A

72. According to the text, early humans________ A. killed animals by exhausting them.

L

B. without shoulders we could not run very fast. C. the movement of our forearms is out of phase. D. our Achilles tendons are an adaptation for running. 71. The pronoun “these” in the third paragraph refers to________ A. legs B. tendons C. muscles D. bones

B. may have evolved big brains for running. D. could probably run before they could

Your answer 67.

68.

69.

70.

71.

72.

73.

74.

75.

QU Y

66.

NH

ƠN

OF

73. Professor Lieberman hopes to________ A. dispel any remaining doubts about the nature of the human body. B. prove conclusively that humans did not always walk in an upright position. C. make people reconsider previously-held ideas about human anatomy. D. inform people of the real reason why humans are able to run and walk. 74. Which of the following spaces can the relative clause “- which came at the expense of the historical ability to live in trees -” fit? A. (A) B. (B) C. (C) D. (D) 75. The word “conferred” in the fourth paragraph can be best replaced by________. A. give out B. bring about C. refer to D. make out

M

Part 3. For questions 76-88, read the following passage and do the tasks that follow. (13 points) A Since 1901, the Nobel Prize has been honoring men and women from all corners of the globe for outstanding achievements in physics, chemistry, medicine, literature, and for work in peace. The foundations for the prize were laid in 1895 when Alfred Nobel wrote his last will, leaving much of his wealth to the establishment of the Nobel Prize. B

DẠ

Y

Alfred Nobel was born in Stockholm on October 21, 1833. His father Immanuel Nobel was an engineer and inventor who built bridges and buildings in Stockholm. In connection with his construction work, Immanuel Nobel also experimented with different techniques for blasting rocks. Successful in his industrial and business ventures, Immanuel Nobel was able, in 1842, to bring his family to St. Petersburg. There, his sons were given a first-class education by private teachers. The training included natural sciences, languages and literature. By the age of 17, Alfred Nobel was fluent in Swedish, Russian, French, English and German. His primary interests were in English literature and poetry as well as in chemistry and physics. Alfred’s father, who wanted his sons to join his enterprise as engineers, disliked Alfred’s interest in poetry and found his son rather introverted. 8

FI CI A

L

C In order to widen Alfred’s horizons, his father sent him abroad for further training in chemical engineering. During a two year period, Alfred Nobel visited Sweden, Germany, France and the United States. In Paris, the city he came to like best, he worked in the private laboratory of Professor T. J. Pclouze, a famous chemist. There he met the young Italian chemist Ascanio Sobrero who, three years earlier, had invented nitroglycerine, a highly explosive liquid. But it was considered too dangerous to be of any practical use. Although its explosive power greatly

exceeded that of gunpowder, the liquid would explode in a very unpredictable manner if subjected to heat and pressure. Alfred Nobel became very interested in nitroglycerine and how it

NH

ƠN

OF

could be put to practical use in construction work. He also realized that the safety problems had to be solved and a method had to be developed for the controlled detonation of nitroglycerine. D After his return to Sweden in 1863, Alfred Nobel concentrated on developing nitroglycerine as an explosive. Several explosions, including one (1864) in which his brother Emil and several other persons were killed, convinced the authorities that nitroglycerine production was exceedingly dangerous. They forbade further experimentation with nitroglycerine within the Stockholm city limits and Alfred Nobel had to move his experimentation to a barge anchored on Lake Malaren. Alfred was not discouraged and in 1864 he was able to start mass production of nitroglycerine. To make the handling of nitroglycerine safer Alfred Nobel experimented with

M

QU Y

different additives. He soon found that mixing nitroglycerine with kieselguhr would turn the liquid into a paste which could be shaped into rods of a size and form suitable for insertion into drilling holes. In 1867 he patented this material under the name of dynamite. To be able to detonate the dynamite rods he also invented a detonator (blasting cap) which could be ignited by lighting a fuse. These inventions were made at the same time as the pneumatic drill came into general use. Together these inventions drastically reduced the cost of blasting rock, drilling tunnels, building canals and many other forms of construction work. E The market for dynamite and detonating caps grew very rapidly and Alfred Nobel also proved himself to be a very skillful entrepreneur and businessman. Over the years he founded factories

DẠ

Y

and laboratories in some 90 different places in more than 20 countries. Although he lived in Paris much of his life he was constantly traveling. When he was not traveling or engaging in business activities Nobel himself worked intensively in his various laboratories, first in Stockholm and later in other places. He focused on the development of explosives technology as well as other chemical inventions including such materials as synthetic rubber and leather, artificial silk, etc. By the time of his death in 18%, he had 355 patents. F Intensive work and travel did not leave much time for private life. At the age of 43, he was feeling like an old man. At this time he advertised in a newspaper “Wealthy, highly-educated elderly gentleman seeks the lady of mature age, versed in languages, as secretary and supervisor of household.” The most qualified applicant turned out to be an Austrian woman, Countess 9

FI CI A

L

Bertha Kinsky. After working a very short time for Nobel she decided to return to Austria to marry Count Arthur von Suttner. In spite of this Alfred Nobel and Bertha von Suttner remained friends and kept writing letters to each other for decades. Over the years Bertha von Suttner became increasingly critical of the arms race. She wrote a famous book, Lay Down Your Arms and became a prominent figure in the peace movement. No doubt this influenced Alfred Nobel when he wrote his final will which was to include a Prize for persons or organizations who promoted peace. Several years after the death of Alfred Nobel, the Norwegian Storting (Parliament) decided to award the 1905 Nobel Peace Prize to Bertha von Suttner. G

NH

ƠN

OF

Alfred Nobel died in San Remo, Italy, on December 10, 1896. When his will was opened it came as a surprise that his fortune was to be used for Prizes in Physics, Chemistry, Physiology or Medicine, Literature and Peace. The executors of his will were two young engineers, Ragnar Sohlman and Rudolf Lilljequist. They set about forming the Nobel Foundation as an organization to take care of the financial assets left by Nobel for this purpose and to coordinate the work of the Prize-Awarding Institutions. This was not without its difficulties since the will was contested by relatives and questioned by authorities in various countries. H Alfred Nobel’s greatness lay in his ability to combine the penetrating mind of the scientist and inventor with the forward-looking dynamism of the industrialist. Nobel was very interested in

M

QU Y

social and peace-related issues and held what were considered radical views in his era. He had a great interest in literature and wrote his own poetry and dramatic works. The Nobel Prizes became an extension and a fulfillment of his lifetime interests. For questions 76-82, decide whether the following statements are True (T), False (F) or Not Given (NG). Write your answers in the corresponding numbered boxes provided Do the following statements agree with the information given in Reading Passage? In boxes 76-81 on your answer sheet, write TRUE if the statement is true FALSE if the statement is false NOT GIVEN if the information is not given in the passage

DẠ

Y

76. The first Nobel Prize was awarded in 1895. 77. Nobel’s father wanted his son to have a better education than what he had had. 78. Nobel was an unsuccessful businessman. 79. Bertha von Suttner was selected by Nobel himself for the first peace prize. 80. The Nobel Foundation was established after the death of Nobel 81. Nobel’s social involvement was uncommon in the 1800s. For questions 82-88, complete the notes below using NO MORE THAN TWO WORDS from the passage. Write your answers in the corresponding numbered boxes provided Education: Having accumulated a great fortune in his business, Nobel’s father determined to give his son the best education and sent him abroad to be trained in 82______ during Nobel’s study in Paris, he 10

worked in a private laboratory, where he came in contact with a young engineer 83______ and

FI CI A

L

his invention nitroglycerine, a more powerful explosiven than 84______ Benefits in construction works: Nobel became really interested in this new explosive and experimented on it. But nitroglycerine was too dangerous and was banned for experiments within the city of 85______. So Nobel had to move his experiments to a lake. To make nitroglycerine easily usable, Nobel invented dynamite along with 86______ while in the meantime 87______ became popular, all of which dramatically lowered the 88______ of construction works.

76.

77.

78.

79.

80.

82.

83.

84.

85.

86.

81.

OF

Your answer:

87.

88.

ƠN

Part 4: In the passage below, seven paragraphs have been removed. Read the passage and choose from paragraphs A-H the one which fits each gap. There is ONE extra pragraph which you do not need to use. Write your answers in the corresponding numbered boxes provided on the answer sheet. (7 points)

NH

“What do you want to be when you grow up?” a class of fresh-faced 12-year-olds were asked upon commencing secondary school. Their new English teacher- Mrs Marcus- asked this question every year and it seemed to fire the imagination of every child. Usually there was a smattering of professions, vocations and trades, along with some interesting surprises. This lot did not disappoint 89.

QU Y

It turns out they were a highly varied lot: doctor, nurse, lawyer, judge, electrician, archeologist, businesswoman, vet, police officer, hairdresser, actor, shop assistant. There was trouble containing their enthusiasm, with some throwing out more than one idea. A few had non-specific ambitions, ‘I don’t know. I want to travel,’ and ‘I just want to go to university.’ All of them had opinions, some stronger than others, but opinions nonetheless. 90.

M

I’m particularly interested in the differences between that generation and the current one. ‘Hopes and dreams,’ she replies immediately. ‘Whether your classmates achieved those things or not is irrelevant. The important thing is you had ideas about your future; you had aspirations. When I have asked that question in recent years, instead if setting their sights on becoming a scientist, a lawyer or an artist, the best some children could think of was going on the dole, being famous, or being the boss of a gang,’ she says.

Y

91.

It is a vicious circle that becomes increasingly difficult to break. It was crucial for my peers

DẠ

and I that we knew people who worked and we could make decisions about our ambitions based on some knowledge. We had the benefit of seeing our parents, relatives and neighbours going to work, returning from work, talking about their jobs, or their time at university. These experiences 11

informed our ideas, ambitions and, let’s face it, our expectations, too. We wouldn’t have dreamt of the alternative. After all, work and study were our means to get ahead and make our way in life.

L

92.

FI CI A

This lies at the very core of a gang’s appeal. The aimlessness of some youths’ experience is replaced by the rigid system of rules, rituals, and codes of behavior that members follow, and which gives them a purpose and adds much-needed structure to their lives. In many cases, the gang becomes a surrogate family, providing security, camaraderie and a sense of belonging. These powerful inducements exert a strange power over vulnerable teenagers. 93.

OF

I ask Margaret what it is that can drive such a change. ‘I’ve known many young boys who have turned over a new leaf,’ she says. ‘The key is intervention at the grassroots level. Community programmes that keep kids off the streets and involve them in pro-social activities are great deterrents. Strong after-school programmes that meet children’s needs for supervision are also successful in reducing attractionto gang-related activities. These cost money, though, and

ƠN

authorities are often not willing to spend,’ she explains, ‘and sadly, some kids fall through the cracks.’ 94.

NH

What chance for rehabilitation do they have, I wonder, when they cling to their gangs even in these circ*mstances? Admittedly, the need for survival plays a role since those in prison rely on their fellow gang members for protection. After all, prison is no picnic and is possibly more dangerous an environment than the outside world. But even in here, there is hope. 95.

QU Y

‘You can’t make anyone succeed, but you can help them to see that success in life is possible outside of the narrow confines of the gang,’ says Margaret. ‘If we give young people opportunities to bring about a change in their circ*mstances, they can build a happy future.’ Let’s hope that the next time Margaret asks ‘the question,’ there will be some scientists, entrepreneurs and plumbers in the group.

M

Paragraphs A. Success, however, can be measured in a myriad of ways, and for those without traditional role models, gang culture becomes increasingly alluring as a way to make something of themselves.

Y

‘They’re not inherently bad kids,’ says Marcus, ‘they just have no direction and no one to look up to. Were they to attach value to work and education, their whole outlook on life would change and they wouldn’t need what gang membership provides.’ B. Though there is no conclusive evidence, many critics of popular media believe exposure to

DẠ

violent films and song lyrics, particularly rap music, has a negative influence; glamorising gang life and encouraging at-risk youths to join gangs or to participate in gang-related crime as a means of gaining a sense of belonging and empowerment. C. Those who do join in a gang inevitably end up in a downward spiral, losing any moral foundation they had and hurtling headlong into a life of violence and crime. And yet, even when 12

FI CI A

L

they are placed in juveline detention centres, or worse- in adult prisons- some maintain their allegiance to the gang and look upon their membership as a badge of honour; a mark of success, not failure. D. Thick and fast came the replies. ‘Teacher,’ sai a bespectacled girl in the front row. Mrs Marcus smiled to herself. The prospective teachers always sat as close as possible to the board, eager and

serious. ‘Football player!’ shouted a tall lad from the back, raising his arms in victory as though he’d just scored a winning goal against Argentina at the World Cup. Mrs Marcus knew he’d be a live wire in class. ‘Prime minister,’ said another, garnering a round of applause as well as ridiculous from his classmates.

NH

ƠN

OF

E. How has it come to this? A recent report has found that children in some areas of the country have so little contact with working people that the concept of employment is almost foreign. They live in the so-called ‘welfare ghettos’ where more than half of the working -age population depends on out-of-work benefits. In many families, unemployment is intergenerational with grandparents and parents living on the dole. F. Thankfully, in many cases the lure if temporary. It becomes nothing more than a phase that plays to their fantasies or rebellion and desire for high drama, in part fueled by pop culture through music and films that glamorise thug life. In time, these wannabe gangsters find other interests and reject the values of the gang. G. Fast forward twenty years and Mrs Margaret Marcus is now teaching at an inner-city school in

Your answer

90.

91.

92.

93.

94.

95.

89.

M

QU Y

a large metropolis. Hoping to get some insight from this forty-year veteran of the education system, I’m interviewing her about the challenges faced by young people today. ‘So you became a journalist instead of a teacher’, she says with a twinkle in the eye. Yes, I was that child in the specs long ago. Before getting down to business, we reminisce for a few moments about my classmates. H. There are many organisations that are working positively with young people in gangs, both inside and outside of detention, and helping them through some very difficult times in their lives. With this help they can stop their slide into crime and violence, and make the tough transition of evolving into productive, responsible and law abiding of society.

DẠ

Y

Part 5: For questions 96-105, you are going to read an extract about food ans cooking. Answer the questions by choosing from the sections of the extract (A - D). The sections may be chosen more than once. Write your answers in the corresponding numbered boxes provided on the answer sheet. (15 points) A. Monosodium Gluctamate Good food is one of the life’s pleasures and even 1,200 years ago, oriental cooks knew that food tasted better when prepared with a soup stock made from a type of seaweed. But it was only in 1908 that Japanese scientists identified the ingredient responsible for enhancing flavour. 13

FI CI A

L

That ingredient is known today by its scientific name, monosodium glutamate. It is often reffered to as MSG and it is an amino acid found in virtually all foods. The bound form is linked to other amino acids in proteins and is manufactured in the human body. The free form of glutamate in foods enhances food flavours. Tomaoes, cheese and mushrooms are just some free glutamate rich foods. Free glutamate content increases during ripening, bringing out a fuller taste in many foods and is made as a flavour enhancer by a fermentation process similar to that used for making soy and vinegar.

People have long known about the four basic tastes-sweet, sour, salty and bitter. But now a fifth basic taste called umani has been recognised. This is imparted to foods by glutamate and is

NH

ƠN

OF

responsible for the savoury taste of many foods, such as tomatoes and cheese. B. Organic Food & Bunisess! Organic farmers pride themselves on fostering sustainable agriculture, but it remains to be seen if the industry’s rapid growth is in fact sustainable. One challenge facing the industry is to bring the price of organic products more in line with those of conventional products. The price of organic ingredients is improving but demand still outpaces suppply. However, supply issues are overshadowed by the fact that the organic foods sector continues to grow faster than the food industry as a whole, fundamentally due to the natural alliance between organic foods and processed foods. Firstly, organic foods earmarked for processing do not have to be as comestically perfect as their fresh counterparts. In addition,

M

QU Y

freezing or tinning reduces many of the shelf-life problems associated with fresh produce. It was only a question of time before mainstream food companies woke up to these synergies. The pioneers of the organic food industry view the growing presence of major food companies in their markets as a mixed blessing. Many smaller companies fear that the ideals of organic agrilculture will be compromised. Other think major food companies will help persuade consumers to buy organic products through the power of their branding. C. Chilli Capsic*ms, commonly known as chillis, come in all dimensions and colours from the tiny, pointed, extremely hot, bird’s eye chilli, to the large, mild, fleshy peppers like the Anaheim. Indigenous to Central and South America and the West Indies, they were cultivated long before

DẠ

Y

the Spanish conquest, which was the eventual cause of their introduction to Europe. Chillis along with tomatoes, avocados, vanilla and chocolate changed the flavours of the known world. Today, there are around 400 different varieties of chillis grown. They are easy to cultivate and are one of the world’s most widely distributed crops, available for sale at most food outlets. In 1902, a method was developed for measuring the strength of a given variety of capsicum, ranking it on predetermined scale. This originally meant tasting the peppers, but nowadays it can be done more accurately with the help of computers to rate the peppers in units to indicate parts per million of capsaicin. This potent chemical not only causes the fiery sensation, but also triggers the brain to produce endorphins, natural painkillers that promote a sense of well-being. D. Writing about cooking 14

FI CI A

L

Two cookery writers are often credited with the present revival of interest in food and cooking. Elizabeth David discovered her taste for good food when she lived with a French family for two years. After returning to England she learnt to cook so that she could reproduce some of the food that she had come to appreciate in France. Her first book appeared when rationing was still in force after the war and most of the ingredients she had so lovingly described were not available. At the time her books was read rather than used, and it created a yearning for good ingredients and for a way of life that saw more in food than mere sustenance. Her late books

confirmed her position as the most inspirational and influential cookery writer in the English language. She shared with Jane Grigson an absorbing interest in the literature of cookery.

OF

Jane Grigson was brought up in the north-east of England, where there is a strong tradition of good eating, but it was not until she began to spend time in France that she became really interested in food. She was renowned for her fine writing on food and cookery, often catching the imagination with a deftly chosen fragment of history or poetry, but never failing to explain the “why” as well as the “how” of cookery.

NH

ƠN

In which section are the following mentioned? 96. a group of foods that changed the way an area of the world cooked 97. a period of time access to food was restricted 98. a comparison of the process of producing a substance with that used for some other foods, too

QU Y

99. the global popularity of a particular food 100. an interest in discovering more about unfamiliar types of food 101. the discrepancy between the amount of a type of food produced and the demand for it 102. a substance that reinforces the savoury aspect of food 103. a way of determining the strength of a foodstuff 104. using literay forms to talk about food dishes 105. worries about the ethical future of a food industry (CPE Practice tests) Your answer 98.

M

97.

99.

100.

101.

102.

103.

104.

105.

96.

Y

D. WRITING (60 points)

DẠ

Part 1. Read the following extract and use your own words to summarize it. Your summary should be about 140 words. You MUST NOT copy the original. (15 pts) Achieving Peak Performance There are seven steps to achieve peak performance. The first step is to lead at well-rounded life. High achievers, according to experts, are obsessed people who take work home and then labor 15

FI CI A

L

over it until bedtime. Furthermore, research has also shown that such people tend to peak early and then go into a decline or level off. They then become addicted to work itself, with less concern for results. High performers, in contrast, are willing to work hard but within strict limits. For them, work is not everything and they know how to relax. They are able to leave work at the office. They value close friendships and family life, and spend a healthy amount of time with their families. The second step is to select a carcer you care about. Studies show that high performers choose work they truly prefer, and spend over two-thirds of their working hours doing it and only onethird on disliked tasks. They want internal satisfaction and not just external results such as pay

NH

ƠN

OF

rises and promotions. In the end, of course, they often have both. Since they enjoy what they do, they produce better work and the rewards are higher. Rehearsing each challenge or task mentally is the third step to achieving peak performance. Before any difficult or important situation - a public presentation, a board meeting, a key tennis match, for example - most peak performers run their desired actions through in their minds over and over again. Nearly all of us day-dream. about important coming events, but idle day-dreaming is not the same as a deliberate mental workout that sharpens the skills to be used in the activity. In order to achieve peak performance, you also have to seek results, not perfection. Many ambitious and hardworking people are so obsessed with perfection that they produce very little work. It has been found that those with perfectionist tendencies earned considerably less a year

M

QU Y

than those who did not have such tendencies. In contrast, high performers are almost always free of the compulsion to be perfect. They do not think of their mistakes as failures, but they learn from mistakes so that they can do better the next time The next step is to be willing to take risks. Most people are willing to settle for jobs which they think are secure, even if that also means mediocrity and boredom, rather than take changes. High performers, on the other hand, are able to take risks because they would carefully consider how they would adjust and how they would salvage the situation if, in reality they did fail. Constructing a 'worst-case' scenario allows them to make a rational choice. The penultimate step to achieving peak performance is not to underestimate your own, potential. Most of us think we know our own limits, but much of what we "know' is not knowledge at all. It

DẠ

Y

could be a belief which is erroneous and self-limiting. These types of beliefs are the biggest barriers to achieving high-level performance. Too many of us set our individual limits far below what we can actually achieve. High performers, on the contrary, are able to ignore artificial barriers. They concentrate instead on their own feelings, on their functioning, on the momentum of their effort and are therefore free to achieve peak levels. Finally, compete with yourself, not with others. High performers focus more on improving on their own previous efforts than on competing with others. Such are the skills of high performers. If you want to make the most of your talents and to live up to your fullest potential, learn to use these skills.

__________________________________________________________________ __________________________________________________________________ 16

__________________________________________________________________

L

__________________________________________________________________ __________________________________________________________________

FI CI A

__________________________________________________________________ __________________________________________________________________

NH

ƠN

OF

__________________________________________________________________ __________________________________________________________________ __________________________________________________________________ __________________________________________________________________ __________________________________________________________________ __________________________________________________________________ __________________________________________________________________ __________________________________________________________________ __________________________________________________________________ __________________________________________________________________ __________________________________________________________________ __________________________________________________________________ __________________________________________________________________

QU Y

__________________________________________________________________ __________________________________________________________________ __________________________________________________________________ __________________________________________________________________ Part 2. The bar chart below illustrates five different industries’ percentage share of Brazil’s

M

economy in 2009 and 2019 with a forecast for 2029. Summarise the information by selecting and reporting the main features, and make

DẠ

Y

comparisons where relevant. (15 points)

17

Industries as a percentage of Brazil's economy

L

45%

FI CI A

40% 35% 30% 25% 20% 15% 10%

0%

Finance

Food processing

Manufacturing 2009

2019

Tourism 2029

OF

5%

Oil, coal and gas

ƠN

__________________________________________________________________ __________________________________________________________________

M

QU Y

NH

__________________________________________________________________ __________________________________________________________________ __________________________________________________________________ __________________________________________________________________ __________________________________________________________________ __________________________________________________________________ __________________________________________________________________ __________________________________________________________________ __________________________________________________________________ __________________________________________________________________ __________________________________________________________________ __________________________________________________________________ __________________________________________________________________ __________________________________________________________________ __________________________________________________________________

DẠ

Y

__________________________________________________________________ __________________________________________________________________ __________________________________________________________________ __________________________________________________________________ __________________________________________________________________ 18

FI CI A

L

Part 3. Essay writing (30 points) Some people enjoy change and look forward to new experiences. Others like their lives to remain the same and do not like any changes. Discuss both views and give your opinion. Give reasons and relevant examples to support your answer. You should write at least 350 words.

__________________________________________________________________ __________________________________________________________________ __________________________________________________________________

OF

__________________________________________________________________ __________________________________________________________________ __________________________________________________________________ __________________________________________________________________ __________________________________________________________________

NH

ƠN

__________________________________________________________________ __________________________________________________________________ __________________________________________________________________ __________________________________________________________________ __________________________________________________________________ __________________________________________________________________ __________________________________________________________________ __________________________________________________________________

QU Y

__________________________________________________________________ __________________________________________________________________ __________________________________________________________________ __________________________________________________________________ __________________________________________________________________

M

__________________________________________________________________ __________________________________________________________________

DẠ

Y

__________________________________________________________________ __________________________________________________________________ __________________________________________________________________ __________________________________________________________________ __________________________________________________________________ __________________________________________________________________ __________________________________________________________________ __________________________________________________________________ 19

__________________________________________________________________

L

__________________________________________________________________ __________________________________________________________________

FI CI A

__________________________________________________________________ __________________________________________________________________

NH

ƠN

OF

__________________________________________________________________ __________________________________________________________________ __________________________________________________________________ __________________________________________________________________ __________________________________________________________________ __________________________________________________________________ __________________________________________________________________ __________________________________________________________________ __________________________________________________________________ __________________________________________________________________ __________________________________________________________________ __________________________________________________________________ __________________________________________________________________

QU Y

__________________________________________________________________ __________________________________________________________________

M

__________________________________________________________________ __________________________________________________________________ __________________________________________________________________ __________________________________________________________________ __________________________________________________________________

DẠ

Y

__________________________________________________________________ __________________________________________________________________ __________________________________________________________________ __________________________________________________________________ __________________________________________________________________ __________________________________________________________________ - THE END -

20

KỲ THI HỌC SINH GIỎI CÁC TRƯỜNG THPT CHUYÊN KHU VỰC DUYÊN HẢI VÀ ĐỒNG BẰNG BẮC BỘ LẦN THỨ XIV, NĂM 2023

FI CI A

MÔN: TIẾNG ANH – LỚP 11

L

TRƯỜNG THPT CHUYÊN TỈNH TUYÊN QUANG

HƯỚNG DẪN CHẤM (Hướng dẫn chấm gồm có 4 trang) A. LISTENING (50 points)

OF

Part 1. For questions 1-5, you will hear a talk about epidemiology. Listen and decide whether the following sentences are true (T) or false (F). Write your answers in the corresponding numbered boxes on the answer sheet. (10 points– 2 points/a correct answer) 1. T

2. F

3. T

4. F

5. T

NH

ƠN

Part 2. For question 6-10, listen to a talk about the future of fashion and answer the questions. WRITE NO MORE THAN FOUR WORDS taken from the recording for each answer in the recording numbered boxes provided. (10 points– 2 points/a correct answer) 6. cognitive computing 7. trendspotting mistakes 8. environmental footprint 9. analyzing images 10. (a) body measurement suit

QU Y

Part 3. You will hear an interview with Maria Stefanovich, co-founder of a creativity group which organises workshop executives. For questions 11-15, choose the answer (A, B, C or D) which fits best according to what you hear. (10 points– 2 points/a correct answer) 11. D

12. B

13. C

14. A

15. D

DẠ

Y

M

Part 4. For questions 16-25, you will listen to a presenter talking abour a phenomenon in the nature. Complete the summary by writing NO MORE THAN THREE WORDS in each gap. Write your answers in the corresponding numbered boxes on the answer sheet. (20 points – 2 points/a correct answer) 16. Nutrients 17. ferlilizer 18. gauge 19. algal bloom 20. impenetrable roof 21. decomposers 22. dissolved oxygen 23. positive feedback loop 24. suppressed 25. eutrophication

1

26. C

27. A

28. D

29. C

30. B

31. B

32. C

33. A

36. B

37. D

38. A

39. C

40. D

41. C

42. D

43. A

FI CI A

L

B. LEXICO – GRAMMAR (30 points) Part 1. For questions 26-45, choose the best option A, B, C or D to complete the following sentences and write your answers in the corresponding numbered boxes provided. (20 points – 1 point/a correct answer) 34. B

35. D

44. B

45. C

Part 2. For questions 46-55, give the correct form of each given word to complete the following

sentences and write your answers in the corresponding numbered provided. (10 points- 1 point/a correct answer) 51. unadulterated

47. streetwise

52. consortium

48. page-turner

53. dehydration

49. inclemency

54. derelict

50. ahistorical

55. unpardonable

ƠN

OF

46. juxtaposition

NH

C. READING (60 points) Part 1. For questions 56-65, fill each of the following numbered blanks with ONE suitable word and write your answers in the corresponding numbered provided. (15 points– 1.5 points/a correct answer) 58. takes

60. span

62. that

57. out

59. doing

61. whilst/ 63. time whereas/ albeit/ while

64. distract 65. interrupt

QU Y

56. happens/occurs

67. A

68. C

66. C

M

Part 2. For questions 66-75, read the passage below and choose the answer A, B, C or D that fits best according to the text. Write your answers in the corrresponding numbered boxes provided. (10 points – 1 point/a correct answer) 69. B

70. D

71. B

72. C

73. C

74. A

75. B

Y

Part 3. For questions 76-88, read the following passage and do the tasks that follow. (13 points– 1 point/a correct answer)

DẠ

76. F

77. NG

82. 83. chemical Ascanio engineering Sobrero

78. F

79. F

80. T

84. gunpower

85. 86. Stockholm detonator

81. T 87. pnematic drill

88. cost

2

89. D

90. G

91. E

92. A

FI CI A

L

Part 4: In the passage below, seven paragraphs have been removed. Read the passage and choose from paragraphs A-H the one which fits each gap. There is ONE extra pragraph which you do not need to use. Write your answers in the corresponding numbered boxes provided on the answer sheet. (7 points – 1 point/a correct answer) 93. F

94. C

95. H

97. A

98. D

99. A

100. D

101. D

102. C

103. B

104. C

105. B

ƠN

96. C

OF

Part 5: For questions 96-105, you are going to read an extract about food ans cooking. Answer the questions by choosing from the sections of the extract (A - D). The sections may be chosen more than once. Write your answers in the corresponding numbered boxes provided on the answer sheet. (15 points – 1 point/a correct answer)

NH

D. WRITING (60 points) Part 1. Read the following extract and use your own words to summarize it. Your summary should be about 140 words. You MUST NOT copy the original. (15 pts) a) Contents (10 points) The summary MUST cover the following points: * Main topic: (2 points)

M

QU Y

* Supporting ideas: (8 points) b) Language use (5 points) The summary: - should show attempts to convey the main ideas of the original text by means of paraphrasing (structural and lexical use) - should demonstrate correct use of grammatical structures, vocabulary, and mechanics (spelling, punctuations, ...) - should maintain coherence, cohesion, and unity throughout (by means of linkers and transitional devices)

Suggested Answer:

Y

There are seven steps which can lead to peak performance. Firstly, have a well-balanced life work hard but know when and how to relax and enjoy your life. The second step is to choose a career you love since internal satisfaction gives better results and rewards. The next step is to rehearse a

DẠ

task mentally before actually doing it. Another step is not to be a perfectionist but to be a risktaker, and to pursue results and learn from mistakes. High performers never underestimate themselves but concentrate on their capabilities. Lastly, compete only with yourself. not with others, (98 words)

3

points) Language use (5 points) The report: - should demonstrate a wide variety of lexical and grammatical structures,

FI CI A

L

Part 2: (15 points) Contents (10 points) The report MUST cover the following points: - Introduce the charts (2 points) and state the overall trends and striking features (2 points) - Describe main features with relevant data from the charts and make relevant comparisons (6

OF

- should have correct use of words (verb tenses, word forms, voice...) and mechanics (spelling,

NH

ƠN

Part 3: (30 points) The mark given to part 3 is based on the following criteria: 1. Organization (5 points) a. Ideas are well organized and presented with coherence, cohesion and unity. b. The essay is well-structured: * Introduction is presented with clear thesis statement. * Body paragraph are written with unity, coherence and cohesion. Each body paragraph must have a topic sentence and supporting details and examples when

M

QU Y

necessary. * Conclusion summarizes the main points and offers personal opinions (prediction, recommendation, consideration ...) on the issue. 2. Content (15 points) a. All requirements of the task are sufficiently addressed. b. Ideas are adequately supported and elaborated with relevant and reliable explanations, examples, evidence.... 3. Language use (10 points) a. Demonstration of a variety of topic-related vocabulary. b. Excellent use and control of grammatical structures (verb tenses, word forms, voice...) and mechanics (spelling, punctuations...)

DẠ

Y

- THE END -

4

Tổng Hợp Đề Đề Xuất Kì Thi Hsg Khu Vực Duyên Hải Và Đồng Bằng Bắc Bộ Môn Tiếng Anh Khối 10, 11 Năm 2023 (Có Đáp Án Và Phần Nghe) - PDFCOFFEE.COM (2024)

References

Top Articles
Latest Posts
Article information

Author: Tuan Roob DDS

Last Updated:

Views: 6553

Rating: 4.1 / 5 (62 voted)

Reviews: 85% of readers found this page helpful

Author information

Name: Tuan Roob DDS

Birthday: 1999-11-20

Address: Suite 592 642 Pfannerstill Island, South Keila, LA 74970-3076

Phone: +9617721773649

Job: Marketing Producer

Hobby: Skydiving, Flag Football, Knitting, Running, Lego building, Hunting, Juggling

Introduction: My name is Tuan Roob DDS, I am a friendly, good, energetic, faithful, fantastic, gentle, enchanting person who loves writing and wants to share my knowledge and understanding with you.